Efficiently Studying Organic Chemistry 3e by Eberhard Breitmaier

You might also like

Download as pdf or txt
Download as pdf or txt
You are on page 1of 323

Eberhard Breitmaier

Efficiently Studying Organic Chemistry


Eberhard Breitmaier
Eberhard Breitmaier
Günther Jung

Efficiently
OrganischeStudying
Chemie Organic Chemistry

Exam Training for Chemists, Biochemists, Pharmacists,


Life and Health Scientists

Third, Revised Edition


Author All books published by WILEY-VCH are carefully
produced. Nevertheless, authors, editors, and
Prof. Eberhard Breitmaier publisher do not warrant the information
Engelfriedshalde 46 contained in these books, including this
72076 Tübingen book, to be free of errors. Readers are advised
Germany to keep in mind that statements, data,
illustrations, procedural details or other items
may inadvertently be inaccurate.

Library of Congress Card No.: applied for

British Library Cataloguing-in-Publication Data


A catalogue record for this book is available
from the British Library.

The chapter Working on Questions is Bibliographic information published by


available as supplementary material on the Deutsche Nationalbibliothek
www.wiley-vch.de/ISBN9783527350568 The Deutsche Nationalbibliothek
lists this publication in the Deutsche
Nationalbibliografie; detailed bibliographic
data are available on the Internet at
<http://dnb.d-nb.de>.

2022 WILEY-VCH GmbH, Boschstraße 12,


69469 Weinheim, Germany

All rights reserved (including those of


translation into other languages). No part of
this book may be reproduced in any form – by
photoprinting, microfilm, or any other means
– nor transmitted or translated into a machine
language without written permission from the
publishers. Registered names, trademarks, etc.
used in this book, even when not specifically
marked as such, are not to be considered
unprotected by law.

Print ISBN: 978-3-527-35056-8


ePDF ISBN: 978-3-527-83849-3

Cover Design SCHULZ Grafik-Design, Fußgönheim, Germany


Typesetting Straive, Chennai, India

Printed on acid-free paper

10 9 8 7 6 5 4 3 2 1
11.6 Rearrangement . ..............................................................................29
Contents 12 Energy Turnover of Chemical Reactions . ....................... 30
12.1 Heat of Reaction, Activation Energy . ...............................................30
12.2 Catalysis . ........................................................................................30
Contents 12.3 Kinetic and Thermodynamic Control . ..............................................31

Contents ....................................................................................... 1 13 Radical Substitution .......................................................... 32


13.1 Photohalogenation of Alkanes. ........................................................32
Preface. ........................................................................................ 6 13.2 Relative Stability of Alkyl Radicals. ..................................................32
13.3 Regioselectivity of Radical Substitution ...........................................33
Organic Chemistry ...................................................................... 7 13.4 Radical Sulfochlorination and Nitration............................................33
1 Atomic Orbitals, Electronic Configurations . ...................... 8 14 Alkenes, Skeletal and Configurational Isomers . ............ 34
1.1 Atomic Orbitals. ...................................................................................8 14.1 Nomenclature and Structural Isomerism ..........................................34
1.2 s and p Orbitals . ..................................................................................8 14.2 Relative Configuration . .....................................................................34
1.3 Electron Spin and PAULI Principle. .......................................................8
1.4 Electronic Configuration of Light Atoms . .............................................9 15 Synthesis of Alkenes......................................................... 36
15.1 -Elimination . ..................................................................................36
2 Covalent Bonding................................................................ 10
15.1.1 Dehydrohalogenation of Haloalkanes . ..........................................36
2.1 Kinds of Chemical Bond . ...................................................................10
15.1.2 Dehydration of Alcohols . ..............................................................36
2.2 Covalent Bonding by Overlapping of Atomic Orbitals ........................10
15.1.3 Reductive Dehalogenation of 1,2-Dihaloalkanes ..........................37
2.3 Overlapping of p Orbitals . .................................................................11
15.2 Alternative Syntheses . ....................................................................37
3 Hybridization of Atomic Orbitals . ...................................... 12 15.2.1 Dehydrogenation of Alkanes . .......................................................37
3.1 Geometry of the Methane Molecule . .................................................12 15.2.2 Partial Hydrogenation of Alkynes . ................................................37
3.2 Hybridization of Atomic Orbitals . .......................................................12 15.2.3 Reductive Coupling of Carbonyl Compounds ...............................37
3.3 Carbon-Hydrogen Bonding in Methane .............................................13 15.2.4 Carbonyl Alkenylations . ................................................................38
15.3 Transformation of Alkenes . .............................................................38
4 Covalent Carbon-Carbon Bonding . ................................... 14 15.3.1 WOHL-ZIEGLER Bromination . .........................................................38
4.1 Ethane, CC Single Bond . ..................................................................14 15.3.2 HECK Reaction . ............................................................................38
4.2 Ethene, CC Double Bond . .................................................................14 15.3.3 Ene Reaction . ..............................................................................39
4.3 Ethyne, CC Triple Bond . ...................................................................15 15.3.4 Alkene Metathesis . .......................................................................39
5 Alkanes ................................................................................. 16 16 Additions to Alkenes ......................................................... 40
5.1 Homologous Series of Alkanes . ........................................................16 16.1 Addition of Hydrogen (Catalytic Hydrogenation) .............................40
5.2 Natural Sources and Preparation . .....................................................16 16.2 Addition of Bromine (Bromination). ..................................................40
5.2.1 Distillation of Petroleum . ................................................................16 16.3 Electrophilic Addition of Hydrogen Halide (Hydrohalogenation) ......41
5.2.2 Catalytic Hydrogenation of Alkenes. ...............................................17 16.4 Electrophilic Addition of Water (Hydration) ......................................41
5.2.3 WURTZ Synthesis Involving Alkylsodium .........................................17 16.5 Halohydrin Formation . .....................................................................42
5.2.4 KOLBE Electrolysis of Carboxylates (Anodic Oxidation) ..................17 16.6 Hydroboration . ................................................................................42
5.3 Alkanes as an Energy Source . ..........................................................17 16.7 Dihydroxylations . .............................................................................42
6 Skeletal Structure, Structural Isomerism . ........................ 18 16.8 1,3-Dipolar Cycloaddition of Ozone (Ozonolysis) ............................43
6.1 Two and More Structures for One Molecular Formula .......................18 17 Dienes ................................................................................. 44
6.2 Skeletal Isomerism . ...........................................................................18 17.1 Cumulation and Conjugation of Double Bonds ................................44
7 Basic Rules of Nomenclature . .......................................... 20 17.2 Molecular Structure . ........................................................................44
7.1 IUPAC Rules . ....................................................................................20 17.2.1 Conformation of 1,3-Butadiene . ...................................................44
7.2 Branched Alkyl Groups . ....................................................................20 17.2.2 Bond Lengths and Resonance Formulas of 1,3-Butadiene ..........44
17.2.3 Molecular Shape of Allene . ..........................................................45
8 Drawing Molecular Structures . .......................................... 22 17.3 Preparation . ....................................................................................45
8.1 Structural Formulas . ..........................................................................22 17.3.1 Catalytic Dehydrogenation of Alkanes to 1,3-Dienes....................45
8.2 Skeletal Formulas . ............................................................................22 17.3.2 Dehydration of Diols to 1,3-Dienes ...............................................45
8.3 LEWIS Formulas . ................................................................................22 17.3.3 Catalytic Dimerization of Ethyne to 1,3-Butadiene........................45
8.4 Projections .........................................................................................23 17.3.4 1,2-Dienes by Elimination. ............................................................45
8.4.1 FISCHER Projection . ........................................................................23
8.4.2 NEWMAN Projection . ........................................................................23
18 Additions and Cycloadditions with 1,3-Dienes . .............46
18.1 1,2- and 1,4-Addition . ......................................................................46
9 Conformation ....................................................................... 24 18.2 Cycloadditions . ................................................................................47
9.1 Conformation, Conformers . ...............................................................24 18.2.1 [4+2]-Cycloaddition (DIELS-ALDER Reaction) ................................47
9.2 Energy Contents and Nomenclature of Conformers ..........................24 18.2.2 [4+1]-Cycloaddition . .....................................................................47
10 Reactive Intermediates ..................................................... 26 19 Alkynes ...............................................................................48
10.1 Radicals . .........................................................................................26 19.1 Homologous Series, Structural Isomerism, Nomenclature ...............48
10.2 Carbenium Ions and Carbanions. ....................................................26 19.2 Preparation . .....................................................................................48
10.3 Carbenes . .......................................................................................27 19.2.1 Partial Oxidation of Methane to Ethyne ........................................48
19.2.2 Carbide Process. ..........................................................................48
11 Basic Types of Organic Reactions . ................................. 28 19.2.3 Double Dehydrohalogenation of 1,1- or 1,2-Dihaloalkanes .........49
11.1 Addition ............................................................................................28 19.2.4 Alkylation of Terminal Alkynes . ....................................................49
11.2 Elimination . .....................................................................................28 19.3 Typical Reactions . ...........................................................................49
11.3 Oxidation ..........................................................................................28 19.3.1 Hydrogenation and Reduction . .....................................................49
11.4 Reduction . .......................................................................................28 19.3.2 Hydroboration . .............................................................................49
11.5 Substitution . ....................................................................................28 19.3.3 Electrophilic Addition of Halogens . ...............................................50
2 Contents

19.3.4 Electrophilic Addition of HX .......................................................... 50 29 Non-benzenoid Aromatic Compounds ............................. 74


19.3.5 CH Acidity of Terminal Alkynes, Alkynylides ................................ 51 29.1 Non-benzenoid Aromatic Ions ......................................................... 74
19.3.6 Oxidative Coupling of Terminal Alkynes....................................... 51 29.1.1 Cyclopropenium Cation................................................................ 74
19.3.7 Cyclotri- and Cyclotetramerization ............................................... 51 29.1.2 Cyclopentadienide Anion ............................................................. 74
20 Cycloalkanes ...................................................................... 52 29.1.3 Cycloheptatrienium Cation ........................................................... 75
20.1 Nomenclature ................................................................................. 52 29.2 [n]Annulenes ................................................................................... 75
20.2 Conformation .................................................................................. 52 30 Alkyl Halides ....................................................................... 76
20.2.1 Cyclopropane............................................................................... 52 30.1 Classification, Nomenclature........................................................... 76
20.2.2 Cyclobutane ................................................................................. 52 30.2 Preparation ..................................................................................... 76
20.2.3 Cyclopentane ............................................................................... 53 30.2.1 Radical Substitution of Alkyl Groups ............................................ 76
20.2.4 Cyclohexane ................................................................................ 53 30.2.2 Addition of Hydrogen Halides and Halogens to Alkenes .............. 76
20.3 Configurational Isomerism .............................................................. 54 30.2.3 Substitution of Hydroxide in Alcohols by Halide ........................... 77
20.3.1 cis- and trans-Disubstituted Cycloalkanes ................................... 54 30.2.4 Fluorination with Antimony Trifluoride .......................................... 77
20.3.2 cis- and trans-Decalin .................................................................. 55 30.2.5 Nucleophilic Iodination of Alkyl Halides (FINKELSTEIN Reaction) ... 77
20.3.3 Cycloalkenes ............................................................................... 55 30.3 Electronegativity and the Inductive Effect ....................................... 78
21 Basic Syntheses of Cycloalkanes and Cycloalkenes .... 56 30.4 Typical Reactions ........................................................................... 78
21.1 Cyclopropane.................................................................................. 56 30.4.1 Nucleophilic Substitution of Halide in Haloalkanes ....................... 78
21.2 Cyclobutane .................................................................................... 56 30.4.2 Dehydrohalogenation (-Elimination)........................................... 79
21.3 Cyclopentene, Cyclopentane .......................................................... 56 30.4.3 Metalation .................................................................................... 79
21.4 Cyclohexane, Cyclohexene ............................................................ 56 31 Mechanisms of Nucleophilic Substitution ....................... 80
21.5 Cycloheptadiene, Cycloheptane ..................................................... 57 31.1 Bimolecular Nucleophilic Substitution (Second-Order) .................... 80
21.6 Larger Rings ................................................................................... 57 31.2 Monomolecular Nucleophilic Substitution (First-Order) ................... 81
22 Reactions of Cycloalkanes and Cycloalkenes................ 58 32 Organometal Compounds ................................................. 82
22.1 Reactions Driven by the Strain of Small Rings ................................ 58 32.1 General Survey ............................................................................... 82
22.2 Alkane-like Reactions ..................................................................... 58 32.2 Preparation ..................................................................................... 82
22.3 Alkene-like Reactions ..................................................................... 58 32.2.1 Metalation of Alkyl and Aryl Halides ............................................. 82
22.3.1 Addition of Bromine...................................................................... 58 32.2.2 Transmetalation ........................................................................... 82
22.3.2 Catalytic Hydrogenation ............................................................... 59 32.2.3 Halogen-Metal Exchange ............................................................. 83
22.3.3 Dihydroxylations .......................................................................... 59 32.2.4 Hydrogen-Metal Exchange .......................................................... 83
23 Benzene, Aromaticity, Aromatic Compounds ................ 60 32.3 Preparative Significance ................................................................. 83
23.1 Structure of Benzene ...................................................................... 60 33 Alcohols ............................................................................... 84
23.1.1 Molecular Shape .......................................................................... 60 33.1 Nomenclature, Classification........................................................... 84
23.1.2 Heat of Hydrogenation ................................................................. 60 33.2 Structure and Physical Properties ................................................... 84
23.1.3 Resonance Energy and Stabilization, Canonical Formulas .......... 60 33.3 Preparation ..................................................................................... 85
23.2 Molecular Orbital Model of Benzene ............................................... 61 33.3.1 Industrial Syntheses of Methanol and Ethanol ............................. 85
23.3 Criteria of Aromaticity...................................................................... 61 33.3.2 Alcoholic Fermentation (Beer, Wine, Distillates) .......................... 85
24 Benzenoid Aromatic Compounds .................................... 62 33.3.3 Hydration of Alkenes .................................................................... 85
24.1 Monosubstituted Benzenes ............................................................. 62 33.3.4 Hydroboration of Alkenes and Oxidation of Trialkylboranes ......... 86
24.2 Multiply Substituted Benzenes ........................................................ 62 33.3.5 Reduction of Carbonyl Compounds by Complex Metal Hydrides . 86
24.3 Preparation of Benzenoid Hydrocarbons ........................................ 63 33.3.6 Nucleophilic Substitution of Alkyl Halides..................................... 86
24.3.1 Fossil Sources ............................................................................. 63 33.3.7 Addition of Alkylmagnesium Halides to Carbonyl Compounds ..... 87
24.3.2 Cyclotrimerization of Alkynes ....................................................... 63 34 Diols, Triols ......................................................................... 88
25 Electrophilic Substitution of Benzene ............................. 64 34.1 Preparation ..................................................................................... 88
25.1 Substituted Benzenes by Electrophilic Substitution: Mechanism..... 64 34.1.1 Dihydroxylation of Alkenes ........................................................... 88
25.2 Electrophilic Halogenation .............................................................. 64 34.1.2 Hydrolysis of Halohydrins ............................................................ 88
25.3 Electrophilic Alkylation (FRIEDEL-CRAFTS Alkylation)........................ 64 34.1.3 Bimolecular Reduction of Carbonyl Compounds .......................... 89
25.4 Electrophilic Acylation (FRIEDEL-CRAFTS Acylation) ......................... 65 34.2 Oxidative Cleavage of Glycols ........................................................ 89
25.5 Electrophilic Nitration ...................................................................... 65 35 Reactions of Alcohols ........................................................ 90
25.6 Electrophilic Sulfonation.................................................................. 65 35.1 Basicity and Acidity ......................................................................... 90
26 Electrophilic Second Substitution of Benzenes ............. 66 35.2 Oxidation ........................................................................................ 90
26.1 Resonance Effects of Substituents at the Benzene Ring ................ 66 35.3 Nucleophilic Substitution ................................................................. 91
26.2 Regioselectivity of Electrophilic Second Substitution of Benzenes.. 66 35.4 Esterification ................................................................................... 91

27 Other Reactions of Benzenoid Aromatics ....................... 68 36 Dehydration of Alcohols .................................................... 92


27.1 Nucleophilic Substitution at the Benzenoid Ring ............................. 68 36.1 Dehydration of Alcohols to Alkenes ................................................ 92
27.2 Radical Substitution at the Side Chain ............................................ 68 36.2 Dehydration of Fully Alkylated 1,2-Diols to Ketones........................ 93
27.3 Hydrogenation, Reduction, Oxidation.............................................. 69 37 Ethers ................................................................................... 94
28 Polycyclic Benzenoid Aromatic Compounds ................. 70 37.1 Nomenclature .................................................................................. 94
28.1 Fusion of Benzenoid Rings ............................................................. 70 37.2 Structure and Physical Properties .................................................... 94
28.2 Preparation of Aromatic Polycycles ................................................ 70 37.3 Preparation ...................................................................................... 94
28.3 Electrophilic Substitution of Naphthalene ........................................ 71 37.3.1 Bimolecular Dehydration of Alcohols............................................ 94
28.4 Oxidation and Reduction of Naphthalene........................................ 72 37.3.2 Alkoxylation of Alkyl Halides (WILLIAMSON Synthesis) .................. 94
28.5 Reactions of Anthracene and Phenanthrene .................................. 72 37.4 Typical Reactions ............................................................................ 95
28.6 Enzymatic Epoxidation of Benzo[a]pyrene ...................................... 73 37.4.1 Formation of Oxonium Salts......................................................... 95
37.4.2 Autoxidation (Insertion of Oxygen) ............................................... 95
Contents 3

37.4.3 Cleavage of Ethers.......................................................................95 46 Diastereomers ...................................................................124


38 Amines................................................................................. 96 46.1 Compounds with Two Different Stereogenic Centers ....................124
38.1 Nomenclature, Classification ...........................................................96 46.2 Compounds with Two Equally Substituted Stereogenic Centers ...125
38.2 Molecular Shape .............................................................................96 47 Aldehydes ..........................................................................126
38.3 Preparation ..................................................................................... 97 47.1 Survey, Nomenclature...................................................................126
38.3.1 Alkylation of Ammonia..................................................................97 47.2 Preparation ................................................................................... 127
38.3.2 Primary Amines by Alkylation of Potassium Phthalimide ..............97 47.2.1 Oxidation of Methyl and Hydroxymethyl Groups.........................127
38.3.3 Primary Amines by Reduction of Nitriles and Nitro Compounds ...97 47.2.2 Hydrolysis of 1,1-Dihaloalkanes .................................................127
39 Reactions of Amines.......................................................... 98 47.2.3 Reduction of Carboxylic Acid Derivatives ...................................127
39.1 Basicity of Alkyl- and Arylamines ....................................................98 47.2.4 Formylation of Arenes ................................................................127
39.2 Diazotization of Primary Amines .....................................................98 47.3 Molecular Shape, Resonance Formulas, Reactivity ......................128
39.3 N-Nitrosation of Secondary Amines ................................................99 47.4 Reactions Specific for Aldehydes ..................................................128
39.4 Exhaustive Alkylation of Amines .....................................................99 47.4.1 Oxidation to Carboxylic Acids, Identification Reactions ..............128
39.5 HOFMANN Elimination of Tetraalkylammonium Hydroxides ..............99 47.4.2 CANNIZZARO Disproportionation of Aromatic Aldehydes ..............129
39.6 Imines from Primary Amines and Carbonyl Compounds ...............100 47.4.3 Addition of Hydrogensulfite ........................................................129
39.7 Enamines from Secondary Amines and Carbonyl Compounds .....100 48 Ketones ..............................................................................130
39.8 Reductive Amination of Carbonyl Compounds to Amines .............101 48.1 Survey, Nomenclature...................................................................130
40 Diazo and Azo Compounds ............................................ 102 48.2 Preparation ................................................................................... 131
40.1 Arenediazonium Salts and Azo Dyes ............................................102 48.2.1 Oxidation of Secondary Alcohols................................................131
40.2 Azoalkanes ...................................................................................103 48.2.2 Catalytic Oxidation of Alkenes by Air (WACKER Process)............131
40.3 Diazoalkanes ................................................................................103 48.2.3 Oxidation of Activated Methylene Groups (RILEY Oxidation) ......131
48.2.4 Acylation of Arenes to Phenones (FRIEDEL-CRAFTS Acylation)....131
41 Carboxylic Acids .............................................................. 106
41.1 Survey, Nomenclature...................................................................106 49 Carbonyl Reactions ..........................................................132
41.2 Carboxy Group: Bonding and Resonance Formulas .....................106 49.1 Reactions with Oxygen and Sufur Nucleophiles ............................132
41.3 Carboxylic Acid Dimers .................................................................107 49.1.1 Hydration (Water as Nucleophile)...............................................132
41.4 Preparation ................................................................................... 107 49.1.2 Formation of Acetals and Ketals (Alcohols as Nucleophiles) ......132
41.4.1 Carbonylation .............................................................................107 49.1.3 Formation of Thioacetals (Mercaptals) .......................................132
41.4.2 Carboxylation .............................................................................107 49.2 Reactions with Nitrogen Nucleophiles ...........................................133
41.4.3 Oxidation of Methyl, Hydroxymethyl, and Aldehyde Groups .......108 49.3 Reactions with Carbon Nucleophiles .............................................133
41.4.4 Hydrolysis of Carboxylic Acid Derivatives ..................................108 49.3.1 Alkynylation ................................................................................133
41.5 Acidity 109 49.3.2 Cyanohydrin and Benzoin Reaction ...........................................134
49.3.3 Addition of GRIGNARD Compounds .............................................134
42 Carboxylic Acid Derivatives ............................................ 110 49.3.4 WITTIG Alkenylation (Carbonyl Alkenylation)...............................134
42.1 Carboxylic Acid Esters ..................................................................110 49.4 Reductions ....................................................................................135
42.2 Carboxylic Acid Halides (Acyl Halides)..........................................111
42.3 Carboxylic Acid Anhydrides...........................................................111 50 CH Acidity of Carbonyl Compounds ..............................136
42.4 Carboxylic Acid Amides, Cyclic Imides..........................................111 50.1 CH Acidity of Carboxylic Acid Esters .............................................136
42.5 Hydrazides, Hydroxamic Acids, Azides .........................................112 50.1.1 CLAISEN Condensation ...............................................................136
42.6 Change of the Carboxy Function...................................................112 50.1.2 DIECKMANN Cyclocondensation...................................................136
42.6.1 Reduction to Primary Alcohols and Aldehydes ...........................112 50.2 CH Acidity of Aldehydes and Ketones ...........................................136
42.6.2 Reductive Coupling of Esters (Acyloin Reaction) .......................113 50.2.1 Aldol Reaction ............................................................................137
42.6.3 Decarboxylation .........................................................................113 50.2.2 CLAISEN Condensation ...............................................................137
42.6.4 Dehydration of Carboxamides to Nitriles and Isonitriles .............113 50.2.3 MANNICH Reaction ......................................................................137

43 Substituted Carboxylic Acids ......................................... 114 51 1,3-Dicarbonyl Compounds .............................................138


43.1 Nomenclature................................................................................114 51.1 CH Acidity .....................................................................................138
43.2 Halo Acids .....................................................................................115 51.2 Typical Reactions ..........................................................................138
43.2.1 Preparation ................................................................................ 115 51.2.1 Alkylation and Cycloalkylation ....................................................138
43.2.2 Reactions ...................................................................................115 51.2.2 Carbonyl Alkenylation (KNOEVENAGEL Alkenylation) ....................139
43.3 Hydroxy Acids ...............................................................................116 51.2.3 Nucleophilic Addition to CC Double Bonds (MICHAEL Addition) ..139
43.3.1 Preparation ................................................................................ 116 51.2.4 Oxo-Enol Tautomerism ..............................................................140
43.3.2 Reactions ...................................................................................117 51.2.5 Cyclizations ................................................................................141

44 Absolute Configuration ................................................... 118 52 Phenols ..............................................................................142


44.1 Stereogenic Center, Enantiomers, Chirality .................................. 118 52.1 Nomenclature................................................................................142
44.2 Optical Activity and Specific Rotation ............................................118 52.2 Resonance Formulas, Acidity Relative to Alcohols .......................142
44.3 Specification of the Absolute Configuration ...................................118 52.3 Preparation ................................................................................... 143
44.3.1 CAHN-INGOLD-PRELOG Convention [(R)- and (S)-Descriptors] .....119 52.3.1 HOCK Process (Synthesis of Phenol and Acetone) .....................143
44.3.2 FISCHER Convention (D- and L-Descriptors) ...............................119 52.3.2 Hydrolysis of Substituted Chlorobenzenes .................................143
44.3.3 Correlation of D,L and R,S Descriptors ......................................120 52.3.3 Catalytic Oxidation of Methylarenes ...........................................143
44.3.4 Racemates and Their Resolution ...............................................121 52.3.4 Melting of Arenesulfonates with Alkali Hydroxides .....................144
44.4 Stereospecificity of the Bimolecular Nucleophilic Substitution .......121 52.3.5 Hydrolysis of Arenediazonium Salts ...........................................144
52.4 Typical Reactions ..........................................................................144
45 Enantiomers without Carbon as Stereogenic Center .. 122 52.4.1 Conversion into Aryl Ethers (WILLIAMSON Synthesis) ..................144
45.1 Heteroatoms as Stereogenic Centers ...........................................122 52.4.2 Esterification and FRIES Rearrangement ....................................144
45.2 Axial Chirality ................................................................................122 52.4.3 Electrophilic Substitution ............................................................145
45.3 Planar Chirality and Helicity ..........................................................123 52.4.4 Oxidation ....................................................................................145
4 Contents

53 Quinones ........................................................................... 146 59.3.2 Syntheses with Silyl Enol Ethers ................................................ 168
53.1 Survey and Nomenclature ............................................................ 146 59.4 Silicones ....................................................................................... 169
53.2 Preparation ................................................................................... 146 60 Heteroalicycles ................................................................. 170
53.2.1 Oxidation of Phenols and Primary Arenamines .......................... 146 60.1 Nomenclature ............................................................................... 170
53.2.2 Oxidation of Polycyclic Aromatics .............................................. 147 60.2 Preparation ................................................................................... 171
53.2.3 FRIEDEL-CRAFTS Acylation of Arenes with Phthalic Anhydride .... 147 60.2.1 Intramolecular Cyclizations ........................................................ 171
53.3 Reactions...................................................................................... 147 60.2.2 Cycloadditions ........................................................................... 171
53.3.1 Reduction-Oxidation Quinone-Hydroquinone Equilibrium .......... 147 60.2.3 Catalytic Hydrogenation of Aromatic Heterocycles .................... 172
53.3.2 Autoxidation of Anthrahydroquinone .......................................... 148 60.3 Reactions...................................................................................... 172
53.3.3 Additions .................................................................................... 148 60.3.1 Heteroatom as Nucleophile ........................................................ 172
53.3.4 Electrophilic Substitutions of Benzenoid Rings ......................... 149 60.3.2 Ring Opening ............................................................................. 173
53.3.5 Carbonyl Reactions.................................................................... 149 60.3.3 Ring Expansion .......................................................................... 173
54 Organosulfur Compounds .............................................. 150 61 Five-Membered Aromatic Heterocycles ......................... 174
54.1 Sulfur in Organic Compounds ....................................................... 150 61.1 Survey and Nomenclature ............................................................ 174
54.2 Organosulfur Compounds with Bivalent Sulfur .............................. 150 61.2 -Excessive Aromatic Heterocycles ............................................. 174
54.2.1 Thiols, Thiophenols, Disulfides .................................................. 150
61.3 Typical Syntheses ......................................................................... 175
54.2.2 Thioethers .................................................................................. 151
61.3.1 Furan, Pyrrole, Thiophene ......................................................... 175
54.2.3 Sulfenic Acid Derivatives ........................................................... 151
61.3.2 Azoles ........................................................................................ 175
54.5.4 Thioaldehydes, Thioketones ...................................................... 152
61.4 Typical Reactions ......................................................................... 176
54.5.5 Thiolic Acids, Thionic Acids, Dithiocarboxylic Acids ................... 152
61.4.1 Basicity and Acidity of Pyrrole .................................................... 176
54.3 Compounds with Tetra- and Hexavalent Sulfur ............................. 152
61.4.2 Electrophilic Substitutions .......................................................... 176
54.3.1 Sulfoxides and Sulfones ............................................................ 152
61.4.3 1,3-Diene Reactions .................................................................. 176
54.3.2 Sulfinic and Sulfonic Acids ......................................................... 152
61.4.4 Nucleophilic Substitutions .......................................................... 177
54.3.3 Sulfonic Acid Derivatives ........................................................... 153
61.4.5 Ring Opening ............................................................................. 177
55 Carbonic Acid Derivatives .............................................. 154 62 Six-Membered Aromatic Heterocycles ........................... 178
55.1 Survey of Derivatives .................................................................... 154
62.1 Survey and Nomenclature ............................................................ 178
55.2 Carbonic Acid Chlorides ............................................................... 154
62.2 -Deficient Aromatic Heterocycles................................................ 178
55.2.1 Phosgene................................................................................... 154
62.3 Typical Syntheses ......................................................................... 178
55.2.2 Carbonic Acid Ester Chlorides ................................................... 154
62.3.1 Pyridines .................................................................................... 178
55.3 Carbonic Acid Esters .................................................................... 155
62.3.2 Pyrimidines ................................................................................ 179
55.3.1 Dialkyl Carbonates, Dialkyl Dicarbonates .................................. 155
62.3.3 Pyrylium Salts ............................................................................ 179
55.3.2 Carbamic Acid Esters (Urethanes) ............................................. 155
62.4 Typical Reactions ......................................................................... 180
55.4 Urea, Thiourea, Guanidine ............................................................ 155
62.4.1 Reactions at the Imino Nitrogen ................................................. 180
55.4.1 Urea ........................................................................................... 155
62.4.2 Nucleophilic Substitutions .......................................................... 180
55.4.2 Guanidine and Thiourea ............................................................ 156
62.4.3 Electrophilic Substitutions .......................................................... 181
55.5 Derivatives of Dithio- and Trithiocarbonic Acid .............................. 157
62.4.4 CH Acidity of Methyl Groups ...................................................... 181
56 Heterocumulenes ............................................................. 158
63 Benzo-Fused Five-Membered Heteroaromatics ............ 182
56.1 Analogues of Carbon Dioxide ....................................................... 158
63.1 Survey and Nomenclature ............................................................ 182
56.2 Carbon Disulfide ........................................................................... 158
63.2 Typical Syntheses ......................................................................... 182
56.3 Isocyanates, Isothiocyanates ........................................................ 158
63.2.1 Benzo[b]furan, Benzo[b]thiophene ............................................. 182
56.4 Carbodiimides ............................................................................... 159
63.2.2 Benzo[b]pyrrole (Indole) ............................................................. 182
57 Rearrangements ............................................................... 160 63.2.3 Benzo-1,3-azoles ....................................................................... 183
57.1 Anionotropic 1,2-Shifts .................................................................. 160 63.2.4 Carbazole .................................................................................. 183
57.1.1 General Mechanisms (Sextet Rearrangements) ....................... 160 63.3 Typical Reactions ......................................................................... 183
57.1.2 1,2-Shifts from Carbon to Carbon .............................................. 161 63.3.1 Electrophilic Substitutions .......................................................... 184
57.1.3 1,2-Shifts from Carbon to Oxygen .............................................. 161 63.3.2 Cycloadditions ........................................................................... 184
57.1.4 1,2-Shifts from Carbon to Nitrogen............................................. 161 63.3.3 Reactions of 2- and 3-Hydroxy Derivatives ................................ 185
57.2 Cationotropic 1,2-Shifts................................................................. 162
64 Benzo-Fused Six-Membered Heteroaromatics.............. 186
57.2.1 FAVORSKII Rearrangement (from Carbon to Carbon) .................. 162
64.1 Survey and Nomenclature ............................................................ 186
57.2.2 STEVENS Rearrangement (from Nitrogen to Carbon) .................. 162
64.2 Typical Syntheses ......................................................................... 186
57.2.3 WITTIG Rearrangement (from Oxygen to Carbon) ...................... 162
64.2.1 Quinolines .................................................................................. 186
57.3 Rearrangements at Benzenoid Rings ........................................... 163
64.2.2 Isoquinolines .............................................................................. 187
57.4 Sigmatropic Rearrangements ....................................................... 163
64.2.3 Benzopyrylium Salts .................................................................. 187
58 Polymers, Polymerization ............................................... 164 64.3 Typical Reactions ......................................................................... 187
58.1 Monomers, Oligomers, Polymers .................................................. 164 64.3.1 Basicity and Reactions at the Imino Nitrogen ............................. 187
58.2 Vinyl and Diene Polymers ............................................................. 164 64.3.2 Catalytic Hydrogenation and Oxidative Ring Opening................ 188
58.3 Polyethers ..................................................................................... 165 64.3.3 Nucleophilic Additions ................................................................ 188
58.4 Polyesters ..................................................................................... 165 64.3.4 Nucleophilic Substitutions .......................................................... 188
58.5 Polyamides ................................................................................... 166 64.3.5 Electrophilic Substitutions .......................................................... 189
58.6 Polyurethanes, Polyureas ............................................................. 167 64.3.6 CH Acidity of Methyl Groups ...................................................... 189
59 Syntheses with Organosilicon Compounds ................. 168 65 Fused Aromatic Heterocycles ......................................... 190
59.1 Comparison of Organosilicon and Organic Compounds ............... 168 65.1 Heterobicycles with Nitrogen as Bridgehead ................................. 190
59.2 Halosilanes ................................................................................... 168 65.2 Purines ......................................................................................... 190
59.3 Preparative Significance of Trimethylsilyl Compounds .................. 168 65.2.1 Survey ....................................................................................... 190
59.3.1 Trimethylsilylation ...................................................................... 168 65.2.2 Syntheses of Purines ................................................................. 191
Contents 5

65.2.3 Oxidative Cleavage of Purines (Uric Acid) ................................. 192 76.3.4 Triterpenes .................................................................................223
65.3 Pteridines ......................................................................................192 76.3.5 Tetraterpenes (Carotenoids) ......................................................223
65.3.1 Survey........................................................................................192 76.3.6 Polyterpenes ..............................................................................223
65.3.2 Syntheses of Pteridines .............................................................192
77 Steroids ..............................................................................224
66 Absorption of Light, Color, Dyes.................................... 194 77.1 Survey, Fusion of the Rings ..........................................................224
66.1 Absorption of Light, Color..............................................................194 77.2 Cholesterol ....................................................................................224
66.2 Dyes and Pigments ....................................................................... 195 77.3 Bile Acids ......................................................................................224
66.3 Basic Types of Dyes .....................................................................196 77.4 Steroid Hormones .........................................................................225
66.3.1 Structural Properties of Dyes: Azo Dyes ....................................196
66.3.2 Polymethine Dyes ......................................................................196
78 Selectivity and Specificity of Organic Reactions ..........226
66.3.3 Triarylmethine Dyes ...................................................................197 78.1 Chemoselectivity ...........................................................................226
66.3.4 Carbonyl Dyes ...........................................................................197 78.2 Regioselectivity .............................................................................226
78.3 Stereoselectivity ............................................................................227
67 Porphyrinoids ................................................................... 198 78.4 Stereospecificity ............................................................................228
67.1 Porphyrins and Phthalocyanines as Polyaza[18]annulenes ..........198
67.2 Porphyrinoids in Blood and Chloroplasts.......................................198
79 Prochirality, Enantioselectivity .......................................230
67.2.1 Heme .........................................................................................198 79.1 Prochirality of Tetrahedral Carbon Atoms .....................................230
67.2.2 Chlorophyll .................................................................................199 79.2 Prochirality of Trigonal Carbon Atoms ...........................................230
79.3 Enantioselectivity ..........................................................................230
68 Amino Acids ..................................................................... 200
68.1 General Survey .............................................................................200
80 Planning Organic Syntheses ...........................................232
68.2 Preparation ................................................................................... 201 80.1 Retrosynthetic Disconnections ......................................................232
68.3 Identification .................................................................................. 201 80.2 Retrons and Synthons...................................................................232
80.3 Designing Selected Syntheses......................................................234
69 Peptides, Proteins ............................................................ 202 80.3.1 2-Ethyl-2-hexenal .......................................................................234
69.1 Amino Acid Sequence ...................................................................202 80.3.2 Ethyl 2,4-Dioxoheptanoate .........................................................234
69.2 Biological Function ........................................................................202 80.3.3 2-(4-Isobutylphenyl)propanoic Acid ............................................235
69.3 Structure of Proteins .....................................................................202 80.3.4 9-Tetrahydrocannabinol ...........................................................235
69.4 Peptide Synthesis .........................................................................203
69.4.1 Protective Groups ......................................................................203 81 Aspects of Molecular Structure ......................................236
69.4.2 Carboxy Activation .....................................................................204 81.1 Molecular Formula and Double Bond Equivalents .........................236
69.4.3 Peptide Coupling ........................................................................205 81.2 Skeletal Structure: Atom Connectivities ........................................236
81.3 Conformation ................................................................................236
70 Alkaloids ........................................................................... 206 81.4 Relative Configuration ...................................................................237
70.1 Origin, Significance, Nomenclature ...............................................206 81.5 Absolute Configuration ..................................................................237
70.2 Biologically Active Alkaloids ..........................................................206
82 Mass Spectrometry ...........................................................238
71 Carbohydrates: Aldoses and Ketoses ........................... 208 82.1 Mass Spectrum .............................................................................238
71.1 Aldoses .........................................................................................208 82.2 Base Ion, Molecular Ion ................................................................238
71.2 Ketoses .........................................................................................208 82.3 Fragment Ions and Atom Connectivities........................................239
71.3 Cyclohemiacetals, Cyclohemiketals: Pyranoses, Furanoses .........209
71.4 Mutarotation ..................................................................................210 83 Infrared Spectroscopy......................................................240
71.5 Typical Reactions ..........................................................................210 83.1 IR Spectrum ..................................................................................240
71.5.1 Glycosides, Glycosidation ..........................................................210 83.2 Molecular Vibrations......................................................................240
71.5.2 O-Alkylation, O-Acylation ...........................................................211 83.3 Identification of Functional Groups and Structural Units ................241
71.5.3 Reduction and Oxidation ............................................................211 84 Nuclear Magnetic Resonance: Proton NMR ..................242
72 Carbohydrates: Oligo- and Polysaccharides ................ 212 84.1 Nuclear Magnetic Resonance .......................................................242
72.1 Oligosaccharides ..........................................................................212 84.2 Chemical Shift ...............................................................................242
72.2 Polysaccharides ............................................................................213 84.3 NMR Spectrum and Integration .....................................................243
84.4 Signal Multiplets and Coupling Constants .....................................243
73 Nucleic Acids: DNA and RNA ......................................... 214 84.4.1 Signal Multiplets .........................................................................243
73.1 Nucleotides, Nucleosides, Nucleobases .......................................214 84.4.2 Coupling Constants and Relative Configuration .........................244
73.2 Base Pairing and Double Helix of DNA .........................................215
85 Nuclear Magnetic Resonance: Carbon-13 NMR ............246
74 Lipids ................................................................................. 216 85.1 Carbon-13 as NMR Probe .............................................................246
74.1 Classification .................................................................................216 85.2 Carbon-13 Chemical Shifts ...........................................................246
74.2 Fatty Acids, Fats, Soaps ...............................................................216 85.3 Carbon-Proton Coupling and Detection of CH Multiplets...............247
74.2.1 Saturated and Unsaturated Fatty Acids......................................216
74.2.2 Waxes, Soaps, Biofuel ...............................................................217 86 Nuclear Magnetic Resonance: Two-Dimensional NMR 248
86.1 Homonuclear Shift Correlation ......................................................248
75 Polyketides ....................................................................... 218 86.1.1 Proton-Proton Shift Correlation (HH COSY) ...............................248
75.1 Polyketide Pathway .......................................................................218 86.1.2 Carbon-Carbon Shift Correlation ................................................248
75.2 Selected Polyketides .....................................................................218 86.2 Heteronuclear Shift Correlation (Carbon-Proton Correlation) ........249
76 Terpenes ........................................................................... 220 Working on Questions..............................................................250
76.1 Survey, Isoprene Rule...................................................................220
76.2 Occurrence, Significance ..............................................................221 Subject Index .............................................................................295
76.3 Selected Terpenes (Flavors, Fragrances, Active Substances) ......221 Periodic Table of the Elements ...............................................314
76.3.1 Hemi- and Monoterpenes ...........................................................221
76.3.2 Sesquiterpenes ..........................................................................222 Selected Reference Sources ...................................................316
76.3.3 Diterpenes..................................................................................222
6 Preface

Preface

Reforms in Europe harmonized the conditions and curricula of the studies of chemistry and other natural sciences, im-
proving the international compatibility of academic degrees (bachelor and master). Therefore, concise, sufficiently de-
tailed, rounded textbooks are increasingly demanded, enabling students to efficiently prepare themselves for written and
oral examinations.

This has been the driving force to structure and to draw up this volume which is not intended to be or to replace a com-
prehensive textbook. Rather, it has been written to facilitate an efficient learning, covering the essentials of organic chemi-
stry in 86 short chapters in a total of about 300 pages, thus enforcing a very disciplined selection of the material and a
succinct style of writing with a minimum of repetition. When adapted to different curricula of universities and individual
demands of instructors, studying can be limited to a selection of the chapters presented in this text.

Based on the author’s teaching and examination experience of more than 40 years, this book and its graphics have been
developed from scripts of his lectures in organic chemistry and molecular spectroscopy held in the USA and in Germany.
It comprises the molecular orbital model to explain covalent bonding in organic molecules, the classes of organic com-
pounds including natural products, polymers and biopolymers, basic concepts (orbital hybridization, resonance, aromatici-
ty), types and mechanisms of organic reactions, and essential aspects of molecular structure such as atom connectivities,
skeletal isomerism, conformation, configuration, and chirality, including a very brief strategic introduction to structure
elucidation by molecular spectroscopy. New chapters deal with selectivity and specificity of organic reactions, prochirality
and enantioselectivity, planning organic syntheses, two-dimensional and carbon-13 NMR in the third revised edition.

Each chapter is designed as a short learning unit, presented at one glance on one or two pairs of pages (two-page view),
ending with at least three questions very closely related to the chapter’s contents. Thus, when answers cannot be given
spontaneously, they are easily found by reading (and understanding) the text once again. These questions offer an option
of self-examination. In few cases, obvious analogous conclusions, variations of reaction equations or the reader’s own
reflections are expected as the answers which are provided in the appendix “Working on Questions” and free of charge
on the website www.wiley-vch.de/ISBN9783527350568.

Many thanks are due to Dr. Kay Greenfield (Brisbane, Australia) for proofreading the first and second editions. - Any
suggestions for corrections or improvement are very welcome for future electronic updating of this text.

Tübingen (Germany), summer 2021 Eberhard Breitmaier

for Stefanie
Organic Chemistry 7

Organic Chemistry
Organic Chemistry
Traditionally, chemistry is appropriately subdivided into inorganic, organic and physical chemistry as well as biochemistry.
Organic chemistry deals with organic compounds. Organic compounds are not only metabolites and natural products
isolated from living organisms and their fossil remains, as was assumed until the beginning of the 19th century, but can
also be prepared by synthesis. WÖHLER´s 1828 preparation of urea, the physiological end-product of nitrogen excretion in
mammals, exemplified the first synthesis of an organic compound from the inorganic salt ammonium isocyanate.

_ NH2
heat
O C N
_ NH4 O C
NH2
ammonium isocyanate urea

All organic compounds contain the element carbon. Thus, organic chemistry can be defined as the chemistry of carbon
compounds, including their chemical structure, properties, syntheses and reactions.
Naturally occurring in the crystalline modifications graphite and diamond, carbon represents the softest and hardest solid
element under normal conditions. Moreover, carbon is one of the most varied elements of the periodic table:
The carbon atom is tetravalent. It is able to form single, double and triple bonds with its own kind as well as with the
atoms of many other elements. Carbon atoms are able to close rings with themselves and with other kinds of atoms, so-
called heteroatoms, thus forming carbocycles and heterocycles. As many as 217 theoretical hydrocarbon structures can
be written for the simple molecular formula C6H6. Bonds originating from the carbon atom are neither too weak nor too
strong, meaning that organic compounds may undergo chemical reactions under moderate or physiological conditions.
This explains the inexhaustibly large number of organic compounds. Several ten millions have already been documented.
About 90 percent are synthetic substances; the remaining compounds have been isolated as natural products (Chapters
68-77), biosynthesized by microorganisms, fungi, plants, and animals.
Apart from carbon (C), organic compounds and natural products predominantly contain the non-metal elements hydro-
gen, oxygen, nitrogen, sulfur, phosphorus (H, O, N, S, P) and halogens. Organometal compounds (Chapter 32) charac-
terized by carbon-metal bonds and useful as highly reactive reagents in organic synthesis are only synthetically available.
Organic compounds are divided into classes, each characterized by a functional group. A functional group is a particular
arrangement of atoms which gives rise to a set of typical physical properties and chemical reactions. Hydroxy groups
(OH), for example, define the reactivity of alcohols (Chapter 33) and phenols (Chapter 52), and carboxy groups
(COOH) that of carboxylic acids (Chapter 41).
CH and CC bonds are the most prominent bonds of organic compounds. Both are covalent bonds. The formation, the
geometry, the interactions, and the polarity of these bonds explain the structure, the properties, and the reactions of
organic compounds. A clear picture of covalent single and multiple bonds is provided by the molecular orbital model
(Chapters 1-4) which has been developed from quantum mechanical calculations.

________________________________________________________________________________________________
This introduction permits answers to the following:
How is the field of organic chemistry defined?
Which elements are the predominant constituents of organic compounds?
What are the reasons behind the large number of organic compounds?
What is a functional group?
8 1 Atomic Orbitals, Electronic Configurations

1 Atomic Orbitals, Electronic Configurations


1.1 Atomic Orbitals
Physical experiments (electron diffraction, COMPTON effect) provide evidence that electrons do not only behave as parti-
cles but as standing waves as well. Mathematically, the wave nature of the electron is described by the SCHRÖDINGER
equation (1). This differential equation correlates the wave function  of the electron with its total energy E.
H = E (1)
(E: total energy, : wave function, H: HAMILTON operator)

The SCHRÖDINGER equation (1) is only resolvable for selected values of the energy E. These values correspond to the
energy levels of the electron in an atom, defined by the quantum numbers 1, 2, 3, ... . Electronic energy is said to be
quantized. The wave function  of an electron on a particular energy level E is the solution of the SCHRÖDINGER equation
for that specific energy E. While this particular wave function  of the electron does not have any concrete meaning, its
square, 2, describes the space around the atomic nucleus where the electron is most likely to be found. This "space of
residence" or simply the "living room" of an electron described by  2 is referred to as the atomic orbital (electron cloud).

1.2 s and p Orbitals


Spherosymmetric atomic orbitals with the atomic nucleus as center are referred to as s orbitals (Fig. 1.1).

y x
Fig. 1.1. s orbital: spherosymmetric distribution of the electron about the atomic nucleus

On the lowest energy level E1 (quantum number n = 1) the electron occupies the 1s orbital; this applies for the hydrogen
atom. The 2s orbital following at the second energy level E2 (n = 2) concentrically envelops the 1s orbital. The 2s orbital is
occupied by an electron in the lithium atom, following the helium atom of the first row of the periodic table (Table 1.1) with
a doubly occupied 1s orbital.
Beginning with the second energy level E2 (n = 2), there are three additional dumpbell-shaped atomic orbitals on energy
levels slightly elevated relative to the 2s orbital (Fig. 1.2). In contrast to the spheric s orbitals, these orbitals are directed,
extending along the axes x, y and z; therefore, they are called the 2px, 2py and 2pz orbitals. All three 2p orbitals possess
the same energy and for this reason they are said to be degenerate.
z z z

y x y x y x

px py pz

Fig. 1.2. p Orbitals (px, py, pz) along the coordinate axes; p orbital halves in the range of negative sign are shaded

1.3 Electron Spin and PAULI Principle


In addition to their negative charge and their impulse to circumrotate the nucleus, electrons possess a spin (electron
spin). They may spin either clockwise or counterclockwise. When two electrons spin in the same manner, they are said to
have parallel spins and this situation is symbolized by two arrows in the same direction (). When, however, two elec-

Efficiently Studying Organic Chemistry: Exam training for chemists, biochemists, pharmacists, life and health scientists,
Third Edition. Eberhard Breitmaier. © 2022 WILEY-VCH GmbH. Published 2022 by WILEY-VCH GmbH.
1.4 Electronic Configuration of Light Atoms 9

trons spin in opposite directions, they are said to have antiparallel spins, and this is symbolized by two arrows in opposite
direction (). Electrons with antiparallel spin are also said to be paired.
Only two electrons can occupy the same orbital, provided they have antiparallel spins (). This restriction is known as
the PAULI principle.

1.4 Electronic Configuration of Light Atoms


The distribution of electrons in the orbitals of an atom is called the electronic configuration of that atom. The electronic
configuration represents the ground state of an atom, that is the state of lowest potential energy and highest stability. The
main constituent atoms C, H, O, N of organic compounds are among the light elements of the periodic table. Their elec-
trons occupy only s and p orbitals. Three rules regulate the occupation of orbitals by electrons:
- The orbitals are occupied in order of increasing energy: first 1s, then 2s, followed by 2px, 2py, 2pz.
- Only up to two electrons can occupy the same orbital. Double occupation requires electronic spins to be anti-
parallel (PAULI principle).
- When a set of degenerate orbitals is available, for instance the three 2p orbitals, single occupation takes place
before any of these orbitals becomes doubly occupied (HUND rule, cf. the electronic configuration of the ele-
ments C, N, O in Table 1.1).
The electronic configuration of an atom (Table 1.1) is described by quoting the occupied orbitals in the order of increasing
energy. The number of electrons in each orbital, 1 or 2, is indicated by the superscript 1 or 2 at the corresponding orbitals;
the 1 for single occupation is usually omitted. Boron as an example has the electronic configuration 1s2 2s2 2p (1s2 2s2
2p1), indicating double occupation for the 1s and 2s orbitals and single occupation for one 2p orbital.

Table 1.1. Electronic configuration of light atoms in the ground state


atom occupation representation
1s 2s 2px 2py 2pz

H 1s

He 1s2

Li 1s2 2s

Be 1s2 2s2

B 1s2 2s2 2p
C 1s2 2s2 2p2

N 1s2 2s2 2p3

O 1s2 2s2 2p4

F 1s2 2s2 2p5


Ne 1s2 2s2 2p6
(1s2 2s2 2px2 2py2 2pz2 )

________________________________________________________________________________________________
Chapter 1 permits answers to the following:
(1.1) What is an atomic orbital?
(1.2) How do p orbitals differ from s orbitals?
(1.3) How do p orbitals differ among themselves?
(1.4) Write the orbital occupancy for the atoms in the first two rows of the periodic table.
(1.5) Is the tetravalency of carbon in accordance with the electronic configuration of carbon in Table 1.1?
10 2 Covalent Bonding

2 Covalent Bonding
2.1 Kinds of Chemical Bond
Ionic and covalent bonds are the main types of chemical bonding. Instead of molecules, ions exist in inorganic salts such
as sodium chloride in which the ions are held together by the ionic bond. Ionic bonding is the electrostatic attraction of
oppositely charged ions. Thus, the sodium ion Na attracts and is attracted by the chloride anions Clin the crystal
lattice of sodium chloride.
In contrast, organic compounds do exist as molecules held together by covalent bonds. Covalent bonding, also referred
to as electron pair bonding, arises from sharing of electrons between two atoms. This is exemplified by the hydrogen
molecule in which two hydrogen nuclei, two protons H, are held together by an electron pair between them:

electron pair or covalent bond


H H H:H H H
two H atoms one H2 molecule

2.2 Covalent Bonding by Overlapping of Atomic Orbitals


In terms of the orbital model, a covalent bond arises from overlapping of atomic orbitals. When two H atoms, for example,
are brought closely enough together, their two singly occupied 1s atomic orbitals overlap (Fig. 2.1 b). A doubly occupied
molecular orbital arises, enclosing both protons H in the hydrogen molecule (Fig. 2.1, HH bond length 74 pm, 1 pm = 1
picometer = 1012 m).

H H H H

74 pm

(a) (b) (c)

Fig. 2.1.  Molecular orbital of the hydrogen molecule H2: (a) shape; (b) cross section; (c) the bonding electrons are most likely to be
found in the red shaded area of overlap

Mathematically, overlapping of atomic orbitals involves an addition and subtraction (linear combination) of the wave
functions 1 and 2 attributed to the overlapping atomic orbitals:
 = N (1 + 2) * = N (1  2)

N is a factor for standardization. The resulting two wave functions are assigned to two molecular orbitals, one more stable
bonding  molecular orbital with lower energy and another less-stable antibonding * molecular orbital with higher energy
(Fig. 2.2).
The square of the wave function, 2, describes the shape of the electron cloud which is the distribution of electron density
about the atomic nuclei. Squaring for the bonding molecular orbital  yields
2 = [N (1 + 2)] 2 = N2 (12 + 22 + 212)

and for the antibonding molecular orbital *


*2 = [N (1  2)] 2 = N2 (12  2 2  212) .

To conclude, an amount of 212 increases the electron density in the bonding  molecular orbital relative to the total of
the atomic orbitals, 12  22. The additional term 212 adopts a maximum where 1 and 2 culminate. This is the area
between the nuclei in the center of the bond where the atomic orbitals overlap (Fig. 2.1 c). The electron cloud accumu-

Efficiently Studying Organic Chemistry: Exam training for chemists, biochemists, pharmacists, life and health scientists,
Third Edition. Eberhard Breitmaier. © 2022 WILEY-VCH GmbH. Published 2022 by WILEY-VCH GmbH.
2.3 Overlapping of p Orbitals 11

lates there, thus overcompensating the electrostatic repulsion of the positively charged atomic nuclei. Overall, the elec-
tron density distribution in the bonding orbital referred to as a  molecular orbital provides the amount of energy which
stabilizes the hydrogen molecule H2 relative to two hydrogen atoms (Fig. 2.2). The antibonding * molecular orbital with
the negative term  212, in contrast, destabilizes the hydrogen molecule in the excited state.

Epot E*
H H * : antibonding

H H

1s 1s

H H  : bonding
E

Fig. 2.2. Overlapping of 1s orbitals of two hydrogen atoms

Generally, two molecular orbitals (MOs) with different energies arise from the overlapping of atomic orbitals, the stable
bonding MO with lower energy, and the antibonding MO on the higher energy level, corresponding to the excited state of
a bonding electron attainable by supply of a defined quantum of energy.

2.3 Overlapping of p Orbitals


The overlapping of p orbitals and other directed atomic orbitals generates  or  molecular orbitals depending on the
manner of overlapping (Fig. 2.3). Molecular orbitals of the  type, for example in the fluorine molecule F2, arise from
endwise overlapping of two singly occupied 2p atomic orbitals (Fig. 2.3 a). Sidewise overlapping of two singly occupied
coaxial 2p atomic orbitals generates  molecular orbitals (Fig. 2.3 b); those explain the formation of multiple bonds in
terms of the molecular orbital model (Chapters 4.2, 4.3).
Endwise or sidewise overlapping of p orbitals generates bonding and antibonding molecular orbitals, depending on the
phase relationship (orbital symmetry) of these orbitals: Bonding molecular orbitals arise from overlapping of lobes which
have the same sign (symmetric overlapping) while overlapping of lobes with opposite sign (antisymmetric overlapping)
results in the formation of antibonding molecular orbitals (Fig 2.3).

antisymmetric antibonding antisymmetric antibonding

 bonding bonding

symmetric bonding symmetric bonding


phase relationship molecular orbitals phase relationship molecular orbitals
(a) bonding by endwise overlapping of p orbitals (b) bonding by sidewise overlapping of coaxial p orbitals

Fig. 2.3. (a) Endwise overlapping of two p orbitals resulting in two  molecular orbitals ( bonding); (b) sidewise overlapping of two
coaxial p orbitals resulting in two  molecular orbitals ( bonding); p orbital lobes with negative sign are shaded
________________________________________________________________________________________________
Chapter 2 permits answers to the following:
(2.1) What is a covalent bond?
(2.2) How is a molecular orbital (MO) generated?
(2.3) How can the covalent bond of the hydrogen molecule be explained?
(2.4) Calculate the particularly high electron density between covalently bonded atomic nuclei in terms of the MO model.
(2.5) Which options of overlapping exist for p orbitals? Which types of bonding are the result?
12 3 Hybridization of Atomic Orbitals

3 Hybridization of Atomic Orbitals


3.1 Geometry of the Methane Molecule
Methane, the simplest hydrocarbon, has the molecular formula CH4. Spectroscopic data indicate the four CH bonds to be
equivalent and directed to the corners of a regular tetrahedron (Fig. 3.1). All CH bonds are equal in length (109 pm, 1 pm
= 1 picometer = 1012 m) and pairwise enclose the tetrahedral bond angle of 109°28'.

Fig. 3.1. Tetrahedral geometry of the methane molecule (tube, ball-spoke and space-filling molecular model)

The equivalence and tetrahedral symmetry of all four CH bonds in methane (CH4) cannot be explained by the electronic
configuration of the carbon atom in the ground state (1s2, 2s2, 2px1, 2py1, Table 1.1). Its two singly occupied p orbitals
could overlap with two s orbitals of two hydrogen atoms; the molecular formula of a hydrocarbon with bivalent carbon
would be CH2. Promotion of one 2s electron into the empty 2pz orbital with the resultant electronic configuration 2s1, 2px1,
2py1, 2pz1 would permit tetravalency and the correct molecular formula CH 4 but does not really solve the problem: two
different kinds of CH bonds would arise, one bond by non-directed ss overlapping and three bonds by directed ps over-
lapping with bond angles of 90°.

3.2 Hybridization of Atomic Orbitals


The inadequacy of the atomic orbital model to explain the methane molecular structure has been removed by the concept
of hybridization (PAULING, SLATER). Hybridization of atomic orbitals involves linear combination of different wave functions,
for example those of s and p orbitals; the resulting equations describe the shapes and directions of hybrid orbitals. Simp-
ly, hybridization is a mixing of different atomic orbitals to form hybrid orbitals which are shaped differently from the parent
atomic orbitals. Hybridization of s and p orbitals may generate sp, sp2 and sp3 hybrid orbitals, depending on the number
of p orbitals (1, 2, 3) involved in the hybridization.
Two sp hybrid orbitals arise from the crossing of one s with one p orbital (Fig. 3.2 a). Due to their origin, they have 50% s
and 50% p character; like their parent p orbital they extend along an axis (linear arrangement), enclosing an interorbital
angle of 180° (Fig. 3.2 b, Table 3.1). Two p orbitals perpendicular to the axis remain unhybridized (Fig. 3.2 c).

(a) (b) (c)

Fig. 3.2. sp hybrid orbitals: (a) hybridization of s and px atomic orbitals; (b) contours of the sp hybrid orbitals along the x axis; (c) remain-
ing p orbitals, py and pz

Three sp2 hybrid orbitals arise from the combination of one s orbital with two p orbitals (Fig. 3.3 a). Due to their origin,
they have 33.3% s and 66.7% p character. All three sp2 hybrid orbitals are coplanar (Fig. 3.3 b); their axes enclose inter-

Efficiently Studying Organic Chemistry: Exam training for chemists, biochemists, pharmacists, life and health scientists,
Third Edition. Eberhard Breitmaier. © 2022 WILEY-VCH GmbH. Published 2022 by WILEY-VCH GmbH.
3.3 Carbon-Hydrogen Bonding in Methane 13

orbital angles of 120° (Table 3.1), forming the apexes of a regular triangle. One p orbital perpendicular to the plane of the
sp2 triangle remains unhybridized (Fig. 3.3 c).

(a) (b) (c)


Fig. 3.3. sp2 hybrid orbitals: (a) hybridization of atomic orbitals s, px, py ; (b) contours of the sp2 hybrid orbitals in the xy plane; (c) remain-
ing pz orbital perpendicular to the plane of the sp2 hybrid orbitals

Four sp3 hybrid orbitals arise from the combination of an s orbital with all three p orbitals (Fig. 3.4). All of the four equiva-
lent sp3 hybrids have 25% s and 75% p character; they are directed towards the corners of a regular tetrahedron, pair-
wise enclosing the tetrahedral angle of 109°28' (Fig. 3.4, Table 3.1).
Table 3.1 compares the properties of sp, sp2 and sp3 hybrid orbitals. It turns out that the hybrid orbitals have a larger
radial extension than s and p orbitals, increasing with their p character (sp < sp2 < sp3), and therefore offer better chances
for overlapping than the parent orbitals.

Table 3.1. Properties of spx hybrid orbitals


hybridizing hybrid geometry interorbital character remaining radius in relation to
orbitals orbitals angle %s %p p orbitals s = 1, p = 1.732
1s 1p 2 sp linear 180° 50 50 2 1.93
1s 2p 3 sp2 coplanar, trigonal 120° 33.3 66.7 1 1.99
1s 3p 4 sp3 tetrahedral 109.5° 25 75 0 2.00

3.3 Carbon-Hydrogen Bonding in Methane


Provided bond angles in carbon compounds correspond to the angles formed by hybrid orbitals, the shape of organic
molecules can be explained. Tetrahedral sp3 hybrid orbitals exactly match the tetrahedral geometry of the methane
molecule (Fig. 3.1): each one of the four CH bonds of methane arises from overlapping of an sp3 hybrid orbital originating
from the carbon atom with an s orbital of a hydrogen atom (Fig. 3.4 c).

(a) (b) (c)

Fig. 3.4. sp hybrid orbitals: (a) all sp hybrid orbitals in the x, y, z coordinate system; (b) cross section of an sp3 hybrid orbital; (c) over-
3 3

lapping of four sp3 hybrid orbitals of carbon and s orbitals of four hydrogens to the four CH  bonds ( molecular orbitals) of methane
________________________________________________________________________________________________
Chapter 3 permits answers to the following:
(3.1) Describe and draw the geometry of the methane molecule.
(3.2) The electronic configuration of the carbon atom does not explain the shape of the methane molecule. Why?
(3.3) What is hybridization of atomic orbitals? Hybrid orbitals offer better chances for overlapping. Why?
(3.4) What is the model explanation of the tetrahedral shape of the methane molecule and the genesis of its CH bonds?
14 4 Covalent Carbon-Carbon Bonding

4 Covalent Carbon-Carbon Bonding


4.1 Ethane, CC Single Bond
Similar to methane (Chapter 3.3), all bonds originating from both carbon atoms of the hydrocarbon ethane (C2H6, Chapter
5.1) are tetrahedral, enclosing bond angles of 109°28´, as illustrated by molecular models (Fig. 4.1).

Fig. 4.1. Tube, ball-spoke and space-filling molecular model of ethane

The molecular orbital model explains the shape of the ethane molecule by invoking sp3 hybridized C atoms. Six CH 
bonds arise from overlapping of sp3 hybrid orbitals of carbon atoms with s orbitals of six hydrogens similar to the MO
model of methane (Fig. 3.4 c). The CC  bond (CC bond length 154 pm) results from an endwise overlapping of two sp3
hybrid orbitals originating from the adjacent carbon atoms (Fig. 4.2).

H H H H
H H
109.5°
sp3 sp3
109.5° C C C C sp3
154 pm 
109 pm 
H H s
H H H H
bond lengths, bond angles overlapping orbitals,  bonds

Fig. 4.2. Bond lengths (nuclear distances), bond angles and  bonds of ethane

4.2 Ethene, CC Double Bond


Ethene (ethylene, H2C=CH2, Chapter 14.1) is the simplest hydrocarbon that contains a CC double bond. It is a planar
molecule with HCH and HCC bond angles of about 120°, as shown by the molecular models in Figs. 4.3 and 4.4 a.
Consequently, hybrid orbitals which explain the shape of the ethene molecule must satisfy two conditions: they must be
coplanar and must enclose interorbital angles of 120°.

Fig. 4.3. Tube, ball-spoke and space-filling molecular model of ethene

H 134 pm H H H H  H
117.5° C C C C C C
H 121° 109 pm H H H H H


(a) (b) (c)

Fig. 4.4. Ethene molecule: (a) trigonal planar molecular shape with bond data; (b) overlapping p orbitals perpendicular to the plane of 
bonds; (c) areas where the  electrons are most likely to be found in the resultant  molecular orbital

Efficiently Studying Organic Chemistry: Exam training for chemists, biochemists, pharmacists, life and health scientists,
Third Edition. Eberhard Breitmaier. © 2022 WILEY-VCH GmbH. Published 2022 by WILEY-VCH GmbH.
4.3 Ethyne, CC Triple Bond 15

These requirements are met by sp2 hybrid orbitals. Two sp2 hybrid orbitals of the two adjacent carbon atoms overlap
endwise, generating the CC  bond of ethene. The four remaining sp2 hybrid orbitals on the two C atoms overlap with
four s orbitals of four hydrogen atoms, generating four CH  bonds. When all five  bonds are coplanar, the unhybridized
2p orbitals on both carbons are coaxial and this situation permits their sidewise overlapping. This overlapping generates a
 molecular orbital and a  bond with the  electron cloud above and below the molecular plane (Fig. 4.4 b);  electrons
are most likely to be found in the overlapping area of the original p orbitals above and below the center of the CC bond
(Fig. 4.4 c). Thus, a CC double bond arises from one  and one  bond.

4.3 Ethyne, CC Triple Bond


Ethyne (acetylene, HCCH, Chapter 19.1), the simplest hydrocarbon with a CC triple bond, is a linear molecule form-
ing HCC bond angles of 180° (Figs. 4.5, 4.6 a).

Fig. 4.5. Tube, ball-spoke and space-filling molecular model of ethyne

In the molecular orbital model, linear sp hybrid orbitals of carbon explain the linear shape of ethyne. The CC  bond
arises from endwise overlapping of two sp hybrid orbitals originating from the adjacent carbon atoms. Each of the two C
atoms forms a CH  bond by overlapping its sp hybrid orbital with an s orbital of hydrogen. Sidewise overlapping of the
coaxial pairs of the remaining 2p orbitals at both carbon atoms generates two  molecular orbitals and two CC  bonds
(Fig. 4.6 b-d);  electrons are most likely to be found in the overlapping areas of the original p orbitals above and below,
and in front and behind, the center of the CC single bond (Fig. 4.6 e). Thus, a CC triple bond arises from one  bond and
two additional  bonds.

120 pm

H C C H H C C H H C C H H C C H H C C H
106 pm 106 pm


(a) (b) (c) (d) (e)

Fig. 4.6. Ethyne molecule: (a) linear molecular shape and bond lengths; (b) overlapping of p orbitals to the first, and (c) to the second 
bond; (d) overlapping of both coaxial pairs of p orbitals to give two  bonds; (e) areas where the  electrons are most likely to be found
in the resultant  molecular orbitals

Since the radial extension of hybrid orbitals decreases with growing s character (sp3 > sp2 > sp), the lengths of carbon-
carbon single, double and triple bonds decrease with increasing bond order (single > double > triple).

CC : 154 pm C=C : 134 pm CC : 120 pm


sp3-sp3 sp2-sp2 sp-sp

________________________________________________________________________________________________
Chapter 4 permits answers to the following:
(4.1) What are the molecular shapes of (a) ethane, (b) ethene, and (c) ethyne?
(4.2) Explain the genesis of a CC single, a CC double and a CC triple bond in terms of the MO model.
(4.3) Draw the areas around a CC double bond where the electrons are most likely to be found.
(4.4) Explain why CC multiple bonds are shorter than a CC single bond.
16 5 Alkanes

5 Alkanes
5.1 Homologous Series of Alkanes
Organic compounds that only contain carbon and hydrogen are called hydrocarbons. Hydrocarbons in which all carbon
atoms are linked exclusively by CC single bonds are referred to as alkanes. All members of the alkane family possess the
general molecular formula CnH2n+2 , thus forming a homologous series. A homologous series includes chemically closely
related compounds with a general molecular formula and common chemical properties; formally, homologues are con-
structed by insertion of a methylene group, CH2 (homologization).

Table 5.1. Homologous series of alkanes (m.p.: melting point; b.p.: boiling point at normal pressure 1013 mbar)
n CnH2n+2 abbreviated structure name m.p. °C b.p. °C

1 CH4 H3CH methane 183 164


2 C2H6 H3CCH3 ethane 183 89
3 C3H8 H3CCH2CH3 propane 190 42
4 C4H10 H3C(CH2)2CH3 butane 138 0
5 C5H12 H3C(CH2)3CH3 pentane 130 36
6 C6H14 H3C(CH2)4CH3 hexane 95 69
7 C7H16 H3C(CH2)5CH3 heptane 90 98
8 C8H18 H3C(CH2)6CH3 octane 59 126
9 C9H20 H3C(CH2)7CH3 nonane 54 151
10 C10H22 H3C(CH2)8CH3 decane 30 174
11 C11H24 H3C(CH2)9CH3 undecane 26 196
12 C12H26 H3C(CH2)10CH3 dodecane 10 216
13 C13H28 H3C(CH2)11CH3 tridecane 6 230
14 C14H30 H3C(CH2)12CH3 tetradecane 6 251
15 C15H32 H3C(CH2)13CH3 pentadecane 10 268
20 C20H42 H3C(CH2)18CH3 eicosane 36
30 C30H62 H3C(CH2)28CH3 triacontane 66

5.2 Natural Sources and Preparation


5.2.1 Distillation of Petroleum
Coal, natural gas, and petroleum are the chief sources of alkanes and other hydrocarbons. These fossil sources of ener-
gy originated from anaerobic decomposition of microorganisms (plankton), plants, and animals in seas and oceans more
than 100 million years ago. Due to their different boiling points (Table 5.1), alkanes and other components of petroleum
can be separated by distillation. The petroleum fractions with their boiling point ranges and their use, predominantly for
the production of energy and as raw materials in chemical industry (petrochemistry), are given in Table 5.2.

Table 5.2. Distillation fractions of refined petroleum


fraction b.p. °C hydrocarbons Cn use
gas fraction < 40 C1 - C6 fuel, heating
petroleum ether 30 - 60 C5 - C6 solvent, fuel
ligroin 60 - 100 C6 - C7 fuel
gasoline 40 - 200 C5 - C10 fuel
kerosine 180 - 230 C11 - C12 fuel for jet engines
gas oil 230 - 300 C13 - C17 diesel and furnace fuel
lubrication oils 300 - 400 C20 - C30 lubrication
paraffin waxes 400 - 500 C20 - C30 vaseline
asphalt distillation polycyclic structures road building, roofing
petroleum coke residues carbon fuel, carbon electrodes

Efficiently Studying Organic Chemistry: Exam training for chemists, biochemists, pharmacists, life and health scientists,
Third Edition. Eberhard Breitmaier. © 2022 WILEY-VCH GmbH. Published 2022 by WILEY-VCH GmbH.
5.3 Alkanes as an Energy Source 17

5.2.2 Catalytic Hydrogenation of Alkenes


Hydrogen adds to the double bonds of alkenes (hydrocarbons unsaturated with hydrogen, Chapter 14.1) in the presence
of a metal (Ni, Pd, Pt) as catalyst which accelerates hydrogenation. Alkanes, saturated with hydrogen, are the products.
R R
R R catalytic hydrogenation
alkene C C R C C R alkane
R R H H
H H
surface of the catalyst (Ni, Pd or Pt)

5.2.3 WURTZ Synthesis Involving Alkylsodium


Alkanes of the type RR are obtained by reacting alkyl halides RX with sodium metal. Initially formed alkylsodium
RNa, an organometal compound (Chapter 32), reacts with alkyl halide to yield the symmetric alkane. In this manner,
hexane (R = CH3CH2CH2) can be prepared from 1-bromopropane (X = Br).

R X + 2 Na R Na + NaX R Na + R X R R + NaX X = Cl, Br, I


alkyl halide alkylsodium symmetric alkane

5.2.4 KOLBE Electrolysis of Carboxylates (Anodic Oxidation)


The electrolysis of carboxylates RCOO, the salts of carboxylic acids (Chapter 41), also yields symmetric alkanes RR.
The anode abstracts an electron from the carboxylate anion, so that the reaction is an anodic oxidation. As an example,
KOLBE electrolysis of propanoate (R = CH3CH2CH2) produces hexane.

O anode O
2R C
(oxidation)
2R C
decarboxylation
2R . dimerization
R R
O
..
.. :
e0 O
..
..
.  2 CO2

carboxylate anion carboxy radical alkyl radical symmetric alkane

5.3 Alkanes as an Energy Source


The alkanes of fuels (natural gas, gasoline, diesel oil, kerosine) can be oxidized by air oxygen to carbon dioxide and
water, as exemplified for methane and ethane.

CH4 + 2 O2 CO2 + 2 H2O H = 883 kJ/mol

2 H 3C CH3 + 7 O2 4 CO2 + 6 H2O  H = 1542 kJ/mol

However, combustion of alkanes requires ignition by a flame or a spark (Chapter 12.1); at room temperature and normal
pressure a mixture of alkane and oxygen (air) does not react.
Combustion of alkanes produces energy because the oxidation is a strongly exothermic reaction (Chapter 12.1), deliver-
ing a high heat of combustion H (reaction enthalpy) to the environment. Since energy is given away in an exothermic
reaction, the sign of H is negative.
This kind of chemical production of energy for heating, driving engines and jet propulsion produces tremendous amounts
of the hothouse gas carbon dioxide (Chapter 56.1). For example, the combustion of 1 mol of methane (16 g / mol or 22.4
L) generates one mol of carbon dioxide (44 g / mol or 22.4 L).
________________________________________________________________________________________________
Chapter 5 permits answers to the following:
(5.1) What is a homologous series of compounds?
(5.2) What natural sources of alkanes exist and how are alkanes produced industrially?
(5.3) Which reactions permit the preparation of specific alkanes?
(5.4) Alkanes are still an important source of energy. Why? Write the complete equation for an appropriate reaction.
18 6 Skeletal Structure, Structural Isomerism

6 Skeletal Structure, Structural Isomerism


6.1 Two and More Structures for One Molecular Formula
There are two different butanes, both with the same molecular formula C4H10: butane, more precisely n-butane (n for
normal) has an unbranched (continuous) chain of carbon atoms with elongated shape (Fig. 6.1); isobutane also referred
to as methylpropane belongs to the branched-chain alkanes, adopting a more spherical shape, as shown by molecular
models (Fig. 6.1).

CH3
H H H H
n-butane H3C CH 2 CH2 CH 3 C H
H C C C C H
H H
H H H H H C

C4H10 CH3
H
H C H CH3
H H CH3
methylpropane
(isobutane) H C C C H H3C CH CH 3 C
H 3C
CH3
H H H H

Fig. 6.1. Tube, ball-spoke and space-filling molecular model (from left to right) of unbranched butane (n-butane, top) and branched
methylpropane (isobutane, bottom)

6.2 Skeletal Isomerism


Butane (n-butane) and methylpropane (isobutane) are referred to as structural isomers (from Greek isos = same, similar;
meros = part). Structural isomers, also known as skeletal isomers or constitutional isomers, have the same molecular
formula but different carbon skeletons (atom connectivities, skeletal structures, constitutions) as exemplified in Fig. 6.1 by
the unbranched n-butane and the branched isobutane which has a methyl group in the side chain. Structural isomers
exhibit different physical properties (melting points, boiling points, refractive indices, molecular spectra). At normal pres-
sure, unbranched n-butane with elongated shape boils at 0 °C while branched isobutane with the more spherical shape
boils at –12 °C. Due to their individual boiling points, structural isomers can be separated (purified) by distillation.

Efficiently Studying Organic Chemistry: Exam training for chemists, biochemists, pharmacists, life and health scientists,
Third Edition. Eberhard Breitmaier. © 2022 WILEY-VCH GmbH. Published 2022 by WILEY-VCH GmbH.
6.2 Skeletal Isomerism 19

Three structural isomers exist for the molecular formula C5H12. These are the unbranched pentane (n-pentane), the singly
branched methylbutane (isopentane), and doubly branched dimethylpropane (neopentane) with methyl groups as side
chains:

unbranched singly branched doubly branched


CH 3 CH 3
C5H12 H3C CH 2 CH 2 CH 2 CH 3 H 3C CH CH 2 CH 3 H3C C CH 3
CH 3
n-pentane methylbutane dimethylpropane
(isopentane) (neopentane)

In going further to higher homologues of alkanes (Table 5.1), the number of isomers increases exponentially. Five struc-
tural isomers exist for hexane C6H14, 75 for decane C10H22, and more than four million for tridecane C30H62.

unbranched singly branched multiply branched


CH3 CH3
C6H14 H3C CH2 CH2 CH2 CH2 CH 3 H 3C CH CH2 CH 2 CH 3 H 3C C CH2 CH3
n-hexane 2-methylpentane CH 3
2,2-dimethylbutane

CH3 CH3 CH3


H 3C CH CH CH2 CH3 H 3C CH CH CH3
3-methylpentane 2,3-dimethylbutane

________________________________________________________________________________________________
Chapter 6 permits answers to the following:
(6.1) What are structural (skeletal) isomers?
(6.2) Which structural isomers exist for (a) butane, (b) pentane, and (c) hexane?
(6.3) Which properties are individual (specific) for structural isomers?
(6.4) Which one of these properties is the basis of a method for the separation and purification of structural isomers?
(6.5) Draw the structures of the skeletal isomers of heptane with (a) one methyl group and (b) two methyl groups in
the side chain.
(6.6) Which of the following compounds (1-6) are structural isomers? Which ones are identical?

CH3
H3C CH 2 CH CH2 CH 2 CH CH3 H 3C CH CH2 CH 2 CH CH3
CH3 CH3 CH2 CH 3
1 2

CH 3 CH3
H3C CH 2 CH CH2 CH CH 2 CH3 H 3C CH CH2 CH 2 C CH3
CH3 CH3 CH3
3 4

CH3 CH3 C2H5


H3C CH 2 C CH 2 CH CH3 (H 3C)2CH CH2 C CH3
CH3 CH3
5 6
20 7 Basic Rules of Nomenclature

7 Basic Rules of Nomenclature


7.1 IUPAC Rules
Organic compounds are named systematically according to the rules established by IUPAC (International Union of Pure
and Applied Chemistry). As shown in Tables 5.1 and 7.1, the first four alkanes have individual names partly derived from
their natural origin (e.g. butane, C4H10, from butyric acid, C4H8O2, in rancid butter). The names of higher members have
Greek prefixes indicating the number of carbon atoms and the ending ane according to their membership in the alkane
family, exemplified by pentane, C5H12, with five carbon atoms (Greek pente = five). Endings ene and yne are given to
hydrocarbons with CC double and triple bonds, belonging to the alkene and alkyne family, respectively (Table 7.1, Chap-
ters 14.1, 19.1).
Following the IUPAC rules, the names of branched alkanes are derived from the names of the unbranched parent al-
kanes in Table 7.1, as demonstrated for a hydrocarbon with the structure drawn in the following manners (Chapter 8):

H3C CH(CH3)2
H3C CH2 C CH CH2 CH2 CH CH2 CH2 CH3
H3C CH2 CH 3

The IUPAC name is obtained as follows:


- The longest continuous carbon chain defines the parent name. Thus, decane is the parent name in this alkane
example.
- The carbon atoms of the parent chain are numbered beginning from that end of the chain which will give the
smallest number to carbons carrying a substituent group or atom other than H on a carbon of the chain; this is
3,4,7- and not 4,7,8- (numbered from the opposite end of the chain) in the example.

H 3C CH(CH 3)2 H 3C CH(CH3)2


1 2 3 4 5 6 7 8 9 10 10 9 8 7 6 5 4 3 2 1
H 3C CH2 C CH CH2 CH2 CH CH 2 CH2 CH 3 and not H 3C CH2 C CH CH 2 CH2 CH CH2 CH 2 CH 3
H 3C CH 2 CH3 H3C CH2 CH3

- Substituents are named and their positions on the carbon chain indicated by prefixes, for example 7-ethyl-.
- The names of substituents are ordered alphabetically; this is ethyl > isopropyl > methyl in the example.
- Two, three, four, five identical substituents are indicated by the prefixes di-, tri-, tetra-, penta-; dimethyl- represents
two CH3 groups in the example.
- Branched alkyl groups are named according to Table 7.2, for example isopropyl.
- Thus, the correct IUPAC name is 7-ethyl-4-isopropyl-3,3-dimethyldecane.

H3C CH(CH3)2 4 6 8 10
1 2 3 4 5 6 7 8 9 10 1 3 5 7 9
H3C CH2 C CH CH2 CH2 CH CH2 CH2 CH3 7-ethyl-4-isopropyl-3,3-dimethyldecane 2

H3C CH2 CH3

All branched structural isomers of pentane and hexane in Chapter 6.2 have been named accordingly.
The methyl group and other alkyl groups formally replacing (substituting) the H atoms in the parent skeleton are referred
to as substituents. Their nomenclature follows Table 7.1.

7.2 Branched Alkyl Groups


When ranking alkyl groups and substituents named according to Table 7.2 alphabetically, hyphenated prefixes such as
sec- and tert- = t- for branched alkyl groups (Table 7.2) are not taken into account.

Efficiently Studying Organic Chemistry: Exam training for chemists, biochemists, pharmacists, life and health scientists,
Third Edition. Eberhard Breitmaier. © 2022 WILEY-VCH GmbH. Published 2022 by WILEY-VCH GmbH.
7.2 Branched Alkyl Groups 21

Therefore, tert-butyl- (t-butyl-) is ranked before ethyl and methyl as exemplified:


CH(CH3)2
1 2 3 4 5 6 7 8 9 10
H3C CH2 CH CH CH2 CH CH2 CH2 CH2 CH3 6-tert-butyl-4-isopropyl-3-methyldecane
CH3 C(CH3)3

Table 7.1. Nomenclature of some parent hydrocarbons and derived alkyl and alkoxy groups (prefix: black, ending: red)

number of h y d r o c a r b o n s u b s t i t u e n t (g r o u p)
C atoms alkane (RH) alkene alkyne alkyl (R) alkoxy (RO)
1 methane methyl methoxy
2 ethane ethene ethyne ethyl ethoxy
3 propane propene propyne propyl propoxy
4 butane butene butyne butyl butoxy
5 pentane pentene pentyne pentyl pentoxy
6 hexane hexene hexyne hexyl hexoxy
7 heptane heptene heptyne heptyl heptoxy
8 octane octene octyne octyl octoxy
9 nonane nonene nonyne nonyl nonoxy
10 decane decene decyne decyl decoxy
11 undecane undecene undecyne undecyl undecoxy
12 dodecane dodecene dodecyne dodecyl dodecoxy
13 tridecane tridecene tridecyne tridecyl tridecoxy
14 tetradecane tetradecene tetradecyne tetradecyl tetradecoxy
15 pentadecane pentadecene pentadecyne pentadecyl pentadecoxy
20 eicosane eicosene eicosyne eicosyl eicosoxy

Table 7.2. Nomenclature of branched alkyl groups

CH 3 CH3 CH 3
H 3C CH H3C CH CH2 H 3C CH CH 2 CH 2
isopropyl- isobutyl- isopentyl-

CH 3
H 3C CH 2 CH CH 3 H 3C C CH2
CH 3
sec-butyl- neopentyl-

CH 3 CH 3
H 3C C H 3C CH 2 C
CH 3 CH 3
tert-butyl- (t-bu) tert-pentyl-

________________________________________________________________________________________________
Chapter 7 permits answers to the following:
(7.1) What are the IUPAC names of all singly and doubly methyl branched isomers of heptane C7H16?
(7.2) Draw the structural formulas of (a) 2,2,4-trimethylhexane, and (b) 5-t-butyl-3,3-diethyloctane.
(7.3) Give the IUPAC names of the alkanes with structures drawn in question 6.6.
(7.4) What is the correct IUPAC name of the following alkane?

CH(CH3)2 C(CH3)3 CH3


H3C CH2 CH2 CH CH CH2 CH CH CH CH2 CH 3
CH3 CH2 CH3

(7.5) What is the structural formula of 2,6,10-trimethyldodecane which occurs in slate oil?
22 8 Drawing Molecular Structures

8 Drawing Molecular Structures


8.1 Structural Formulas
Planar projections of the three-dimensional (e.g. tetrahedral) molecules are generally drawn for convenience in several
variations, depending on whether convenience, time saving, and clearness take preference or electronic configurations
have to be presented in reaction equations.
The structural or valence bond formula including all bonds presents the complete atom connectivities of the molecule, as
shown for the alkanes butane and decane and the cycloalkanes cyclobutane and cyclohexane.
H H
H H H H
H H H H H H H H H H H H H H C
H C C H H C C H
H C C C C H H C C C C C C C C C C H
H C C H H CC H
H H H H H H H H H H H H H H C
H H H H
H H
butane decane cyclobutane cyclohexane

Condensed structural formulas arise from removal of all CH bonds and permit reduced drawing time.

H 3C CH2 CH2 CH 2 CH2 CH 2 CH2 CH 2 CH2 CH 3 H2


C
or H 2C CH 2 H2C CH2
H 3C CH2 CH 2 CH3
H3C (CH2)8 CH3 H 2C CH 2 H2C CH2
C
H2
butane decane cyclobutane cyclohexane

8.2 Skeletal Formulas


By not depicting all C and H atoms, the drawing is further shortened to the clear skeletal formula. Alkanes are simply
represented as zigzag chains, and the six-membered ring of cyclohexane is a hexagon. Each corner represents a meth-
ylene group (CH2); each open end corresponds to a methyl group (CH3). The skeletal formula is particularly useful for
the presentation of larger organic molecules like cholestane, the parent hydrocarbon of cholesterol (Chapter 77.2).

butane decane cyclobutane cyclohexane cholestane

Condensed and skeletal formulas are generally used to write reaction equations.

8.3 LEWIS Formulas


Non-bonding electron pairs at heteroatoms or functional groups frequently explain the reactivity and the origin of new
bonds. Electron pairs are indicated by non-bonding lines (one line for each electron pair) or colons (one dot for each
electron) in the LEWIS formula taking into account the octet rule.
..
NH H3C CH 2 CH 2 CH2 NH 2 or NH 2 H3C CH2 CH 2 CH2 NH 2
.. 2
1-aminobutane (n-butylamine)

.. ..
Br: Br:
H3C CH 2 CH 2 CH2 .. or Br I H3C CH2 CH 2 CH2 Br I
..
1-bromobutane (n-butyl bromide)

In reaction equations LEWIS formulas are frequently used to indicate the origin of newly formed bonds and non-bonding
electron pairs. Nucleophilic substitution (Chapters 11.5, 31) of 1-bromobutane, for example, involves an attack of a non-

Efficiently Studying Organic Chemistry: Exam training for chemists, biochemists, pharmacists, life and health scientists,
Third Edition. Eberhard Breitmaier. © 2022 WILEY-VCH GmbH. Published 2022 by WILEY-VCH GmbH.
8.4 Projections 23

bonding electron pair of the hydroxide anion at the brominated carbon atom of 1-bromobutane, thus providing the bond-
ing electron pair of the newly formed covalent bond. Thereby, the arrow depicts the direction of electronic motion: it
points, by convention, from the donor of the electron pair (the hydroxide anion as the nucleophile, Chapters 11.5, 31) to
the acceptor (the brominated C atom as the electrophile):

H 3C CH 2 CH 2 CH2 Br I + IOH H 3C CH 2 CH 2 CH2 OH + IBr I


1-bromobutane hydroxide anion 1-butanol bromide anion

8.4 Projections
8.4.1 FISCHER Projection
Two methods are suitable for drawing the spatial representation of a tetravalent carbon atom with tetrahedral bonds. In
the planar FISCHER projection, horizontal bonds are above and vertical bonds behind the paper plane. In the tetrahedral
projection, full lines indicate bonds in the plane, dotted lines represent bonds behind the plane, and wedge lines empha-
size bonds rising above the plane of the paper.
FISCHER projection tetrahedral projections
H H H CH3
H H H
H C H H C H C C C H
H H H C
H H H H H H
CH 3
methane methane butane
arrangement of bonds: : in , : above , : behind the paper plane

8.4.2 NEWMAN Projection


The NEWMAN projection clearly visualizes the angular relationships of groups attached to the carbons of a CC single
bond. In this projection, the observer looks into the direction of the selected CC single bond so that one C atom lies
behind the other. A circle surrounding the front carbon symbolizes both carbon atoms and the CC single bond connecting
them. The additional three bonds originating from the front carbon atom enclose angles of 120° in the projection; the real
tetrahedral angle is 109°28´ (Chapter 3.2). The three additional bonds connecting groups with the rear carbon atom
originate from the circle line, also enclosing angles of 120° in the projection. The tetrahedral projection of butane is
converted into the NEWMAN projection of the central CC bond in this manner. As this drawing shows, the substituents of
the tetrahedron in front and behind are not eclipsed but staggered (Chapter 9.1) and enclose dihedral angles (torsional
angles) of 60° (CH3 and H) and 180° (both CH3 groups) in the projection:
CH 3
CH 3 butane
H H
H
direction of view C H
H C
H H H
CH 3 CH 3
tetrahedral projection NEWMAN projection

________________________________________________________________________________________________
Chapter 8 permits answers to the following:
(8.1) Draw the skeletal formulas for cyclopropane, cyclopentane, cycloheptane, and cyclooctane.
(8.2) Draw the condensed and skeletal structural formula of 2,6,10-trimethyldodecane.
(8.3) Draw the condensed and skeletal structural formula of 3-methylhexane and number the longest carbon chain.
(8.4) Draw one of the tetrahedral projections of the CH carbon (C-3) in 3-methylhexane.
(8.5) Write the reaction equation for 1-bromobutane with sodium hydroxide by means of LEWIS formulas. What is the
origin of the CO bond in the product 1-butanol?
(8.6) Draw NEWMAN projections of the central CC bond of hexane with staggered ethyl groups.
24 9 Conformation

9 Conformation
9.1 Conformation, Conformers
There is free rotation about the CC single bonds of alkanes; alkane molecules such as butane (R = CH3 in the formulas to
follow) can adopt an infinite number of arrangements of their atoms and groups (H and CH3) in space resulting from
rotation about CC single bonds (rotational states). The term conformation describes the total of these arrangements. A
conformer is one specific arrangement.
The NEWMAN projection (Chapter 8.4.2) simply and precisely visualizes the spatial arrangement of groups in conformers.
Two kinds of conformers can be distinguished as borderline cases: those with an eclipsed and those with a staggered
arrangement of groups, such as H and CH3 in butane. Conformers with groups arranged somewhere between eclipsed
and staggered are called skew.
R R
R R R
H R
H
C R C R
H C H C
H H H
H HH H H
H H H H
conformer with eclipsed alkyl groups and H atoms conformer with staggered alkyl groups and H atoms

Clearly, repulsion because of crowding of groups is stronger for an eclipsed arrangement. To conclude, conformers with
staggered groups will be more stable, occupying a lower energy level than those with eclipsed groups.

9.2 Energy Contents and Nomenclature of Conformers


Fig. 9.1 portrays the full rotation about a CC single bond, the relative energy contents, and the names of the conformers
depending on the dihedral angle (torsional angle).

RR R RH R RH R RR
H R H H R H
conformer
H HH H H RH H R HH H H HH
H H H H H H H
H R H
dihedral angle 0° 60° 120° 180° 240° 300° 360°

Epot
[kJ/mol]

27.7
14.3

3.8

name fully gauche partially fully partially gauche fully


eclipsed staggered eclipsed staggered eclipsed staggered eclipsed

KLYNE-
PRELOG system +
 syn-periplanar + syn-clinal + anti-clinal  anti-periplanar
+  anti-clinal  syn-clinal +
 syn-periplanar
abbreviation +
 sp + sc + ac  ap
+  ac  sc +
 sp

Fig. 9.1. Rotation about the C-2C-3 bond of butane (R = CH3): potential energy and nomenclature of conformers

Efficiently Studying Organic Chemistry: Exam training for chemists, biochemists, pharmacists, life and health scientists,
Third Edition. Eberhard Breitmaier. © 2022 WILEY-VCH GmbH. Published 2022 by WILEY-VCH GmbH.
9.2 Energy Contents and Nomenclature of Conformers 25

Eclipsed groups with bonds enclosing dihedral angles of 0° in the NEWMAN projection suffer from overcrowding in space;
in this situation they develop the most intense steric interaction, strongly repulsing each other. Thus, conformers with fully
and partially eclipsed groups occupy energy maxima and have the lowest stability, as shown in Fig. 9.1.
Staggered groups with bonds enclosing dihedral angles of 60° in the NEWMAN projection do not disturb one another as
strongly. As a result, conformers with staggered substituents are more stable and occupy energy minima (Fig. 9.1). In
these conformers the staggered substituents are able to adopt two different arrangements denoted as syn and anti.
In syn conformers the groups R are adjacent to each other, their bonds enclosing dihedral angles of 60°. This kind of
staggered arrangement is referred to as syn-clinal or gauche:
R 60°
R
H R
H syn-clinal alkyl groups R
C R
H C (dihedral angle 60°)
H H H
H H

In anti conformers the groups R are as far apart as possible, their bonds enclosing dihedral angles of 180°. This ar-
rangement is referred to as antiperiplanar or fully staggered. There is practically no steric interaction between the groups
R in this state which is therefore the most stable and favored one, with the lowest potential energy (Fig. 9.1).
R
R
H H
H anti-periplanar alkyl groups R
C H 180°
H C (dihedral angle 180°)
H H H
R R

In contrast to structural isomers, which can be separated and purified by distillation (Chapter 6.2), the conformers of
alkanes cannot be isolated in general because the energy barrier between them is too small (< 30 kJ/mol between con-
formers with fully eclipsed and fully staggered groups, Fig. 9.1) under normal conditions. At room temperature the kinetic
energy of molecules is much more than large enough to stimulate the rotation about CC single bonds. At very low tem-
peratures, however, conformers can be detected by some spectroscopic methods, the bulkier the substituents R the
better.
________________________________________________________________________________________________
Chapter 9 permits answers to the following:
(9.1) Why do alkanes form conformers? Explain the terms conformation and conformer.
(9.2) How do conformers differ from structural isomers?
(9.3) Draw NEWMAN projections of the C-2C-3 bond of butane for all conformers with staggered methyl groups.
(9.4) Draw NEWMAN projections of the C-1C-2 bond of (a) 1-bromopropane and (b) 1,2-dibromoethane for all
conformers with staggered groups.
(9.5) Draw the energy diagram of rotation about the central CC bond of butane and use NEWMAN projections to present
all conformers.
(9.6) Draw NEWMAN projections of the following conformers of 2-methylhexane and specify the spatial arrangements
of the alkyl groups.
C 2H 5 C 2H 5
H H
C CH(CH3) 2 C H
H C H C
H H
H CH(CH3) 2

(9.7) What are steric interactions? Which conformers particularly suffer from these interactions?
26 10 Reactive Intermediates

10 Reactive Intermediates
Organic reactions are described as proceeding by way of reactive intermediates such as radicals, ions, and carbenes.
Reactive intermediates, not to be confused with the collision adducts initiating reactions and called transition states
(Chapter 12), usually cannot be isolated apart from some exceptions, but physical methods provide evidence for their
existence.

10.1 Radicals
Homolysis of a  bond cleaves a molecule into two radicals. Radicals are molecular fragments or atoms with unpaired
electrons. The bonding electrons of the original  bond are shared equally among both resulting radicals. Homolysis of
one CH bond in methane, for example, generates a methyl radical (. CH3) and a hydrogen radical (. H, H atom):
homolysis
H 3C H H3C + H
methyl radical

The unpaired electron gives rise to paramagnetism of the radical. Physical methods provide evidence for this. Spectro-
scopic measurements have revealed that the methyl radical is flat, forming HCH angles of about 120° (Fig. 10.1 a). In
the molecular orbital model, this shape can be explained by an sp2 hybridized carbon atom which uses its triangular sp2
hybrid orbitals to form three coplanar CH  bonds. The unpaired electron occupies the 2p orbital of the carbon atom
perpendicular to the CH3 plane (Fig. 10.1 b). This exposed cloud of the unpaired electron explains the short lifetime
(about 108 s) and the high reactivity of radicals.

50%

H H
H C H C
120°
H H C
50% H H
H
(a) (b) (c)

Fig. 10.1. (a, b) Methyl radical: planar shape and distribution of the unpaired electron on a p orbital; (c) methyl anion: pyramidal shape
with an electron pair which occupies an sp3 hybrid orbital

10.2 Carbenium Ions and Carbanions


Heterolysis of the  bond in ethane formally leads to a methyl cation and a methyl anion:
heterolysis
H 3C CH 3 H 3C + ICH 3
ethane methyl cation methyl anion
a carbenium ion a carbanion

Fig. 10.2 summarizes the generation of radicalic and ionic intermediates by formal homolytic and heterolytic cleavages of
the methane and ethane molecule.
Spectroscopy has shown that the methyl cation H3C, representing the simplest carbenium ion, is planar. Thus, its mo-
lecular orbital model is that of the methyl radical, but with an empty 2p orbital (Fig. 10.1 b). Its vacant 2p orbital causes
the carbenium ion to be a strong electron-pair acceptor, a typical electrophile.
Carbocations are divided into two kinds according to OLAH: In the so-called classical carbenium ions, such as the methyl
cations, the positively charged carbon atom adopts the coordination number 3, for example R3C; in the non-classical
carbenium ions, also referred to as carbonium ions, the positively charged carbon atom exhibits delocalized bonds and a
higher coordination number, for example R5C.

Efficiently Studying Organic Chemistry: Exam training for chemists, biochemists, pharmacists, life and health scientists,
Third Edition. Eberhard Breitmaier. © 2022 WILEY-VCH GmbH. Published 2022 by WILEY-VCH GmbH.
10.3 Carbenes 27

The methyl anion with LEWIS formula ICH3 or :CH3 represents the simplest carbanion. In its molecular orbital model,
carbon is pictured as using three sp3 hybrid orbitals to form three CH  bonds. An electron pair occupies the remaining
sp3 hybrid orbital (Fig. 10.1 c), causing the carbanion to be a strong electron- pair donor, a typical nucleophile.

methane
CH4

sp3

H CH3 H CH3 H CH3

 H: heterolysis H homolysis H heterolysis

e0 e0
CH3 CH 3 :CH 3
ionization ionization
reactive intermediate methyl cation (oxidation) methyl radical (reduction) methyl anion
(carbenium ion) (carbanion)

electronic configuration
sp2 sp2 sp2 p sp2 sp2 sp2 p sp3 sp3 sp3 sp3

H H
molecular orbital model H C H C C
H H H H
H
cation radical anion
trigonal trigonal pyramidal
LEWIS acid (electrophile) LEWIS base (nucleophile)

heterolysis homolysis heterolysis

H 3C CH3 H 3C CH3 H 3C CH 3
ethane

Fig. 10.2. Formal generation of reactive intermediates from methane and ethane

10.3 Carbenes
Heating or ultraviolet irradiation of diazomethane (Chapter 40.3) generates carbene with LEWIS formula ICH2 or :CH2 as
reactive intermediate, also referred to as methylene.
_ heat or h
H2C N NI H 2C: + IN NI
diazomethane carbene (methylene)

Carbene and substituted derivatives (R2C: or R2CI) are highly reactive intermediates occuring in many organic reactions.
Due to the sextet of electrons at their reactive carbon atom they have, similar to the nitrenes (RN: or RNI), a lack of
electrons, thus representing another kind of strong electrophiles.
Spectroscopic studies have revealed two types of carbenes: a low-energy carbene with an HCH bond angle of 136°
and two unpaired electrons at the carbon atom, and a high-energy, more reactive ("hot") carbene with an HCH bond
angle of 103° and paired electrons at the carbon atom.
________________________________________________________________________________________________
Chapter 10 permits answers to the following:
(10.1) Give appropriate examples for a homolysis and a heterolysis of a  bond.
(10.2) What are radicals, carbenium ions, carbanions, and carbenes? How are these species formed?
(10.3) Draw the molecular orbital models of the methyl radical and the methyl cation. How do they differ?
(10.4) Define the terms electrophile and nucleophile and classify carbenium ions, carbanions, and carbenes as such.
(10.5) Suggest a molecular orbital model for the low-energy carbene with unpaired electrons.
28 11 Basic Types of Organic Reactions

11 Basic Types of Organic Reactions


There are six basic types of organic reactions.

11.1 Addition
In a typical addition (Chapter 16), a molecule HX adds to a multiple bond to give an adduct without any byproduct (a
"clean" reaction). An addition is the reverse of an elimination.
H
C C + HX C C
X

11.2 Elimination
In a typical elimination (Chapter 15), a molecule HX is abstracted from a compound, generating a multiple bond. An
elimination is the reverse of an addition. -Eliminations involve the loss of H (as H) and X (as X) from neighboring
carbon (or other) atoms.
H
 
C C C C + HX
X

11.3 Oxidation
In an oxidation, a molecule loses electrons or H atoms (equivalent to protons H and electrons e0), exemplified by the
oxidation of an alcohol to a carbonyl compound. The oxidation number (of the C atom) increases. An oxidation is the
reverse of a reduction.
H
alcohol C C + 2H + 2 e0
OH O

11.4 Reduction
In a reduction, a molecule gains electrons or H atoms (equivalent to protons H and electrons e0), exemplified by the
reduction of a carbonyl compound to an alcohol. The oxidation number (of the C atom) decreases. A reduction is the
reverse of an oxidation.
H
carbonyl compound C + 2H + 2 e0 C
O OH

11.5 Substitution
A substitution displaces one substituent (including an H atom) of a compound by another group. The compound to be
substituted is called the substrate. Substitutions enable the introduction and exchange of functional groups which deter-
mine the properties and reactions of a compound.
In a nucleophilic substitution, abbreviated as SN (Chapter 31), the new substituent is introduced by an electron-pair donor
called the nucleophile. The nucleophile :Nu donates an electron pair to the substrate, using it to tie the new bond. The
leaving group, also called the nucleofuge, moves away with the former bonding electron pair. The nucleophile may be an
anion (e.g. the cyanide anion :CN) or a neutral molecule (e.g. ammonia :NH3) which provides the electron pair.

R X + :Nu R Nu + :X

Efficiently Studying Organic Chemistry: Exam training for chemists, biochemists, pharmacists, life and health scientists,
Third Edition. Eberhard Breitmaier. © 2022 WILEY-VCH GmbH. Published 2022 by WILEY-VCH GmbH.
11.6 Rearrangement 29

In an electrophilic substitution, abbreviated as SE (Chapter 25), the new substituent is introduced by a cation as an elec-
tron-pair acceptor called the electrophile E. Typically, the substrate is an aromatic hydrocarbon ArH which provides the
electron pair for the new bond. Thus, the leaving group, also called the electrofuge is a proton H.

Ar H + E Ar E + H

In a radical substitution, abbreviated as SR (Chapter 13), the new substituent is introduced by a radical. A radical is a
reactive intermediate with an unpaired electron (e.g. a halogen atom such as Cl.), generated by a starting reaction for
"ignition". Each propagating step of the reaction process produces a new radical.
Cl + H R HCl + R
R + Cl2 R Cl + Cl

11.6 Rearrangement
In a rearrangement, which may be an isomerization (Chapter 57), a substituent such as an alkyl group frequently mi-
grates from one to another atom, usually to the adjacent position so that a 1,2-shift of the group takes place. An existing
bond is cleaved and a new bond is formed. Rearrangements, marked by a loop on the arrow in reaction equations, mostly
involve carbenium ions or carbenes with carbanions that migrate (anionotropic rearrangements), or carbanions with
cations that migrate (cationotropic rearrangements).
Anionotropic rearrangements
The substituent, for example an alkyl group R, takes the bonding electron pair and migrates as an anion, exempli-
fied by the WOLFF rearrangement of an acylcarbene to a ketene. Cleavage of a diazo ketone generates nitrogen
and the intermediate acylcarbene.
R R R
_ _
C CH N NI C CH O C C
N 2 1 2 1 2
O O H
diazo ketone acylcarbene ketene

Cationotropic rearrangements
The substituent, for example an alkyl group R, leaves the bonding electron pair and migrates as a cation, exempli-
fied by the WITTIG rearrangement of an ether to an alkoxide anion via an intermediate carbanion.
2 1 base 1 2 _1
R CH 2 O R CH
_ O R CH OI
_
 [H ] 2
R' R' R'
ether carbanion alkoxide anion

Sigmatropic rearrangements
A sigmatropic rearrangement involves concerted shift or reorganization of  and  bonds, exemplified by the
COPE rearrangement of 1,5-dienes.
R R

1,5-diene rearranged 1,5-diene

________________________________________________________________________________________________
Chapter 11 permits answers to the following:
(11.1) Which kind of reactions are additions and eliminations? Formulate typical equations.
(11.2) Explain the term -elimination.
(11.3) Which kind of reactions are oxidations and reductions? Formulate typical equations.
(11.4) Which kind of substitution reactions exist? Formulate typical equations.
(11.5) What kind of rearrangements do you know? Formulate typical equations.
30 12 Energy Turnover of Chemical Reactions

12 Energy Turnover of Chemical Reactions


12.1 Heat of Reaction, Activation Energy
Every reaction liberates or requires a net amount of energy called the heat of reaction. Strictly speaking, this is the en-
thalpy of the reaction H which is essentially the difference in bond energies between the reactants (A + B) and the
product (C) of the reaction
A + B C ( + or  H )

Energy differences, versus a reaction coordinate, so-called energy diagrams, are plotted in order to visualize the turnover
of energy during the reaction progress, as exemplified in Fig. 12.1.
If the reactants have a higher energy than the product, the reaction delivers energy to the environment, thus warming it:
The heat of reaction H is negative because energy is withdrawn from the reacting system, and the reaction said to be
exothermic (Fig. 12.1 a).
In contrast, a reaction is said to be endothermic when the heat of reaction H is positive because the reactants have a
lower energy than the product, so that the reacting system takes energy from the environment which is cooled as a result
(Fig. 12.1 b).
A negative H does not necessarily mean that a reaction takes place in a reasonable period of time. The oxidation of
alkanes, for example, liberating a high amount of energy, and thus having a very large negative H, requires ignition by a
flame (Chapter 5.3). Triggering the reactants A and B to produce C (the ignition) requires an activation energy EA. In
addition to the thermal energy of the reactants, this is an extra amount of energy which has to be added in order to reach
the transition state of the reaction. The transition state is not a reactive intermediate but the collision complex [AB] of the
reactants at maximum energy, from which further reaction to the product C takes place (Fig. 12.1).

E [A B]
E
[A B]
transition state transition state

EA1
EA1
[A B cat]

EA2
H endothermic

exothermic H

a A + B C b A + B C
reaction coordinate reaction coordinate

Fig. 12.1. Energy diagram (a) of an exothermic reaction and influence of a catalyst, (b) of an endothermic reaction

12.2 Catalysis
The activation energy may be too high for a reaction to proceed in a reasonable time. An appropriate catalyst, usually
optimized empirically, may decrease this barrier considerably, as shown in Fig. 12.1 a: Upon collision with the reactants,
the catalyst (cat) forms a transition state [AB cat] with the activation barrier EA2 , which is much lower than the barrier

Efficiently Studying Organic Chemistry: Exam training for chemists, biochemists, pharmacists, life and health scientists,
Third Edition. Eberhard Breitmaier. © 2022 WILEY-VCH GmbH. Published 2022 by WILEY-VCH GmbH.
12.3 Kinetic and Thermodynamic Control 31

EA1 of the collision complex [AB] of the reactants themselves. As a result, the catalyst accelerates the reaction. The
catalyst is regenerated in each cycle of catalysis:

A + B + cat C + D
cycle of catalysis
[ AB cat ]

12.3 Kinetic and Thermodynamic Control


Two reactants A and B may react to give two different products C and D because of two competing reactions. In this
case, the thermodynamic conditions (cooling or heating, reaction time) may control which one of the products will be
predominantly formed. Fig. 12.2 shows the energy diagram of a reaction producing a more stable product C requiring a
large activation energy EA(C) and a less stable product D by way of another reaction requiring a lower activation barrier
EA(D).
If neither reaction is reversible, product D will predominate because it is formed faster due to the lower activation barrier
EA(D). This reaction yielding the less stable product D is said to be kinetically controlled.
If both reactions are reversible and quenched before equilibrium has been reached, they remain kinetically controlled
because more of the faster-formed compound D has been produced. If equilibrium is allowed to be approached, however,
for example by heating or waiting, the more stable product C will predominate. Under these conditions, the faster-formed
product D reverts to the reactants A and B because of the low energy barrier, while the more stable product C reverts
much less because of the higher energy barrier (Fig. 12.2). In this case, the reaction is said to be thermodynamically
controlled.
Frequently, the more stable product is also formed faster so that the product of thermodynamic control is also the product
of kinetic control.

transition states

EA(C)

EA(D)

C A + B D

thermodynamic control kinetic control

Fig. 12.2. Thermodynamic and kinetic control of a reaction

________________________________________________________________________________________________
Chapter 12 permits answers to the following:
(12.1) What is the transition state of a reaction?
(12.2) Explain the terms activation energy and heat of reaction.
(12.3) What are the effects of an exothermic and an endothermic reaction?
(12.4) What are the characteristics of a kinetically and a thermodynamically controlled reaction? Draw an energy diagram
for explanation.
(12.5) Characterize the action of a catalyst.
32 13 Radical Substitution

13 Radical Substitution
13.1 Photohalogenation of Alkanes
Alkanes react with halogens X2 (mostly (X = Cl, Br) to produce haloalkanes RX also called alkyl halides. Formally, a
halogen atom replaces (substitutes) a hydrogen atom of the alkane RH.
R H + X2 R X + HX X = Cl, Br
alkane haloalkane (alkyl halide)
Halogenation of alkanes involves radicals as reactive intermediates and is therefore a radical substitution. Since the initial
radicals are generated by photodissociation of halogen molecules, the reaction is also referred to as photohalogenation of
alkanes. Three steps contribute to the reaction mechanism:
-initiating step UV light cleaves the halogen molecule into halogen atoms as initiating radicals.
h(photodissociation)
Cl2 2 Cl

-propagating steps One halogen atom cleaves the CH bond of an alkane (e.g. methane) leaving an alkyl radical
(methyl radical) and hydrogen halide (HCl). The alkyl radical propagates the reaction in cleav-
ing another halogen molecule, thus forming a haloalkane (chloromethane) and another halo-
gen radical which goes on to react with another alkane molecule. Each step generates a new
reactive radical in this chain reaction.
Cl + CH 4 HCl + CH 3
CH 3 + Cl2 CH3Cl + Cl

-terminating steps All reactions involving productive or non-productive combination of radicals to give unreactive
molecules (including the product) will terminate this chain reaction.
non-productive non-productive productive
Cl + Cl Cl2 H3C + CH3 H3C CH 3 H3C + Cl H3C Cl
product

Overall, photohalogenation of methane produces a mixture of chloromethane CH3Cl (methyl chloride) , dichloromethane
CH2Cl2 (methylene chloride), trichloromethane CHCl3 (chloroform), and tetrachloromethane CCl4 (carbon tetrachloride),
as shown by the following reaction equations, which only relate the molar amounts of the reactants (left of the arrow) to
those of the products (right of the arrow) and provide no information concerning the mechanism.
H [kJ/mol]
CH 4 + Cl2 CH 3Cl + HCl  104
chloromethane
CH 3Cl + Cl2 CH 2Cl2 + HCl  103
dichloromethane
CH 2Cl2 + Cl2 CHCl3 + HCl  99
trichloromethane
CHCl3 + Cl2 CCl4 + HCl  94
tetrachloromethane

All of these reactions exhibit a negative enthalpy of reaction H; they are exothermic, thus heating the environment so
that cooling is required to control the reaction.

13.2 Relative Stability of Alkyl Radicals


The stability of radicals (and their lifetime) increases with increasing alkylation due to two factors: First, adjacent alkyl
groups shield and thereby protect the unpaired electron. Second, sidewise overlapping of the 2p obital occupied by the
unpaired electron with an sp3 hybrid orbital of an adjacent alkyl C atom (referred to as hyperconjugation) delocalizes the
unpaired electron, as shown in Fig. 13.1.

Efficiently Studying Organic Chemistry: Exam training for chemists, biochemists, pharmacists, life and health scientists,
Third Edition. Eberhard Breitmaier. © 2022 WILEY-VCH GmbH. Published 2022 by WILEY-VCH GmbH.
13.3 Regioselectivity of Radical Substitution 33

p p
H

H H
C C H C
H H

. . . .
H

H < H CH 3 H
H C H 3C C < H3C C < H 3C C CH3
H H H CH 3 sp3 sp2 sp2
methyl ethyl 2-propyl t-butyl radical ethyl radical: methyl radical
sidewise sp -p overlapping
3

Fig. 13.1. Relative stability of alkyl radicals and hyperconjugation which delocalizes the unpaired electron

13.3 Regioselectivity of Radical Substitution


Free-radical fluorination and chlorination of propane proceeds very violently, producing a 1:1 mixture of the two possible
products, the regioisomers 1-halopropane and 2-halopropane. The position of functional groups (e.g. halogen atoms)
differs in regioisomers. Radical bromination of propane, however, proceeds smoothly, predominantly producing 2-
bromopropane (yield 97 %), and iodine does not react at all with propane. Bromination of propane is thus said to be
regioselective. To conclude, high reactivity (F2, Cl2) implies low regioselectivity; moderate reactivity, however, results in
high selectivity (Br2), bromine predominantly reacting with the higher alkylated and therefore longer-living radical (i.e. the
2-propyl radical).

intermediate radicals:
less stable radical
H 3C CH 2 CH2
. .
more stable radical
H 3C CH CH3
X
H3C CH2 CH3 + X2 H 3C CH 2 CH2 X and / or H 3C CH CH3
propane 1-halopropane 2-halopropane
X =F 0°C 50 % 50 % unselective
X = Cl 25 ° C 45 % 55 % weakly selective
X = Br 130 ° C 3% 97 % selective
reactivity X =I no reaction 0% regioselectivity 0%

13.4 Radical Sulfochlorination and Nitration


Photosulfochlorination of alkanes with sulfur dioxide and chlorine to produce alkanesulfonic acid chlorides (Chapter
54.4.2) involves a radical substitution initiated by photodissociation of the chlorine molecule.
O
h / base
R H + SO2 + Cl2 R S Cl + HCl
O
alkanesulfonic acid chloride

Nitroalkanes (e.g. nitromethane, R = CH3) are obtained by radical nitration of alkanes with nitric acid.
> 400 °C
R H + HNO3 R NO2 + HOH
nitroalkane

________________________________________________________________________________________________
Chapter 13 permits answers to the following:
(13.1) Formulate all equations to describe the mechanism of the radical halogenation of alkanes.
(13.2) Which kind of substituents can be introduced into alkanes by radical substitutions?
(13.3) The 2-butyl radical is more stable than the 1-butyl radical. Why?
(13.4) Define the term regioselectivity. How are regioselectivity and reactivity related to each other in photohalogenation
of alkanes?
(13.5) What product is expected from the bromination of butane? Explain, write equations and give the product’s name.
34 14 Alkenes, Skeletal and Configurational Isomers

14 Alkenes, Skeletal and Configurational Isomers


14.1 Nomenclature and Structural Isomerism
Alkenes (olefins) are hydrocarbons with CC double bonds and planar molecular shape (Figs. 4.3, 4.4). Ethene, H2C=CH2,
also named ethylene, is the parent representative. Similar to alkanes, alkenes form a homologous series, partly reviewed
in Table 14.1, with the general molecular formula CnH2n which indicates that they contain two hydrogen atoms less than
the corresponding alkanes (CnH2n+2). Thus, alkenes are unsaturated with respect to hydrogen and therefore also called
unsaturated hydrocarbons, converted into saturated alkanes by addition of hydrogen (hydrogenation). 1-Alkenes are also
referred to as terminal alkenes or vinyl compounds.

Table 14.1. Alkenes: homologous series and structural isomers differing in the position of the double bond
molecular formula 1-alkene structural isomers

C 2H 4 H 2C=CH 2
ethene

C 3H 6 H 2C=CHCH 3
propene

C 4H 8 H 2C=CHCH 2CH 3 H 3CCH=CHCH 3


1-butene 2-butene

C 5H 10 H 2C=CHCH 2CH 2CH 3 H 3CCH=CHCH 2CH 3


1-pentene 2-pentene

C 6H 12 H 2C=CHCH 2CH 2CH 2CH 3 H 3CCH=CHCH 2CH 2CH 3 H 3CCH 2CH=CHCH 2CH 3
1-hexene 2-hexene 3-hexene

Table 14.1 shows that structural isomers with different position of the double bond are found for alkenes with carbon
chains longer than that of propene. Three such structurally isomeric alkenes with an unbranched chain of six carbon
atoms exist for C6H12 , named 1-, 2-, and 3-hexene (Table 14.1).
The survey also illustrates the application of IUPAC rules (Chapter 7) to the nomenclature of alkenes:
- The longest possible carbon chain including both C atoms of the double bond defines the parent name.
- Numbering of C atoms starts at the end of the chain closest to the double bond.
- The ending ane of the alkane chain equal in length is replaced by ene.
Thus, the double bond in the parent skeleton takes preference and defines the positions of side chains as exemplified by
the substituted decene drawn as the condensed structural formula and skeletal formula:

H 3C CH(CH3)2 1
10 4 1 10 8 4
H 3C CH 2 C CH CH2 CH 2 CH CH 2 CH CH 2 7
8 7
H 3C CH 2 CH3 4-ethyl-7-isopropyl-8,8-dimethyl-1-decene

14.2 Relative Configuration


The  bond of alkenes (Chapter 4.2) prevents rotation about the CC double bond. Free rotation would be theoretically
possible only in an excited state after selective homolysis of the  bond, leaving a biradical with an unpaired electron at
both of the alkene C atoms and a single bond between them, permitting rotation.
Therefore, 2-butene and all non-terminal alkenes (alkenes with a double bond not located at the end of the chain) exist as
isomers with different arrangement of substituents in relation to the CC double bond. For this reason they are said to

Efficiently Studying Organic Chemistry: Exam training for chemists, biochemists, pharmacists, life and health scientists,
Third Edition. Eberhard Breitmaier. © 2022 WILEY-VCH GmbH. Published 2022 by WILEY-VCH GmbH.
14.2 Relative Configuration 35

differ in their relative configuration and are specified as configurational isomers. Configurational isomers are classified as
stereoisomers due to their different spatial structure.
Descriptors are used to specify these stereoisomers. If both groups are located on the same side of the double bond, the
isomer is called the (Z)- or cis-alkene (Z from Ger. zusammen = together or Lat. cis = on this side). If both substituents lie
on different sides of the double bond, the isomer is called the (E)- or trans-alkene (E from Ger. entgegen = opposite or
Lat. trans = on the other side).
H H X H
C C C C
X X H X
substituents on the same side substituents on different sides
descriptors: (Z)- or cis- descriptors: (E)- or trans-

Thus, (Z)/(E)- or cis-trans-isomers exist for 1,2-dichloroethene, 2-butene and 2-hexene. These configurational isomers
exhibit different physical properties such as melting and boiling points. At normal pressure, cis-1,2-dichloroethene boils at
a higher temperature (60 °C) than the trans-isomer (48 °C). Thus, in contrast to the conformers of alkanes, the configura-
tional isomers of alkenes can be separated, for example by distillation, and kept as pure compounds.
H H Cl H H H H3C H H H H 3C H
C C C C C C C C C C C C
Cl Cl H Cl H3C CH3 H CH3 H 3C CH 2 CH 2 CH3 H CH 2 CH 2 CH3
(Z)- or cis- (E)- or trans- (Z)- or cis- (E)- or trans- (Z)- or cis- (E)- or trans-
1,2-dichloroethene 2-butene 2-hexene

Molecular models in Fig. 14.1 illustrate the stretched shape of (E)-2-butene and the compact shape of the (Z)-isomer
showing that the hydrogen atoms of the methyl groups approach each other quite closely.

Fig. 14.1. Configurational isomerism: tube, ball-spoke and space-filling molecular models (from left to right) of (Z)-2-butene (top) and the
(E)-isomer (bottom)

________________________________________________________________________________________________
Chapter 14 permits answers to the following:
(14.1) Which unbranched structural isomers exist for the molecular formula C8H16? Give their IUPAC names.
(14.2) There is no free rotation about the  bond of alkenes. Look at Fig. 4.4 and provide an explanation.
(14.3) What are configurational isomers? How do they differ? How are they specified? Why do they differ from
conformers?
(14.4) Which of the alkenes (a) propene, (b) 1-bromopropene, (c) 1-butene, (d) 2-butene, and (e) 3-hexene exist as
configurational isomers? Draw condensed structural and skeletal formulas (Chapters 8.1, 8.2) of all compounds.
(14.5) Draw condensed structural and skeletal formulas of all configurational isomers of the non-terminal octenes
C8H16 and name them.
36 15 Synthesis of Alkenes

15 Synthesis of Alkenes
15.1 -Elimination
-Elimination (Chapter 11.2) of hydrogen halide from haloalkanes (dehydrohalogenation), of water from alcohols (dehy-
dration), and of halogens from ,-dihaloalkanes (reductive dehalogenation) are general methods to prepare alkenes.

15.1.1 Dehydrohalogenation of Haloalkanes


The -elimination of hydrogen halide from an alkyl halide (dehydrohalogenation) to give an alkene requires the presence
of hydroxide (OH) or another base. This facilitates the abstraction of the proton at the -position (producing H2O), such
that the transition state is converted into the alkene.
H HO H
C C + OH C C C C
X ,  H2O
X X
haloalkane (X = Cl, Br) transition state alkene

According to the SAYTZEFF rule, the more stable alkene with higher alkylation at the double bond is predominantly (regio-
selectively) formed because protons are more easily removed from higher alkylated carbon atoms of longer-chain al-
kanes. Dehydrobromination of 2-bromopentane, for example, yields 2-pentene (trans- and cis-isomers) as the major
product.
KOH in methanol
H5C2 CH 2 CH CH3 H5C2 CH CH CH 3 and H5C2 CH2 CH CH2
 HBr
Br
2-bromopentane 2-pentene (major) 1-pentene (minor)

The reaction proceeds bimolecularly because two compounds react (alkyl halide and base) in one step involving a transi-
tion state (no intermediates). The reaction rate  depends on the concentrations [RX] and [OH] of both reactants, thus
following a second-order rate law ( = k [RX][OH]); the factor k is a rate constant which is individual for each reaction.
This type of elimination is therefore said to have an E2 mechanism (E for elimination, 2 for bimolecular and second
order).
Highly branched haloalkanes, however, such as 2-bromo-2-methylpropane (t-butyl bromide), eliminate hydrogen halide in
two steps: The first and slow step involves dissociation of a bromide anion, leaving a highly alkylated carbenium ion (t-
butyl cation) which is, compared with unbranched alkyl cations, quite stable and longer-living. Once formed, this carbeni-
um ion loses a proton in a fast second step.

CH 3  Br CH 3 CH 3
slow fast
C C C
H 3C Br H H 3C CH 2
H 3C H 3C CH 2 H

t-butyl bromide t-butyl cation (a carbenium ion) methylpropene (isobutene)


positive charge = vacant p orbital

Overall, the reaction cannot proceed faster than the slowest step; thus, the first step is rate-determining. Only one mole-
cule (t-butyl bromide) reacts in this step which is therefore monomolecular. The rate depends only on the concentration
[RX] of the alkyl halide, thus obeying a first-order rate law ( = k [RX]). This kind of elimination is said to follow the E1
mechanism (E for elimination, 1 for monomolecular and first order).

15.1.2 Dehydration of Alcohols


Dehydration of alcohols opens another access to alkenes. This dehydration involves a -elimination of water and is
performed by heating an alcohol in the presence of a strong acid (e.g. H2SO4) which protonates the OH group. The
intermediate oxonium ion eliminates water, thus leaving a carbenium ion which loses one proton at a -position generat-

Efficiently Studying Organic Chemistry: Exam training for chemists, biochemists, pharmacists, life and health scientists,
Third Edition. Eberhard Breitmaier. © 2022 WILEY-VCH GmbH. Published 2022 by WILEY-VCH GmbH.
15.2 Alternative Syntheses 37

ing the double bond of the alkene. Provided the elimination of water from the protonated alcohol is the rate-determining
step, the dehydration also follows an E1 mechanism.
fast slow fast
OH OH2
+ [H ] H2O
C C C C C C C C
 [H ]
H H H
alcohol oxonium ion carbenium ion alkene

Acid-catalyzed dehydration of 2,3-dimethyl-2-butanol to give 2,3-dimethyl-2-butene is a typical example.


H 3C OH H3C CH3
[H ]
H3C C C CH 3 C C
 H2O
H CH3 H3C CH3
2,3-dimethyl-2-butanol 2,3-dimethyl-2-butene

15.1.3 Reductive Dehalogenation of 1,2-Dihaloalkanes


Dihaloalkanes with halogens at adjacent carbon atoms eliminate halide anions when reacted with zinc dust or other
reducing metals as donors of electrons (reductive dehalogenation). The addition of a halogen such as bromine to an
alkene is the reverse of this reaction (Chapter 16.2).
Br R H R R Br
+ Zn in ethanol,  ZnBr 2 C
e0 C
R CH CH R C C and C C Zn + ZnBr2
+ Br 2 e0 C C
Br H R H H Br
,-dibromoalkane trans- and cis-alkene

15.2 Alternative Syntheses


15.2.1 Dehydrogenation of Alkanes
Lower alkenes are produced industrially by catalytic dehydrogenation (elimination of H2) of the corresponding alkanes at
high temperatures (> 450 °C). Higher alkane chains are cleaved in part such that shortened alkenes are obtained ("crack-
ing"). Mixtures of oxides (Al2O3, Cr2O3 , SiO2) are used as catalysts.
cat.
H 3C CH 3 H 2C=CH2 + H2
cat.
2 H3C CH2 CH 3 H3C CH=CH 2 + H2C=CH 2 + CH4 + H2

15.2.2 Partial Hydrogenation of Alkynes


Partial hydrogenation of alkynes is a general method to prepare alkenes. Catalytic hydrogenations of alkynes with deac-
tivated catalysts (LINDLAR catalysts, e.g. Pd/BaSO4) yield cis-alkenes while trans-alkenes are obtained by reduction with
sodium as the donor of electrons and ammonia as provider of hydrogen (Chapter 19.3.1).
+ 2 Na, + 2 NH3
R R + H2 (LINDLAR cat.)
R H
 72 °C
(Z)-alkene C C R C C R C C (E)-alkene
 2 NaNH2
H H alkyne H R

15.2.3 Reductive Coupling of Carbonyl Compounds


Reductive coupling of carbonyl compounds (aldehydes and ketones, Chapters 47, 48) with low-valent titanium involving
intermediate ketyl radicals opens access to bulky substituted alkenes (MCMURRY reaction).
aldehyde (R' = H) or ketone Ti(I) Ti... Ti R = R' = CH(CH3)2 : tetraisopropylethene
TiCl4 O O R R
R O Li Hg R O
2 Ti(0) + 2 C 2 C C C C C
R R
R' R'  2 TiO
R' R' R' R'
ketyl radical titanium(I) pinacolate alkene
38 15 Synthesis of Alkenes

15.2.4 Carbonyl Alkenylations


Carbonyl alkenylations of aldehydes and ketones (carbonyl compounds) with carbon nucleophiles such as activated
methylene compounds are applied for the synthesis of alkenes.
carbonyl alkenylation
C O + Z2 C C C
Z2O
carbonyl compound C nucleophile Z2 = H2, R3P alkene

- KNOEVENAGEL Alkenylation
Aldehydes (R1 = alkyl, R2 = H) and ketones (R1 = R2 = alkyl) undergo dehydration with activated methylene compounds in
the presence of bases, yielding alkenes (KNOEVENAGEL alkenylation, Chapter 51.2.2).
R1 base R1
C O + H2C C C
H 2O
R2 R2
carbonyl compound activated methylene alkene
compound

- WITTIG Alkenylation
Driven by the formation of stable PO double bonds, alkylidenephosphoranes react with aldehydes and ketones to provide
alkenes and phosphane oxides (WITTIG alkenylation, Chapter 49.3.4).
R1 R R1 R
C O + (C6H5)3P C C C
 (C6H5) 3P=O
R2 R R2 R
carbonyl compound alkylidene phosphorane alkene

15.3 Transformation of Alkenes


15.3.1 WOHL-ZIEGLER Bromination
Alkenes such as propene are brominated at the allylic position ( to the double bond) by N-bromosuccinimide, prepared
from succinimide (Chapter 42.4), bromine and aqueous sodium hydroxide:
O O
CCl4
H2C CH CH3 + N Br H2C CH CH2 Br + N H

O 3-bromo-1-propene O
propene N-bromosuccinimide (allyl bromide) succinimide

The reaction follows the pattern of radical substitution (Chapter 13.1) with intermediate allyl radicals, and bromine arising
from the reaction of N-bromosuccinimide with hydrogen bromide generated in step 1.
step 1 O O
R H + Br R + HBr
step 2
R + Br2 R Br + Br N Br + HBr N H + Br2

R = H2C CH CH2 (ally l radical) O O

15.3.2 HECK Reaction


The HECK reaction of an alkenyl or aryl halide (RX) with an alkene in the presence of a palladium(0) complex with the
ligand L as catalyst and a base results in substitution of an alkene hydrogen by an alkenyl or aryl group (Chapter 24.1).
H base , L2Pd(0)
R
R X + C C C C e. g. R= ,
 HX
R = alkenyl, aryl; X = Br, I alkenyl aryl
15.3 Transformation of Alkenes 39

15.3.3 Ene Reaction


An alkene containing an H atom at the alkyl group  to the double bond (an allylic hydrogen), called the ene, may react
with a compound containing a multiple bond, called the enophile. In the simplest case of this ene reaction, propene as
ene and ethene as enophile react to give 1-pentene when heated under pressure or in the presence of a LEWIS acid. The
mechanism involves a concerted shift of the allylic hydrogen with a cyclic transition state equally distributing the six
electrons of both  bonds and the CH  bond among all atoms.
H H H
+

propene ethene transition state 1-pentene


ene enophile

15.3.4 Alkene Metathesis


In the presence of an oxide or an alkylidene complex of some transition metals (Mo, W, Re, and Ru) as a catalyst, two
different alkenes may equilibrate to new alkenes by interchanging their alkylidene groups (R1R2C= and R3R4C=). This
reaction is referred to as a cross-metathesis:
R1 R1

R1 R2 R1 R2 C C PR3
C C cat. R2 R2 Cl
+ + e.g. cat. = Ru
C C R3 R3 Cl
R3 R4 R3 R4 R'
C C PR3
R4 R4

Catalytic cross-metathesis is used industrially to produce alkenes with averaged chain lengths from symmetrically substi-
tuted alkenes with shorter and longer chains:
C4 C 18 C 11
H 3C H H nC8H 17 MoO3 / Al2O3 H 3C nC8H 17
+ 2
H 3C H H nC8H 17 H H
(Z)-2-butene (Z)-9-octadecene 2-undecene, (E)- and (Z)-

Terminal alkenes such as 2-methyl-1-butene undergo catalytic self-metathesis, liberating ethene as a volatile gas and
leaving a symmetric alkene with longer alkylidene groups, such as liquid 3,4-dimethyl-3-hexene:
cat.
+ + cat. = WCl 6
ethanol

2-methyl-1-butene 3,4-dimethyl-3-hexene ethene


(E)- and (Z)-

________________________________________________________________________________________________
Chapter 15 permits answers to the following:
(15.1) What mechanisms exist for the dehydrohalogenation of alkyl halides? Formulate appropriate equations.
(15.2) What is the mechanism for the acid-catalyzed dehydration of an alcohol? Write an appropriate equation.
(15.3) Dehydrobromination of 2-bromobutane predominantly yields 2-butene. Why?
(15.4) 2,3-Dimethyl-2-butanol dehydrates to give 2,3-dimethyl-2-butene and not to 2,3-dimethyl-1-butene. Why?
(15.5) Formulate the general conversion of a carbonyl compound into an alkene.
(15.6) 1-Butene reacts with N-bromosuccinimide. What product do you expect? Formulate the reaction.
(15.7) Methylpropene and ethene are heated under pressure. What product is likely to be formed?
(15.8) Design a synthesis of (a) 3,4-diisopropyl-2,5-dimethyl-3-hexene and (b) cis- and trans-4,5-dimethyl-4-octene.
40 16 Additions to Alkenes

16 Additions to Alkenes
Additions of molecules such as HX or X2 to the CC double bond are typical reactions of alkenes and the reverse of elimi-
nations (Chapters 11.1, 11.2, 15.1).

16.1 Addition of Hydrogen (Catalytic Hydrogenation)


Catalytic hydrogenation of alkenes to provide alkanes involves the addition of hydrogen to a CC double bond in the
presence of a group 10 metal (Ni, Pd, Pt) as catalyst. As illustrated in Fig. 12.1, the catalyst decreases the activation
barrier of hydrogenation and thus the reaction proceeds more rapidly. The hydrogen molecule, activated at the surface of
the catalyst, collides as a whole with the CC double bond, forming a cyclic transition state to induce a four-center reac-
tion. Thus, after having added from one side of the CC double bond, both hydrogen atoms are attached at the same side
of the CC double bond meaning that they will adopt a syn arrangement (syn- or cis-addition). This stereospecificity,
however, is undetectable in open-chain alkanes because of free rotation about the CC single bond arising from the hy-
drogenation.

C C
C C
H H H
H
H H
Ni, Pd or Pt

H
H H
H
H2 on the surface of the catalyst cyclic transition state of a H atoms in the alkane
adsorbed and activated four-center reaction are arranged syn

16.2 Addition of Bromine (Bromination)


The addition of brownish-red bromine to an alkene giving an ,-dibromoalkane decolorizes the solution and thereby
indicates the presence of a CC double bond. The mechanism of bromination starts with a polarization of the bromine
molecule upon collision with the alkene, resulting in a transition state which is converted into an intermediate three-
membered cyclic bromonium ion with loss of a bromide anion. This bromide anion adds from the backside of the bulky
bromonium ion (anti- or trans-addition), opening the strained three-membered ring to yield an ,-dibromoalkane.

Br
C C C C + Br C C
+ Br
 Br
Br Br 
Br Br  bromonium ion ,-dibromoalkane

The stereospecificity of this reaction is not detectable for open-chain alkenes because of free rotation about the newly
formed CC single bond, so that 2,3-dibromobutane is obtained by bromination of both configurational isomers of 2-
butene:
H3C H Br Br H 3C CH3
+ Br2 + Br2
C C H 3C CH CH CH3 C C
H CH3 H H
trans-2-butene 2,3-dibromobutane cis-2-butene

Chlorination of alkenes also follows the mechanism of bromination, involving an intermediate three-membered halonium
ion. Fluorine is too reactive for a controlled reaction: all bonds are attacked and thus mixtures of compounds are pro-
duced. Iodinations proceed too slowly to be of synthetic significance.

Efficiently Studying Organic Chemistry: Exam training for chemists, biochemists, pharmacists, life and health scientists,
Third Edition. Eberhard Breitmaier. © 2022 WILEY-VCH GmbH. Published 2022 by WILEY-VCH GmbH.
16.3 Electrophilic Addition of Hydrogen Halide (Hydrohalogenation) 41

16.3 Electrophilic Addition of Hydrogen Halide (Hydrohalogenation)


Hydrogen halides (HCl, HBr, HI) add to alkenes yielding alkyl halides. This reaction, called hydrohalogenation of alkenes,
follows a stepwise ionic mechanism involving the proton of the hydrogen halide as an electrophile: In the first step, the
electrophilic proton adds to the CC double bond (electrophilic addition), thus generating an intermediate carbenium ion
which reacts with the halide anion in a fast reaction of the ions to produce the haloalkane (alkyl halide).
H H
C C + H Br C C + Br C C
Br
carbenium ion bromoalkane

Ethene (H2C=CH2) is readily hydrochlorinated to give chloroethane (H3CCH2Cl). Hydrohalogenation of an unsymmetri-


cally substituted alkene, such as propene in the simplest case, may produce two structural isomers of the resulting alkyl
halides, with different positions of the halogens, called regioisomers. Thus, hydrobromination of propene may produce the
regioisomers 2-bromopropane and 1-bomopropane:

H 3C H
C C + H Br H 3C CH CH 3 and H 3C CH 2 CH2 Br H 3C CH CH3 more stable than H 3C CH 2 CH 2
H H Br
2-bromopropane (major) 1-bromopropane (minor)

The regioselectivity of hydrohalogenation, favoring one and discriminating against the other product, is predicted by the
MARKOVNIKOV rule: Electrophilic addition preferentially involves the most stable carbenium ion. Since the relative stability
of radicals (Chapter 13.2) as well as of carbenium ions increases with increasing alkylation, the 2-propyl cation is more
stable than the 1-propyl cation. As a result, hydrobromination of propene predominantly yields 2-bromopropane (isopropyl
bromide).

16.4 Electrophilic Addition of Water (Hydration)


Water adds to the CC double bond of alkenes in the presence of an acid to give alcohols (Chapter 33.3.3). This hydration
of alkenes, the reverse of dehydration of alcohols to alkenes (Chapters 15.1.2, 36.1), also proceeds as an electrophilic
addition, because the electrophilic proton of the acid adds to the double bond in the first and slow step. Water adds to the
resulting intermediate carbenium ion to give an intermediate oxonium ion which deprotonates to yield the alcohol.
H H H
C C + H3O C C + H 2O C C C C
 [H ]
OH 2 OH
carbenium ion oxonium ion alcohol

Again, MARKOVNIKOV´s rule predicts the regioselectivity of hydration taking into account the relative stabilities of the
intermediate carbenium ions. Thus, hydration of methylpropene (isobutene) almost exclusively yields 2-methyl-2-propanol
(t-butyl alcohol) because the t-butyl cation is much more stable and longer-living than the isomeric methylpropyl cation
with a terminal positive charge.

H3C CH 3 CH3 CH 3 CH3


H3O , 20 °C
C CH2 + H2O H 3C C CH3 and not H 3C CH CH2 OH H 3C C CH3 more stable than H3C CH CH2
H3C OH
methylpropene 2-methyl-2-propanol 2-methyl-1-propanol
(isobutene) (t-butyl alcohol)
42 16 Additions to Alkenes

16.5 Halohydrin Formation


Halohydrins (2-haloalcohols) are formed by addition of halogen and water to alkenes. For example, 2-bromoethanol
(ethylene bromohydrin) is obtained by reacting ethene, bromine and water.

H 2C CH 2 + Br2 + H2O Br CH 2 CH2 OH + HBr


2-bromoethanol
(ethylene bromohydrin)

Halohydrin formation involves an electrophilic addition of the positive halogen end of hypohalous acid to the double bond.
The resulting intermediate ion pair, a -halocarbenium ion and hydroxide anion, quickly reacts to provide the halohydrin:

slow fast
Br2 + H2O HBr + HOBr HO Br + C C Br C C + IOH Br C C OH
hypobromous
acid -bromocarbenium ion bromohydrin

16.6 Hydroboration
Diborane, B2H6, adds to alkenes. The monomer borane, BH3, is assumed to act as the reactive compound. This addition,
referred to as hydroboration, involves a four-center mechanism, with electrophilic addition of BH2 and nucleophilic
addition of the hydride part of borane occurring simultaneously. A trialkylborane is produced in three successive steps.

R 2C CH2 R 2CH CH 2 + R2 C CH 2 + R2C CH 2


+ (R2CH CH 2 )2 BH (R 2CH CH2 )3 B
H BH2 BH2
alkylborane dialkylborane trialkylborane

The addition is controlled by the spatial requirements of groups (thus said to be stereocontrolled) so that the bulkier BH2
attacks at the less hindered part of the alkene. Methylpropene (R = CH3), as an example, exclusively reacts to provide
triisobutylborane.

16.7 Dihydroxylations
- cis-Dihydroxylation with Osmium Tetroxide or Potassium Permanganate
Osmium tetroxide or potassium permanganate undergo a [3+2]-cycloaddition (2 atoms of the alkene, and 3 of the oxide)
with alkenes.Cyclic esters (osmates and manganates) are the primary products, liberating 1,2-diols (alcohols with two
adjacent hydroxy groups) with syn arrangement of OH groups upon hydrolysis.

+ 2 H2O
O O 0 °C O O OH
C C ( Na2SO3 ) C
+ Os Os
C C O  H2OsO4 C
O O O OH
osmium tetroxide
cyclic esters 1,2-diols (with syn OH groups)

O O 0 °C O O OH
C C + 2 H2O C
+ Mn Mn
C O C O HMnO4 C
O O OH
 OH
permanganate

- trans-Dihydroxylation with Peroxy Acids Involving Oxiranes


Peroxy acids convert alkenes into oxiranes (epoxides, PRILEZHAEV epoxidation), a process which involves a [2+1]-
cycloaddition (2 for two carbon atoms of the alkene, 1 for the adding oxygen of the peroxy acid). The oxirane ring is
16.8 1,3-Dipolar Cycloaddition of Ozone (Ozonolysis) 43

opened upon protonation and backside attack of water (similar to the opening of bromonium ions described Chapter
16.2), yielding 1,2-diols with OH groups arranged anti to each other.

O R OH OH
C C C + H3O C C  [H ] C
+ O O H2O + O H
C O  RCO2H C C C C
H H 2O HO

alkene peroxycarboxylic acid oxirane protonated oxirane trans-addition 1,2-diol

The stereospecificity of these dihydroxylations is not detectable for non-cyclic alkenes due to free rotation about the
newly formed CC single bond, and both cis- and trans-2-butene are dihydroxylated to yield 2,3-butanediol:
1. OsO4, 2. H2O 1. OsO4, 2. H2O
or OH OH or
H 3C H 1. RCO3H, 2. H3O 1. RCO3H, 2. H3O
H 3C CH 3
C C H 3C CH CH CH 3 C C
H CH 3 H H
trans-2-butene 2,3-butanediol cis-2-butene

16.8 1,3-Dipolar Cycloaddition of Ozone (Ozonolysis)


Canonical formulas depict ozone as a 1,2- and 1,3-dipole. As a 1,3-dipole, ozone adds to alkenes, and this kind of [2+3]-
cycloaddition is called a 1,3-dipolar cycloaddition. An initially formed five-membered primary ozonide rearranges to the
corresponding ozonide. In the presence of a reducing reagent, water cleaves the ozonide to yield carbonyl compounds
(aldehydes or ketones, Chapters 47, 48).
ozone
1,3-dipole 1,2-dipole
O O
O O O O
+ O
O O O O O O + H2O
R R R
( Zn, H3O )
C C C C R 2C CR 2 R2C CR 2 2 C O
R R O O  H2 O2
R R R R R
alkene primary ozonide ozonide carbonyl compound
(one R = H : aldehyde
both R = alkyl : ketone)

The resulting carbonyl compounds permit reconstruction of the original alkene for structure elucidation. Acetone (pro-
panone) and butanal as products of ozonolysis and subsequent hydrolysis, for example, identify 2-methyl-2-hexene as
the original alkene.
H3C H O3 / Zn / H3 O
H 3C H
C O + O C C C
H3C CH2 CH2 CH 3 H 3C CH 2 CH 2 CH 3
propanone (acetone) butanal 2-methyl-2-hexene
a ketone an aldehyde

________________________________________________________________________________________________
Chapter 16 permits answers to the following:
(16.1) Suggest the preparation of (a) 2,3-dimethylpentane and (b) 2,3-dibromohexane from appropriate alkenes.
(16.2) What is an electrophilic addition? Write mechanisms for the bromination and hydrobromination of alkenes.
(16.3) What products are obtained by hydrobromination of (a) 2-methyl-2-butene and (b) 1-hexene?
(16.4) What products arise from (a) hydration and (b) hydroboration of 2-methyl-1-butene?
(16.5) Which methods can be used to prepare 1,2-diols from alkenes? Formulate general reaction equations.
(16.6) Stereospecificity of hydrogenation, bromination and dihydroxylation for non-cyclic alkenes is not detectable. Why?
(16.7) Which alkenes give only (a) propanone and (b) butanal upon ozonolysis followed by hydrolysis?
44 17 Dienes

17 Dienes
17.1 Cumulation and Conjugation of Double Bonds
Dienes, trienes, tetraenes, and polyenes contain two, three, four, and more CC double bonds. In polyenes, CC double
bonds may be arranged cumulated (originating from one C atom), conjugated (alternating with single bonds), and isolated
(two or more single bonds between double bonds). Three structurally isomeric pentadienes, differing in the positions of
the double bonds, demonstrate this.
1 2 3 4 5 1 2 3 4 5 1 2 3 4 5
H2C C CH CH 2 CH 3 H 2C CH CH CH CH3 H 2C CH CH 2 CH CH2
1,2-pentadiene 1,3-pentadiene 1,4-pentadiene
cumulated double bonds conjugated double bonds isolated double bonds

1,3-Butadiene is an industrially produced representative of the conjugated dienes. 1,2-Dienes are also known as cu-
mulenes or allenes, referring to the simplest representative 1,2-propadiene called allene.
H 2C CH CH CH 2 or 1,3-butadiene H 2C C CH 2 1,2-propadiene (allene)

17.2 Molecular Structure


17.2.1 Conformation of 1,3-Butadiene
There is free rotation around the central CC single bond of 1,3-butadiene. Two conformers therefore exist, the more
stable s-trans-conformer favored by the molecules and the less stable s-cis-conformer (s comes from single bond).
Epot
s-cis
s-trans (more populated by molecules)
s-trans s-cis

17.2.2 Bond Lengths and Resonance Formulas of 1,3-Butadiene


The central CC single bond of 1,3-butadiene is shorter than the CC single bond of ethane, and the CC double bonds of
1,3-butadiene are longer than that of ethene.
bond length [pm] 154 134 137 148
H 3C CH 3 H 2C CH 2 H 2C CH CH CH 2
ethane ethene 1,3-butadiene

Clearly, formula a (Fig. 17.1) does not correctly describe the structure of 1,3-butadiene because compression of the
central single bond implies a partial double bond, as portrayed by the biradical formula b or the dipolar (zwitterionic)
formula c.

Epot a b c
C C C C C C H H
C H
E = 14.7 kJ/mol (resonance energy) C C
C C C C C C H C H
H

a b c 1,3-butadiene (genuine molecule) d

Fig. 17.1. a-c: Resonance formulas and resonance energy of 1,3-butadiene; double-headed arrows between formulas a-c symbolize
resonance; d: molecular orbital model: continuous sidewise overlapping of p orbitals resulting in delocalized  bonding

Not one but all three formulas (a, b, c) together account more accurately for the real state of the bonds in 1,3-butadiene.
This feature of conjugated dienes is referred to as resonance. 1,3-Butadiene is said to be a hybrid of the three canonical
formulas (a-c), describing delocalized double bonds arising from continuous sidewise overlapping of p orbitals (Fig. 17.1
d). Hydrogenation of 1,3-butadiene liberates 14.7 kJ/mol less than twice the heat obtained for ethene with one localized
double bond. Thus, the genuine 1,3-butadiene has a lower energy than a hypothetical 1,3-butadiene with two localized
double bonds (formula a, Fig. 17.1): it is stabilized. The stabilizing energy difference (14.7 kJ/mol) is attributed to reso-
nance and therefore called the resonance energy of 1,3-butadiene (Fig. 17.1); each of the three canonical formulas is
less stable.

Efficiently Studying Organic Chemistry: Exam training for chemists, biochemists, pharmacists, life and health scientists,
Third Edition. Eberhard Breitmaier. © 2022 WILEY-VCH GmbH. Published 2022 by WILEY-VCH GmbH.
17.3 Preparation 45

17.2.3 Molecular Shape of Allene


Two  bonds originate from the central carbon atom in allene (1,2-propadiene) and other cumulated dienes. The MO
model explains these bonds by sp hybridization of the central C atom, thus providing two p orbitals for sidewise overlap-
ping to two  bonds. Since the axes of these p orbitals are perpendicular, and only coaxial p orbitals are able to overlap
to  bonds, the p orbitals at the terminal carbon atoms must also extend along axes perpendicular to each other. The
same applies for the planes of the sp2 hybrid orbitals overlapping to give the  bonds of allene so that these bonds ar-
range themselves in a stretched tetrahedron (Chapter 45.2).

allene H 
H
H  H
molecular orbital model C C C C C C molecular shape
  H
H H
H

17.3 Preparation
17.3.1 Catalytic Dehydrogenation of Alkanes to 1,3-Dienes
1,3-Butadiene as starting chemical for the production of polymers (rubber substitutes, Chapter 58) is prepared by catalytic
dehydrogenation of butane via 1- and 2-butene. Analogous procedures yield other 1,3-dienes.

cat., heat
H3C H H3C CH 3 cat., heat
H3C CH 2 CH2 CH3
 H2
H2C CH CH 2 CH3 , C C , C C
 H2
H 2C CH CH CH 2
H CH 3 H H
butane 1-butene 2-butene, trans- and cis- 1,3-butadiene

17.3.2 Dehydration of Diols to 1,3-Dienes


Acid-catalyzed dehydration of 1,2-, 1,3-, or 1,4-diols affords 1,3-dienes. 2-Methyl-1,3-butadiene, known as isoprene, the
structural unit of terpenes including caoutchouc (Chapter 76.3.6) and starting material to produce the latter industrially, is
obtained by heating () 3-methyl-1,3-butanediol in the presence of an acid.
CH3 CH 3
 , [H ]  , [H ]
H3C C CH 2 CH2 OH H 2C C CH CH2 H 3C CH CH2 CH 2 CH CH 3 H 3C CH CH CH CH CH 3
 2 H2O  2 H2O
OH 2-methyl-1,3-butadiene OH OH 2,4-hexadiene
3-methyl-1,3-butanediol (isoprene) 2,5-hexanediol

17.3.3 Catalytic Dimerization of Ethyne to 1,3-Butadiene


In the presence of cuprous chloride and ammonium chloride (NIEUWLAND catalyst), ethyne (acetylene) dimerizes to give 1-
buten-3-yne (vinylacetylene) which yields 1,3-butadiene on partial catalytic hydrogenation.
(NH4Cl / CuCl) + H2 (cat.)
2H C C H H C C CH CH2 H2C CH CH CH2
ethyne (acetylene) 1-buten-3-yne 1,3-butadiene

17.3.4 1,2-Dienes by Elimination


1,2-Propadiene (allene) arises from dehydrohalogenation (Chapter 15.1.1) of allyl bromide (Chapter 15.3.1) or by reduc-
tive dechlorination (Chapter 15.1.3) of 2,3-dichloro-1-propene with zinc dust.
Cl
base + Zn (ethanol)
H2C CH CH 2 Br H 2C C CH2 H2C C CH 2 Cl
 HBr  ZnCl2
3-bromo-1-propene (allyl bromide) 1,2-propadiene (allene) 2,3-dichloro-1-propene
________________________________________________________________________________________________
Chapter 17 permits answers to the following:
(17.1) Which structural isomeric hexadienes exist and how are these classified? Write the formulas.
(17.2) What are the appropriate formulas of 1,3-butadiene? What is the meaning of the term resonance in this context?
(17.3) There are several methods to prepare 1,3-butadiene. Formulate the equations for these reactions.
(17.4) Suggest a resonable synthesis of 1,2-hexadiene.
46 18 Additions and Cycloadditions with 1,3-Dienes

18 Additions and Cycloadditions with 1,3-Dienes


18.1 1,2- and 1,4-Addition
The addition of bromine or hydrogen bromide to 1,3-dienes yields a mixture of 1,2- and 1,4-addition products. At low
temperatures the 1,2-addition is preferred, whereas at high temperatures the 1,4-addition predominates.
1 2 3 4 1 2 3 4
H2C CH CH CH 2 + Br2 Br CH 2 CH CH CH 2 and Br CH 2 CH CH CH2 Br
Br (E + Z)
1 2 3 4 1 2 3 4
H2C CH CH CH 2 + HBr H 3C CH CH CH 2 and H 3C CH CH CH 2 Br
Br (E + Z)
1,2-addition 1,4-addition

The electrophilic addition to 1,3-dienes involves resonance-stabilized carbenium ions. Hydrobromination of 1,3-butadiene,
for example, starts with the electrophilic addition of a proton to one of the double bonds, generating an intermediate
carbenium ion which can be depicted as a hybrid of two resonance formulas, indicating a distribution (delocalization) of
positive charge between positions 2 and 4:

H2C CH CH CH2 + H Br H CH2 CH CH CH 2 H CH2 CH CH CH 2 or H CH2 CH ...CH ...CH 2


1,3-butadiene  Br

explaining the term allyl, allylic: H2C C CH2 R CH2 CH CH2 R CH CH CH2 R CH CH CH2
allene allyl group a resonance-stabilized allyl cation

The stability of this allyl cation is comparable to that of a tertiary carbenium ion because delocalization of the positive
charge lowers its potential energy level, hence stabilizing it (resonance stabilization).
In the subsequent step of hydrobromination, the resonance-stabilized carbenium ion is subjected to a temperature-
dependent equilibrium with the bromide anion (Fig. 18.1). Because C-2 and C-4 share the positive charge, the bromide
ion has the choice to add to one of these two C atoms. As shown in Fig. 18.1, 1,2-addition requires the lower barrier of
activation, therefore proceeds faster and is favored at low temperature (kinetic control). At higher temperatures, however,
1,4-addition takes preference and the more stable 1,4-addition product is formed (thermodynamic control) which cannot
as easily reequilibrate to the ions because of the higher barrier of activation (Fig. 18.1, Chapter 12.3).

Epot

transition states

EA

EA

CH2 CH... CH... CH2


H + Br
CH2 CH CH CH 2
H Br

CH2 CH CH CH 2
H Br
1,4-addition 1,2-addition
favored at high temperature favored at low temperature

Fig. 18.1. Competition of 1,2- and 1,4-addition to 1,3-dienes (hydrobromination of 1,3-butadiene)

Efficiently Studying Organic Chemistry: Exam training for chemists, biochemists, pharmacists, life and health scientists,
Third Edition. Eberhard Breitmaier. © 2022 WILEY-VCH GmbH. Published 2022 by WILEY-VCH GmbH.
18.2 Cycloadditions 47

1,3-Dienes react more readily than alkanes (Chapter 13) with halogen radicals (atoms) arising from photodissociation
because the intermediate allylic radicals are resonance-stabilized, similar to the allyl cations:
1 2 3 4
H2C CH CH CH2 + Cl Cl CH2 CH CH CH2 Cl CH2 CH CH CH2
or
... ...
Cl CH 2 CH CH CH2

The carbon atoms C-2 and C-4 share the unpaired electron, so both 1,2- and 1,4-addition are possible in the subsequent
reaction steps. Radical addition of chlorine to 1,3-butadiene thus produces 3,4-dichloro-1-butene and 1,4-dichloro-2-
butene (trans- and cis-isomers):
... ... + Cl2 Cl CH 2 CH CH CH2 and Cl CH2 CH CH CH2 Cl
Cl CH2 CH CH CH2
 Cl
Cl trans- and cis-

18.2 Cycloadditions
A cycloaddition involves addition of two reactants with double or triple bonds, with closure of carbon rings to give cyclic
compounds such as cycloalkanes and cycloalkenes.

18.2.1 [4+2]-Cycloaddition (DIELS-ALDER Reaction)


The reaction of a 1,3-diene (C4 reactant providing 4  electrons) with an alkene (C2 reactant providing 2  electrons),
called the dienophile, produces a cyclohexene ring. This process is known as the [42]-cycloaddition or DIELS-ALDER
reaction which offers an efficient access to six-membered rings (Chapter 21.4). In the transition state, all six  electrons
are distributed equally among the six carbon atoms involved.

X X X X
+ 
6 or
Y Y Y Y
1,3-diene dienophile transition state X,Y-disubstituted cyclohexene
4 electrons 2 electrons  electrons

The relative configuration of the substituents in the reactants is conserved in the cycloaddition product. If X and Y adopt
the cis-configuration in the dienophile as shown, they will keep their cis-configuration in the cycloaddition product. There-
fore, the DIELS-ALDER reaction is said to be stereospecific (Chapter 78.4), converting a stereochemically pure reactant
(e.g. a cis-alkene) into a stereochemically pure product (e.g. a cis-disubstituted cyclohexene).

18.2.2 [4+1]-Cycloaddition
1,3-Dienes (C4 reactant) undergo cycloaddition with sulfur dioxide (S1 reactant) providing a non-bonding electron pair (2 n
electrons) for closure to a five-membered ring. A sulfolane (2,5-dihydrothiophene 1,1-dioxide) is the product. This [4+1]-
cycloaddition is reversible meaning that the liquid sulfolanes are storable precursors of 1,3-dienes, for example for per-
forming DIELS-ALDER reactions with unstable 1,3-dienes.

O
cold O
+ IS S
heat O
O
1,3-diene sulfolane
4 electrons 2 n electrons

________________________________________________________________________________________________
Chapter 18 permits answers to the following:
(18.1) What products are obtained by bromination and hydrobromination of 1,3-butadiene?
(18.2) How can the selectivity of hydrobromination of 1,3-butadiene be controlled?
(18.3) Write the equation for a [42]-cycloaddition. What is the preparative significance of this reaction?
(18.4) What product is expected from the reaction of 1,3-butadiene and trans-1,2-dibromoethene? Write the equation.
48 19 Alkynes

19 Alkynes
19.1 Homologous Series, Structural Isomerism, Nomenclature
Alkynes are hydrocarbons containing CC triple bonds. Ethyne, also named acetylene, is the parent representative (Table
19.1). Similar to alkanes and alkenes, alkynes form a homologous series, partly reviewed in Table 19.1, with the general
molecular formula CnH2n2 which indicates that they contain four hydrogen atoms less than the corresponding alkanes
(CnH2n+2), thus representing another kind of unsaturated hydrocarbons.
The nomenclature of alkynes follows the IUPAC rules of alkanes (Chapters 7.1, 14.1, Table 7.1), the ending yne replac-
ing ane; 1-, 2-, and 3-hexyne are structural isomers, differing in the position of their triple bond. 1-Alkynes are also re-
ferred to as terminal alkynes.

Table 19.1. Homologous series and structural isomers of alkynes


molecular formula 1-alkyne s t r u c t u r a l i s o m e r (s)
C 2H 2 H C C H
ethyne
C 3H 4 H C C CH 3
propyne
C 4H 6 H C C CH 2 CH 3 H 3C C C CH 3
1-butyne 2-butyne
C 5H 8 H C C CH 2 CH 2 CH 3 H 3C C C CH 2 CH 3
1-pentyne 2-pentyne
C 6H 10 H C C CH 2 CH 2 CH 2 CH 3 H 3C C C CH 2 CH 2 CH 3 H 3C CH 2 C C CH 2 CH 3
1-hexyne 2-hexyne 3-hexyne
C 7H 12 H C C CH 2 CH 2 CH 2 CH 2 CH 3 H 3C C C CH 2 CH 2 CH 2 CH 3 H 3C CH 2 C C CH 2 CH 2 CH 3
1-heptyne 2-heptyne 3-heptyne

The linear shape of alkynes is explained by sp hybridization of the C atoms in the molecular orbital model (Chapter 4.3).
This shape prevents configurational isomerism of 2-butyne and other 1,2-dialkylethynes, in contrast to alkenes (Chapter
14.2). Alkynes are structural (skeletal) isomers of dienes, exemplified by butynes and butadienes with the molecular
formula C4H6:
C4H6 H C C CH2 CH3 H3C C C CH3 H2C C CH CH3 H2C CH CH CH2
1-butyne 2-butyne 1,2-butadiene 1,3-butadiene

19.2 Preparation
19.2.1 Partial Oxidation of Methane to Ethyne
Ethyne (acetylene), carbon monoxide and hydrogen as basic chemicals are produced industrially by partial oxidation of
methane at very high temperature.
1500 °C
6 CH4 + O2 2H C C H + 10 H2 + 2 CO

19.2.2 Carbide Process


The hydrolysis of calcium carbide, calcium ethynediide as the salt of the CH acid ethyne (Chapter 19.3.5), is an older
process for the production of ethyne. Calcium carbide is produced from calcium oxide (burnt limestone) and carbon
(coke) at high temperatures.
coke, calcium oxide
2000-3000 °C
CaO + 3C CO + CaC2 (calcium carbide Ca 2 IC CI )
CaC2 + 2 H2O H C C H + Ca(OH)2

Efficiently Studying Organic Chemistry: Exam training for chemists, biochemists, pharmacists, life and health scientists,
Third Edition. Eberhard Breitmaier. © 2022 WILEY-VCH GmbH. Published 2022 by WILEY-VCH GmbH.
19.3 Typical Reactions 49

19.2.3 Double Dehydrohalogenation of 1,1- or 1,2-Dihaloalkanes


Terminal alkynes (1-alkynes) are formed by removal of two equivalents of hydrogen halide from 1,2- or 1,1-dihaloalkanes
with a strong base such as sodium amide.
NaNH2 NaNH2
H2C CH R or Br CH CH 2 R HC CH R H C C R
HBr HBr
Br Br Br Br
1,2-dibromoalkane 1,1-dibromoalkane 1-bromoalkene 1-alkyne

19.2.4 Alkylation of Terminal Alkynes


Alkynes with a central triple bond are obtained by alkylation of alkynylides (Chapter 19.3.5) with alkyl halides containing
unbranched alkyl groups R2 (which do not tend to undergo base-catalyzed dehydrohalogenation).

R 1 C CI Na + X R2 R1 C C R2 + Na X
alkynylide X = Cl, Br, I dialkylethyne

19.3 Typical Reactions


19.3.1 Hydrogenation and Reduction
- cis-Hydrogenation with Deactivated Catalysts
Partial catalytic hydrogenation of non-terminal alkynes yields (Z)-alkenes because both H atoms of the H2 molecule,
adsorbed and activated on the surface of the catalyst, add to the triple bond from one side. Deactivation of the catalyst
(Pd / Pb, CaCO3 or Pd/ BaSO4, LINDLAR catalyst) prevents total hydrogenation to alkanes.

(LINDLAR cat.)
R R
alkyne R C C R C C (Z)-alkene
+ H H
H H
catalyst surface

- trans-Reduction with Sodium in liquid Ammonia


Reduction of non-terminal alkynes with sodium as reducing reagent and liquid ammonia as proton donor yields (E)-
alkenes. In the first step, sodium reduces the alkyne to a radical anion, with the (E)-configuration favored as it suffers less
than the (Z)-isomer from electrostatic repulsion of the electrons (an electron pair and an unpaired electron). Addition of a
proton from ammonia yields a radical, which is further reduced by sodium to the anion which, finally, abstracts another
proton from ammonia to produce the (E)-alkene.

+ Na
R + NH 3
R + Na
R + NH 3
R H
R C C R C C C C C C C C
Na NH2 Na NH2
R H R H R H R
alkyne radical anion radical anion (E)-alkene

Partial catalytic hydrogenation and reduction of disubstituted ethynes are stereospecific reactions which, as dictated by
the mechanisms, produce one of two possible stereoisomers [e.g. (E)- or (Z)-].

19.3.2 Hydroboration
The addition of sodium borohydride NaBH4 to alkynes is another stereospecific addition, preferentially yielding (Z)-
trialkenylboranes which are converted into (Z)-alkenes upon hydrolysis in the presence of an acid.

NaBH4 , BF3
R R + 3 H 2O (H 3 O ) R R
3 R C C R + BH4 C C B 3 C C
 H  B(OH) 3
H 3 H H
(Z)-trialkenylborane (Z)-alkene
50 19 Alkynes

19.3.3 Electrophilic Addition of Halogens


Halogens must be activated by LEWIS acids, such as BF3, AlCl3, and FeCl3, before being able to add to alkynes. LEWIS
acids are electron-pair-accepting reagents, equilibrating with halogens to provide electrophilic halonium ions (e.g. chloro-
nium ions Cl) which add to alkynes, yielding 1,2-dihaloalkenes. In contrast, halogenation of alkenes does not require
any activation (Chapter 16.2).
 
FeCl3 + Cl2 Cl3Fe Cl Cl or [Cl3FeCl] Cl
R Cl R Cl
[Cl3FeCl] Cl + R C C H [Cl3FeCl] + C C C C + FeCl3
H Cl H
terminal alkyne chloroalkenyl cation (E)-1,2-dichloroalkene

19.3.4 Electrophilic Addition of HX


- Addition of Hydrogen Halide (X = Br, Cl)
Electrophilic addition of hydrogen halide (providing the proton as electrophile) to alkynes yields alkenyl halides and, in a
second step at higher temperature, ,-dihalides.
R R X H X
+ HX , heat
R C C R' + H X C C + X C C R C C R'
H R' H R' H X

- Addition of Hydrogen Cyanide (X = CN)


In analogy to electrophilic hydrohalogenation, electrophilic hydrocyanation of ethyne with hydrogen cyanide yields acrylo-
nitrile (cyanoethene) as a starting chemical for the production of polymers (Chapter 58).
N
Cu(I) salt
H C
H C C H + H C N C C acrylonitrile
H H

- Addition of Alcohols (X = OR)


Base-catalyzed addition of alcohols (e.g. methanol) to alkynes (e.g. ethyne) affords alkenyl ethers (e.g. methyl vinyl ether,
an enol ether).
CH3 CH3
CH3O Na , pressure
H C C H + H O H2C CH O
ethyne methanol methyl vinyl ether, an enol ether

- Addition of Water (Hydration, X = OH)


Water adds to ethyne in the presence of mercuric hydrogensulfate (HgSO4/H2SO4). The intermediately formed vinyl
alcohol equilibrates to the stable acetaldehyde (ethanal).
H OH H O H O
Hg(HSO4) 2
H C C H + H2O C C C C H3C C
H H H H H
enol enolate
vinyl alcohol (ethenol) ethanal (acetaldehyde)

Vinyl alcohol (an alkenol or enol) and acetaldehyde (an oxo compound containing the carbonyl or keto group =C=O) are
tautomers; the equilibrium between them is referred to as oxo-enol or keto-enol tautomerism. Tautomers are isomers
arising from each other by a 1,3-shift of protons (from O to the C atom at the -position).
Hydration converts an ethynyl group into an acetyl group. Thus, hydration of terminal alkynes other than ethyne in the
presence of mercuric acetate produces methyl ketones.
O
Hg salt
R C C H + H2O R C
CH3
terminal alkyne methyl ketone
19.3 Typical Reactions 51

Water and mercuric salts (hydrogensulfate, sulfate, acetate) as catalysts add to the terminal alkyne, forming an interme-
diate alkenyl cation; this is hydrated to give the hydroxyalkenyl derivative which finally regenerates mercuric hydrogensul-
fate in producing the methyl ketone.
R H R H
C C + HOH + HSO4 C C + H 2SO4
HgOSO3H HO HgOSO3H

R
R C C H + HOH + Hg(HSO4)2 C CH 3 + Hg(HSO4)2
O

19.3.5 CH Acidity of Terminal Alkynes, Alkynylides


The electron pair of an alkynylide anion occupies a compact sp hybrid orbital and is more strongly attracted by the carbon
nucleus than in an alkenyl anion where this pair would occupy a more extended sp2 hybrid orbital. Therefore, terminal
alkynes much more readily lose protons than alkenes. Alkali alkynylides are prepared by reacting a terminal alkyne with a
strong base such as sodium amide (NaNH2). Heavy metal ions such as Ag precipitate salts (alkynylides) when reacted
with terminal alkynes. These salts are explosive in the dry state; their treatment with mineral acid liberates the alkyne.

R C C H + Na NH2 R C CINa + NH3

R C C H + [Ag(NH3)2] NO3 R C CIAg + NH4 + NO3 + NH3

R C C H + [Cu(NH3)2] Cl R C CICu + NH4 + Cl + NH3


terminal alkynes alkynylides

19.3.6 Oxidative Coupling of Terminal Alkynes


Terminal alkynes undergo oxidative coupling in the presence of cupric salts and a base, producing symmetrical ,-
diynes (GLASER coupling). The base deprotonates the alkyne; the resulting alkynylide anion reduces cupric to cuprous by
one-electron transfer; the remaining alkynyl radicals dimerize to provide the diyne.
2
2 R C C H + 2 OH + 2 Cu R C C C C R + 2 H2O + 2 Cu
,-diyne

19.3.7 Cyclotri- and Cyclotetramerization


In the presence of metal catalysts, which arrange the alkyne molecules in metal complexes as templates, specific cycliza-
tions may occur. Benzene and substituted derivatives can be made by cyclotrimerization of alkynes with complexes of Ni
or Co, and cyclotetramerization of ethyne to cyclooctatetraene is catalyzed by Ni(CN)2.
R' + R'
R Ni or Co H + H H + H
R complexes R R Ni(CN) 2
+
R' R' R'
R' H + H H + H
+ R
R hexaalkylbenzene cyclooctatetraene

________________________________________________________________________________________________
Chapter 19 permits answers to the following:
(19.1) cis-trans-Isomers do not exist for alkynes. Why?
(19.2) What structurally isomeric alkynes exist for the molecular formula C8H14? Name them.
(19.3) Which dihaloalkanes are starting materials for the preparation of 1-hexyne? Formulate the equations for this.
(19.4) How can the (E)- and (Z)-isomer of 2-hexene be prepared? Formulate the equations.
(19.5) Terminal alkynes are CH acids. Why? Suggest a reasonable synthesis of 3,5-octadiyne from a terminal alkyne.
52 20 Cycloalkanes

20 Cycloalkanes
20.1 Nomenclature
Cycloalkanes are cyclic hydrocarbons without CC multiple bonds; CC double bonds are present in cycloalkenes, and CC
triple bonds in cycloalkynes. Their names are derived from those of alkanes with the same number of carbon atoms
(Table 7.1), the prefix cyclo- indicating a ring.

cyclopropane cyclobutane cyclopentane cyclohexane cycloheptane cyclooctane cyclooctene cyclooctyne


smallest ring containing
a CC triple bond

Two and more rings are connected in bi- and polycycloalkanes. Depending on the manner of connection, bicycles are
classified as fused when having one common bond, bridged when containing an alkyl bridge, and spirocyclic when having
one common carbon atom.

fused bridged spirocyclic

The nomenclature of polycycles follows from


- the total number of ring carbon atoms (e.g. nonane, heptane, decane from left to right)
- the number of rings (representative prefixes are bicyclo-, tricyclo- , and spiro-),
- the number of carbon atoms between the bridgeheads in square brackets and in decreasing order
(e.g. [4.3.0], [2.2.1], and [5.4]):

bicyclo[4.3.0]nonane bicyclo[2.2.1]heptane spiro[5.4]decane

20.2 Conformation
20.2.1 Cyclopropane
The three-membered ring of cyclopropane is planar, the carbon atoms forming a triangle with internal CCC bond
angles of 60° which are much smaller than 109°28', the tetrahedral angle of  bonds arising from overlapping of sp3
hybrid orbitals of C atoms (Table 3.1). This large angle strain along with the unfavorable eclipsed arrangement of the CH
bonds are driving forces of ring-opening reactions (Chapter 22.1). In the molecular orbital model, the CC bonds of cyclo-
propane arise from endwise overlapping of sp3 hybrid orbitals above the CC axes (bent or banana bond model), resulting
in some  character of these bonds.

H H H
overlapping
cyclopropane molecular shape sp3 hybrid orbitals
H H ("banana bonds")
H

20.2.2 Cyclobutane
Authentic cyclobutane is not planar. Its angle strain (90° instead of 109°28') and the eclipsed arrangements of CH bonds
are relieved in folded conformers which rapidly interconvert.
fast
cyclobutane, conformers

Efficiently Studying Organic Chemistry: Exam training for chemists, biochemists, pharmacists, life and health scientists,
Third Edition. Eberhard Breitmaier. © 2022 WILEY-VCH GmbH. Published 2022 by WILEY-VCH GmbH.
20.2 Conformation 53

20.2.3 Cyclopentane
A planar cyclopentane would have almost no angle strain, but all CH bonds would be eclipsed. True cyclopentane re-
lieves this by adopting five rapidly interconverting envelope conformers, each with one methylene group bent out of the
plane of the others.

interconverting envelope conformers


of cyclopentane

20.2.4 Cyclohexane
As shown by energy-minimized molecular models (Fig. 20.1), cyclohexane adopts a chair conformer.

Fig. 20.1. Tube, ball-spoke and space-filling molecular model (from left to right) of cyclohexane (chair conformer)

Tube and ball-spoke models in Fig. 20.1 show two types of differently directed (staggered) bonds in the chair conformer:
Axial bonds are aligned parallel to the molecular axis while equatorial bonds are directed to the equator of the molecule,
as drawn in Fig. 20.2.

molecular axis
chair conformer 1 1
with equatorial and
equator of the molecule axial bonds 4 4

front view NEWMAN projection


staggered bonds, no angle strain, stable

Fig. 20.2. Axial and equatorial bonds in the chair conformer of cyclohexane

An individual chair conformer of cyclohexane cannot be isolated, because rapid interconversion of two chair conformers I
and II takes place at room temperature (Fig. 20.3).

Epot
boat
kJ / mol
half-chair
21
5-6

twist-boat

21 - 25

H'
H
ring inversion
chair
H'
I II

Fig. 20.3. Ring inversion of cyclohexane

This so-called ring inversion of cyclohexane interchanges axial and equatorial bonds of hydrogen atoms or substituents in
the chair conformers I and II, as shown in Fig. 20.3. The chair conformers without any kind of strain are most stable, with
54 20 Cycloalkanes

the lowest potential energy. Ring inversion involves other less-stable, more-strained states such as the half-chair con-
former on the highest energy level, and slightly more-stable states including boat and twist-boat conformers, as illustrated
in the diagram (Fig. 20.3). The energy requirement of this process (45 kJ/mol) is covered by the kinetic energy of cyclo-
hexane molecules at room temperature.

20.3 Configurational Isomerism


20.3.1 cis- and trans-Disubstituted Cycloalkanes
Multiply substituted cycloalkanes exist as configurational isomers, differing in the relative configuration of substituents
with respect to the ring plane and specified by the descriptors cis- and trans-. This is exemplified by cis- and trans-
dimethylcyclopropane, the trans-isomer additionally forming non-superimposable isomers behaving like image and mirror
image (Chapter 44).
H 3C H CH 3 H H CH3 H3C H H
dimethylcyclopropane
H H H H
H 3C CH 3
H H H
cis- trans- trans-
non-superimposable mirror images

Projection formulas and chair conformers of 1,2-, 1,3-, and 1,4-disubstituted cyclohexanes demonstrate different presen-
tations of configurational isomers in Table 20.1.

Table 20.1. Projection formulas and chair conformers of disubstituted cyclohexanes


relative
configuration projection formula chair conformers

R 1
R 2
cis-1,2- 1
2
1
2

R R
R R
equatorial-axial (ea) axial-equatorial (ae)

1 R
R
R
trans-1,2- 1 2 R 2 1
2

R R
diequatorial (ee) diaxial (aa)

R 1
R
cis-1,3- 1 3
3
1
3
R
R R
R diequatorial (ee) diaxial (aa)

R
R 1
R
trans-1,3- 1
R 3
1
3
3
R
R equatorial-axial (ea) axial-equatorial (ae)

R 1 4
R R
cis-1,4- 1
4 1
4

R R R
equatorial-axial (ea) axial-equatorial (ae)
R
R 1
4
trans-1,4- 1 R
4 R 1
4
R R
diequatorial (ee) diaxial (aa)
20.3 Configurational Isomerism 55

20.3.2 cis- and trans-Decalin


Two cyclohexane rings are fused in bicyclo[4.4.0]decane, also referred to as decalin. Two configurational isomers with
different boiling points can be separated by distillation, differing in the fusion of the two cyclohexane rings and, as a
result, in the relative configuration of the bridgehead hydrogen atoms. These configurational isomers are called cis- and
trans-decalin with - and -fusion of the cyclohexane rings;  specifies that both bridgehead hydrogen atoms adopt the
cis-configuration while  indicates their trans-configuration. cis-Decalin and trans-decalin are reference compounds for
steroids (Chapter 77.1).
H
H
cis-decalin
1
10  H
b.p. 195.7 °C (1011 mbar) 5

H

H H
1
trans-decalin 10

b.p. 187.3 °C (1011 mbar) 5

H  H

20.3.3 Cycloalkenes
Because all bonds originating from a CC double bond are coplanar, cyclohexene is not able to adopt a chair conformation
like cyclohexane. Molecular spectra have revealed that cyclohexene exists as a half-chair conformer.

half-chair conformer of cyclohexene H


H

Cycloalkenes up to cyclohexene and cycloheptene occur exclusively as cis-isomers because a trans-configuration of the
CC double bond would distort the tetrahedral geometry of the singly bonded carbon atoms too severely. Higher cycloal-
kenes, however, beginning with cyclooctene, do indeed exist as cis- and trans-isomers:
H

cis-cyclooctene trans-cyclooctene
H H H

________________________________________________________________________________________________
Chapter 20 permits answers to the following:
(20.1) Draw skeletal formulas of (a) bicyclo[4.4.0]decane, (b) bicyclo[3.3.1]nonane, and (c) spiro[5.2]octane.
(20.2) Name the bicycles (1-5). Which kind of isomers are these cyclic hydrocarbons?

1 2 3 4 5

(20.3) Draw conformers of (a) cyclopentane, (b) cyclohexane, and (c) cyclohexene, and describe their shapes.
(20.4) Which isomers exist for (a) dimethylcyclopropane, (b) 1,3-dimethylcyclobutane, (c) 1,2-, 1,3-, and 1,4-
dimethylcyclohexane, and (d) bicyclo[4.4.0]decane? Draw skeletal formulas of the isomers and specify them.
(20.5) Cyclohexane undergoes ring inversion at room temperature. Which conformers are involved? What happens
to axial and equatorial substituents?
(20.6) Bromocyclohexane with equatorial Br and bromocyclohexane with axial Br cannot be separated by distillation.
Why?
56 21 Basic Syntheses of Cycloalkanes and Cycloalkenes

21 Basic Syntheses of Cycloalkanes and Cycloalkenes


21.1 Cyclopropane
Cyclopropane can be prepared by reductive cyclodehalogenation of 1,3-dihalides with zinc (GUSTAVSON synthesis):
+ HBr + Zn
CH 2Br
H 2C CH CH 2Cl H 2C
CH 2Cl  ZnBrCl
3-chloro-1-propene (allyl chloride) 1-bromo-3-chloropropane cyclopropane

[21]-Cycloaddition of a carbene (C1 reactant) to an alkene (C2 reactant) opens a general pathway to cyclopropanes.
Intermediate carbene is generated in situ by heating diazomethane (Chapter 40.3) as the carbene precursor. The relative
configurations of the starting alkene and the resulting cyclopropane are identical (stereospecificity). A cis-alkene (e.g. cis-
2-butene) reacts to provide a cis-disubstituted cyclopropane (e.g. cis-dimethylcyclopropane):
R R R R
C C + [ ICH 2 ]
H H carbene CH 2
cis-alkene  N2
cis-dialkylcyclopropane

diazomethane ICH 2 N NI

21.2 Cyclobutane
Reductive cyclodehalogenation of 1,4-dihalides gives moderate yields of cyclobutanes (GUSTAVSON synthesis). The [22]-
cycloaddition of alkenes (as C2 reactants), mostly induced by irradiation of the alkene with UV light (photocycloaddition),
more efficiently produces cyclobutanes. Alkenes substituted with halogens or cyano groups, such as acrylonitrile (Chap-
ter 19.3.4), readily undergo [2+2]-cycloaddition upon heating:
1,3-butadiene
acrylonitrile
H
CN trans- cis-
C CH2
CH CH2 H2C C CN CN
H2C C
heat H H heat H H
+ H F F
+ and
CN H CN
F 2C CF 2 F F H2C C CN H
tetrafluoroethene 1,1,2,2-tetrafluoro-3-vinylcyclobutane H 1,2-dicyanocyclobutane

21.3 Cyclopentene, Cyclopentane


Driven by the strain of the cyclopropane ring, cyclopentenes are formed by ring-expanding sigmatropic rearrangement
(Chapter 11.6) of the  and a  bond in ethenylcyclopropanes (vinylcyclopropanes) upon heating under pressure. Cyclo-
pentenes can be catalytically hydrogenated to give cyclopentanes. Bicyclo[3.3.0]octane is prepared in this way from 1,1-
dicyclopropylethene.
heat heat heat
pressure pressure pressure + H2 (Pd)

vinyl-
cyclopropane cyclopentene 1,1-dicyclopropylethene 1-cyclopropylpentene bicyclo[3.3.0]oct-1-ene bicyclo[3.3.0]octane

21.4 Cyclohexane, Cyclohexene


Substituted cyclohexanes are prepared by catalytic hydrogenation of appropriately substituted benzenes:
+ 3 H2 (Pd)
C(CH 3)3 C(CH 3)3

t-butylbenzene t-butylcyclohexane

Efficiently Studying Organic Chemistry: Exam training for chemists, biochemists, pharmacists, life and health scientists,
Third Edition. Eberhard Breitmaier. © 2022 WILEY-VCH GmbH. Published 2022 by WILEY-VCH GmbH.
21.5 Cycloheptadiene, Cycloheptane 57

The [42]-cycloaddition of a 1,3-diene (C4 reactant with 4  electrons) to an alkene as dienophile (C2 reactant with 2 
electrons) opens access to cyclohexene(s) which can be hydrogenated to cyclohexane(s).
X X X
+ or
Y Y Y
1,3-diene dienophile X,Y-disubstituted cyclohexene
4 electrons 2 electrons (preserving relative configuration of dienophile)

[4+2]-Cycloadditions (DIELS-ALDER reactions, Chapter 18.2.1) often proceed spontaneously under mild conditions, exem-
plified by the dimerization of 2-methyl-1,3-butadiene (isoprene) to the stereoisomers of limonene (a terpene fragrance of
mandarins and oranges, Chapter 76.3.1),

+ and

1,3-diene dienophile
isoprene (2-methyl-1,3-butadiene) limonene (two stereoisomers)

as well as the dimerization of cyclopentadiene at room temperature to dicyclopentadiene, by this way explaining the
nomenclature of a tricyclic compound. Upon heating (distillation), the tricyclic dimer regenerates cyclopentadiene by
undergoing cycloreversion to the 1,3-diene and the dienophile (retro-DIELS-ALDER reaction, the reverse of cycloaddition).
5 H
H 7
room temperature 4
6
+ 10 8
1
heat 3
H 9
1,3-diene dienophile 2 H
cyclopentadiene dicyclopentadiene (tricyclo[4.3.0.1 2,5]deca-3,7-diene)

21.5 Cycloheptadiene, Cycloheptane


1,4-Cycloheptadiene arises from COPE sigmatropic rearrangement (Chapter 11.6) of instable divinylcyclopropane, anoth-
er concerted ring-expanding rearrangement of  and  bonds, driven by the strain of the cyclopropane ring. Divinylcyclo-
propane is prepared by cyclopropanation of 1,3,5-hexatriene with diazomethane as carbene precursor. Catalytic hydro-
genation of 1,4-cycloheptadiene affords cycloheptane.

COPE rearrangement + 2 H2 (cat.)


+ [ ICH 2 ]
carbene
from diazomethane
1,3,5-hexatriene divinylcyclopropane 1,4-cycloheptadiene cycloheptane

21.6 Larger Rings


Catalytic ring-closing metathesis of terminal dienes such as 1,11-dodecadiene opens access to larger cycloalkenes such
as cyclodecene. Liberation of ethene gas shifts the equilibrium in favor of the cycloalkene.
PR3
cat. Cl
+ cat. = Ru
Cl
R'
PR3
1,11-dodecadiene cyclodecene

________________________________________________________________________________________________
Chapter 21 permits answers to the following:
(21.1) Formulate equations to suggest preparations of (a) bicyclo[4.1.0]heptane, (b) cyclopentene, and (c) cyclononene.
(21.2) Trans-1,2-dibromoethene is irradiated with UV light. What product is expected?
(21.3) A structural isomer of limonene could be a minor byproduct of the dimerization of isoprene. Which one?
(21.4) cis-1,2-Dibromoethene reacts with 1,3-cyclohexadiene. What product is expected? Write the equation.
(21.5) Write an equation to suggest a synthesis of 1,5-cyclooctadiene. What kind of reaction occurs?
58 22 Reactions of Cycloalkanes and Cycloalkenes

22 Reactions of Cycloalkanes and Cycloalkenes


22.1 Reactions Driven by the Strain of Small Rings
The ring strain of cyclopropane is the driving force for ring-opening reactions. Thus, catalytic hydrogenation of cyclopro-
pane yields propane (less expensively obtained from natural gas). With increasing ring size, much higher temperatures
are required for this reaction to occur:

H2 / Ni , 80 °C H2 / Ni , 180 °C H2 / Ni , 300 °C

H3C CH2 CH3 H3C CH2 CH2 CH3 H3C CH2 CH2 CH2 CH3

Typical electrophiles, such as bromine, hydrogen halides, and sulfuric acid, are also able to open cyclopropane rings by
1,3-additions, involving carbenium ions (Chapter 16.3).
+ Br 2 + H 2SO4
Br CH 2 CH 2 CH 2 Br H 3C CH 2 CH 2 OSO3H
1,3-dibromopropane 1-propyl hydrogensulfate

Br
+ HBr
H 3C H 3C CH CH 2 CH 3
methylcyclopropane 2-bromobutane

22.2 Alkane-like Reactions


Apart from the three- and four-membered rings, cycloalkanes and cycloalkenes react like alkanes and alkenes. Radicalic
chlorination and bromination (photohalogenations, Chapter 13) of cyclohexane yield chloro- and bromocyclohexane,
respectively:
h
+ Br2 + HBr
Br
bromocyclohexane

22.3 Alkene-like Reactions


22.3.1 Addition of Bromine
Bromine adds to all alkenes (Chapter 16.2) including cycloalkenes. Bromination of cyclohexene yields trans-1,2-
dibromocyclohexane, primarily by generation of an intermediate bicyclic bromonium ion, followed by backside attack of a
bromide anion (anti- or trans-addition, Chapter 16.2).
Br
2 1
+ Br2 + Br Br 2
1
Br Br
Br
cyclohexene trans-1,2-dibromocyclohexane

Due to this mechanism, bromination of cycloalkenes is stereospecific, converting a stereochemically pure reactant (cy-
clohexene with cis-configuration) into a stereochemically pure product (trans-1,2-dibromocyclohexane, not the cis-
isomer). This stereospecificity is not detectable for open-chain alkenes because of free rotation about the newly formed
CC single bond (Chapter 16.2).

Efficiently Studying Organic Chemistry: Exam training for chemists, biochemists, pharmacists, life and health scientists,
Third Edition. Eberhard Breitmaier. © 2022 WILEY-VCH GmbH. Published 2022 by WILEY-VCH GmbH.
22.3 Alkene-like Reactions 59

22.3.2 Catalytic Hydrogenation


Both hydrogen atoms of the H2 molecule, activated at the surface of a catalyst, approach from one side of the CC double
bond of the cycloalkene. Thus, catalytic hydrogenation of cycloalkenes is also stereospecific, as exemplified by 1,2-
dimethylcyclohexene which is hydrogenated to yield cis-1,2-dimethylcyclohexane (syn- or cis-addition, Chapter 16.1).
H
CH3 CH3
CH3 CH3
CH3 HH CH3
H
H H
cis-1,2-dimethylcyclohexane
Ni, Pd or Pt

22.3.3 Dihydroxylations
Dihydroxylations of cycloalkenes yield 1,2-cycloalkanediols. Depending on the mechanism controlled by the reagent, the
cis- or the trans-isomer is obtained.

- cis-Dihydroxylation with Osmium Tetroxide or Potassium Permanganate


[3+2]-Cycloaddition of osmium tetroxide or potassium permanganate to alkenes gives cyclic esters which are converted
into 1,2-diols with hydroxy groups in the syn- or cis-position (Chapter 16.7). With cyclohexene (and other cycloalkenes)
this stereospecific reaction affords a bicyclic ester which hydrolyzes to cis-1,2-cyclohexanediol (cis-1,2-cycloalkanediols).
O
O
Os
O OH OH
O + 2 H2O
+ OsO4 OH
 H2OsO4
OH
cyclohexene cis-1,2-cyclohexanediol cis-1,2-cyclohexanediol
osmium acid ester

- trans-Dihydroxylation with Peroxy Acids via Oxiranes


Alkenes undergo [2+1]-cycloaddition with the oxygen of peroxy acids (PRILEZHAEV epoxidation). The resulting oxiranes
open their rings upon addition of aqueous acids to yield 1,2-diols with their hydroxy groups in anti- or trans-configuration
because water attacks from the back of the protonated oxygen of the oxirane (Chapter 16.7). When subjected to this
sequence of reactions, cyclohexene, necessarily having cis-configuration of the double bond, is stereospecifically con-
verted into trans-1,2-cyclohexanediol via cyclohexene oxide.
+ H 3O
+ OH2 OH 2
O O H OH
cyclohexene oxide
+ H 2O  H3 O
+ RCO3H  RCO2H
OH

OH
OH
OH
cyclohexene trans-1,2-cyclohexanediol

________________________________________________________________________________________________
Chapter 22 permits answers to the following:
(22.1) Hydrobromination of methylcyclopropane yields 2-bromobutane as the chief product. Write the mechanism and
explain the regioselectivity. Look at Chapter 16.3.
(22.2) Bromine reacts with (a) cyclopropane, (b) cyclohexane, and (c) cyclohexene: What products are obtained?
(22.3) Write the mechanism of photochlorination of cyclohexane to give chlorocyclohexane. Look at Chapter 13.1.
(22.4) Suggest a one-step reaction to prepare pure cis-1,2-dimethylcyclopentane.
(22.5) Write equations to suggest the preparations of cis- and trans-1,2-cyclopentanediol.
60 23 Benzene, Aromaticity, Aromatic Compounds

23 Benzene, Aromaticity, Aromatic Compounds


23.1 Structure of Benzene
23.1.1 Molecular Shape
As found by X-ray diffraction and molecular spectroscopy, benzene with the molecular formula C6H6 forms a planar,
regular hexagon of six carbon atoms (Fig. 23.1). All bond angles are 120°; all CH bonds have lengths of 109 pm. Each C
atom of the ring is 139 pm from both of its nearest neighbors; this is a distance between the lengths of a CC single (154
pm) and a CC double bond (134 pm). A planar hexagon with three cyclically conjugated CC double bonds was proposed
as the structural formula by KEKULÉ in 1865 (Fig. 23.2).

H H
120°
H 120° H
109 pm
139 pm
H H

Fig. 23.1. Molecular shape of benzene: tube, ball-spoke, space-filling molecular model, and bond data

23.1.2 Heat of Hydrogenation


Considering benzene as a "1,3,5-cyclohexatriene" with three "normal" CC double bonds, hydrogenation of benzene
should liberate three-fold the heat of hydrogenation obtained for cyclohexene with one CC double bond (120 kJ/mol).
Instead of the expected value (3 x 120 = 360 kJ / mol), 209 kJ / mol has been measured. The deficit (360209 = 151 kJ /
mol) obviously reflects the particular stability of true benzene.
benzene
1,3,5-cyclohexatriene cyclohexene
+ 3 H2 (cat.)  kJ/mol
H
H H + 3 H2 (cat.) + H2 (cat.)
154 pm 134 pm
360 kJ/mol  120 kJ/mol
H H (expected)
H cyclohexane

23.1.3 Resonance Energy and Stabilization, Canonical Formulas


Taking into account its heat of hydrogenation, benzene has considerably less potential energy and is therefore much
more stable than the hypothetical 1,3,5-cyclohexatriene. The large difference (151 kJ/mol) between the expected and
the measured heat of hydrogenation is attributed to the stabilization or resonance energy Hres of benzene (Fig. 23.2).

Epot KEKULÉ formulas DEWAR formulas

Hres =  151 kJ / mol


6 benzene

Fig. 23.2. Resonance formulas of benzene and their potential energies in relation to true benzene; double-headed arrows between
formulas symbolize resonance

Efficiently Studying Organic Chemistry: Exam training for chemists, biochemists, pharmacists, life and health scientists,
Third Edition. Eberhard Breitmaier. © 2022 WILEY-VCH GmbH. Published 2022 by WILEY-VCH GmbH.
23.2 Molecular Orbital Model of Benzene 61

Benzene has one structure which, however, cannot be drawn by one structural formula with localized double bonds. More
adequately, benzene is represented as a hybrid of resonance formulas, each with localized  and  bonds. These include
two KEKULÉ and three DEWAR formulas with cyclically oscillating  and  bonds (Fig. 23.2). By themselves, none of these
canonical formulas correctly represents benzene; all together as a whole they do and describe resonance stabilization
and delocalized  bonding which characterizes benzene, graphically pictured by the molecular orbital model.

23.2 Molecular Orbital Model of Benzene


The CC  bonds of benzene arise from endwise overlapping of sp2 hybrid orbitals originating from the ring C atoms in the
molecular orbital model, matching to the bond angles of 120° (Fig. 23.1). Six coaxial 2p orbitals with axes perpendicular
to the ring plane remain for cyclic sidewise overlapping (Fig. 23.3 a). Therefore, the resulting  bonds are not localized
but cyclically delocalized (Fig. 23.3 b), providing space for the benzenoid  electron sextet. Linear combination of the
wave functions of all six coaxial 2p orbitals leads to six molecular orbitals. Three of those are the bonding  orbitals, the
most stable one on the lowest energy level cyclically extending above and below the ring plane, as pictured in Fig. 23.3 c.


H H H H
C C C  C
H C C H H C C H
C C C C
H H H H

a b c
Fig. 23.3. Molecular orbital model of benzene: (a) cyclically overlapping 2p orbitals; (b) areas of  and  bonds in benzene; (c) most
stable (of six)  molecular orbitals of benzene, depicting the distribution of  electrons in the benzene ring

Usually, one of the KEKULÉ formulas is drawn to represent the benzene molecule; more appropriately, to account for the
molecular orbital model, benzene is drawn as a regular hexagon with an inscribed circle that symbolizes the delocalized 
orbitals and the benzenoid  electron sextet (Fig. 23.2).

23.3 Criteria of Aromaticity


Four essential properties characterize benzene:
-It is a planar molecule (coplanarity).
-Its  bonds are cyclically delocalized (cyclically conjugated CC double bonds, two KEKULÉ formulas).
-It has a high resonance energy (151 kJ/mol).
-The number of  electrons (six) follows (4N + 2) for N = 0, 1, 2, 3,... (N = 1 for benzene). This HÜCKEL rule applies
for (non-fused) monocyclic compounds.
The total of these characteristics is described as aromaticity and all cyclically conjugated compounds which share these
four properties with benzene are classified as aromatic compounds or aromatics; aromatic hydrocarbons are also called
arenes. Four classes exist; these are benzenoid (Chapter 24), polycyclic benzenoid (Chapter 28), non-benzenoid (Chap-
ter 29), and heterocyclic (Chapter 61-65) aromatic compounds.
________________________________________________________________________________________________
Chapter 23 permits answers to the following:
(23.1) Which facts indicate that benzene is more than just a 1,3,5-cyclohexatriene?
(23.2) Explain the term resonance energy and its physical (thermodynamic) origin.
(23.3) Draw all formulas describing the benzene molecule and explain the term delocalized  bonding.
(23.4) Which properties characterize an aromatic compound?
(23.5) Draw skeletal formulas of (a) cyclobutadiene C4H4 and (b) cyclooctatetraene C8H8. Are these compounds
aromatic?
62 24 Benzenoid Aromatic Compounds

24 Benzenoid Aromatic Compounds


24.1 Monosubstituted Benzenes
Benzenoid aromatic compounds include all substituted benzenes. Monosubstituted benzenes are named by the substitu-
ent’s name as prefix followed by the suffix benzene. Important benzene derivatives are frequently called by their common
names derived from their natural origin. Examples are toluene, first found in Balsamum tolutanum, aniline, originally
prepared from the natural blue dye indigo (Chapter 63.3.3), from the Arabian word anil = blue, and benzoic acid occurring
in gum benzoin.
O H O OH
CH 3 C 2H 5 CH(CH 3)2 C(CH 3)3 X NH 2 OH CH 2OH C C

X=
toluene ethylbenzene cumene t-butylbenzene F : fluorobenzene aniline phenol benzyl alcohol benzaldehyde benzoic acid
(methylbenzene) (isopropyl- Cl : chlorobenzene (amino- (hydroxy- (-hydroxy- (benzene-
benzene) Br : bromobenzene benzene) benzene) toluene) carboxylic acid)
I : iodobenzene
NO2 : nitrobenzene

As a substituent, the benzene ring is called phenyl, belonging to the aryl residues, symbolized as Ph or Ar in formulas.
Alkyl-, alkenyl-, and alkynylbenzenes with longer carbon chains or more than one phenyl group may be named phenylal-
kanes, phenylalkenes, phenylalkynes, diphenylalkanes, diphenylalkenes, and diphenylalkynes. Common names such as
styrene for phenylethene, stilbene for 1,2-diphenylethene, and tolane for diphenylethyne are often used. Biphenyl and
terphenyl are classified as polyphenyls.

H
H
H H
H

3-phenylpropene phenylethene (E)-diphenyl- (Z)-diphenyl- phenylethyne diphenylethyne diphenyl- 1,2-diphenyl- biphenyl terphenyl
allylbenzene vinylbenzene ethene ethene ethynylbenzene tolane methane ethane
styrene trans-stilbene cis-stilbene

24.2 Multiply Substituted Benzenes


Benzene derivatives with two identical or different substituents which occupy different positions of the benzenoid ring are
skeletal isomers, also called regioisomers; they are distinguished by using ortho- (o-) for 1,2-, meta- (m-) for 1,3-, and
para- (p-) for 1,4-disubstitution. The highest ranking substituent defines the parent compound; the element number in the
periodic table (e.g. Br > Cl, O > Cl > F) and the oxidation state (CO2H > CH2OH) determine the ranking. Whenever availa-
ble, common names take preference.

CH3 CH3 CH 3 OH OH OH Br Br Br
CH 3 CH 3 NO2

CH 3 CH 3 NO2
CH 3 CH 3 NO2
1,2- or o- 1,3- or m- 1,4- or p- 1,2- or o- 1,3- or m- 1,4- or p- 1,2- or o- 1,3- or m- 1,4- or p-
xylene (dimethylbenzene) cresol (methylphenol) bromonitrobenzene

In multiply substituted benzenes too, the highest ranking substituent determines the name of the parent compound and
occupies the 1-position. The smallest position numbers are given to the other substituents. When available, a common

Efficiently Studying Organic Chemistry: Exam training for chemists, biochemists, pharmacists, life and health scientists,
Third Edition. Eberhard Breitmaier. © 2022 WILEY-VCH GmbH. Published 2022 by WILEY-VCH GmbH.
24.3 Preparation of Benzenoid Hydrocarbons 63

name of the disubstituted benzene is the preferred parent compound to derive the name of a multiply substituted ben-
zene, exemplified by salicyclic acid (2-hydroxybenzoic acid from Latin salix = willow).

F NH2 CH 3 CO2H CO2H CO2H


NO2 CN O2N NO2 CH2Br OH OH

F F O2N NO2
NO2 C 2H5 NO2 2-bromomethyl- 3,5-dinitro- salicylic acid
2,4-dinitrofluorobenzene 2-cyano-4-ethylaniline 2,4,6-trinitrotoluene 3,5-difluorobenzoic acid salicylic acid (2-hydroxy-
parent compound fluorobenzene aniline toluene benzoic acid salicylic acid benzoic acid)

24.3 Preparation of Benzenoid Hydrocarbons


24.3.1 Fossil Sources
Benzene, toluene, xylene isomers and other alkylated benzenes, as well as polycyclic aromatic compounds (Chapter 28),
are obtained by distillation or extraction of coal tar which is produced by coking of coal at 1000-1300 °C.
Crude petroleum which is predominantly distilled to fractions used for the production of energy (Chapter 5.2) also con-
tains a variety of aromatics depending on the origin of the oil. These amounts, however, do not satisfy the requirements
of chemical industry. Therefore, large quantities of aromatic compounds are prepared industrially by catalytic dehydro-
genation and cyclization (cyclodehydrogenation) of petroleum fractions containing alkanes. Benzene is produced by
passing n-hexane at high temperature and under pressure over catalysts containing chromic oxide and alumina; corre-
spondingly, catalytic cyclodehydrogenation of n-heptane yields toluene. Hydrogen is the attractive byproduct of these
platforming processes.
CH3 CH3
Cr 2 O3 / Al2 O3 Pt
CH3 500 °C, pressure CH3 500 °C, pressure
CH3  4 H2  4 H2
n-hexane benzene n-heptane toluene

24.3.2 Cyclotrimerization of Alkynes


Polyalkylated benzenes are prepared by catalytic cyclotrimerization of appropriately substituted alkynes (Chapter 19.3.7).

R R
R Ni(CO) 4 R R
R pressure, heat
R
R R
R
R R
R = CH3 : 2-butyne R = CH3 : hexamethylbenzene

________________________________________________________________________________________________
Chapter 24 permits answers to the following:
(24.1) Draw the structural formulas of (a) 2,4,6-trinitrotoluene, (b) 4,4´-dihydroxydiphenylmethane, and (c) 4-nitro-o-cresol.
(24.2) What are other names for the industrial chemical known as bisphenol A and the contact insecticide known as DDT?
CH3 CCl3
4 1 1' 4' 4 1 1' 4'
bisphenol A HO C OH DDT Cl C Cl
CH3 H

(24.3) Which structurally isomeric aromatic compounds with an OH group and the molecular formula C 7H8O exist?
(24.4) What kind of chemicals are produced by the so-called platforming processes?
(24.5) Suggest a synthesis of hexaethylbenzene from an alkyne. Formulate the equation.
64 25 Electrophilic Substitution of Benzene

25 Electrophilic Substitution of Benzene


25.1 Substituted Benzenes by Electrophilic Substitution: Mechanism
Due to its  electron sextet the benzene molecule is a nucleophile and reacts with electrophiles. Upon collision with
benzene, the electrophile E is loosely held by the nucleophilic  electron cloud of benzene in a  complex in which the
electrophile attaches to a particular ring C atom, thereby changing the latter from sp2 to sp3 hybridization. The resulting
phenonium ion, also called a  complex, represents a hybrid of three resonance formulas which distribute the positive
charge of the attached electrophile to the 2-, 4-, and 6-positions. Loss of the  proton from the phenonium ion regener-
ates the benzenoid  electron sextet (rearomatization), involving a  complex loosely holding the proton and finally pro-
ducing the benzene substituted by E. The intermediate phenonium ions can be isolated as salts in some cases, providing
evidence for this mechanism of electrophilic substitution (SE).

E E E
E E
H H H
+E E H + [H ]

 complex  complex: a resonance-stabilized phenonium ion  complex

25.2 Electrophilic Halogenation


LEWIS acids (AlCl3, FeBr3) polarize halogen molecules to provide electrophilic halonium ions (E = Cl, Br):
 
X X + AlCl3 X X : AlCl3 [X ] + [XAlCl3]
 
Br Br + ZnCl2 Br Br : ZnCl2 [Br ] + [BrZnCl2]

Probably, the halogen molecule is not polarized before forming the  complex, which subsequently is converted into the
chlorophenonium ion ( complex), before finally producing the aryl halide chlorobenzene.
Cl Cl AlCl3 Cl Cl
fast slow H
+ Cl2 + AlCl3 [ClAlCl3]
HCl
H AlCl 3
 complex chlorophenonium ion chlorobenzene
three resonance formulas

25.3 Electrophilic Alkylation (FRIEDEL-CRAFTS Alkylation)


Carbenium ions as electrophiles for the preparation of alkylbenzenes by FRIEDEL-CRAFTS alkylation are generated in situ
by dissociation of alkyl halides in the presence of a LEWIS acid (AlCl3, AlBr3),
 
H 3C CH 2 Br AlBr3 [H 3C CH 2] [AlBr4] [H 3C CH 2] + [AlBr4]
polar complex ion pair dissociated ions

as well as by protonation of alkenes with hydrogen halide (Chapter 16.3) or sulfuric acid:
H H
C + H2SO4 C + HSO4
H3C CH2 H3C CH2 H
propene propylium cation (a carbenium ion)

Ethylbenzene is prepared by electrophilic alkylation of benzene with bromoethane in the presence of aluminum bromide,
involving an ethylium ion as electrophile,
H 3 C CH 2 Br , AlBr 3
H + [H 3C CH 2] CH 2 CH 3
 [H ]
ethylium cation ethylbenzene

Efficiently Studying Organic Chemistry: Exam training for chemists, biochemists, pharmacists, life and health scientists,
Third Edition. Eberhard Breitmaier. © 2022 WILEY-VCH GmbH. Published 2022 by WILEY-VCH GmbH.
25.4 Electrophilic Acylation (Friedel-Crafts Acylation) 65

and isopropylbenzene (cumene) arises from electrophilic alkylation of benzene with propene in the presence of sulfuric
acid which generates the isopropylium ion (2-propylium ion) as the intermediate electrophile.
H
H 2C CH CH3 , H2SO4
H + C CH(CH 3)2
H3C CH3  [H ]
2-propylium cation isopropylbenzene (cumene)

25.4 Electrophilic Acylation (FRIEDEL-CRAFTS Acylation)


Acyl cations as electrophiles for the acylation of aromatics are formed in situ from carboxylic acid anhydrides (Chapter
42.3) and sulfuric acid or from carboxylic acid halides (Chapter 42.2) and aluminum chloride:
O O O
O
C C C
H3C O CH3 + 2 H2SO4 H3C C OI + H3C C + 2 HSO4 H3C Cl + AlCl3 H3C C OI + [ AlCl4 ]
acetic anhydride acetyl cation OH2 acetyl chloride
(a carboxylic acid anhydride) (an acylium ion) (a carboxylic acid halide)

Acetylbenzene (= acetophenone = methyl phenyl ketone, Chapter 48.1) is prepared by electrophilic acylation (FRIEDEL-
CRAFTS acylation) of benzene with acetic acid anhydride in the presence of sulfuric acid or with acetyl chloride in the
presence of aluminum chloride as catalyst.
_
OI O
H + H 3C C OI C C
CH3  [H ] CH3
acetyl cation as electrophile acetylbenzene (acetophenone)
from acetic anhydride or acetyl chloride

25.5 Electrophilic Nitration


A mixture of concentrated nitric and sulfuric acid (HNO3 + H2SO4) is used to nitrate benzene. This mixture generates the
intermediate nitronium ion as the active electrophile of nitration.

H 2SO4 + HONO2 H 2O NO2 + HSO4 [ O=N=O ] + HSO4 + H 2O


nitronium ion

Electrophilic nitration of benzene involving the nitronium ion as electrophile produces nitrobenzene according to the
mechanism discussed in Chapter 25.1 (replacing E by NO2).
HNO3 / H2SO4
H + [ O=N=O ] NO2
nitronium ion  [H ]
nitrobenzene

25.6 Electrophilic Sulfonation


Sulfur trioxide (SO3) occurring in fuming sulfuric acid (oleum) is an electrophilic molecule due to its positive charge at the
sulfur atom, as depicted by three resonance formulas.
O O O O
O S O S O S O S
O O O O

Therefore, benzene can be sulfonated to benzenesulfonic acid by treatment with fuming sulfuric acid. Electrophilic sul-
fonation is reversible. The intermediate phenonium ion is depicted as a hybrid of three zwitterionic resonance formulas,
distributing the positive charge among the 2-, 4-, and 6-positions of the benzenoid ring.
O
O O O
fast S SO3 SO3 slow fast
+ SO3 S O + [H ] S OH
HO H H reversible
O O
benzenesulfonate benzenesulfonic acid
66 26 Electrophilic Second Substitution of Benzenes

26 Electrophilic Second Substitution of Benzenes


26.1 Resonance Effects of Substituents at the Benzene Ring
Substituents with non-bonding electron pairs such as an amino group (INH2) are electron-releasing and called donors
(ID). In contrast, the nitro group (NO2) with two electron-attracting oxygen atoms attached to nitrogen is an electron-
withdrawing substituent; such substituents are called acceptors A. Interactions of donor and acceptor substituents with
the  bonds of aromatics, alkenes, and alkynes are called resonance or mesomeric effects (M effects). Donor substitu-
ents (ID) release electron pairs to attached  bonds [()-M effect] while acceptor substituents (A), in contrast, withdraw
electron pairs from attached  bonds [()-M effect].
Donors D also referred to as ()-M substituents such as halogens (Cl, Br, I), hydroxy, and amino groups, all with non-
bonding electron pairs (ID), release one pair into the benzenoid ring, thus increasing the  electron density at the o-, o´-
and p-positions, as shown by the dipolar (zwitterionic) resonance formulas of aniline.
INH2 INH2 NH2 NH2 NH2 INH2

aniline

KEKULÉ formulas zwitterionic resonance formulas

Acceptors A also called ()-M substituents such as nitro- (NO2), aldehyde (CH=O), and cyano (CN) groups, all with
polar double bonds to heteroatoms, withdraw one  electron pair from the benzenoid ring, thus decreasing the  electron
density at the o-, o´- and p-positions, as shown by the dipolar (zwitterionic) resonance formulas of nitrobenzene.
O O O O O O O O O O
N N N N N NO2

nitrobenzene

KEKULÉ formulas zwitterionic resonance formulas

Resonance or mesomeric effects of substituents polarize the  bonds of benzene, controlling regioselectivity of electro-
philic substitution in determining where an electrophile most likely attacks a substituted benzene.

26.2 Regioselectivity of Electrophilic Second Substitution of Benzenes


As shown by the resonance formulas of aniline, a donor like the amino group increases electron density at the o-, o´-, and
p-positions, activating them for electrophilic substitution. Thus, a donor substituent (Cl, Br, I, OH, NH 2) at the benzene
ring directs the electrophile to the (nucleophilic) o-, o´-, and p-positions, but not to the m-position, making the reaction
regioselective. Electrophilic nitration of phenol with nitric acid and concentrated sulfuric acid at 10 °C yields the regioi-
somers o- and p-nitrophenol, but only traces of the m-isomer.
OH HNO3 / H 2SO4 OH OH
 10 °C
+ [ O=N=O ] and
nitronium ion  H
NO2 O2N
o-nitrophenol p-nitrophenol

The donor additionally contributes to the distribution of the positive charge introduced by the electrophile in the intermedi-
ate  complex. As shown by resonance formulas for the nitration at the o- and p-positions of phenol, the donor (OH) also
accepts the positive charge which is distributed among four positions as a result. This delocalization of charge additional-
ly stabilizes the  complex and decreases the activation barrier of its formation.
OH OH OH OH OH OH
NO2 NO2 NO2 NO2
NO2
+ NO2 H H H H
 H
phenol donor- and resonance-stabilized  complex arising as intermediate from the nitration of phenol at the o-position o-nitrophenol

Efficiently Studying Organic Chemistry: Exam training for chemists, biochemists, pharmacists, life and health scientists,
Third Edition. Eberhard Breitmaier. © 2022 WILEY-VCH GmbH. Published 2022 by WILEY-VCH GmbH.
26.2 Regioselectivity of Electrophilic Second Substitution of Benzenes 67

Attachment of the nitronium ion para to the hydroxy group also yields an intermediate  complex in which the donor
additionally accepts and thereby delocalizes the positive charge:
OH OH OH OH OH

 H

H NO2 H NO2 H NO2 H NO2 NO2


donor- and resonance-stabilized  complex arising as intermediate from the nitration of phenol at the p position p-nitrophenol

Entering at the m-position, however, the nitronium ion would generate a  complex in which the hydroxy group would not
be able to additionally stabilize the phenonium ion by accepting the positive charge:
OH OH OH

H H H
NO2 NO2 NO2
 complex arising as intermediate from the nitration of phenol at the m-position

Conversely, the resonance formulas of nitrobenzene (Chapter 26.1) demonstrate that the nitro group as an acceptor A
with ()-M effect decreases electron density at the o-, o'-, and p-positions, deactivating these positions for electrophilic
substitution. Thus, an acceptor substituent directs electrophilic second substitution to the non-deactivated m-position.
Nitration of nitrobenzene with fuming nitric acid regioselectively produces m-dinitrobenzene:
NO2 NO2

HNO3 (fuming)
+ [ O=N=O ]
nitronium ion  H
NO2
nitrobenzene m-dinitrobenzene

Typical donor substituents such as dimethylamino, N(CH3)2, and hydroxy, OH, increase the reactivity while typical
acceptor substituents such as carboxy, CO2H, and nitro, NO2, decrease the reactivity of the substituted benzene
relative to benzene as reference.
O O
substituent N(CH3)2 OH H C N
OH O
relative reactivity 107 103 1 103 108

________________________________________________________________________________________________
Chapters 25 and 26 permit answers to the following:
(25.1) Formulate a detailed reaction mechanism for the electrophilic substitution of benzene by an electrophile E.
(25.2) Write this mechanism for (a) a bromination to give bromobenzene and (b) an alkylation to give isopropylbenzene.
(25.3) Starting with benzene, how would you prepare (a) t-butylbenzene, (b) nitrobenzene, and (c) acetophenone?
(25.4) Sulfur trioxide is electrophilic at the sulfur atom. Why? Write the mechanism of the electrophilic sulfonation of benzene.
________________________________________________________________________________________________
(26.1) Explain the ()-M and ()-M effect. In which manner do they influence the reactivity of substituted benzenes?
(26.2) Phenol reacts with (a) nitric acid and sulfuric acid and (b) fuming nitric acid. Which products are formed?
(26.3) Salicyclic acid (o-hydroxybenzoic acid) is nitrated. Which products are expected?
(26.4) Which alkene is an appropriate precursor of an intermediate electrophile to prepare 2,4,6-tri-t-butylphenol?
(26.5) Which products arise from the nitration of (a) bromobenzene and (b) nitrobenzene?
(26.6) Which reactions are suitable for the preparation of 3-nitrobenzenesulfonic acid from benzene?
(26.7) Alkyl groups at benzene direct electrophiles to the o- and p-positions. How would you prepare 2,4,6-trinitrotoluene?
68 27 Other Reactions of Benzenoid Aromatics

27 Other Reactions of Benzenoid Aromatics


27.1 Nucleophilic Substitution at the Benzenoid Ring
The carbon-halogen bond in halobenzenes is shorter and stronger than in alkyl halides. Therefore, the halide anion
cannot be so easily removed from the benzenoid ring, and nucleophilic substitution of aryl halides does not take place
under mild conditions. Conversion of chlorobenzene into phenoxide with sodium hydroxide (OH as nucleophile) requires
high temperature, pressure, and a catalyst (RASCHIG process).
360 °C, 300 bar, Cu _
Cl + 2 NaOH Ol
_ Na + NaCl + H 2O

sodium phenoxide

However, as shown by resonance formulas of o-chloronitrobenzene, an acceptor substituent such as a nitro group ortho
(or para) to a halogen in the benzenoid ring accepts the negative charge of the nucleophile (OH) in the  complex,
stabilizing this complex and lowering the activation barrier for its formation. As a result, nucleophilic substitution of halide
by hydroxide proceeds under milder conditions.

O O O O O O O O
NO2 N N N N NO2
Cl Cl Cl Cl
Cl + OH OH + Cl
slow OH OH OH OH fast

o-chloronitrobenzene resonance-stabilized carbanion ( complex) o-nitrophenol

Thus, nucleophilic substitution of chloride in o-chloronitrobenzene by hydroxide yielding o-nitrophenol occurs upon gentle
heating, and 1-chloro-2,4-dinitrobenzene is converted into 2,4-dinitrophenol with sodium hydroxide solution at 30 °C.
30 °C
O2N Cl + OH O2N OH + Cl

NO2 NO2
1-chloro-2,4-dinitrobenzene 2,4-dinitrophenol

27.2 Radical Substitution at the Side Chain


Alkylbenzenes are halogenated at the side chain by halogens (Cl2, Br2) upon heating and irradiation with UV light, ac-
cording to the mechanism of radical substitution (Chapter 13.1). Bromination of ethylbenzene regioselectively yields -
bromoethylbenzene and only traces of the  isomer.
+ Br 2 , h
CH 2 CH 3 CH CH 3 CH2 CH 2 Br
HBr
Br
ethylbenzene -bromoethylbenzene -bromoethylbenzene
major product traces

The regioselectivity is explained by resonance stabilization of the phenylethyl radical having the unpaired electron at the
 carbon atom called the benzylic position. In this situation, the electron is distributed among the 2-, 4-, and 6-positions of
the benzene ring, as pictured by the molecular orbital model and resonance formulas. This delocalization and stabilization
is impossible when the unpaired electron occupies the -position.

H CH 3 H CH 3 H CH3 H CH 3 H CH 3

H

CH 3

sidewise overlapping of p orbitals resonance formulas of the phenyl ethyl radical with the unpaired electron
in the phenyl ethyl radical in the benzylic  position

Efficiently Studying Organic Chemistry: Exam training for chemists, biochemists, pharmacists, life and health scientists,
Third Edition. Eberhard Breitmaier. © 2022 WILEY-VCH GmbH. Published 2022 by WILEY-VCH GmbH.
27.3 Hydrogenation, Reduction, Oxidation 69

At low temperatures and in the presence of a LEWIS acid as catalyst, electrophilic halogenation at the benzene ring takes
place. A bulky alkyl group favorably directs the electrophilic halonium ion (Chapter 26.2) to the p-position. Ethylbenzene
undergoes electrophilic bromination to provide p-bromoethylbenzene.
Br
+ Br 2 , FeCl3
CH2 CH 3 Br CH2 CH3 CH2 CH3
HBr
ethybenzene p-bromoethybenzene o-bromoethybenzene
major product traces

27.3 Hydrogenation, Reduction, Oxidation


Catalytic hydrogenation of alkylbenzenes yields the correspondingly substituted cyclohexanes. Xylenes (o-, m-, p-), for
example, are hydrogenated to cis- and trans-dimethylcyclohexanes.
Pt , Pd or Ni CH3
H3C CH3 + 3 H2 CH3 and
H3C
CH3
p-xylene trans- (major) cis- (minor)
1,4-dimethylcyclohexane

The BIRCH reduction of benzene with sodium in liquid ammonia, usually in the presence of an alcohol ROH as proton
donor, opens access to 1,4-cyclohexadiene. As a reducing reagent, the alkali metal abstracts electrons which are solvat-
ed in liquid ammonia. One of these electrons reduces the benzenoid ring to an intermediate radical anion which is proto-
nated by alcohol to give a radical. Another electron reduces this radical to a carbanion which finally undergoes protona-
tion to provide 1,4-cyclohexadiene.
H H H H H H
reduction NH 3 / ROH reduction NH 3 / ROH
+ Na + [H ] + Na + [H ]

Na Na

H H H H H H H H
benzene radical anion radical carbanion 1,4-cyclohexadiene

Oxidation of the side chain of alkylbenzenes with a hot solution of potassium permanganate or sodium dichromate
(providing CrO3 as the oxidation reagent) produces benzoic acid, as demonstrated for toluene. The benzenoid ring resists
these oxidation conditions.

CH 3 + 2 CrO3 CO2H + Cr2O3 + H 2O

toluene benzoic acid

________________________________________________________________________________________________
Chapter 27 permits answers to the following:
(27.1) In aryl halides, carbonhalogen bonds are shorter than in alkyl halides. Look at Chapters 3.2, 26.1, and 27.1
and explain why.
(27.2) 1-Fluoro-2,4-dinitrobenzene (2,4-dinitrofluorobenzene) reacts with (a) water and (b) ammonia. What products are
formed? Draw resonance formulas to explain the influence of the nitro groups on the reactive intermediate.
(27.3) Propylbenzene reacts with bromine in the presence of UV radiation (a) or ferric chloride in the cold (b). Outline
the mechanisms and explain the regioselectivity of both reactions.
(27.4) Formulate equations to suggest the preparation of t-butylcyclohexane from benzene and methylpropene.
(27.5) Write equations to describe the reactions of (a) benzene with sodium in liquid ammonia and a proton donor, and
(b) of toluene with chromium trioxide (CrO3).
70 28 Polycyclic Benzenoid Aromatic Compounds

28 Polycyclic Benzenoid Aromatic Compounds


28.1 Fusion of Benzenoid Rings
Two benzene rings are fused in naphthalene which respresents the simplest polycyclic benzenoid aromatic compound.
Three benzene rings are fused linearly to anthracene and angularly in the isomeric phenanthrene. Four benzene rings
are fused linearly in tetracene (naphthacene) and angularly in the isomers benzo[a]anthracene and chrysene (benzo[a]-
phenanthrene). Pyrene is a benzo[d,e,f]-fused phenanthrene.

naphthalene
a a
d a b
a f e
b a d
d c b c
anthracene phenanthrene tetracene (naphthacene) benzo[a]anthracene chrysene pyrene benzo[a]pyrene

Naphthalene forms a planar molecule like benzene. Its  bonds arise from endwise overlapping of sp2 hybrid orbitals
originating from the ten ring carbon atoms. Sidewise overlapping of the remaining coaxial 2p orbitals perpendicular to the
molecular plane generates  molecular orbitals which describe a cloud of ten  electrons delocalized above and below
this plane (Fig. 28.1 d). The structure of naphthalene can be described as a hybrid of three resonance formulas a, b, and
c (Fig. 28.1).


a b c d

Fig. 28.1. Resonance formulas (a - c) and overlapping of coaxial p orbitals to give the  molecular orbitals of naphthalene (d)

Each resonance formula of naphthalene (Fig. 28.1, a, b, c) possesses only one autonomous  electron sextet. Therefore,
a formula with two inscribed circles is not correct.

or not correct:

The relative stability of higher fused aromatic compounds increases with the number of autonomous  electron sextets
(CLAR’s rule). To conclude, anthracene with one autonomous  electron sextet is less stable, as reflected in a lower
resonance energy, than phenanthrene with two autonomous benzenoid rings.

anthracene phenanthrene
linearly fused, one autonomous  electron sextet angularly fused, two autonomous  electron sextets
resonance energy 360 kJ/mol resonance energy 416 kJ/mol

28.2 Preparation of Aromatic Polycycles


Polycyclic aromatic compounds such as naphthalene, anthracene, phenanthrene and some of their alkyl derivatives are
obtained industrially by working up petroleum coke (Table 5.2) or by distillation or extraction of coal tar which is produced
by coking of coal at 1000-1300 °C. Tars including those of tobacco smoke contain several carcinogenic polycyclic aro-
matic hydrocarbons such as benzo[a]pyrene (Chapter 28.1).

Efficiently Studying Organic Chemistry: Exam training for chemists, biochemists, pharmacists, life and health scientists,
Third Edition. Eberhard Breitmaier. © 2022 WILEY-VCH GmbH. Published 2022 by WILEY-VCH GmbH.
28.3 Electrophilic Substitution of Naphthalene 71

Aromatic polycycles are prepared by attaching benzenoid rings to other arenes. Such sequences of reactions, frequently
involving electrophilic acylations, are referred to as benzoannelations (from Lat. anulus = ring). Methylated arenes such as
m-xylene, for example, are acylated with benzoyl chloride (the acid chloride of benzoic acid, Chapter 42.2) in the presence
of aluminum chloride to give a mixture of ketones (dibenzoyl-m-xylene by FRIEDEL-CRAFTS acylation). In the presence of a
catalyst and at high temperature (pyrolysis), these ketones undergo cyclodehydration and dehydrogenation to yield
polycyclic arenes such as pentacene (ELBS reaction).

H3C CH3 H 3C CH3 Cu


AlCl3 400 °C 380 °C
2
Cl +  2 HCl  H 2 O  H2
C
O O O
benzoyl chloride m-xylene dibenzoyl-m-xylene 6,13-dihydropentacene pentacene

28.3 Electrophilic Substitution of Naphthalene


Because one autonomous  electron sextet is shared among two rings (Chapter 28.1), naphthalene is less stable than
benzene. Therefore, it more readily undergoes electrophilic substitutions such as nitration, bromination, sulfonation and
acylation (Fig. 28.2), producing attractive intermediates for some industrial syntheses.

Br Br 2 / CCl 4 SO3H
reflux SO3H
no cat. conc. H2SO4
and

-bromonaphthalene - -
naphthalenesulfonic acid
O CH 3 O
NO2 C
HNO3 / H 2SO4 naphthalene CH 3COCl / C
50 - 60 °C AlCl3 CH3
and

1- acetylnaphthalene 2-
-nitronaphthalene

Fig. 28.2. Selected electrophilic monosubstitutions of naphthalene

Resonance formulas of the intermediate naphthonium ions explain why the substitution by an electrophile E at the 1-
position is energetically favored: Two of three resonance formulas of the naphthonium ion arising from 1-substitution do
not disrupt the benzenoid  electron sextet, while two of three resonance formulas do so in the naphthonium ion resulting
from 2-substitution. Thus, formation of the 1- or -naphthonium ion requires a lower activation barrier. Under mild condi-
tions (kinetic control), 1- or -substitution takes preference, while higher temperatures favor substitution at the 2- or -
position (thermodynamic control).

H
E E E E
H H H
2-substitution
H H
 electron sextet of both rings disrupted
less stable naphthonium ion
E H E H E H
H H
1-substitution

H
undisrupted  electron sextet of one ring
more stable naphthonium ion

The regioselectivity of electrophilic second substitution can be predicted by following the rules outlined for substituted
benzenes (Chapter 26.2), as demonstrated for naphthalenes substituted by donor and acceptor groups D and A, the
arrows indicating the positions attracting electrophiles. Donors D direct the electrophile to the ortho (or para) position of
72 28 Polycyclic Benzenoid Aromatic Compounds

the same ring, while electron-withdrawing acceptor substituents A guide the electrophile to the other ring. Thus, nitration
of 1-nitronaphthalene produces a mixture of 1,5- and 1,8-dinitronaphthalene.
D A
D (A)
8 1
2
5 4

28.4 Oxidation and Reduction of Naphthalene


Catalytic oxidation of naphthalene with air at high temperature yields phthalic anhydride, needed for the production of
anthraquinone dyes (Chapter 53.2.3). Stronger reagents (CrO3) oxidize naphthalene to 1,4-naphthoquinones (Chapter
53.2.2).
O
V2 O5, 470 °C
+ 9/2 O2 O + 2 CO2 + 2 H 2O

O
phthalic anhydride

Sodium as reducing reagent in an alcohol as proton donor reduces naphthalene to tetralin (from Ger. Tetrahydronaphtha-
lin), thus preserving one of the two benzenoid rings. BIRCH reduction (Chapter 27.3) with sodium in liquid ammonia pro-
duces isotetralin, disrupting both benzenoid rings. Catalytic hydrogenation of naphthalene and tetralin yields a mixture of
cis- and trans-decalin (from Ger. Decahydronaphthalin, Chapter 20.3.2).

Na, liquid NH3 Na, ethanol H 2 / Pt or Ni


 78 °C  78 °C heat, pressure and

isotetralin naphthalene tetralin cis- and trans-decalin

28.5 Reactions of Anthracene and Phenanthrene


Reactions involving the meso-positions (C-9 and C-10) of phenanthrene and anthracene generate or preserve two ben-
zenoid rings in the products. Oxidations produce the 9,10-quinones, and both tricyclic arenes favorably undergo addition
reactions in 9,10-position as exemplified by reductions, hydrogenations, and brominations all proceeding under mild
conditions.

H H O
8 1
+ 2 Na , + 2 C 2H5OH 9 + 3/2 O2 (vapor phase)

10
 2 C2H 5ONa  H2O

H H O
9,10-dihydroanthracene 9,10-anthraquinone
H H O O
H H
8 10 9 1
+ H2 , Cu , Cr 2O3 + 2 CrO3

Cr 2O3
9,10-dihydrophenanthrene 9,10-phenanthrenequinone

Bromine adds to phenanthrene like an alkene, preserving two autonomous  electron sextets. The resulting dibromoad-
duct undergoes dehydrobromination to give 9-bromophenanthrene which is also formed by electrophilic bromination of
the parent arene. Driven by the formation of a second benzenoid  electron sextet, anthracene reacts like a 1,3-diene,
28.6 Enzymatic Epoxidation of Benzo[a]pyrene 73

with bromine adding to provide the 9,10-dibromoadduct which is also subject to dehydrobromination, yielding 9-
bromoanthracene.
+ Br 2 (FeBr 3) ,  HBr (substitution)
H Br Br
Br H

+ Br 2 KOH, 100 °C

HBr
9,10-dibromo-9,10-dihydrophenanthrene 9-bromophenanthrene
Br H Br

+ Br 2 KOH, 100 °C

HBr

Br H
9,10-dibromo-9,10-dihydroanthracene 9-bromoanthracene

Like a 1,3-diene, anthracene undergoes [4+2]-cycloadditions (DIELS-ALDER reactions) with typical dienophiles such as
maleic anhydride (Chapter 42.3) or p-benzoquinone (Chapter 53.3.3), thereby bridging the 9,10-positions. Benzene and
naphthalene are not susceptible to such cycloadditions.
O
H O
O H
H O
9
+ O
10
H
O 2,3:5,6-dibenzobicyclo[2.2.2]octane-
maleic anhydride 7,8-dicarboxylic acid anhydride

28.6 Enzymatic Epoxidation of Benzo[a]pyrene


Polycyclic arenes, such as benzo[a]pyrene occurring in exhaust and tobacco smoke, are enzymatically epoxidized in
mammalian organisms by oxygen. The resulting oxirane opens upon nucleophilic addition of water, giving a trans-diol
(Chapters 16.7, 22.3.3). Subsequent epoxidation of the adjacent CC double bond generates an oxiranediol as a carcino-
gen which adds to the nucleobase guanine of DNA (Chapter 73). The chemically changed DNA may induce the growth of
tumor cells.
H H OH H OH
O H H
H H
7 9 HO HO
O
H
enzyme enzyme
+ 1/2 O2 + H2O + 1/2 O2
5 11

3 1

benzo[a]pyrene 7,8-epoxybenzo[a]pyrene trans-7,8-dihydroxy- 9,10-epoxy-7,8-dihydroxy-


benzo[a]pyrene benzo[a]pyrene

________________________________________________________________________________________________
Chapter 28 permits answers to the following:
(28.1) Draw the formulas of tetracene and its isomers containing benzenoid rings with autonomous electron sextets.
(28.2) What products are obtained when naphthalene is reacted with electrophiles under mild conditions? Give reasons
and examples.
(28.3) What is the driving force behind the DIELS-ALDER reaction of anthracene with maleic anhydride?
(28.4) Anthracene and phenanthrene react with (a) bromine, (b) oxidizing reagents, and (c) reducing reagents. Write
equations for these reactions.
(28.5) Polycyclic arenes are known to be carcinogenic. Which reactions produce the carcinogens? Formulate
these reactions.
74 29 Non-benzenoid Aromatic Compounds

29 Non-benzenoid Aromatic Compounds


Non-benzenoid aromatic compounds contain cyclic carbon skeletons other than that of benzene. They are planar, and
may be neutral molecules, anions or cations, all having cyclically conjugated  bonds. Their number of  electrons follows
HÜCKEL’s (4N + 2) rule (Chapter 23.3), and they exhibit chemical and physical properties similar to those of benzene.
Table 29.1 reviews a small selection of non-benzenoid aromatics.

Table 29.1. Selection of non-benzenoid aromatic compounds


aromatic ions
2 6 6 10
_ 2
2

cyclopropenium cation cyclopentadienide anion cycloheptatrienium cation cyclooctatetraenediide dianion


number of
electrons 2 6 6 10
(4N + 2) , N = 0 1 1 2
[n]annulenes

H
H

[10]annulene [14]annulene [18]annulene [22]annulene


number of
electrons 10 14 18 22
(4N + 2) , N = 2 3 4 5

29.1 Non-benzenoid Aromatic Ions


29.1.1 Cyclopropenium Cation
The cyclopropenium cation as the hexachloroantimonate (salt) is obtained by reacting 3-chlorocyclopropene with antimo-
ny pentachloride. Reductive dechlorination of tetrachlorocyclopropene with tributyltin hydride yields the precursor 3-
chlorocyclopropene.
Cl Cl (C4H 9) 3SnH + SbCl5
+ 3 HI [ SbCl6 ]
 3 Cl
Cl Cl Cl H cyclopropenium hexachloroantimonate

29.1.2 Cyclopentadienide Anion


Deprotonation of cyclopentadiene with alkali hydroxide as base produces the cyclopentadienide anion. It can be de-
scribed as a hybrid of five equivalent resonance formulas or as a carbon pentagon with an inscribed circle symbolizing
the  electron sextet. Strong resonance stabilization due to aromaticity of the cyclopentadienide anion is the reason
behind the CH acidity of cyclopentadiene.
H H

or 6
_ H H
 [H ]
H H H H cyclopentadienide anion: resonance formulas

Reacting cyclopentadienide with transition-metal salts yields metal  complexes, also known as metallocenes. For exam-
ple, orange-red ferrocene is obtained from cyclopentadienide and ferrous chloride.
frontal view view from above
+ FeCl 2 (THF, N 2 )
2 6 Fe Fe
Na
 2 NaCl

ferrocene

Efficiently Studying Organic Chemistry: Exam training for chemists, biochemists, pharmacists, life and health scientists,
Third Edition. Eberhard Breitmaier. © 2022 WILEY-VCH GmbH. Published 2022 by WILEY-VCH GmbH.
29.2 [n]Annulenes 75

The antiprismatic structure of ferrocene and other metallocenes, embedding the metal cation between two staggered
cyclopentadienide pentagons, referred to as a sandwich complex, has been confirmed by crystal structure determination.
Similar to benzene, ferrocene undergoes electrophilic sulfonation and acetylation.

29.1.3 Cycloheptatrienium Cation


The abstraction of a hydride anion from cycloheptatriene yields a cycloheptatrienium salt. Carbenium ions, obtained in
situ from alkyl halides such as t-butyl chloride (Chapter 30.1) in the presence of a LEWIS acid, are used as hydride accep-
tors. Alternatively, 1,4-addition of bromine to cycloheptatriene yields 3,6-dibromo-1,4-cycloheptadiene, which is dehydro-
brominated upon heating to provide cycloheptatrienium bomide.
(H3C)3CCl , AlCl3 Br H
+ (H3C)3C 6  + Br 2
X H Br
 (H3C)3CH  HBr

H H X = Cl , Br
cycloheptatriene cycloheptatrienium halide 3,6-dibromo-1,4-cycloheptatriene cycloheptatriene

Five ring C atoms share six  electrons in the cyclopentadienide anion (Chapter 29.1.2), which is a -electron-excessive
aromatic compound attracting electrophiles. In contrast, the cycloheptatrienium ion has six  electrons shared by seven
ring C atoms, and therefore is a -electron-deficient aromatic compound which reacts with nucleophiles.
+ CN H
6 Br
 Br CN
7-cyano-1,3,5-cycloheptatriene

29.2 [n]Annulenes
Because of strong steric interaction of its inner hydrogen atoms, [10]annulene (Table 29.1) is not stable enough to exist
under normal conditions. [14]Annulene, however, can be prepared by oxidative cyclodehydrogenation of 4,10-tetradeca-
diene-1,7,13-triyne (GLASER coupling, Chapter 19.3.6), followed by catalytic hydrogenation.
13

10
H 1.) Cu(CH3CO2) 2 , pyridine
2.) KOC(CH3) 3 , HOC(CH3) 3  H2 (Pd) HH
7  H2 HH
H1
4 1

[18]Annulene is obtained by oxidative cyclodehydrogenation (G LASER coupling, Chapter 19.3.6) of three equivalents of
1,5-hexadiyne to give 1,3,7,9,13,15-cyclooctadecahexayne, which isomerizes to the fully conjugated cyclooctadecahex-
aenetriyne in the presence of a base. Partial catalytic hydrogenation yields the aromatic [18]annulene, existing as brown-
ish-red crystals.

Cu(CH 3CO2 ) 2 / pyridine KOC(CH 3) 3 + 3 H2


3
 6 [H ] ,  6 e0 (CH3) 3 COH / C6H 6 Pd / C in C 6H6
H H

________________________________________________________________________________________________
Chapter 29 permits answers to the following:
(29.1) Which types of non-benzenoid aromatic compounds do you know? Draw formulas.
(29.2) Write equations describing preparations of (a) potassium cyclopentadienide and (b) cycloheptatrienium bromide.
(29.3) What are -electron-excessive and -electron-deficient aromatics? Give examples and draw formulas.
(29.4) (a) [10]Annulene does not exist. Why? (b) Formulate equations describing preparations of [14]- and [18]annulene.
76 30 Alkyl Halides

30 Alkyl Halides
30.1 Classification, Nomenclature
A halogen atom X (X = F, Cl, Br, I) is connected to an alkyl group R in alkyl halides (RX), also referred to as haloalkanes.
Nomenclature follows the IUPAC rules outlined for alkanes (Table 30.1). Depending on the number of alkyl groups at-
tached to the halogenated carbon atom, primary, secondary, and tertiary alkyl halides differ from one another as shown in
Table 30.1 for the regioisomeric butyl bromides all having the molecular formula C4H9Br.

Table 30.1. Primary, secondary, and tertiary alkyl halides (regioisomers with the molecular formula C4H9Br)
alkane (parent hydrocarbon) alkyl halide (haloalkane): example IUPAC and common name alkyl halide: class
CH3 CH 2 CH2 CH3 butane CH 3 CH2 CH2 CH 2 Br 1-bromobutane
(n-butyl bromide) R CH 2 X primary
CH 3 CH 3
H3C C CH3 methylpropane H3C CH CH2 Br 1-bromo-2-methylpropane
(isobutyl bromide)
H
CH 3 CH2 CH CH3 2-bromobutane R CH X secondary
(sec-butyl bromide) R
Br
CH3 R
2-bromo-2-methylpropane
H3C C CH3 (t-butyl bromide) R C X tertiary
Br R

Alkyl halides give a positive BEILSTEIN test: Copper wire treated with a drop of alkyl halide held into the flame of a BUNSEN
burner changes the color of the flame to green due to the formation of volatile cupric halides. Tertiary and some second-
ary alkyl halides react with silver nitrate to give silver halide precipitates.

30.2 Preparation
30.2.1 Radical Substitution of Alkyl Groups
Photohalogenation of alkanes or alkylbenzenes, following the mechanism of radical substitution (Chapter 13.1), yields
alkyl halides. Benzyl halides (-halotoluenes) are prepared from toluene in this manner.
Cl
h + Cl2 , h
CH3 + Cl2 CH2 Cl CH Cl
 HCl  HCl
toluene -chlorotoluene ,-dichlorotoluene
(benzyl chloride) (benzal chloride)

Utilized as a bromine reservoir, N-bromosuccinimide brominates alkenes at the allylic position (  to the double bond) and
not at the double bond (WOHL-ZIEGLER bromination, Chapter 15.3.1). Allyl bromide is obtained from propene (Chapter
15.3.1), and 3-bromocyclohexene from cyclohexene.
O O
CCl4
reflux Br
CH2 + N Br + N H
H
O O
cyclohexene 3-bromocyclohexene

30.2.2 Addition of Hydrogen Halides and Halogens to Alkenes


Alkyl halides are prepared by electrophilic addition of hydrogen halides (HCl, HBr, HI) to alkenes involving the most stable
intermediate carbenium ion according to MARKOVNIKOV’s rule (hydrohalogenation of alkenes, Chapter 16.3). Therefore,
terminal alkenes such as 1-butene (R = CH2CH3) are regioselectively hydrohalogenated to provide 2-haloalkanes.
X
R CH CH 2 + H X R CH CH 3 + X R CH CH 3 ( X = Cl , Br , I )

Efficiently Studying Organic Chemistry: Exam training for chemists, biochemists, pharmacists, life and health scientists,
Third Edition. Eberhard Breitmaier. © 2022 WILEY-VCH GmbH. Published 2022 by WILEY-VCH GmbH.
30.2 Preparation 77

Halogens (Cl2 and Br2) add to alkenes yielding ,-dihaloalkanes (halogenation of alkenes and cycloalkenes, Chapters
16.2, 22.3.1).
X
C C + X2 C C X = Cl, Br
X
, -dihaloalkane

30.2.3 Substitution of Hydroxide in Alcohols by Halide


Iodo-, bromo- and chloroalkanes can be prepared from alcohols (ROH) and phosphorus halides, by displacing hydrox-
ide by halide. Thionyl chloride converts alcohols into chloroalkanes.
pyridine
3 R OH + PBr3 ( P + Br2 ) 3 R Br + H 3PO3 pyridine
pyridine R OH + SOCl2 R Cl + SO2 + HCl
3 R OH + PI3 ( P + I2 ) 3R I + H 3PO3 thionyl chloride
phosphorus halides

Methyl iodide (iodomethane) is prepared from highly toxic dimethyl sulfate, the ester of sulfuric acid and methanol (Chap-
ter 35.4).
O
H 2O, CaCO3
H 3CO S OCH 3 + 2K I 2 CH 3  + K2SO4
O methyl iodide
dimethyl sulfate (iodomethane)

30.2.4 Fluorination with Antimony Trifluoride


Fluorination of alkanes with fluorine proceeds violently, leading to perfluoroalkanes via scission of CC bonds. Chloroal-
kanes can be fluorinated with antimony trifluoride. Dichlorodifluoromethane, known as Freon and formerly used as a
refrigerant and aerosol propellant, was prepared from carbon tetrachloride and antimony trifluoride.
SbCl 5
3 CCl4 + 2 SbF 3 3 CCl2F 2 + 2 SbCl3
dichlorodifluoromethane (Freon)

Ultraviolet sunlight cleaves chlorofluoroalkanes in the stratosphere, generating chlorine radicals there which subsequently
react with ozone O3:
h
CCl2F 2 CClF 2 + Cl Cl + O3 ClO + O2
ClO + O O2 + Cl

As a result, the ozone concentration has decreased in the stratosphere, particularly above the arctic and antarctic re-
gions. Because ozone efficiently absorbs UV radiation of the sun, which not only induces skin cancers but also warms the
stratosphere, the production of "ozonekillers" such as chlorofluoroalkanes including Freon has been phased out since the
late 1970s.

30.2.5 Nucleophilic Iodination of Alkyl Halides (FINKELSTEIN Reaction)


Iodoalkanes, not available by iodination of alkanes, are prepared by nucleophilic substitution (Chapter 31) of chloride or
bromide in chloro- or bromoalkanes by iodide (FINKELSTEIN reaction).
NaI , acetone
R X + I R I + X ( X = Cl, Br )
78 30 Alkyl Halides

30.3 Electronegativity and the Inductive Effect


When a  bond joins two identical atoms (e.g. HH, ClCl, H3CCH3 ), both atoms attract the  electrons equally and
these electrons will concentrate in the center of the bond. When a  bond connects two different atoms (e.g. HCl,
H3CCl), one atom attracts  electrons more strongly and is said to be more electronegative than the other. Electronega-
tivity is a relative measure of the power with which an atom attracts bonding electrons. In the periodic table, electronega-
tivity increases from left to right in the rows and from down to up in the groups. Electronegativities as defined by PAULING
(Table 30.2) refer to the most electronegative element fluorine, having a value of 4 by convention.

H 2.2
Li 1.0 C 2.6 N 3.0 O 3.4 F 4.0
Na 0.9 Si 1.9 P 2.2 S 2.6 Cl 3.2
Br 3.0
I 2.7
Table 30.2. PAULING electronegativities of some elements

When two elements of different electronegativity are connected, the more electronegative one attracts the bonding elec-
trons at the expense of the other; the bonding electrons are no longer focused in the center of the bond but approach the
more electronegative atom. This polarization of a  bond by the more electronegative atom is called the inductive effect
[()-I effect]. Thus, a halogen atom in an alkyl halide RX polarizes the carbon-halogen  bond; the halogen acquires a
small negative charge, symbolized by  (or ), leaving a small positive charge at the attached carbon atom, symbolized
by + (or +):
   
R X (X = F, Cl, Br, I) H 3C Br

Molecules with this property are said to be dipoles. Having a positive and a negative end, dipolar molecules suffer a
torque when placed in an electric field, called the dipole moment .

30.4 Typical Reactions


30.4.1 Nucleophilic Substitution of Halide in Haloalkanes
In alkyl halides, the polarization of the carbon-halogen  bond determines the reactivity: The carbon atom attached to
halogen in an alkyl halide is polarized (+) to provide an electrophile (a latent carbenium ion, Chapter 30.3), able to attract
nucleophiles. Thus, nucleophilic substitution (SN, Chapters 11.5, 31) is the typical reaction of alkyl halides: A nucleophile
INu approaches the electrophilic carbon atom, displacing the halogen which leaves as a halide anion.

substrate nucleophile product nucleofuge


entering group, new substituent leaving group (halide anion)
 
R X + INu R Nu + X
R X + INu R Nu + X
R X + INu H R NuH + X

Nucleophiles are anions (hydroxide, cyanide) or neutral molecules (ammonia) reacting, for example, with 1-bromobutane
as substrate. All nucleophiles contain a non-bonding electron pair which forms the new bond in the product.

substrate nucleophile product nucleofuge


entering group, new substituent leaving group (halide anion)

CH3 CH 2 CH 2 CH2 Br + I OH CH3 CH 2 CH 2 CH 2 OH + Br


1-bromobutane 1-butanol

CH3 CH 2 CH 2 CH2 Br + I CN CH3 CH 2 CH 2 CH 2 CN + Br


1-cyanobutane ( n-butyl cyanide)

CH3 CH 2 CH 2 CH2 Br + I NH 3 CH3 CH 2 CH 2 CH 2 NH3 Br


n-butylammonium bromide
30.4 Typical Reactions 79

Nucleophilic substitutions of alkyl halides obtained from alkanes and alkenes introduce new substituents into an alkyl
chain, thus having preparative significance.

30.4.2 Dehydrohalogenation (-Elimination)


The -elimination of hydrogen halide from an alkyl halide in the presence of a base (dehydrohalogenation) has been
described as a method to prepare alkenes (Chapter 15.1). Eliminations from alkyl halides compete with nucleophilic
substitutions of alkyl halides. Which one of the two reactions takes preference depends on the basicity of the nucleophile
according to the HSAB principle (hard soft acid base, soft with soft, hard with hard). Hard bases (nucleophiles) are com-
pact ions with high charge density, such as the hydroxide anion. Soft bases (nucleophiles) are voluminous ions, such as
the iodide anion, or polarizable ions with multiple bonds, such as the cyanide anion (ICNI. Alkyl halides are weak
bases (electrophiles). To conclude, substitutions of alkyl halides take preference with weak bases such as iodide and
cyanide, while eliminations predominate with strongly basic nucleophiles such as hydroxide (hard base):
H H
+ hard base B:  + soft base B:
H B + X + + X
C C C C C C
elimination E nucleophilic substitution SN
X B
alkyl halide

30.4.3 Metalation
Alkyl halides react with magnesium metal (turnings) in ether solution. This reaction produces alkylmagnesium halides
(GRIGNARD compounds). Alkylmagnesium halides equilibrate with dialkylmagnesium (SCHLENK equilibrium). Alkylmagne-
sium halides and dialkylmagnesium contain carbon-metal bonds, thus representing a class of organometal compounds
(Chapter 32).
+  ether  ++  SCHLENK  ++ 
2 R X + 2 Mg 2 R Mg X R Mg R + MgX2
equilibrium
alkyl halide ( X = Cl , Br , I ) alkylmagnesium halide dialkylmagnesium

The reaction represents a metalation. Magnesium metalates iodomethane to give methylmagnesium iodide, while 2-
bromopropane yields isopropylmagnesium bromide.
+  ether  ++  +  ether  ++ 
H3C I + Mg H 3C Mg I (H3C)2CH Br + Mg (H3C)2CH Mg Br
methylmagnesium iodide isopropylmagnesium bromide

As indicated by the partial charges  and +, metalation reverses the polarity (umpolung) of the electrophilic  carbon
atom of the alkyl halide to provide a carbon nucleophile in the organometal compound, accounting for the preparative
significance of alkylmagnesium halides (Chapters 32.3, 33.3.7, 49.3.3).
Alkylmagnesium halides react with sufficiently acidic compounds such as H2O or alcohols ROH to give alkanes RH, thus
detecting "active hydrogen". Upon addition of methylmagnesium iodide to a compound containing one acidic hydrogen
atom in the molecule, one equivalent of methane is evolved (ZEREWITTENOFF reaction):
 +  ++   +  ++ 
R X H + H 3C Mg I R XMg I + CH 4 O H + H 3C Mg I HO + I + Mg + CH4
H
________________________________________________________________________________________________
Chapter 30 permits answers to the following:
(30.1) Draw all structural isomers of (a) C3H7Br and (b) C4H9Cl, and classify them as primary, secondary, or tertiary.
(30.2) 2-Bromopentane can be prepared by substitution and addition. Write the equations for these reactions.
(30.3) What kind of compounds are the so-called ozonekillers? Write equations for the ozone-killing reactions.
(30.4) Explain the term inductive effect and describe its influence on the reactivity of alkyl halides.
(30.5) 2-Bromopentane reacts with (a) potassium iodide, (b) potassium cyanide, and (c) ammonia. Write the equations.
(30.6) 2-Bromobutane reacts with potassium hydroxide. Which products can be formed?
(30.7) Formulate the reaction of 1-bromobutane with magnesium in ether including the change of polarity of C-1.
(30.8) Can the ()-I effect of halogen influence the regioselectivity of electrophilic substitution of halobenzenes?
80 31 Mechanisms of Nucleophilic Substitution

31 Mechanisms of Nucleophilic Substitution


Nucleophilc substitutions (SN) may follow two different mechanisms.
The bimolecular mechanism (SN2) proceeds in one step involving both reactants, the substrate as well as the nucleo-
phile.
The monomolecular mechanism (SN1) involves two steps: first, the substrate slowly loses the leaving group, thus forming
a carbenium ion; second, the intermediate carbenium ion rapidly reacts with the nucleophile to give the product.

31.1 Bimolecular Nucleophilic Substitution (Second-Order)


The SN2 mechanism is a one-step reaction involving a transition state (Fig. 31.1). The formation of the transition state
upon collision of the substrate (RX) and the nucleophile (INu) represents a bimolecular reaction, the rate r depending
on both concentrations [RX] and [INu], thus following a second-order rate law:
r = k [RX] [INu].
The rate constant k depends on the activation barrier EA of the transition state (Fig. 31.1): the lower the activation
barrier the higher k and the rate r of reaction.

Epot
Nu C X transition state

EA

NuI + C X

H

Nu C + X
reaction
coordinate
NuI + R X Nu R + X
Fig. 31.1. Energy diagram of an SN2 reaction

The central C atom of the substrate adopts the coordination number 5 (becoming pentavalent) in the transition state (Fig.
31.1). The molecular orbital model explains this by sp2 hybridization, the orbitals of nucleophile and nucleofuge overlap-
ping with the halves of the unhybridized 2p orbital perpendicular to the plane of sp2 hybrid orbitals forming the bonds with
the other substituents R, R’, and R’’.

R
R R
+  slow   fast
NuI + C X Nu C X Nu C + X
R' R'
R'
R" R"
R"
nucleophile substrate product nucleofuge

transition state with electronic balance of charge

The nucleophile INu attacks the substrate molecule from the side opposite to the leaving group X, as shown in the
equation and in Fig. 31.1, because electrostatic repulsion would hinder frontal attack (from the same side). As the nucleo-
phile approaches, the leaving group departs the transition state (push-pull mechanism), the central carbon changing from
trigonal sp2 back to tetrahedral sp3 hybridization. For this reason, the configuration of the substituents R, R’, and R’’ is
inverted in the product (WALDEN inversion), and this is detectable when all four substituents are different (Chapter 44.4).

Efficiently Studying Organic Chemistry: Exam training for chemists, biochemists, pharmacists, life and health scientists,
Third Edition. Eberhard Breitmaier. © 2022 WILEY-VCH GmbH. Published 2022 by WILEY-VCH GmbH.
31.2 Monomolecular Nucleophilic Substitution (First-Order) 81

31.2 Monomolecular Nucleophilic Substitution (First-Order)


Tertiary alkyl halides such as t-butyl bromide able to form stable carbenium ions such as t-butyl cations may undergo
nucleophilic substitutions involving two steps. First, the alkyl halide slowly loses a halide anion X, leaving an intermedi-
ate carbenium ion R. Second, the nucleophile (e.g. INu = OH) can attack the electron-deficient carbon from either
side of the planar carbenium ion, which contains a vacant 2p orbital at the sp2 hybridized carbon atom on both sides of
the plane of bonds in the molecular orbital model.

1st step H3C 2nd step


slow dissociation CH3 fast ion reaction
H3C
of the substrate
H3C
+   Br  OH
C Br C (H3C)3C OH

H3C
CH3
2-bromo-2-methylpropane 2-methyl-2-propanol
t-butyl bromide t-butyl alcohol
c a r b e n i u m io n
vacant 2p orbital (positive charge)

Because the second step is a fast ion reaction requiring a lower activation barrier EA2 than the first step (EA1 > EA2 ,
Fig. 31.2), the first and monomolecular step is rate-determining, solely dependent on the concentration of the substrate,
thus following a first-order rate law:
r = k [RX]

Epot R X
transition state 1

R Nu
transition state 2
EA2
EA1
R
solvated
+ X ions
+ INu

R X
H

R Nu
reaction
coordinate
R X R + X R + INu R Nu
Fig. 31.2. Energy diagram of an SN1 reaction step 1 step 2

Usually, nucleophilic substitutions of tertiary alkyl halides follow the SN1 mechanism. Primary and secondary alkyl halides
are substituted by way of the SN2 mechanism, provided dissociation of the alkyl halide does not leave a particularly stable
carbenium ion such as the resonance-stabilized benzyl cation (similar to the benzyl radical in Chapter 27.2), exemplified
by benzyl halides which can follow the SN1 mechanism.
 Br  OH
CH 2 Br CH 2 CH 2 CH 2 CH 2 CH 2 OH
slow fast
-bromotoluene (benzyl bromide) resonance-stabilized benzyl cation benzyl alcohol

________________________________________________________________________________________________
Chapter 31 permits answers to the following:
(31.1) Draw equations to outline the mechanisms and rates of nucleophilic substitution.
(31.2) Formulate the reactions of (a) t-butyl bromide with iodide (SN1), and (b) 1-bromobutane with hydroxide (SN2).
(31.3) Which mechanism is probable for the reaction of -bromoethylbenzene with sodium hydroxide?
(31.4) What happens in the so-called WALDEN inversion? Why does this not play a role in substitutions with an
SN1 mechanism?
82 32 Organometal Compounds

32 Organometal Compounds
32.1 General Survey
An organometal compound contains a carbon-metal bond, connecting an alkyl or aryl group with a metal. Therefore,
organometal compounds are called alkyl- and arylmetals or, in the case of GRIGNARD compounds (Chapter 30.4.3), alkyl-
and arylmetal halides:

H3C CH2 CH2 CH2 Li Li Mg Br CH2 Mg Br

butyllithium phenyllithium phenylmagnesium bromide benzylmagnesium bromide


GRIGNARD compounds

Prefixes di-, tri-, tetra- indicate the number (two, three, four) of groups attached to the metal; similar to the alkanes, or-
ganometal compounds containing group IV metals are also named metallanes (e.g. plumbanes):

(H 3C CH 2 CH2 CH2)2 Zn CH 2 Cd CH 2 (H 3C CH2 CH2)3 Al (H 3C)4Sn (H 3C CH 2)4 Pb


dibutylzinc dibenzylcadmium tripropylaluminum tetramethylstannane tetraethyllead
(tetraethylplumbane)

Organometal compounds include  complexes of transition-metals with aromatics such as benzene (dibenzenechromium)
and the cyclopentadienide anion (ferrocene, Chapter 29.1.2),  bonding arising from sidewise overlapping of the aromatic
 clouds with hybrid orbitals of the metal.

metallocene
transition-metal  complexes Cr dibenzenechromium M
M = Fe: ferrocene

32.2 Preparation
32.2.1 Metalation of Alkyl and Aryl Halides
Metalation of alkyl and aryl halides with reducing metals yields organometal compounds and metal halides, iodides and
bromides reacting most readily (I > Br > Cl >> F). Many organometallic compounds react violently with water and oxygen,
igniting in air, including diethylzinc. Thus, controlled preparations and reactions have to be performed in unreactive (inert)
solvents such as hydrocarbons or ethers and under a protecting gas like argon or nitrogen (absence of oxygen). Rea-
gents for organic synthesis such as butyl- and phenyllithium, diethylzinc and alkylmagnesium halides (Chapter 30.4.3) are
obtained by metalation of alkyl or aryl halides.
ether
H 3C CH2 CH2 CH2 Br + 2 Li . H3C CH 2 CH 2 CH2 Li + LiBr
1-bromobutane 1-butyllithium

Br + 2 Li . Li + LiBr
bromobenzene phenyllithium
2 H 3C CH 2 I + 2 Zn .. H3C CH 2 Zn CH 2 CH3 + ZnI2
iodoethane diethylzinc
(H3C)2CH Br + Mg .. (H 3C)2CH MgBr
2-bromopropane isopropylmagnesium bromide

32.2.2 Transmetalation
The reaction of a GRIGNARD reagent or another organometallic compound with metal halides in an inert solvent is an
alternative preparation. This transmetalation involves an equilibration between two organometallics in favor of the com-
pound containing the less electropositive metal. Diethylcadmium is obtained by reacting ethylmagnesium bromide with
cadmium bromide.
2 C2H5 MgBr + CdBr2 (C2H5)2 Cd + 2 MgBr2
ethylmagnesium bromide diethylcadmium

Efficiently Studying Organic Chemistry: Exam training for chemists, biochemists, pharmacists, life and health scientists,
Third Edition. Eberhard Breitmaier. © 2022 WILEY-VCH GmbH. Published 2022 by WILEY-VCH GmbH.
32.3 Preparative Significance 83

Transmetalation also opens access to tetraalkylated group IV elements such as silanes and stannanes.
4 CH 3 MgCl + SiCl4 (H 3C)4 Si + 4 MgCl2 4 R MgCl + SnCl4 R 4 Sn + 4 MgCl2
tetramethylsilane R = H3C CH 2 CH 2 CH 2
(TMS) tetrabutylstannane

32.2.3 Halogen-Metal Exchange


Aryl, alkenyl, and alkynyl groups are more closely and strongly bonded to metals than alkyl groups due to the shorter sp2
and sp hybrid orbitals originating from carbon, as outlined in the molecular orbital model. For this reason, aryl, alkenyl,
and alkynylmetals can be prepared by halogen-metal exchange with alkyl metals:

R X + R' Li R Li + R' X R = aryl-, alkenyl-, alkynyl- ; R' = alkyl- ; X = I > Br > Cl

Following this route, p-chlorophenyllithium can be prepared from p-bromochlorobenzene and butyllithium:

Cl Br + H 3C CH 2 CH 2 CH2 Li Cl Li + H3C CH 2 CH 2 CH2 Br

32.2.4 Hydrogen-Metal Exchange


Hydrogen-metal exchange permits the preparation of GRIGNARD compounds from CH acidic hydrocarbons such as termi-
nal alkynes (Chapter 19.3.5) and cyclopentadiene (Chapter 29.1.2) by reaction with readily available alkylmagnesium
halides:
H
CH2 + H3C CH2 MgBr + C2H6 R C C H + H3C CH2 MgBr R C C MgBr + C2H 6
MgBr
terminal alkyne alkynylmagnesium
cyclopentadiene cyclopentadienyl- bromide
magnesium bromide

32.3 Preparative Significance


As shown for the formation of GRIGNARD compounds and indicated by the partial charges  and +, metalation reverses
the polarity of the electrophilic  carbon atom of the alkyl halide (Chapter 30.4.3). Organometal compounds are therefore
carbon nucleophiles, readily reacting with carbon electrophiles to form carbon-carbon bonds.
Alkyl or aryl halides, R´X, with an electrophilic carbon atom attached to halogen, and alkyl- or arylmagnesium halides,
RMgX, with a nucleophilic carbon attached to the metal react to form a new carbon-carbon bond, producing the hydro-
carbon RR´. In this manner, phenylcyclopentane can be prepared from bromocyclopentane and phenylmagnesium
bromide or from bromobenzene and cyclopentylmagnesium bromide:

+   
Br + Br Mg Mg Br + Br +
 MgBr2  MgBr2
bromocyclopentane phenylmagnesium phenylcyclopentane cyclopentylmagnesium bromobenzene
bromide bromide

Nucleophilic addition of organometallic compounds to electrophilic carbon-heteroatom multiple bonds, as present in


carbonyl compounds (X = O), is widely applied in organic syntheses (Chapter 33.3.7, 49.3.3).
R R
+ H2O
C X + R M C XM C XH X = O , S , NR' ; M = Li , MgBr
 MOH

________________________________________________________________________________________________
Chapter 32 permits answers to the following:
(32.1) Which reactions open access to organometallic compounds? Formulate general equations.
(32.2) Suggest reactions to prepare (a) butyllithium, (b) phenyllithium, and (c) butynylmagnesium bromide.
(32.3) Formulate all reactions necessary to prepare allylmagnesium bromide from propene. Look at Chapter 15.3.1.
(32.4) Suggest syntheses of (a) allylcyclopropane and (b) phenylcyclopentadiene.
84 33 Alcohols

33 Alcohols
33.1 Nomenclature, Classification
Alcohols contain the hydroxy group as function and have the general formula ROH, where R represents any alkyl
group. Their IUPAC name is derived from the name of the longest continuous carbon chain containing the hydroxy group,
the ending ol replacing the ending e of the parent alkane, alkene, or alkyne. 2-Propen-1-ol (H2C=CHCH2OH) with
common name allyl alcohol originates from the parent propene (H2C=CHCH3) in this manner. A homologous series of
alkanols with molecular formula CnH2n+2O, beginning with methanol H3COH as the simplest representative, is related to
the homologous series of alkanes with molecular formula CnH2n+2 (Table 5.1).
The number (1, 2, or 3) of alkyl groups attached to the carbon atom bearing the hydroxy group defines the class of alco-
hol, denoted as primary (1°), secondary (2°) or tertiary (3°). Four structurally isomeric alcohols with molecular formula
C4H10O, for example (Table 33.1), include two primary alcohols, and one secondary and one tertiary alcohol.

Table 33.1. Primary, secondary and tertiary alcohols (regioisomers with the molecular formula C4H10O)
IUPAC name
alkane (parent compound) alkanol (alcohol) : example (common name) alkanol (alcohol): structural type

CH3 CH2 CH 2 CH3 butane CH 3 CH2 CH2 CH 2 OH 1-butanol


(n-butyl alcohol) R CH2 OH primary
CH3 CH3
H 3C C CH3 methylpropane H3C CH CH2 OH 2-methyl-1-propanol
(isobutyl alcohol)
H
CH 3 CH2 CH CH 3 2-butanol R CH OH secondary
(sec-butyl alcohol)
OH R
CH3 R
2-methyl-2-propanol
H 3C C CH 3 (t-butyl alcohol) R C OH tertiary
OH R

Cycloalkanols are hydroxycycloalkanes. Cyclohexanol is a secondary alcohol. 1-Methylcyclohexanol, a tertiary alcohol, is


one of four structurally isomeric (regioisomeric) methylcyclohexanols, the secondary alcohols 2-, 3-, and 4-methylcyclo-
hexanol existing as cis- and trans-isomers (configurational isomers).
regioisomers
1-methylcyclohexanol 2-methylcyclohexanol 3-methylcyclohexanol 4-methylcyclohexanol
OH CH 3
CH3 OH OH OH OH OH OH
CH 3 H3C
H3C H3C
OH CH3
cis- trans- cis- trans- cis- trans-
cyclohexanol configurational isomers (stereoisomers)

33.2 Structure and Physical Properties


As shown for methanol and ethanol (Fig. 33.1), alkanols possess COH bond angles close to the tetrahedron angle of
109°28´, matching to sp3 hybridized oxygen in the molecular orbital model. CO single bonds are shorter (143 pm, Fig.
33.1) than CC single bonds (154 pm, Fig. 4.2). Due to its high electronegativity (inductive effect, Chapter 30.3), oxygen
polarizes the CO and OH  bonds of alcohols.

143 pm  
O CH 3 O CH 3
92 pm 107°
H  H

Fig. 33.1. Molecular models of ethanol (tube, ball-spoke, space-filling), bond data, and polarity of the methanol molecule

Efficiently Studying Organic Chemistry: Exam training for chemists, biochemists, pharmacists, life and health scientists,
Third Edition. Eberhard Breitmaier. © 2022 WILEY-VCH GmbH. Published 2022 by WILEY-VCH GmbH.
33.3 Preparation 85

Alcohols do not exist as single molecules in the liquid state. The hydroxy hydrogen of alcohols is held by a polarized
covalent bond to the oxygen and by electrostatic forces to the oxygen of another molecule. Such hydrogen bridges cause
association of many molecules (clusters). Vaporization of alcohols requires an extra amount of energy to cleave the
hydrogen bridges. As a result, alcohols have boiling points much higher than those of isomeric ethers (Chapter 37.2) that
are unable to associate by hydrogen bridging.
R R
hydrogen bridges
H CH 3
O O
H  H H H H 3C CH 2 O H 3C O
O  O ethanol dimethyl ether
R R boiling points (1011 mbar) 78.2 °C  24 °C

33.3 Preparation
33.3.1 Industrial Syntheses of Methanol and Ethanol
Toxic methanol, causing dimness of vision and blindness, and used as an industrial solvent and raw material for synthe-
sis, is produced from carbon monoxide and hydrogen at high temperatures, under pressure, and in the presence of a
metal oxide catalyst. Catalytic hydrogenation of acetaldehyde (Chapter 19.3.4) is applied to produce ethanol.
ZnO / Cr2O3 O
heat, pressure + H2 / RANEY-Ni
CO + 2 H2 H3C OH H3C C H3C CH2 OH
methanol H
ethanal (acetaldehyde) ethanol

33.3.2 Alcoholic Fermentation (Beer, Wine, Distillates)


Ethanol is the intoxicating and addictive component of alcoholic beverages which are aqueous solutions of ethanol gen-
erated by alcoholic fermentation of glucose (Chapter 71) in water by yeast.
enzymatic hydrolysis yeast
starch C6H12O6 2 C2H5OH + 2 CO2
(in fruits, cereals, potatoes) glucose (in fruits) ethanol carbon dioxide

Wines from grapes and other fruits are prepared in this manner and contain up to 15 % ethanol. Beers containing less
than 6 % ethanol are brewed from barley (and hops) by enzymatic hydrolysis of the cereal starch to glucose (Chapters
71.1, 72.2) followed by alcoholic fermentation. Distillates of must and mash such as brandy and whisky (whiskey) contain
up to 45% ethanol.

33.3.3 Hydration of Alkenes


Hydration of alkenes yields alcohols (Chapter 16.4), exemplified by ethanol which is industrially produced by hydration of
ethene with aqueous sulfuric acid:
H H H H H H H H
H2SO4
C C + H3O H C C + H2O H C C H H C C H
H H H  [H ]
H H OH2 H OH
ethene ethylium ion ethoxonium ion ethanol

Hydration of unsymmetric alkenes proceeds regioselectively through the most stable carbenium ion according to MAR-
KOVNIKOV’s rule (Chapter 16.4). Hydration of methylpropene, for example, involves the higher alkylated t-butyl cation
rather than the isomeric less-stable 2-methyl-1-propyl cation with terminal positive charge, therefore predominantly yield-
ing t-butyl alcohol:
H3C H3C OH2 OH
+ H2O
C CH2 + [H ] C CH3 H3C C CH3 H3C C CH3
 [H ]
H3C H3C CH3 CH3
methylpropene more stable carbenium ion 2-methyl-2-propanol
(isobutene) (t-butyl alcohol)
86 33 Alcohols

33.3.4 Hydroboration of Alkenes and Oxidation of Trialkylboranes


Hydroboration of alkenes (addition of diborane B2H6 as BH3 to CC double bonds) to provide trialkylboranes (Chapter
16.6) involves synchronous addition of electrophilic boron (BH2) and nucleophilic hydride to the CC double bond.
R 2C CR 2 R 2CH CR 2 + R 2C CR2 + R2 C CR2
+ (R2CH CR2 )2 BH (R 2CH CR2 )3 B
H BH2 BH2
alkylborane dialkylborane trialkylborane

Oxidation of the trialkylborane with hydrogen peroxide leads to an alcohol and boric acid via trialkyl borate.
(R2CH CR2 )3 B + 3 H2O2 (R2CH CR2 O )3 B + 3 H2O 3 R2CH CR2 OH + B(OH)3
trialkyl borate (ester of boric acid)

Hydroboration proceeds stereocontrolled, the bulky BH2 adding to the less substituted terminal C atom of the double
bond, yielding the anti-MARKOVNIKOV alcohol upon reaction with hydrogen peroxide. Thus, when performed with
methylpropene, the hydroboration-oxidation reaction sequence yields 2-methyl-1-propanol (isobutyl alcohol) and not t-
butyl alcohol.
H 3C H 3C + 3 H2O2
H3C
C CH 2 CH CH 2 BH2 [(H 3C)2CH CH2] 3B CH CH2 OH
 B(OH) 3
H 3C + H 3C H3C
2-methyl-1-propanol
H BH2
(isobutyl alcohol)

33.3.5 Reduction of Carbonyl Compounds by Complex Metal Hydrides


Electron-pair withdrawal [()-M effect] by the carbonyl oxygen atom of the carbonyl group present in aldehydes and
ketones polarizes the carbonyl carbon to an electrophile, as shown by the resonance formulas of the carbonyl group
(carbonyl resonance, Chapters 47.3, 49). Therefore, nucleophilic hydride anions add to the carbonyl carbon atom, pro-
ducing an alkoxide anion (alcoholate).
_ _
C O C OI
_ + IH C OI
_
H
carbonyl compound, resonance formulas nucleophilic alkoxide anion (alcoholate)
electrophilic carbonyl carbon hydride anion

Providing hydride ions, lithium aluminum hydride LiAlH4 and other complex metal hydrides reduce carbonyl compounds to
alcohols in anhydrous (absolute) ether as solvent. Primary alcohols are obtained by reduction of aldehydes; ketones are
reduced to secondary alcohols in this manner.
O + 4H2 O
ether (absolute)
4R C + Li AlH4 (R CH2 O)4 Al Li 4 R CH2 OH
 Al(OH) 3 ,  LiOH
H primary alcohol
aldehyde

O H H
ether (absolute) + 4H2 O
4R C + Li AlH4 (R C O)4 Al Li 4 R C OH
 Al(OH) 3 ,  LiOH
R´ R´ R´
ketone secondary alcohol

33.3.6 Nucleophilic Substitution of Alkyl Halides


Nucleophilic substitution (SN) of the halide anion of alkyl halides by hydroxide as nucleophile is a general method to
prepare alcohols. 1-Butanol is obtained from 1-bromobutane (Chapter 30.4.1), and allyl alcohol (2-propen-1-ol) from allyl
bromide (3-bromo-1-propene).
SN1
H 2C CH CH 2 Br + OH H 2C CH CH 2 OH + Br
allyl bromide allyl alcohol
33.3 Preparation 87

33.3.7 Addition of Alkylmagnesium Halides to Carbonyl Compounds


Organometal compounds are carbon nucleophiles (Chapters 30.4.3, 32.3). Therefore, the addition of the nucleophilic C
atom of an alkyl- or arylmagnesium halide (GRIGNARD compound) to the electrophilic C atom of a carbonyl compound
(aldehyde or ketone) yields a magnesium halide alcoholate and, upon hydrolysis, an alcohol (GRIGNARD reaction):
_    _ + H2O
C O C OI
_ + R Mg X C OI
_ C OH
2
Mg , X  OH
R R
carbonyl compound, resonance formulas alkylmagnesium halide alcoholate alcohol
electrophilic carbonyl carbon nucleophilic alkyl carbon

For example, isopropylmagnesium bromide, obtained in situ from 2-bromopropane (isopropyl bromide) and magnesium
turnings in absolute ether, permits the preparation of primary, secondary, and tertiary alcohols, depending on the kind of
carbonyl compound employed as electrophile, thus reacting

-with formaldehyde to give 2-methyl-1-propanol (a primary alcohol),


O
ether + H2O
H C + H 3C CH MgBr CH 2 OMgBr CH 2 OH
 Mg(OH)Br
H CH 3 CH CH 3 CH CH 3

isopropylmagnesium CH 3 CH 3
formaldehyde bromide 2-methyl-1-propanol
primary alcohol

- with benzaldehyde to give 2-methyl-1-phenylpropanol (a secondary alcohol),


O
ether + H 2O
C + H3C CH MgBr CH OMgBr CH OH
H  Mg(OH)Br
CH3 CH CH3 CH CH3
CH3 CH3
benzaldehyde 2-methyl-1-phenylpropanol
secondary alcohol

- and with acetophenone (methyl phenyl ketone) to give 3-methyl-2-phenyl-2-butanol (a tertiary alcohol).

O CH 3 CH 3
ether + H 2O
C + H 3C CH MgBr C OMgBr C OH
CH 3  Mg(OH)Br
CH 3 CH CH 3 CH CH 3

acetophenone CH 3 CH 3
3-methyl-2-phenyl-2-butanol
tertiary alcohol

________________________________________________________________________________________________
Chapter 33 permits answers to the following:
(33.1) 1-Butanol has a higher boiling point (74 g/mol, 118 °C) at normal pressure than pentane (72 g/mol, 36 °C). Why?
(33.2) Draw the formulas of all structurally isomeric alcohols (7) with the molecular formula C 5H12O and classify them
as primary, secondary, or tertiary.
(33.3) Draw formulas of all structurally and configurationally isomeric methylcyclopentanols and name them.
(33.4) Write equations describing the preparations of (a) t-butyl alcohol, (b) 2-methyl-1-propanol, and (c) allyl alcohol.
(33.5) Suggest syntheses of (a) 1-pentanol (n-pentyl alcohol) and (b) 2-pentanol from 1-pentene.
(33.6) Write equations describing the preparations of (a) primary and (b) secondary alcohols by reduction.
(33.7) Which alcohols are obtained by reacting ethylmagnesium bromide with (a) benzaldehyde and (b) acetophenone?
Write equations.
(33.8) Nucleophilic hydroxylation of allyl bromide follows an SN1 mechanism. Why? Look at Chapter 31.2.
88 34 Diols, Triols

34 Diols, Triols
Diols, triols, and polyols contain two, three, and more hydroxy groups as indicated by the suffix -diol or -triol in the IUPAC
name derived from the alkane. 1,2,3-Propanetriol with the common name glycerol is the alcohol component of fats (Chap-
ter 74.2). Polyols with four and more hydroxy groups (m = 2, 3, 4) are reduction products of carbohydrates (Chapter
71.5.3). Cycloalkanediols exist as cis- and trans-isomers.
diols triols polyols
CH 2 OH CH 2 OH CH 2 OH OH OH CH 2 OH CH 2 OH CH 2OH
OH
CH 2 OH CH OH CH 2 CH OH CH OH (HC OH) m
CH 3 CH 2 OH OH CH 2 OH CH OH CH 2OH
cis- trans-
glycol glycerol CH 3
(ethanediol) 1,2-propanediol 1,3-propanediol 1,2-cyclohexanediol 1,2,3-propanetriol 1,2,3-butanetriol

34.1 Preparation
34.1.1 Dihydroxylation of Alkenes
cis-Cycloalkanediols are prepared by cis-dihydroxylation of cycloalkenes with osmium tetroxide or potassium permanga-
nate,
O
O
Os
O OH OH
O + 2 H2O
+ OsO4 OH
 H2 OsO4
OH
cyclohexene cis-1,2-cyclohexanediol- cis-1,2-cyclohexanediol
osmic acid ester

and trans-cycloalkanediols by trans-dihydroxylation of cycloalkenes to bicyclic epoxides (oxiranes) followed by hydrolysis


of the epoxides in a weakly acidic medium:

OH
+ RCO3 H + H 3O + H 2O
+ OH2 OH2 OH
 RCO2H  H3 O OH
O O H OH OH
cyclohexene cyclohexene oxide trans-1,2-cyclohexanediol

The stereospecificity of both types of dihydroxylation is not detectable for non-cyclic alkenes due to free rotation about the
newly formed CC single bond (Chapter 16.7).

34.1.2 Hydrolysis of Halohydrins


Glycerol is produced by saponification of fats (Chapter 74.2.2). An industrial synthesis begins with radical chlorination of
propene to give allyl chloride. Subsequent nucleophilic substitution by hydroxide yields allyl alcohol which is reacted with
sodium hypochlorite (halohydrin formation, Chapter 16.5). Another nucleophilic hydroxylation of the resulting glycerol
chlorohydrin finally produces glycerol.

propene H 3C CH CH 2 CH2 CH CH 2 glycerol


OH OH OH
500 - 600 °C , + Cl2  HCl + OH (NaOH) Cl

+ OH (NaOH) + HOCl
CH2 CH CH 2 CH2 CH CH 2 CH2 CH CH 2
 Cl
Cl OH OH OH Cl
allyl chloride (3-chloro-1-propene) allyl alcohol (2-propen-1-ol) glycerol chlorohydrin (3-chloro-1,2-propanediol)

Efficiently Studying Organic Chemistry: Exam training for chemists, biochemists, pharmacists, life and health scientists,
Third Edition. Eberhard Breitmaier. © 2022 WILEY-VCH GmbH. Published 2022 by WILEY-VCH GmbH.
34.2 Oxidative Cleavage of Glycols 89

34.1.3 Bimolecular Reduction of Carbonyl Compounds


A bimolecular reduction of ketones (Chapter 48) with magnesium yields magnesium -diolates which undergo hydroly-
sis to provide -diols (pinacol reaction).
R R R R
R R' R R' benzene R' R' + 2 H2O
C C C C R' C C R'
O + Mg + O O O  Mg(OH) 2
HO OH
Mg
ketone magnesium diolate ,-diol

Following this path, 2,3-dimethyl-2,3-butanediol (pinacol) can be prepared from acetone (propanone):
1.) Mg in benzene
H 3C CH3
acetone 2 H3C CH3 2.) + 2 H2O ,  Mg(OH) 2 2,3-dimethyl-2,3-butanediol
(propanone) C H 3C C C CH 3 (pinacol)
O HO OH

34.2 Oxidative Cleavage of Glycols


,-Diols (glycols) undergo oxidative cleavage of the bond between the hydroxylated carbon atoms upon treatment with
periodic acid (MALAPRADE cleavage) or lead tetraacetate (CRIEGEE cleavage), yielding two carbonyl compounds (alde-
hydes and/or ketones):
R2 R3 R2 R3
HI O4 or (CH3CO2) 4Pb
R1 C C R4 C + C
 2 [H ] ,  2 e0 R1 O O R4
HO OH
, -diol carbonyl compounds

As an example, pinacol (R1 = R2 = R3 = R4 = CH3) is cleaved to the ketone acetone (propanone). Cyclic 1,2-diols with cis-
configuration of the hydroxy groups are more easily cleaved than the trans-isomers, providing evidence that cyclic perio-
dates or lead salts are involved as intermediates:

O O
IO3H Pb 2 2 CH3CO2
O O

Therefore, oxidative cleavage of a 1,2-diol with lead tetraacetate probably proceeds in three steps:

C OH
+ Pb(O2C CH 3)4 2 C O
C OH

(1) CH 3CO2H (3)  Pb (CH3 CO2) 2

C O Pb (O2C CH 3)3 (2) C O


Pb (O2C CH 3)2
C OH CH 3CO2H C O

________________________________________________________________________________________________
Chapter 34 permits answers to the following:
(34.1) Six isomeric cyclohexanediols exist (structural and configurational). Draw all of them; look at Table 20.1 for
reference.
(34.2) Formulate a synthesis of (a) trans- and (b) cis-1,2-cyclopentanediol from cyclopentene.
(34.3) Write equations to describe a synthesis of glycerol from propene.
(34.4) What product is formed by reacting magnesium with acetone (propanone)? Write the equation.
(34.5) Formulate the cleavage of an ,-diol with lead tetraacetate. Which products (formulas) do you expect from
the oxidative cleavage of (a) 1,2-cyclobutanediol, (b) 1,2-cyclopentanediol, and (c) 1,2-cyclohexanediol?
90 35 Reactions of Alcohols

35 Reactions of Alcohols
35.1 Basicity and Acidity
Due to the unshared pairs of electrons at the oxygen atom of the hydroxy group, alcohols are electron donors (LEWIS
bases) and proton acceptors. They are protonated by hydrogen halides to give alkyloxonium ions.

_H _H
_
R O + [H ] R O alkyloxonium ion
H

Ethyloxonium chloride is formed by passing hydrogen chloride gas through ethanol in the absence of water which would
regenerate the reactants by hydrolysis.

H3C CH2 OH + HCl H3C CH2 OH 2 Cl ethyloxonium chloride

Alcohols react as OH acids with strongly electropositive metals such as alkali and alkaline earth metals, yielding metal
alkoxides (alcoholates) and hydrogen.
_ _
H3C CH2 OH + Na _ Na + 1/2 H 2
H 3C CH 2 OI (H3C)3C OH + K (H3C)3C OI _ K + 1/2 H 2
ethanol sodium ethanolate (ethoxide) t-butyl alcohol potassium t-butoxide

However, alcohols are much weaker acids than water. Therefore, alkoxides are readily hydrolyzed in water, yielding
alcohols and metal hydroxides.
_
R OI
_ + H2O R OH + OH

35.2 Oxidation
In the presence of oxidation reagents, primary alcohols are oxidized to carboxylic acids by way of aldehydes. Ketones are
the oxidation products of secondary alcohols.
O O R R
+ 1/2 O2
R CH 2 OH R C R C CH OH C O
 2 [H ] ,  2 e0  2 [H ] ,  2 e0
H OH R R
primary alcohol aldehyde carboxylic acid secondary alcohol ketone

The aldehyde can be isolated if removed continuously by distillation before further oxidation to the carboxylic acids oc-
curs. Ecologically undesirable oxidation reagents include alkaline permanganate (KMnO4), dichromate (K2Cr2O7) in
concentrated sulfuric acid (JONES reagent), and pyridinium chlorochromate (COLLINS reagent). Oxidation by air oxygen in
the presence of copper metal as catalyst at elevated temperatures is more environment-friendly. Cuprous and cupric
oxide (Cu2O and CuO) transfer the oxygen and are reduced to copper by the alcohol in these catalytic cycles:

+ 1/2 O2

O
R CH2 OH + Cu2O 2 Cu + H2O + R C
H
300 - 500 °C
O
R CH2 OH + CuO Cu + H2O + R C
H
+ 1/2 O2

Efficiently Studying Organic Chemistry: Exam training for chemists, biochemists, pharmacists, life and health scientists,
Third Edition. Eberhard Breitmaier. © 2022 WILEY-VCH GmbH. Published 2022 by WILEY-VCH GmbH.
35.3 Nucleophilic Substitution 91

35.3 Nucleophilic Substitution


Alcohols react with hydrogen halides, yielding alkyl halides by way of nucleophilic substitutions (SN). Phosphorus trihal-
ides (PBr3, PI3) are frequently used as reagents to transfer the halide (Chapter 30.2.3).
NaBr , H 2SO4
reflux
H 3C CH 2 CH 2 CH 2 CH 2 OH + HBr H 3C CH 2 CH 2 CH 2 CH 2 Br + H 2O
1-pentanol 1-bromopentane
HBr gas
OH + HBr Br + H 2O
cyclohexanol bromocyclohexane

35.4 Esterification
Alcohols and mineral or carboxylic acids (Chapter 41) react to give esters and water. This esterification is a dynamic
equilibrium. The reverse reaction involving hydrolysis of the ester is traditionally called saponification because hydrolysis
of fats, which are esters of glycerol and fatty acids, yields soaps (Chapter 74.2.2).
O O
esterification
R OH + HO C R O C + H2O
saponification
R' R'
alcohol carboxylic acid alkyl carboxylate (carboxylic acid ester)

Ethanol and acetic acid equilibrate to provide ethyl acetate (acetic acid ethyl ester); the equilibrium is shifted in favor of the
ester by continuously removing it from the reaction mixture by distillation. The monoethyl ester of sulfuric acid, ethyl
hydrogensulfate, arises from esterification of ethanol with concentrated sulfuric acid.
O O
esterification
H 3C CH 2 OH + HO C H 3C CH 2 O C + H 2O
saponification
CH3 CH 3
ethanol acetic acid ethyl acetate (acetic acid ethylester)
O
esterification
H 3C CH 2 OH + HO SO3H H 3C CH 2 O S OH + H 2O
saponification
O
sulfuric acid ethyl hydrogensulfate (sulfuric acid monoethylester)

Toxic and carcinogenic dimethyl sulfate, (H3CO)2SO2, the dimethyl ester of sulfuric acid used as a methylating reagent, is
produced by distillation of a mixture of fuming sulfuric acid and methanol. Glycerol reacts with nitric acid in the presence
of concentrated sulfuric acid, yielding glyceryl trinitrate (1,2,3-propanetriol trinitrate), better known as nitroglycerin. The
latter name is misleading for an ester of nitric acid, as the nitro groups are attached to oxygen and not to carbon such as
in nitrobenzene. The highly explosive liquid is adsorbed by silica (diatomaceous earth) to give the blasting material known
as dynamite (NOBEL, 1867).
O O
CH2 OH CH2 O N CH2 ONO2 alkyl nitrate O N
O O
(H2SO4)
O
ester of nitric acid
HO CH + 3 HO N N O CH O2NO CH R O
O
O  3 H2O O
CH2 OH CH2 O N CH2 ONO2 O
O nitro compound R N
glycerol nitric acid glyceryl trinitrate, an ester of nitric acid O
________________________________________________________________________________________________
Chapter 35 permits answers to the following:
(35.1) Hydrogen chloride dissolves in cold ethanol. Which reaction takes place? What happens upon heating this
mixture? Look at Chapter 15.1.2.
(35.2) Anhydrous ethanol reacts with sodium metal. Write the reaction equation. What happens upon addition of water?
(35.3) Formulate the oxidation of (a) a primary alcohol (e.g. 1-pentanol) and (b) a secondary alcohol (e.g. 3-pentanol).
(35.4) Formulate the catalytic oxidation of a primary alcohol with copper metal as catalyst.
(35.5) Pure methanol is reacted with (a) acetic acid and (b) fuming sulfuric acid. What products are obtained?
(35.6) The name nitroglycerin (nitroglycerol) is misleading. Draw the formula in order to explain this.
92 36 Dehydration of Alcohols

36 Dehydration of Alcohols
36.1 Dehydration of Alcohols to Alkenes
Whenever protonation of an alcohol by a mineral acid, followed by loss of water (dehydration), generates a sufficiently
stable carbenium ion, both substitution by a nucleophile and  elimination of a proton, yielding an alkene (Chapter
15.1.2), may occur. Since dehydration of the protonated alcohol is the slow and thus rate-determining step in the case of
tertiary and some secondary alcohols, nucleophilic substitution and  elimination are considered monomolecular, follow-
ing the SN1 and E1 mechanism, respectively.

+ X
C C alkyl halide
 H2 O SN1
+ [H ] slow H X
C C C C C C
H OH H OH2 H E1
C C alkene
 [H ]

Which one of these reactions occurs depends on the presence of nucleophiles and the conditions. 2-Methyl-2-propanol (t-
butyl alcohol) undergoes dehydration to give methylpropene (E1) when slightly heated with diluted sulfuric acid. Nucleo-
philic substitution to yield t-butyl chloride, however, occurs upon shaking the alcohol with concentrated hydrochloric acid
and removing the alkyl halide by distillation.
conc. HCl H 3C
+ Cl
H 3C C Cl
H 3C CH 3 SN1
H 3C H3C  H 2O
+ [H ] slow H 3C
H 3C C OH H 3C C OH C
2
H 3C H3C CH2 H H 3C
E1
C CH 2
 [H ]
dil. H2SO4 H 3C

Due to increased stabilization of the intermediate carbenium ions, either by resonance (benzyl and allyl cations) or by
sterically protecting alkyl groups (t-butyl cation), the reactivity of alcohols towards dehydrating reagents such as sulfuric
acid and hydrogen halides increases in the order
primary < secondary < tertiary < benzyl and allyl alcohols.

Thus, dehydration of t-butyl alcohol to methylpropene requires much milder conditions (90 °C, aqueous sulfuric acid 20
%) than the dehydration of 1-butanol to 1-butene (170 °C, sulfuric acid 90 %).
When  protons are available at different  carbon atoms in the carbenium ion, regioselective dehydration to the more
stable alkene takes place. This is the higher alkylated alkene according to the SAYTZEFF rule (Chapter 15.1.1) or the
conjugated compound in the case of dienes and phenylalkenes.

+ [H ]
H 3C CH 2 CH CH 3 H 3C CH2 CH CH 3 H 3C CH CH CH3 and H3C CH 2 CH CH 2
 H 2O  [H ]
OH 2-butene ( E + Z ) 1-butene
2-butanol higher alkylated
major

+ [H ]
CH2 CH CH 3 CH 2 CH CH 3 CH CH CH 3 and CH 2 CH CH 2
 H 2O  [H ]
OH 1-propenylbenzene ( E + Z ) 2-propenylbenzene
1-phenyl-2-propanol conjugated (allylbenzene)
major

Efficiently Studying Organic Chemistry: Exam training for chemists, biochemists, pharmacists, life and health scientists,
Third Edition. Eberhard Breitmaier. © 2022 WILEY-VCH GmbH. Published 2022 by WILEY-VCH GmbH.
36.2 Dehydration of Fully Alkylated 1,2-Diols to Ketones 93

Finally,  elimination of a proton involves the most stable carbenium ion. Whenever dehydration of the protonated alcohol
generates a carbenium ion which can rearrange to a more stable ion, this will occur by anionotropic 1,2-alkyl shift (Chap-
ter 11.6), known as the WAGNER-MEERWEIN rearrangement:
anionotropic 1,2-alkyl shift
(WAGNER-MEERWEIN
R R rearrangement) R R R
+ [H ]
R C CH R R C CH R R C CH R C C
 H 2O  [H ]
R OH R R R R
less alkylated carbenium ion higher alkylated carbenium ion
less stable more stable

WAGNER-MEERWEIN rearrangements changing the carbon skeleton of the reacting alcohol frequently occur with cycloalka-
nols which are fully alkylated at the  carbon atom. For example, 2,2-dimethylcyclohexanol undergoes dehydration to
give the rearranged 1,2-dimethylcyclohexene:
anionotropic 1,2-alkyl shift
(WAGNER-MEERWEIN
OH rearrangement) H
+ [H ] CH 3
CH3 CH3 CH3
 H2O H CH  [H ] CH 3
H 3
CH3 CH 3
2,2-dimethylcyclohexanol 1,2-dimethylcyclohexene

36.2 Dehydration of Fully Alkylated 1,2-Diols to Ketones


Fully alkylated 1,2-diols, known als pinacols (Chapter 34.1.3), undergo acid-catalyzed dehydration to yield ketones with
the involvement of another kind of anionotropic 1,2-alkyl shift known as the pinacol rearrangement.
H3C CH3 H 3C CH3 CH3 O
[H ] [H ]
H3C C C CH3 C C CH 3 CH 3 CH3
 H2O  H 2O
HO OH O CH3 HOOH CH 3
2,3-dimethyl-2,3-butanediol 3,3-dimethyl-2-butanone 1,2-dimethyl-1,2- 2,2-dimethyl-
(pinacol) (pinacolone, t-butyl methyl ketone) cyclohexanediol cyclohexanone

Pinacol dehydration starts with the protonation of one OH group, followed by loss of water (dehydration). The resulting
carbenium ion undergoes an anionotropic 1,2-alkyl shift by way of an intermediate in which the migrating alkyl group
leaves carbon 1 while simultaneously attaching itself to the electron-deficient carbon 2. A protonated ketone arises which
finally deprotonates to provide the rearranged ketone.

R OH + [H ] R OH 2 R
R C C R R C C R R C C R
 H2O
HO R HO R HO R
fully alkylated 1,2-diol
(pinacol) R R R R
1 2
R C C R R C C R C C R
 [H ]
HO R HO R O R
rearranged protonated ketone

________________________________________________________________________________________________
Chapter 36 permits answers to the following:
(36.1) Write the mechanism of alcohol dehydration.
(36.2) Which one of the alcohols (a) t-butyl alcohol or (b) 1-butanol undergoes dehydration more readily? Why?
(36.3) Three products are expected from the dehydration of 2-pentanol. Which one will be a minor product?
(36.4) What product is obtained by the acid-catalyzed dehydration of 3,3-dimethyl-2-butanol? Formulate the mechanism.
(36.5) Suggest a method to prepare t-butyl methyl ketone from acetone? Which reactions are needed?
94 37 Ethers

37 Ethers
37.1 Nomenclature
Formally, ethers are dialkyl derivatives (ROR) of water (H2O). Following IUPAC rules, they are named as alkoxy
derivatives of hydrocarbons (Table 7.1). In the common system, the suffix ether is added to the names of both alkyl or
aryl groups attached to oxygen (dialkyl ethers). The names of cyclic ethers can be derived from the cycloalkane (oxacyc-
lopentane) or from the common name of a heterocycle (tetrahydrofuran).
CH3
O O
H3C O CH3 H3C CH2 O CH2 CH3 H3C O CH CH3 O CH3 O CH3

dimethyl diethyl isopropyl methyl cyclopropyl methyl methyl phenyl ether tetrahydrofuran furan

37.2 Structure and Physical Properties


As illustrated by the molecular models of dimethyl ether (Fig. 37.1), ethers are angular molecules, possessing COC
bond angles very close to the tetrahedron angle of 109°28´ (sp3 hybridization of the O atom).

Fig. 37.1. Molecular models (tube, ball-spoke, space-filling) of dimethyl ether

With no OH bonds, in contrast to alcohols, ethers cannot associate by hydrogen bridging (Chapter 33.2). Consequently,
their boiling points are considerably lower than those of alcohols with comparable molecular size. Examples are diethyl
ether (b.p. 34.2 °C) and 1-butanol (b.p. 118 °C at 1011 mbar), both having the same molecular formula C4H10O and
respresenting functional isomers.

37.3 Preparation
37.3.1 Bimolecular Dehydration of Alcohols
Acid-catalyzed bimolecular dehydration of alcohols is a common method to prepare symmetrical ethers. Thus, diethyl
ether is produced by heating ethanol with concentrated sulfuric acid (byproduct: ethene).
H 2SO4 , 140 °C
2 H 3C CH 2 OH H 3C CH 2 O CH 2 CH 3
 H2O
ethanol diethyl ether

Bimolecular dehydration involves nucleophilic substitution of H2O in the alkyloxonium ion arising from protonation of one
alcohol molecule by a second alcohol molecule as nucleophile:
R OH + [H ]

_ R R
SN 2
R O
_ + R OH
_ 2 R O R O
 H2O  [H ]
H H
alcohol alkyloxonium ion dialkyloxonium ion dialkyl ether

37.3.2 Alkoxylation of Alkyl Halides (WILLIAMSON Synthesis)


Sodium alkoxides (alcoholates) as nucleophiles readily substitute the halide anion in alkyl halides:
_  
R'
SN
R OI
_ Na + X R' R O + Na X
sodium alkoxide alkyl halide dialkyl ether
(alcoholate) X = Cl , Br , I

Efficiently Studying Organic Chemistry: Exam training for chemists, biochemists, pharmacists, life and health scientists,
Third Edition. Eberhard Breitmaier. © 2022 WILEY-VCH GmbH. Published 2022 by WILEY-VCH GmbH.
37.4 Typical Reactions 95

This nucleophilic substitution, known as the WILLIAMSON synthesis, opens access to a large variety of ethers:
_
_ OI
_ Na Br O
(H3C)2CH OI_ Na + I CH 3 (CH 3)2CH O CH3 +
NaI  NaBr
sodium isopropoxide isopropyl methyl ether
(from isopropyl alcohol) sodium phenoxide allyl bromide allyl phenyl ether
(from phenol)

37.4 Typical Reactions


37.4.1 Formation of Oxonium Salts
With unshared electron pairs at their oxygen, ethers are LEWIS bases, reacting with proton donors and electron-deficient
compounds such as boron trifluoride and other LEWIS acids. Dialkyloxonium compounds are formed.
R R R R
O + HCl O
_ H Cl O + BF 3 O
_ BF3
R R R R
dialkyl ether dialkyloxonium chloride boron trifluoride boron trifluoride etherate

37.4.2 Autoxidation (Insertion of Oxygen)


Ethers, particularly those with branched alkyl groups such as isopropyl, readily undergo radical insertion of a peroxy
group with biradicalic air oxygen. The resulting hydroperoxides are explosive. This autoxidation can be prevented by
storing ethers over reducing reagents (ferrous salts), with protection from light in dark bottles.

_air _ +R O C H
h + O
_ O _
R O C H R O C R O C O O R O C O OH
H
 R O C
ether peroxy radical ether hydroperoxide

37.4.3 Cleavage of Ethers


Hydrogen halides, particularly HI and HBr, cleave ethers to give an alcohol and an alkyl halide:
R
reflux
O CH 3 + HI R OH + CH3 I

Protonation of the oxygen initiates the mechanism, by providing a dialkyloxonium ion. Depending on the stability of the
intermediate carbenium ion R, nucleophilic substitution by the halide anion follows the SN1 mechanism (ethers cleaving
to intermediate tertiary or otherwise stabilized carbenium ions), or proceeds via the SN2 pathway (ethers with primary
alkyl groups).

_ X
H SN1
O
_ R + HX O
_ R R carbenium ion
R R ROH
ether + X + X

H SN2
transition state O R R X
ROH
X R alkyl halide

________________________________________________________________________________________________
Chapter 37 permits answers to the following:
(37.1) Formulate the synthesis of diethyl ether from ethanol. Which reaction, producing ethene, competes?
(37.2) Write equations to suggest the preparation of (a) ethyl phenyl ether and (b) allyl propyl ether.
(37.3) Ethers should be stored protected from light and air. If not, which reaction may take place?
(37.4) What products are expected from the cleavage of (a) diisopropyl ether and (b) diallyl ether with hydrogen iodide?
96 38 Amines

38 Amines
38.1 Nomenclature, Classification
Amines are alkyl- or aryl-substituted derivatives of ammonia. They are classified as primary, secondary and tertiary
amines accounting for one, two, or three alkyl or aryl groups attached to the nitrogen. Fully alkylated ammonium salts
(tetraalkylammonium salts) are called quaternary. Cyclic amines such as piperidine (azacyclohexane) and its N-methyl
derivative are secondary and tertiary amines belonging to the heteroalicyclic class of compounds (Chapter 60).
NH 3 R NH2 R NH R R N R R4N X cyclic amines (nitrogen heteroalicycles)
R
ammonia primary secondary tertiary amine quaternary secondary tertiary
ammonium salt N N
H CH3
NH2 NHR NR2
primary secondary tertiary amino group piperidine N-methylpiperidine

In keeping with IUPAC rules, amines are named as amino derivatives of the hydrocarbons (aminomethane, methana-
mine, or methylamine for CH3NH2). Often, common names are preferred, which list the number (di-, tri-, tetra- for two,
three, four identical residues), and the name of the groups attached to nitrogen in alphabetical order, followed by the
ending amine.

H3C NH2 H3C NH CH 3 (H3C)3N (H3C)3N I CH3 CH2 CH2 CH2 NH CH 3


methyl- dimethyl- trimethylamine tetramethylammonium iodide n-butylmethylamine
(methanamine) (N-methylmethanamine) (N,N-dimethylmethanamine) (N-methylbutanamine)

Aminobenzenes are better known as anilines (Chapter 24.1), and regioisomeric aminotoluenes as toluidines:
NH 2 NHCH 3 N(CH 3)2 N(CH 3)2 NH 2 NH 2 NH 2

O2N CH 3 H 3C
CH 3
aniline N-methylaniline N,N-dimethylaniline p-nitro- o- m- p-toluidine
(aminobenzene) N,N-dimethylaniline (2-) (3-) (4-methylaniline)

Di- and polyamines such as ethylenediamine and diethylenetriamine, containing amino groups attached to two adjacent
carbon atoms, are bi- and multidentate ligands for metal cations, forming deeply colored metal chelates (Chapter 51.2.4)
with them. Diaminobenzenes are known as phenylenediamines.
CH 2 CH 2 NH 2 NH 2 NH 2 NH 2
H 2N CH 2 CH 2 NH 2 HN
CH 2 CH 2 NH 2 NH 2 H2N
NH 2
ethylenediamine diethylenetriamine
o- m- p-phenylenediamine
(1,2-diaminoethane) [bis(2-aminoethyl)amine]
(1,2-) (1,3-) (1,4-diaminobenzene)

38.2 Molecular Shape


Amines have a pyramidal shape (Fig. 38.1). The CNC bond angle of trimethylamine is 108° close to the tetrahedral
angle of 109°28'. Thus, sp3 hybrid orbitals of nitrogen overlap to form three  bonds with attached carbon and hydrogen
atoms in the molecular orbital model; the non-bonding electron pair occupies the remaining fourth sp3 hybrid orbital (Fig.
38.1).

N N  N 
H3C 108° CH3 H 3C CH 3

CH3 CH3
pyramidal shape sp3 hybrid orbitals  bonds originating from N
and non-bonding electron pair

Fig. 38.1. Models (tube, ball-spoke, space-filling) of trimethylamine; sp3 hybridization of the N atom accounting for the pyramidal shape

Efficiently Studying Organic Chemistry: Exam training for chemists, biochemists, pharmacists, life and health scientists,
Third Edition. Eberhard Breitmaier. © 2022 WILEY-VCH GmbH. Published 2022 by WILEY-VCH GmbH.
38.3 Preparation 97

38.3 Preparation
38.3.1 Alkylation of Ammonia
Alkyl halides undergo nucleophilic substitution (SN) with ammonia. In four steps, ammonia is alkylated to primary, sec-
ondary, tertiary amines, and tetraalkylammonium salts. A mixture of all for products is obtained.
R R
  + OH
C + INH3 X C NH3 R CH2 NH3 X R CH2 NH2
X H  H2O ,  X
H H H
alkyl halide alkylammonium salt primary amine

+ OH
R NH2 + R X R2NH2 X R2NH
 H2 O ,  X
secondary amine
+ OH
R2NH + R X R3NH X R3N
 H2 O ,  X tertiary amine
R3N + R X R4N X
tetraalkylammonium salt (quaternary ammonium salt)

38.3.2 Primary Amines by Alkylation of Potassium Phthalimide


Pure primary amines are prepared from alkyl halides and potassium phthalimide (GABRIEL synthesis), involving nucleo-
philic substitution of the halide anion by the phthalimide anion. Cleavage of the resulting phthaloylamines either by aque-
ous hydrochloric acid or by hydrazine hydrate liberates the primary amines.

+ 2 H2O , + HX
CO2H
O O + X H 3N R
heat, pressure
CO2H alkylammonium salt
N K + R X N R phthalic acid
 KX O
60-80°C
O O + H2N NH2 NH
potassium phthalimide phthaloylamine + H 2N R
NH
primary amine
O
phthalhydrazide

38.3.3 Primary Amines by Reduction of Nitriles and Nitro Compounds


Catalytic hydrogenation or reduction of nitriles with reducing reagents (lithium aluminum hydride) yields primary amines.
Industrial production of aniline involves reduction of nitrobenzene with iron (scrap).

CH 3 CH 3
+ 4 [H ] + 4 e0 H2SO4
C N CH 2 NH 2 4 NO2 + 9 Fe + 4 H 2O 4 NH 2 + 3 Fe3O4

2-methylbenzonitrile 2-methylbenzylamine nitrobenzene aniline

________________________________________________________________________________________________
Chapter 38 permits answers to the following:
(38.1) Draw structural formulas of all isomeric amines with the molecular formula C5H13N and classify them.
(38.2) Explain the geometry of the trimethylamine molecule. Where is the non-bonding electron pair?
(38.3) Alkylation of ammonia is not a clean method to prepare amines. Why?
(38.4) N-Methylpiperidine (Chapter 38.1) reacts with iodomethane. Formulate the reaction. What product is obtained?
(38.5) Suggest a method to prepare pure n-butylamine.
(38.6) How would you prepare benzylamine (-aminotoluene) from (a) benzonitrile and (b) benzyl bromide?
98 39 Reactions of Amines

39 Reactions of Amines
39.1 Basicity of Alkyl- and Arylamines
Since their nitrogen atom provides an unshared electron pair, ammonia and amines act as bases, accepting protons and
forming ammonium ions. As shown for trimethylamine, the unshared electron pair which occupies an sp3 hybrid orbital
overlaps with the empty s orbital of a proton abstracted from water, forming a new NH bond in a trimethylammonium ion
with tetrahedral shape:
H

 N  + H 2O N + OH
H 3C CH 3 H 3C CH3

CH 3 CH3
trimethylamine trimethylammonium ion

As a result, aqueous solutions of all amines in water are alkaline. The equilibrium constant of this reaction is called the
basicity constant Kb:
_ c(R NH3 ) c(OH )
R NH 2 + H 2O R NH 3 + OH Kb =
c(R NH2)

The basicity of amines depends on the number and kind of substituents attached to nitrogen. Dialkylamines are stronger
bases than alkylamines. Trialkylamines are weaker bases than dialkylamines because crowding of the alkyl groups at
nitrogen sterically hinders protonation. Alkylamines (Kb  104) are stronger bases than arylamines such as aniline (Kb 
109) because the amino nitrogen releases its unshared pair to the benzenoid ring, as indicated by the dipolar resonance
formulas of aniline [()-M effect, Chapter 26.1], making the unshared pair at nitrogen less available for protonation.

I NH 2 I NH 2 NH2 NH2 NH 2 NH 3 NH3


+ [H ]

aniline anilinium ion

39.2 Diazotization of Primary Amines


Nitrous acid converts primary alkyl- and arylamines into diazonium salts (diazotization), which involves an electrophilic
nitrosonium cation (N=O) generated from nitrous acid or dinitrogen trioxide N2O3. The cation attaches itself to the nucle-
ophilic amino function. The intermediate N-nitrosoammonium ion then loses water.
primary amine nitrosonium cation
nucleophile electrophile H O tautomerism H OH tautomerism OH 2
R NH 2 + N O R N N N N N N
H R R

 H2O  NO2  H2O

O H O O
+ [H ]
O N O N O N N N R N N
H H O N R = alkyl or aryl R
alkane- or arenediazonium ion
nitrous acid dinitrogen trioxide resonance formulas

Alkanediazonium salts decompose spontaneously, so that evolution of nitrogen on addition of sodium nitrite in mineral
acid solution detects the presence of a primary alkylamine (VAN SLYKE test tube reaction).

R NH2 R N N X + H 2O N2 + HX + R OH ( + alkene )

alkanediazonium salt (unstable)

Efficiently Studying Organic Chemistry: Exam training for chemists, biochemists, pharmacists, life and health scientists,
Third Edition. Eberhard Breitmaier. © 2022 WILEY-VCH GmbH. Published 2022 by WILEY-VCH GmbH.
39.3 N-Nitrosation of Secondary Amines 99

Arenediazonium ions are resonance-stabilized. Their positive charge is distributed (delocalized) among the benzenoid
carbon atoms ortho and para to the diazonium residue so that three additional resonance formulas can be drawn. There-
fore, arenediazonium salts are sufficiently stable to be isolated in some cases:
NI
N
_ N NI N NI
_ N NI
_ N NI
_ BF4

resonance formulas of the benzenediazonium ion (in the stable tetrafluoroborate)

39.3 N-Nitrosation of Secondary Amines


Secondary amines as nucleophiles undergo N-nitrosation with the electrophilic nitrosonium ion from nitrous acid, yielding
N-nitrosoamines which are yellow and oily carcinogenic compounds.
H3C H 3C O
NaNO2 , HCl
N H + HNO2 N N + H 2O
H3C H 3C
dimethylamine N-nitrosodimethylamine

N-Nitrosation of tertiary alkyl amines yields mixtures of N-nitrosodialkylamines and oxidation products. Tertiary aryla-
mines undergo regioselective electrophilic nitrosation of the benzenoid ring para to the bulky substituent. For example,
nitrosation of N,N-dimethylaniline yields green crystals of p-nitroso-N,N-dimethylaniline:
H3C H3C O
NaNO2 , H2SO4
N H + N O N N
 [H ]
H3C H3C
N,N-dimethylaniline p-nitroso-N,N-dimethylaniline

39.4 Exhaustive Alkylation of Amines


Alkyl halides alkylate primary amines in three steps, finally providing tetraalkylammonium salts (Chapter 38.3.1):
+ RX + RX + RX
R NH 2 R2NH R 3N R 4N X
 HX  HX
primary secondary tertiary amine tetraalkylammonium salt

The last step of N-alkylation to the quaternary ammonium salt is referred to as quaternization or exhaustive alkylation.
Exhaustive methylation of tertiary amines is carried out with an excess of methyl iodide:
CH 3 CH3
 
CH 2 CH2 NI + CH 3 I CH2 CH 2 N CH 3 I
CH 3 methyl iodide CH3
(2-cyclopropylethyl) dimethylamine (iodomethane) (2-cyclopropylethyl) trimethylammonium iodide

39.5 HOFMANN Elimination of Tetraalkylammonium Hydroxides


Ion exchange of tetraalkylammonium halides with the hydroxide anion provided by silver hydroxide (suspension of Ag2O
in water) or an anion exchanger yields tetraalkylammonium hydroxides.

CH 3 ion exchanger CH 3
+ OH
CH2 CH 2 N CH 3 I CH 2 CH2 N CH3 OH
 I
CH 3 CH 3
(2-cyclopropylethyl)trimethylammonium iodide (2-cyclopropylethyl)trimethylammonium hydroxide

Tetraalkylammonium hydroxides are strong bases, comparable with alkali hydroxides, and stable at room temperature.
When heated, those containing a hydrogen atom  to the ammonium nitrogen undergo a bimolecular elimination (E2),
100 39 Reactions of Amines

yielding an alkene, a tertiary amine (usually trimethylamine) and water. This reaction is known as the HOFMANN elimina-
tion and provides an additional method to prepare alkenes (Chapter 15).
CH 3
> 100 °C
CH CH 2 N CH 3 C CH 2 + N(CH 3)3 + H 2O
H + OH CH 3 H
(2-cyclopropylethyl) trimethylammonium hydroxide ethenylcyclopropane (vinylcyclopropane)

Alkyl groups  to the ammonium nitrogen sterically hinder the hydroxide anion from abstracting the  proton. To con-
clude, a  proton is eliminated from CH3 rather than from RCH2, and from the latter rather than from R2CH (HOF-
MANN rule). For this reason, HOFMANN elimination is regioselective when there is a choice to abstract protons from differ-
ently substituted -positions, as shown for 2-butyltrimethylammonium hydroxide, arising from exhaustive methylation of 2-
butylamine, followed by ion exchange:
150 °C
H3C CH2 CH CH 3 OH H3C CH2 CH CH 2 and H3C CH CH CH 3
 N(CH 3)3
N(CH 3)3  H 2O 1-butene (95 %) 2-butene (cis- and trans-, 5 %)

39.6 Imines from Primary Amines and Carbonyl Compounds


Imines (azomethines, SCHIFF bases), containing carbon nitrogen double bonds, arise from nucleophilic addition of primary
amines to the electrophilic carbonyl carbon atom of aldehydes and ketones (Chapter 49.2). Primarily, an -hydroxyamine
(hemiaminal) is formed; this undergoes acid-catalyzed dehydration to yield the imine, involving an intermediate carbeni-
um-iminium ion:
R R R R R R
_ _ + [H ]
C OI + H2N R C N H C N C N C N C N C N
_
H2 O  [H ]
O
_ H OH H OH2 H H H
aldehyde primary amine -hydroxyamine carbenium-iminium ion imine
or ketone (hemiaminal)

Benzaldehyde and n-hexylamine, for example, react to provide N-hexylbenzaldimine.


benzaldehyde
H H
toluene
C O + H 2N C N
 H2O
n-hexylamine N-hexylbenzaldimine
(1-aminohexane)

39.7 Enamines from Secondary Amines and Carbonyl Compounds


Enamines, containing an amino group attached to a CC double bond, are formed by nucleophilic addition of secondary
amines to the electrophilic carbon atom of aldehydes or ketones (Chapter 49.2). Because no H atom is available at the
dialkylamino nitrogen in the intermediate -hydroxyamine, the proton at the carbon atom adjacentto the OH group is
abstracted upon dehydration to give an enamine.
carbonyl compound R
R R R N R
C C + IN R C C N R C C N C C
 H2O
H O H H O H H OH R enamine
secondary amine -hydroxyamine 1-(N,N-dialkylamino)alkene

1-Piperidinocyclohexene as an enamine is obtained by nucleophilic addition of piperidine to cyclohexanone:

O + HN N N
 H2O
piperidine 1-piperidinocyclohexene

Resonance formulas of the enamine indicate negative polarization (nucleophilicity) of the carbon atom  to nitrogen due
to electron release from the dialkylamino group [-(M) effect]. This nucleophilic carbon atom is able to substitute the
39.8 Reductive Amination of Carbonyl Compounds to Amines 101

halide anion in alkyl halides, as demonstrated by the reaction of 1-piperidinocyclohexene and allyl bromide yielding 2-
allylcyclohexanone upon hydrolysis of the intermediate iminium bromide (STORK enamine alkylation).

 H2O
N + Br N Br O + H 2N Br

1-piperidinocyclohexene allyl bromide iminium bromide 2-allylcyclohexanone

39.8 Reductive Amination of Carbonyl Compounds to Amines


Reductive amination of carbonyl compounds (aldehydes and ketones, Chapters 47-49) opens access to primary, sec-
ondary, and tertiary amines involving imine and enamine intermediates.
Aldehydes and ketones react with nitrogen nucleophiles such as ammonia and primary amines to give imines (Chapter
39.6) and with secondary amines to give enamines (Chapter 39.7). If the reaction is performed under hydrogen pressure
in the presence of a hydrogenation catalyst, the intermediate imines and enamines are hydrogenated to provide the
amines. Primary amines are thus obtained from ammonia, secondary amines from primary amines, and tertiary amines
from secondary amines via enamines. The imino and enamino intermediates can also be isolated and then reduced with
reducing reagents such as complex metal hydrides.
H
+ H2 , Ni , pressure
C O + H3N C NH C NH 2 primary amine
 H2 O
imine
H
+ H2 , Ni , pressure
C O + H 2N R C NR C NHR secondary amine
 H2 O
N-alkylimine
R R
+ H2 , Ni , pressure H
C O + HN C N C NR2 tertiary amine
 H2 O
C R H C R C
H H
H enamine H

Reductive amination of cyclohexanone with ammonia, ethylamine, and diethylamine yields the primary, secondary, and
tertiary aminocyclohexanes (cyclohexylamines):
Ni , 8 bar , 70 °C , ethanol
+ NH3 + H2 NH 2 cyclohexylamine
 H2O
O

Ni , 8 bar , 70 °C , ethanol
+ H 2N C2H5 + H2 N H N-ethylcyclohexylamine
 H2O C2H5
O

Ni , 100 bar , 120 °C N C2H5


+ HN(C 2H5)2 + H2 N,N-diethylcyclohexylamine
O  H2O C2H5

________________________________________________________________________________________________
Chapter 39 permits answers to the following:
(39.1) Aniline is a much weaker base than cyclohexylamine and other alkylamines. Why?
(39.2) Formulate the mechanism of diazotization of primary amines. Explain the stability of arenediazonium ions.
(39.3) Which simple test tube reaction can distinguish between (a) cyclohexylamine and (b) aniline?
(39.4) What products are expected from nitrosation of (a) N-methylaniline and (b) N,N-dimethylaniline with nitrite and acid?
(39.5) Starting from -aminoethylcyclopropane, formulate equations for all steps to synthesize vinylcyclopropane.
(39.6) What products are expected from the reactions of cyclohexanone with (a) ethylamine and (b) diethylamine?
(39.7) Draw resonance formulas of an enamine to explain the nucleophilicity of the carbon  to the nitrogen.
(39.8) Using the concept of reductive amination of carbonyl compounds, draw equations to outline the syntheses of
(a) N-isopropylcyclohexylamine and (b) N,N-diethylcyclohexylamine.
102 40 Diazo and Azo Compounds

40 Diazo and Azo Compounds


40.1 Arenediazonium Salts and Azo Dyes
Arenediazonium salts are formed by diazotization of primary arylamines (Chapter 39.2). They are sufficiently stable to be
kept for reactions such as the reduction to arylhydrazines with sodium sulfite (Na2SO3):
Ar
+ 4 [H ] + 4 e0
arenediazonium salt Ar N NI N
_ NI X Ar NH NH3 X arylhydrazinium salt

Arenediazonium cations are nitrogen electrophiles due to their positively charged diazonium residue, consequently react-
ing with donor-substituted and therefore nucleophilic aromatics to produce arylazo compounds (substituted azobenzenes,
azo dyes). This electrophilic substitution, usually directed to the p-position by the bulky donor substituent (dialkylamino,
hydroxy, and alkoxy groups), is called azo coupling.
Ar
Ar _ N N Ar N _
SE
Ar N NI N
_ NI + H D D N D
H [H ]
arenediazonium cation nucleophilic aromatic intermediate  complex arylazo compound (dye)

The preparation of 4-(N,N-dimethylamino)azobenzene-4´-sulfonic acid from diazotized sulfanilic acid (electrophile) and
N,N-dimethylaniline (nucleophile) is an example:

O3S
O3S NH 3 sulfanilic acid
O3S

+ HNO2  2 H2 O diazotization
N N
azo
_ coupling N N
O3S N2 + N(CH3)2 N(CH 3)2
H
NH(CH 3)2
diazotized sulfanilic acid N,N-dimethylaniline 4-(N,N-dimethylamino)azobenzene-
(zwitterionic) 4'-sulfonic acid (zwitterionic)

The sodium salt of this p,p´-disubstituted azobenzene is known as the indicator methyl orange, which changes color from
yellow (pH > 4.4) to red (pH < 3.1) due to the formation of a resonance-stabilized zwitterion:

O3S O3S O3S

[H ]
N N N N N N
[H ] H H

IN(CH3)2 IN(CH3)2 N(CH3)2


benzenoid resonance formula quinoid resonance formula
yellow anion red zwitterion
pH > 4.4 pH < 3.1

Similar to alkenes with CC double bonds, azobenzenes with NN double bonds exist as configurational isomers (cis- and
trans-) with different physical properties (melting point, light absorption max, Chapter 66.1). Azo coupling reactions yield
trans-azobenzenes which isomerize to cis-isomers when irradiated with UV light.

trans-azobenzene h cis-azobenzene
 = 0 Debye ; m.p. 68 °C N N N N  = 3.0 Debye ; m.p. 71.5 °C
max = 330 nm max = 255 nm

Efficiently Studying Organic Chemistry: Exam training for chemists, biochemists, pharmacists, life and health scientists,
Third Edition. Eberhard Breitmaier. © 2022 WILEY-VCH GmbH. Published 2022 by WILEY-VCH GmbH.
40.2 Azoalkanes 103

The preformed nitrogen molecule in arenediazonium salts facilitates nucleophilic and radicalic substitutions. Thermolysis
of benzenediazonium tetrafluoroborates yields fluorobenzenes (BALZ-SCHIEMANN reaction, SN). Substitution by (cuprous)
cyanide to give benzonitriles (SANDMEYER reaction) involves aryl radicals.

R + CuCN R + HF , + BF3 R heat R


CN N NI Cl N NI BF 4 F
N2 ,  CuCl HCl N2 ,  BF3
benzonitriles benzenediazonium tetrafluoroborates fluorobenzenes

40.2 Azoalkanes
Similar to alkenes and azobenzenes, azoalkanes and their derivatives with the general formula RN=NR exist as cis-
and the more stable trans-isomers:

_ _ _ R
N N cis- N N
_ trans-azoalkane
R R R

They are prepared by oxidation of 1,2-disubstituted hydrazines, exemplified by the formation of diethyl azodicarboxylate
(DEAD) from diethyl hydrazine-1,2-dicarboxylate:
R H5C 2O2C H H 5C 2O2C
R NH NH R N N N N N N
 2 [H ] ,  2 e0  2 [H ] ,  2 e0
R H CO2C 2H5 CO2C 2H 5
dialkylhydrazine azoalkane diethyl hydrazine-1,2-dicarboxylate diethyl azodicarboxylate

Azobisisobutyronitrile (AIBN) is prepared in three steps from acetone, via cyanohydrin formation (Chapter 49.3.2), hydra-
zinolysis of the cyanohydrin, and oxidation of the resulting hydrazine with bromine:
H3C H3C
CN CN
H3C + 2 HCN + H2NNH2 C H + Br 2 (OH ) C
H3C CN H3C H3C
2 C O 2 C N N N N
H3C OH  2 H2O CH3  2 HBr CH3
H3C H
C C
NC NC
CH3 CH3
acetone acetone cyanohydrin 1,2-bis(2-cyano-2-propyl)hydrazine azobisisobutyronitrile

Non-aromatic diazonium ions and diazoalkanes readily decompose by elimination of nitrogen (Chapters 39.2, 40.3),
leaving intermediate carbenium ions and carbenes. Azoalkanes are more stable, generating alkyl radicals at elevated
temperatures by homolysis of their carbon-nitrogen bonds. Azomethane (R = CH3) is cleaved at 400 °C, and azobisisobu-
tyronitrile below 100 °C due to steric protection of the resulting radicals by methyl groups and delocalization of the un-
paired electron by resonance.
H3C
CN
R C H 3C H3C
heat H 3C > 60 °C
N N 2R + N2 N N C C NI C C NI
CH 3  N2
R C H 3C H3C
NC
CH 3

Therefore, azobisisobutyronitrile is used as a precursor to generate the radicals initiating polymerizations of alkenes
(Chapter 58.2).

40.3 Diazoalkanes
Diazoalkanes (R2C=N2) are precursors of carbenes which, upon generation, undergo subsequent reactions of preparative
significance. Their bonding state is described by three 1,2- and 1,3-dipolar canonical formulas:
R R R
_ _ _
resonance formulas of diazoalkanes
C N NI C N NI C N NI
_
( R = H , alkyl , aryl )
R R R
1,3- 1,2- 1,2-dipolar
104 40 Diazo and Azo Compounds

Diazomethane (R = H) is prepared from precursors containing an N-methyl-N-nitroso unit such as N-methyl-N-nitroso-p-


toluenesulfonamide. Diazomethane explodes in its pure state; therefore, preparative work is performed with ether solu-
tions.
H
_
H 2C N H 2C N
N O N OH OC 2H5
tautomerism + OC2H 5 _
O S O O S O O S O + H2C N NI
 HO
diazomethane

N-methyl-N-nitroso-
CH3 p-toluenesulfonamide CH 3 CH 3

Reactions of diazoalkanes and carbenes for synthetic purposes include four basic types:

- Carbene Insertions
Driven by the formation of the stable nitrogen molecule N2, diazomethane generates the intermediate carbene (meth-
ylene, :CH2 or ICH2, Chapter 10.3) upon heating or irradiation with UV light (thermolysis or photolysis). Reactive carbene
inserts itself into CH bonds. This reaction homologizes alkyl chains.
R H + ICH 2 R CH 2 H

h or heat  N2

ICH 2 N NI

- Carbene Cycloadditions ([2+1]-Cycloadditions)


Generated in situ by thermolysis or photolysis of diazomethane, carbene as the C1 reactant undergoes [2+1]-
cycloaddition with alkenes and alkynes as the C2 reactants, forming cyclopropanes and cyclopropenes.

C C
+ ICH2 CH 2 + ICH 2 CH 2
C C
alkene (C2) carbene (C1) cyclopropane alkyne cyclopropene

Cyclopropanations with carbene permit attractive syntheses, exemplified by the [2+1]-cycloaddition of carbene to ben-
zene, primarily yielding norcaradiene which undergoes COPE rearrangement (Chapters 11.6, 21.5) to give 1,3,5-cyclo-
heptatriene.
CH2N 2
+ ICH2 CH2 CH 2

bicyclo[4.1.0]hepta-2,4-diene (norcaradiene) 1,3,5-cycloheptatriene

-Alkylation of Acidic Compounds


Compounds containing sufficiently acidic hydrogen, RYH, react with diazomethane to give the methanediazonium ions
(as salts) which undergo elimination of nitrogen to yield the Y-methylated derivatives:

R Y H + ICH2 N NI R YI + CH 3 N NI methanediazonium ion

 N2

R YI + CH 3 R Y CH 3

O-Methylation of carboxylic acids and phenols proceeds spontaneously:


carboxylic acid R CO2H + CH2N 2 R CO2CH 3 methyl carboxylate (ester)
 N2
phenol OH + CH2N 2 OCH 3 anisole (methyl phenyl ether, an aryl methyl ether)
 N2
40.3 Diazoalkanes 105

Alkylation of alcohols and amines, however, requires the presence of a LEWIS acid such as boron trifluoride to acidify the
OH or NH group.
R R
BF3 BF3
R OH + CH2N2 R OCH 3 alkyl methyl ether N H + CH2N2 N CH 3 N-methylamine
 N2  N2
R R

- Nucleophilic Addition to Carbon-Heteroatom Multiple Bonds


As nucleophilic part of the molecule, the carbanionic carbon atom of diazomethane adds to the polarized carbon atom of
carbon-heteroatom multiple bonds (carbonyl and imino groups).

Y YI
R C + ICH2 N2 R C CH2 N2
R' R'

Nitrogen is removed from the dipolar diazo compound. The resulting intermediate is stabilized either by 1,2-alkyl shift,
regenerating the CY double bond, or by formation of a three-membered heterocyclic ring (an oxirane for Y = O, an azirane
for Y = NR).
YI YI 1,2-alkyl shift YI
R C CH 2 N 2 R C CH 2 R C CH 2 R´
 N2
R´ R´
cyclization

Y Y
R C CH 2 C
R CH 2 R´

Homologizations of aldehydes (Chapter 47) and ketones including cycloalkanones (Chapter 48) by diazomethane in
competition with the formation of oxiranes (epoxides, Chapter 60) are examples.
_
O IOI O
R C + ICH 2 N 2 R C CH 2 N 2 C
 N2 R CH 2 R'
R' R'
ketone or aldehyde  N2
homologized ketone ( R' = alkyl )
( R' = alkyl or H ) homologized aldehyde ( R' = H )
O
R C CH 2 oxirane
R'

________________________________________________________________________________________________
Chapter 40 permits answers to the following:
(40.1) Outline the mechanism of azo coupling of an arenediazonium chloride with a donor-substituted benzene.
(40.2) Outline a synthesis of methyl orange and explain the function of this compound as a pH indicator.
(40.3) Which steps are necessary to synthesize phenylhydrazine from benzene as starting material? Write equations.
(40.4) Formulate all equations to describe the preparation of azobisisobutyronitrile (AIBN) from acetone.
(40.5) What happens upon heating AIBN?
(40.6) Write an equation to outline a preparation of diazomethane.
(40.7) Which resonance formulas describe the bonding state of diazomethane and which one of these explains
the thermolysis to generate carbene?
(40.8) Formulate the reaction of diazomethane with (a) phenol, (b) 1-butanol, and (c) benzene.
106 41 Carboxylic Acids

41 Carboxylic Acids
41.1 Survey, Nomenclature
Carboxylic acids contain a carboxy group (COOH or CO2H). The IUPAC name of a carboxylic acid arises from its
longest continuous carbon chain containing the carboxy group which is given the position 1 in the chain. The ending e of
the corresponding alkane, alkene, or alkyne is replaced by oic acid for monocarboxylic acids and by edioic acid for
acids with two carboxy groups (Table 41.1).
Frequently used common names of carboxylic acids (Table 41.1) are related to their natural origin. Selected examples
are formic acid from the secretion of ants (Lat. formica = ant), acetic acid from vinegar (Lat. acetum = vinegar), butyric acid
from rancid butter (Lat. butyrum = butter), valeric acid from valerian (Lat. valeriana), caproic acid from goat fat (Lat. caper
= goat), benzoic acid from gum benzoin (Lat. benzoe) and salicylic acid from willow (Lat. salix).

Table 41.1. Carboxylic acids (alkanoic, alkenoic, alkynoic, benzoic, and dicarboxylic acids)
alkanoic acid alkenoic acid alkynoic acid
O
methanoic
H C (formic)
OH
O
ethanoic
CH 3 C (acetic)
OH HO
O H C O O
propanoic propenoic propynoic
CH 3 CH 2 C C C (acrylic) H C C C (propiolic)
(propionic)
OH H H OH
HO
O H C O O
butanoic (E)-2-butenoic 2-butynoic
CH 3 CH 2 CH 2 C (butyric) C C H 3C C C C
(crotonic)
OH H 3C H OH
O
pentanoic
CH 3 CH 2 CH 2 CH 2 C (valeric)
OH
O
6 5 4 3 2 1 hexanoic
CH 3 CH 2 CH 2 CH 2 CH 2 C (caproic)
OH
dicarboxylic acids
O O O O O O H CO2H H H O O
C C C CH 2 C C CH 2 CH2 C C C C C C C C C
HO OH HO OH HO OH HO2C H HO2C CO2H HO OH
ethanedioic propanedioic butanedioic (E)-butenedioic (Z)-butenedioic butynedioic
(oxalic) (malonic) (succinic) (fumaric) (maleic)

a r e n e c a r b o x y l i c a c i d s = b e n z o i c a c i d s (aromatic and heteroaromatic mono- and dicarboxylic acids)


O O O
C C C CO2H CO2H CO2H
OH OH OH

OH H 3CO CO2H HO2C


CO2H
benzoic o-hydroxybenzoic p-methoxybenzoic m-
o- p-
(salicylic) (p-anisic) benzenedioic
phthalic isophthalic terephthalic

41.2 Carboxy Group: Bonding and Resonance Formulas


Carboxy groups have planar shape (Fig. 41.1). The CO single bond in the carboxy group (136 pm) is shorter than in
methanol (143 pm), and the CO double bond (123 pm) is longer than in formaldehyde (HCHO, 120 pm). Thus, the

Efficiently Studying Organic Chemistry: Exam training for chemists, biochemists, pharmacists, life and health scientists,
Third Edition. Eberhard Breitmaier. © 2022 WILEY-VCH GmbH. Published 2022 by WILEY-VCH GmbH.
41.3 Carboxylic Acid Dimers 107

carboxy group neither has a pure CO double bond nor a pure CO single bond, and this situation is represented by two
resonance formulas indicating delocalization of the CO double bond. Perfect delocalization of this  bond is realized in
the carboxylate anion (CO2) of the salts of carboxylic acids so that the CO single and the CO double bond are equal in
length (127 pm), as shown by X-ray diffraction analysis of sodium formate (the sodium salt of formic acid).

O O O
carboxy group: resonance formulas C C or C
O H O H O H


O
123 pm
C C 120°
136 pm
O H

Fig. 41.1. Molecular models (tube, ball-spoke, space-filling) and bond data of acetic acid

41.3 Carboxylic Acid Dimers


Carboxylic acids boil at significantly higher temperatures than alcohols of comparable molecular mass. Molecular weight
determinations of lower carboxylic acids by the vapor density method give twice the formula weight because such acids
exist as dimers, strongly held together by two hydrogen bridges, even in the vapor state. In aqueous carboxylic acid
solutions, hydrogen bridges are formed between dissolved acid and water molecules.

O H O O H O
carboxylic acid hydrogen bridges between
R C C R O C H
dimer carboxylic acids and water O H
O H O H R
O C
R

41.4 Preparation
41.4.1 Carbonylation
The introduction of the carboxy group by insertion of carbon monoxide is referred to as carbonylation.

- Carbonylation of hydroxide yields a formate, the salt of formic acid:


100 °C, pressure
HO Na + CO H CO2 Na sodium formate

- Carbonylation of alcohols yields alkanecarboxylates, the salts of alkanoic acids:


100 °C, pressure
RO Na + CO R CO2 Na sodium carboxylate

- Catalytic carbonylation and hydration of alkenes is an industrial method used to produce alkanoic acids:
250 °C, Ni(CO)4 , 150 bar
H 2C CH 2 + CO + H 2O H 3C CH 2 CO2H
ethene propanoic acid (propionic acid)

41.4.2 Carboxylation
As shown by resonance formulas, the carbon atom of carbon dioxide is electrophilic. It therefore reacts with carbon
nucleophiles such as organometal compounds or nucleophilic aromatics, introducing the carboxy group. These reactions
involve electrophilic carboxylations.
108 41 Carboxylic Acids

- Electrophilic carboxylation of alkynylides (Chapter 19.3.5) produces alkynoic acids:


O O O
sodium salt of an
R C CI Na + C C R C C C alkynoic acid
sodium alkynylide O O O Na

- Electrophilic carboxylation of alkylmagnesium halides (GRIGNARD reagents) yields alkanoic acids:


(CH3)3C Br

+ Mg ether

CH 3 O O CH3 O CH 3
+ H2 O
H3C C MgBr + C C H 3C C C H3C C CO2H
O  Mg2 ,  OH ,  Br
CH 3 O CH3 OMgBr CH 3
t-butylmagnesium 2,2-dimethylpropanoic acid
bromide (pivalic acid)

- Electrophilic carboxylation of phenoxide, obtained from phenol and sodium hydroxide (Chapter 52.2), affords the
sodium salt of salicylic acid (KOLBE-SCHMITT carboxylation):
_ _ _ _ _ _
IOl IO IO IO IO IOI
H
O O C
O
Na + C C Na
O O
resonance formulas of the phenoxide anion electrophile
nucleophile
+ NaOH  H2O

_
IOH OH
CO2 Na
phenol sodium salicylate

41.4.3 Oxidation of Methyl, Hydroxymethyl, and Aldehyde Groups


Catalytic oxidation of methyl and ethyl groups attached to benzenoid rings by air produces substituted benzoic acids.
Co , Pb , Mn(II)acetate , 150-180 °C
Cl CH 2 CH 3 + 3 O2 Cl CO2H + CO2 + 2 H 2O

p-chloroethylbenzene p-chlorobenzoic acid

Primary alcohols undergo oxidation to give carboxylic acids via the intermediate aldehydes:
CH 3 CH 3 O CH 3 O
MnO4 , OH + 1/2 O2
H 3C CH 2 CH CH 2 OH H 3C CH 2 CH C H 3C CH 2 CH C
 2H ,  2 e0
H OH
2-methyl-1-butanol 2-methylbutanal 2-methylbutanoic acid
(-methylbutyric acid)

Strong oxidation reagents oxidize cycloalkenes, opening the ring to provide dicarboxylic acids:
MnO4 , OH CO2H
+ 2 O2
CO2H
cyclohexene adipic acid (hexanedioic acid)

41.4.4 Hydrolysis of Carboxylic Acid Derivatives


All carboxylic acid derivatives (Chapter 42), such as carboxylic acid amides (Chapter 42.4), undergo hydrolysis to give
carboxylic acids. Amides can be prepared by hydrolysis of nitriles (alkyl or aryl cyanides RCN) in aqueous acids.
41.5 Acidity 109

Nitriles are formed by the KOLBE synthesis, involving nucleophilic substitution (SN) of halide in alkyl halides by (potassium)
cyanide as nucleophile.
O O
+ CN + H2O (H3O ) + H2O
R X R C N R C R C
 X NH2  NH3 OH
alkyl halide nitrile
( X = Cl, Br, I ) carboxylic acid amide carboxylic acid

41.5 Acidity
In aqueous solution, carboxylic acids equilibrate with water to give the carboxylate anion (the anion of the salts which are
named carboxylates) and the hydronium cation, H3O, thus producing an acidic reaction:
R CO2H + H2O R CO2 + H3O
carboxylic acid carboxylate anion

The equilibrium constant Ka (ionization constant, acidity constant) is defined as the quotient of the product of the concen-
trations of the ions to the concentration of the acid:
c (RCO2 ) c (H3O ) lg Ka = lg c(RCO2 ) + lg c(H3O ) lg c(RCO2H)
Ka = or in logarithmic form:
c(RCO2H) pKa = 2 pH lg c(RCO2H) because c(RCO2 ) = c(H3O )

With acidity constants of Ka  105,


carboxylic acids are rather weak, a 1 N aqueous carboxylic acid solution [c(RCO2H) = 1,
one normal, one molar] thus setting up a pH of about 2.5 as calculated for this equilibrium constant (Ka  105, lg Ka   5,
pKa  5). Halogens and other electronegative substituents  to the carboxy group increase the acidity, stabilizing the
carboxylate anion by inducing a positive charge at the -position, as shown for chloroacetic acid. Acceptor substituents
ortho or para to the carboxy group increase the acidity of a benzoic acid due to their ()-M effect, exemplified by p-
nitrobenzoic acid. Donor substituents decrease acidity due to their ()-M effect, as demonstrated by p-hydroxybenzoic
acid.
5 5 5 5 5
Ka = 1.7 x 10 136 x 10 Ka = 6.8 x 10 2.9 x 10 40 x 10
O O O O
OH OH N N
O  
O
CH 3 C Cl CH 2 C
O O
acetic acid chloroacetic acid: C C C C C
(acetate anion) ()-I effect stabilizes O O O O O O O O O O
the chloroacetate anion benzoic acid p-hydroxybenzoic acid p-nitrobenzoic acid
(benzoate anion) ()-M effect destabilizes ()-M effect stabilizes
the p-hydroxybenzoate anion the p-nitrobenzoate anion

Dissociation of dicarboxylic acids occurs in two steps. Two acidity constants Ka1 and Ka2 are therefore measurable, as
demonstrated by oxalic acid:
5
(1) HO2C CO2H + H 2O HO2C CO2 + H 3O Ka1 = 5400 x 10
5
(2) HO2C CO2 + H 2O O2C CO2 + H 3O Ka2 = 5.2 x 10

The acidity constant of the second step Ka2 is significantly smaller than Ka1 of the first step (Ka1 > Ka2) because more
energy is required to separate a proton from an anion than from a molecule.
________________________________________________________________________________________________
Chapter 41 permits answers to the following:
(41.1) Draw the structural formula of the analgesic and antipyretic 2-(4-isobutylphenyl)propanoic acid (ibuprofen).
(41.2) Explain why the CO single bond of a carboxylic acid is shorter than the same bond in an alcohol.
(41.3) Formulate equations to describe three general methods to prepare carboxylic acids without starting from CO2.
(41.4) Write equations to suggest the preparations of carboxylic acids, starting from CO2.
(41.5) Calculate the pH value of a 1 N carboxylic acid with Ka  105 using the acidity constant formula.
(41.6) Arrange (a) acetic and chloroacetic acid, and (b) benzoic, p-hydroxybenzoic, and p-nitrobenzoic acid in order of
increasing acidity and explain your reasoning.
110 42 Carboxylic Acid Derivatives

42 Carboxylic Acid Derivatives


The hydroxy group of the carboxy function is substituted by other groups in carboxylic acid derivatives:
O
R C X = OR´, Cl, Br, NH2, NHR´
X

42.1 Carboxylic Acid Esters


Esterification of a carboxylic acid with an alcohol in the presence of small amounts of an acid (mineral or LEWIS) yields a
carboxylic acid ester and water by way of an equilibrium between esterification and saponification (Chapters 35.4, 74.2.2)
with constant K.
c(ester) c(H2O)
K =
c(acid) c(alcohol)

In order to increase the yield of ester, the equilibrium must be shifted in favor of the ester by using an excess of one of
the reactants or by continuously removing by distillation one of the products during the reaction. Esters of carboxylic acids
with alcohols are called alkyl alkanoates; some of them have pleasant fruity odors and are used as fragrances in per-
fumery and as flavors for food. Ethyl butanoate, for example, smells like pineapples.
O O O O
esterification esterification
R C + HO R' R C + H 2O C + HO C + H 2O
saponification saponification
OH O R' OH O
carboxylic alcohol alkyl alkanoate butanoic acid ethanol ethyl butanoate
acid (ester) (butyric acid) (butyric acid ethyl ester)

The mechanism of acid-catalyzed esterification involves protonation of the carbonyl oxygen, generating a resonance-
stabilized cation with electrophilic carbon susceptible to nucleophilic attack by an alcohol. The intermediate orthocarbox-
ylic acid monoester undergoes protonation, loss of water to give the protonated ester and, finally, deprotonation to pro-
vide the ester. All steps are reversible.

O OH
O  [H ] OH OH + H R'
R C R C R C R C OH
 [H ]
OH OH OH O
protonated carboxylic acid H R'
 [H ]

OH
saponification esterification orthocarboxylic acid monoester R C OH
OR'

 [H ]
protonated ester
O OH OH  H2 O OH2
 [H ]
R C R C R C R C OH
OR' OR' OR' OR'

Fats are esters (triglycerides, Chapter 74.2.2) of glycerol and carboxylic acids with long saturated or unsaturated carbon
chains, called fatty acids (Chapter 74.2). Soaps are manufactured by hydrolysis of fats with aqueous sodium hydroxide or
aqueous potassium hydroxide, yielding the sodium salts (hard soaps) or potassium salts (soft soaps) of fatty acids. For
this reason, ester hydrolysis is usually referred to as saponification.

Efficiently Studying Organic Chemistry: Exam training for chemists, biochemists, pharmacists, life and health scientists,
Third Edition. Eberhard Breitmaier. © 2022 WILEY-VCH GmbH. Published 2022 by WILEY-VCH GmbH.
42.2 Carboxylic Acid Halides (Acyl Halides) 111

42.2 Carboxylic Acid Halides (Acyl Halides)


Carboxylic acids are converted into acyl halides by inorganic halides (PCl5, PCl3, PBr3); thionyl halides (SOCl2, SOBr2)
produce volatile gases (SO2, HCl, HBr) as byproducts, leaving almost pure acid halides. The halide of acyl halides is
readily substituted by nucleophiles; therefore, acyl halides are acylation reagents.
O2N O2N
O O O O
H 2C CH C + SOCl2 H2C CH C + SO2 + HCl C + PCl5 C + POCl3 + HCl
OH Cl OH Cl
acrylic (propenoic) acid acryloyl chloride O2N O2N
3,5-dinitrobenzoyl chloride

42.3 Carboxylic Acid Anhydrides


Symmetric carboxylic acid anhydrides such as acetic or trifluoroacetic anhydride are obtained by bimolecular dehydration
of the corresponding carboxylic acid with a dehydrating reagent.
O O O O
trifluoroacetic P4O10 trifluoroacetic
acid F 3C C + C CF 3 F 3C C O C CF 3 anhydride
 H 2 O
OH HO

The nucleophilic oxygen of alcohols and phenols (Chapter 52) adds to one of the electrophilic carbonyl carbon atoms of
anhydrides to form esters. The secondary alcohol menthol (Chapter 76.3.1), for example, is acetylated by acetic anhy-
dride to give menthyl acetate, the menthyl ester of acetic acid:
O
H3C C CH3
O H + O O CH3 + H3C CO2H
C
CH(CH3)2 C CH3
O
menthol O menthyl acetate

Acylation of carboxylates by acyl halides yields mixed anhydrides (from two different carboxylic acids):
O O O O
R C + C R' R C O C R'
 NaX
X NaO
X = Cl , Br mixed carboxylic acid anhydride

Dicarboxylic acids undergo intramolecular thermal dehydration to give cyclic anhydrides if a five- or six-membered ring
can be formed, exemplified by the formation of succinic, maleic and phthalic anhydride:
O O O O O O
heat
C C C
H 2C OH or P4O10 OH heat OH heat
O O O
H 2C OH  H2O OH  H2O OH  H2O
C C C
O O O O O O
succinic anhydride maleic anhydride phthalic anhydride

42.4 Carboxylic Acid Amides, Cyclic Imides


Carboxylic acids react with ammonia to give ammonium carboxylates which are dehydrated to provide the carboxamides
(carboxylic acid amides) upon heating. Alternatively, nucleophilic substitution of acyl halides (or esters with X = OR2) by
ammonia and primary and secondary amines yields amides and their N-alkyl- and N,N-dialkyl derivatives.
O O O O O
heat + H NR 2
H 3C C + NH 3 H 3C C H 3C C R1 C R1 C
 H 2O  HX
OH O NH 4 NH 2 X NR 2
ammonium acetate acetamide (a carboxamide) acid halide N,N-dialkylcarboxamide
112 42 Carboxylic Acid Derivatives

Cyclic acid imides are obtained from diammonium salts or cyclic anhydrides of dicarboxylic acids:
O O O O
C heat heat
H 2C O NH 4
NH succinimide O + NH 3 NH phthalimide
H 2C O NH 4  2 H2O  H2O
C  NH3
O O O O

The amide or peptide bond (NHCO) is the structural feature of peptides and proteins (Chapter 69.1).

42.5 Hydrazides, Hydroxamic Acids, Azides


Nucleophilic substitution of the halide anion in acyl halides by hydrazine as nucleophile yields the hydrazide of acetic acid
which is also formed by reacting the esters with hydrazine.
O O O
H 3C C + 2 H 2N NH2 H 3C C H3C C + H 2N NH 2
 H2 N NH 3 Cl  C2H5OH
Cl NH NH 2 OC 2H 5
acetyl chloride hydrazine acetic acid hydrazide ethyl acetate

Nucleophilic substitution of an acyl halide with hydroxylamine as nucleophile leads to a hydroxamic acid. Hydroxamic
acids equilibrate with their oxime tautomers by a hydrogen shift and react with ferric salts to give deep red metal chelates.
This color reaction can be used to identify carboxylic acids.
H
O O OH R O
tautomerism + 1/3 Fe3 C
R C + 2 H2N OH R C R C Fe/3
 HO NH 3 Cl [H ] N
Cl NH OH N OH O
hydroxamic acid oxime iron(III) chelate (purple)

Similarly, nucleophilic substitution of the halide anion in acyl halides by the azide anion of sodium azide (NaN3) produces
carboxylic acid azides such as benzoyl azide (benzoic acid azide):
O _ _ O O
benzoyl chloride resonance formulas
C + Na N N N
_ _ C _ C_ of benzoyl azide
 Na Cl
Cl N
_ N N
_ N
_ N N
_

Acyl azides readily lose nitrogen, leaving intermediate acylnitrenes which rearrange to isocyanates (RN=C=O) by an-
ionotropic 1,2-alkyl shift (CURTIUS rearrangement, Chapter 57.1.4).

42.6 Change of the Carboxy Function


42.6.1 Reduction to Primary Alcohols and Aldehydes
Carboxylic acids and their esters are reduced with lithium aluminum hydride to primary alcohols by way of intermediate
aldehydes, involving nucleophilic addition of the hydride anion to the carbonyl group:
LiAlH4 H H
O H
or NaBH 4 + HI + H2O
R C + IH R C O R C R C O R CH 2 OH
 R'O  OH
OR' OR' O H
carboxylic acid ester aldehyde primary alcohol

In the presence of deactivated catalysts, hydrogenation of acyl halides yields aldehydes (ROSENMUND reduction). N,N-
Dialkylcarboxamides are reduced by complex metal hydrides to aldehydes (Chapter 47.2.3).
O
O O O LiAlH4 O R'
+ Pd (BaSO4 ) + H2 C R' tetrahydrofuran Li N
R C R C R C R N + Li H C +
R H R'
 Pd,  HCl R'
Cl PdCl H
acyl halide aldehyde N,N-dialkylcarboxamide aldehyde
42.6 Change of the Carboxy Function 113

42.6.2 Reductive Coupling of Esters (Acyloin Reaction)


Carboxylic acid esters undergo reductive coupling when heated with sodium as reducing reagent (electron donor) in ether
or toluene. -Hydroxy ketones, also known as acyloins, are obtained. The mechanism involves reduction of the ester to a
radical anion which dimerizes to provide a dianion. Elimination of alkoxide yields a diketone which is reduced to the dieno-
late. The dienol arising from hydrolysis tautomerizes to give the acyloin.
O . O Na NaO ONa O O . NaO ONa HO OH HO O
+ 2 Na + 2 Na + 2 H 2O
2R C 2R C. C C C C C C C C C C
R'O OR'  2 NaOR'  2 NaOH H
OR' OR' R R R R R R R R R R
ester radical anion (salt) dianion (salt) diketone dienolate dianion dienol acyloin

42.6.3 Decarboxylation
Carboxylic acids eliminate carbon dioxide when heated to 200 °C, yielding an alkane. Decarboxylation of trichloroacetic
acid (R = CCl3) to chloroform and of malonic acid to acetic acid already occurs below 140 °C.
O
heat O OH 136 °C H 2C OH tautomerism H 3C OH
R C R H + CO2
 CO2
O H O O O O
H H
malonic acid acetic acid (enol) acetic acid

42.6.4 Dehydration of Carboxamides to Nitriles and Isonitriles


Nitriles (alkyl or aryl cyanides) are not only prepared by nucleophilic substitution of alkyl halides by cyanide (KOLBE syn-
thesis, Chapter 41.4.4) but also by dehydration of carboxamides (except formamide). This dehydration represents a -
elimination because H and OH are abstracted from adjacent atoms (C- and N-).
  SN P4O10
OH tautomerism O
R X + IC NI R C N R C R C
 X  H2 O
alkyl halide nitrile (cyanide) N NH2
( X = Cl, Br, I ) H carboxamide

Isonitriles (isocyanides) are prepared from N-alkylformamides (amides of formic acid) involving dehydration of the -
hydroxy imino tautomer. This dehydration exemplifies an -elimination because H and OH are abstracted from the
same atom, the former carbonyl carbon of the N-alkylformamide.
R O tautomerism OH R
POCl3 , KOR
N C N C R N CI
  H2 O
H H H isonitrile (isocyanide)
N-alkylformamide (tautomers) R = CH3 : methyl isocyanide

Nucleophilic addition of hydrogen chloride to nitriles yields imidoesters by way of intermediate imidoyl chlorides. Ammo-
nolysis of the imidoesters produces amidines as nitrogen analogues of carboxylic acids:
NH NH2 Cl NH 2 NH 2
+ NH3
R1 C N + HCl + R2 OH R1 C + R2 OH R1 C R1 C R1 C Cl
 R2 OH
nitrile Cl O R2 NH 2 NH 2
(R1 = CH3 : acetonitrile) imidoyl chloride imidoester (hydrochloride) R1 = CH3 : acetamidine (hydrochloride)
________________________________________________________________________________________________
Chapter 42 permits answers to the following:
(42.1) (a) Write the mechanism of the acid-catalyzed esterification; (b) suggest three options to prepare n-butyl acetate.
(42.2) Outline the preparation of (a) acetyl chloride and (b) acryloyl chloride by using thionyl chloride as a reagent.
(42.3) Formulate the preparation of the mixed anhydride of formic and acetic acid (two options).
(42.4) What products are formed by heating (a) maleic acid, (b) phthalic acid, and (c) ammonium succinate?
(42.5) Formulate the reaction of phthalic anhydride with one equivalent of ethanol. What product is obtained?
(42.6) Outline the steps of a test tube color reaction to identify a carboxylic acid.
(42.7) What are the decarboxylation products of (a) trichloroacetic, (b) malonic, and (c) cyanoacetic acid? Formulate.
(42.8) Formulate the preparation of (a) 2,4,6-trimethylphenyl cyanide and (b) n-butyl isocyanide with suitable reagents.
114 43 Substituted Carboxylic Acids

43 Substituted Carboxylic Acids


43.1 Nomenclature
Substituted carboxylic acids contain functional groups in addition to the carboxy group. In order to name such multifunc-
tional compounds, the IUPAC rules define a ranking of substituents (Table 43.1) partly related to the oxidation state of the
functional group. Accordingly, the carboxy group has the highest ranking, followed by carboxylic acid derivatives, alde-
hydes, ketones, alcohols, and ending with halogens.

Table 43.1. Nomenclature and ranking of substituents in multifunctional compounds


ranking formula name prefix suffix
1 COOH ( CO2H) carboxylic acid carboxy- -oic acid
2 COOR ( CO2R) (carboxylic acid) ester alkoxycarbonyl- -oic acid alkyl ester
3 COCl acyl chloride chlorocarbonyl- -oyl chloride
4 CONH 2 carboxamide aminocarbonyl- (carboxamido-, carbamoyl-) -oic acid amide
5 C N nitrile (cyanide) cyano- -nitrile (cyanide)
6 CHO aldehyde formyl -al
7 CO ketone oxo- (keto-) -one
8 SO3H sulfonic acid sulfo- -sulfonic acid
9 OH alcohol hydroxy- -ol
10 SH thiol (mercaptan) mercapto- -thiol
11 NH 2 amine amino- -amine
12 N imine imino- -imine
13 OR ether alkoxy- (e.g. methoxy- for R = CH3) ether
14 SR thioether (sulfide) alkythio- (e.g. methythio- for R = CH3) sulfide
15 X (X = F, Cl, Br, I) halide halo- halide

In keeping with this ranking, any multifunctional organic compound containing a carboxy group is named as a carboxylic
acid (e.g. pentanoic acid, hexanoic acid). Lower ranking substituents are inserted as prefixes (Table 43.1) in alphabetical
order; their position is indicated by the lowest possible number for the highest ranking substituent, the carboxy group
always taking position 1 (e.g. 6-aminohexanoic acid). The correct name for a pentanoic acid with a hydroxy and an oxo
function in the chain is therefore 5-hydroxy-3-oxopentanoic acid (not 3-oxo-5-hydroxypentanoic acid).

O
   1 5 1
N C CH 2 CO2H H 2N CH 2 CO2H H2N CH2 CH2 CH2 CH 2 CH 2 CO2H HO CH2 CH2 C CH2 CO2H
3

1
O 1
5
NC CO2H H2N CO2H H 2N CO2H CO2H
6 4 2
HO 3

cyanoacetic acid glycine -aminocaproic acid 5-hydroxy-3-oxopentanoic acid


(cyanoethanoic acid) (aminoethanoic acid) (6-aminohexanoic acid)

Frequently, common names are used. In this case, Greek letters (, , , , ) indicate the positions of substituents,
beginning with  for the carbon atom adjacent to the carboxy group (e.g. -aminocapronic acid). Numbering is unneces-
sary if the substituent occupies an unambiguous position, as is the case in substituted acetic acids (cyanoacetic acid,
aminoethanoic acid = glycine).

Efficiently Studying Organic Chemistry: Exam training for chemists, biochemists, pharmacists, life and health scientists,
Third Edition. Eberhard Breitmaier. © 2022 WILEY-VCH GmbH. Published 2022 by WILEY-VCH GmbH.
43.2 Halo Acids 115

43.2 Halo Acids


43.2.1 Preparation
Similar to alkanes, alkanoic acids undergo radical photohalogenation. Acetic acid and phenylacetic acid are photochlorin-
ated with chlorine under UV light, yielding the -halo acids. Carboxylic acids with longer alkyl chains are additionally
halogenated at other positions, thus yielding product mixtures.
h , reflux
H 3C CO2H + Cl2 Cl CH2 CO2H + HCl
chloroacetic acid
h
C 6H5 CH2 CO2H + Cl2 C6H 5 CH CO2H + HCl
Cl
-chlorophenylacetic acid

In the presence of phosphorus halides or catalytic amounts of red phosphorus, halogenation with chlorine and bromine
regioselectively yields -halo acids (HELL-VOLHARD-ZELINSKY halogenation). The mechanism probably involves the enol
tautomer of the acyl halide as intermediate, which undergoes addition of halogen. Hydrolysis of the resulting -
haloorthocarboxylic acid dihalide liberates the -halo acid.
O OH OH OH
PX3 or P tautomerism + X2 + H 2O
R CH 2 CO2H + X 2 R CH 2 C R CH C R CH C X R CH C
 HX  2 HX
X = Cl , Br X X X X X O
acyl halide enol tautomer -haloorthocarboxylic acid dihalide -halo acid

As an example, HELL-VOLHARD-ZELINSKY bromination of valeric acid produces -bromovaleric acid:


Br
+ Br 2 , P
H3C CH 2 CH 2 CH2 CO2H H 3C CH 2 CH 2 CH CO2H
 HBr
valeric acid (pentanoic acid) -bromovaleric acid (2-bromopentanoic acid)

Electrophilic hydrohalogenation of ,-unsaturated carboxylic acids regioselectively leads to -halo acids:


H3C CH CH CO2H + HBr H3C CH CH 2 CO2H
Br
(E-) or (Z-)-2-butenoic acid 3-bromobutanoic acid

Regioselectivity arises from the positive polarization of the carbon atom at the -position due to electron withdrawal [()-
M effect] by the carboxy group in the intermediate oxonium-carbenium ion, as shown by the resonance formulas:

OH
R CH CH C
OH
OH OH O
+ H Br
R CH CH CO2H R CH CH C + Br R CH CH C R CH CH 2 C
OH Br OH Br OH
OH unstable enol tautomer -bromo acid
R CH CH C
OH
oxonium-carbenium ion

43.2.2 Reactions
Due the inductive effect of the halogen substituents, halo acids are more acidic than the parent alkanoic acids (Chapter
41.5). They undergo neutralization to salts and all derivatizations known from alkanoic acids, as well as all reactions
typical for alkyl halides such as nucleophilic substitutions and eliminations. Nucleophilic substitutions of the halide anions
in halo acids open access to amino-, hydroxy-, and cyano acids and, by hydrolysis of the latter, malonic acids (Fig. 43.1).
116 43 Substituted Carboxylic Acids

alkyl halide reactions carboxylic acid derivatizations


+ NaOH ,  NaCl
R CH 2 CH CO2H
hydrolysis + NaOH (cold),  H2O
OH R CH2 CH CO2 Na
neutralization
2-hydroxyalkanoic acid X
+ 2 NH3 ,  NH4Cl
R CH 2 CH CO2 sodium 2-haloalkanoate
ammonolysis
NH 3 + R'OH ,  H 2O [H ]
O
2-aminoalkanoic acid R CH 2 CH CO2H R CH2 CH C
(amino acid) esterification
X X OR'
+ KCN ,  KCl
R CH 2 CH CO2H 2-haloalkanoic acid ester
KOLBE nitrile synthesis 2-haloalkanoic acid
CN + SOCl2 ,  SO2 ,  HCl
O
2-cyanoalkanoic acid R CH2 CH C
 HX acid halogenation
R CH CH CO2H X Cl
dehydrohalogenation 2-haloacyl chloride
2-alkenoic acid

Fig. 43.1. Reactions of 2-haloalkanoic acids

43.3 Hydroxy Acids


43.3.1 Preparation
Nucleophilic substitution of the halide located at any position of halo acids permits the preparation of hydroxy acids (Fig.
43.1):
H H
+ OH
-bromobutyrate CH3 CH2 C CO2 Na CH3 CH 2 C CO2 Na -hydroxybutyrate
 Br
Br OH

-Hydroxyalkanoic acids are obtained from -halo esters and carbonyl compounds such as aldehydes or ketones by the
REFORMATSKY reaction. Initiated by the metalation of the -halo ester by zinc in an inert solvent to provide an alkylzinc
halide of the ester, a REFORMATSKY synthesis formally follows the GRIGNARD metalation of alkyl halides by magnesium and
the 1,2-addition of the alkylmagnesium halides to carbonyl compounds (Chapter 33.3.7). Accordingly, 1,2-addition of the
organozinc halide to the carbonyl group of an aldehyde or ketone yields the alcoholate of a -hydroxy ester which is
finally liberated by hydrolysis.
-halo ester R1 CH CO2R 2
( X = Cl , Br , I )
X
+ Zn in ether or toluene

R1 R3 R1 R3 R1
R4 R3 R4 R3 1,2-addition + H2O
C C +  CH CO2R 2 R 4 C CH CO2R 2 R 4 C CH CO2R2
 ZnX(OH)
O O  ZnX OZnX OH
carbonyl compound
aldehyde (R = H) or ketone (R = alkyl or aryl)
3 3
-hydroxy ester

As an example, the REFORMATSKY reaction of ethyl bromoacetate with benzaldehyde affords ethyl 3-hydroxy-3-
phenylpropanoate (-hydroxy--phenylpropionic acid ethyl ester) which readily undergoes acid- catalyzed dehydration to
the ,-unsaturated ester ethyl cinnamate:
ethyl bromoacetate
Br CH 2 CO2C 2H 5 Ph =
+ Zn
H H H Ph H
+ H2O [H ]
Ph C + CH 2 CO2C 2H 5 Ph C CH 2 CO2C 2H 5 Ph C CH 2 CO2C 2H 5 C C
O  ZnX(OH)  H2O
ZnBr OZnBr OH H CO2C 2H 5
benzaldehyde ethyl 3-hydroxy-3-phenylpropanoate ethyl cinnamate
(trans-)

More reactive alkylmagnesium halides would also attack the ester function and are, therefore, not suitable for the prepa-
ration of -hydroxy esters.
43.3 Hydroxy Acids 117

43.3.2 Reactions
Apart from reactions specific for carboxy and hydroxy groups such as derivatizations, substitutions, and oxidation to keto
acids, which are more efficiently prepared by other methods (Chapter 50.1), hydroxy acids display ambident reactivity
arising from the presence of both functional groups.
Thus, lactic acid (2-hydroxypropanoic acid) undergoes intermolecular cyclodehydration upon heating to give bis-lactones
also referred to as lactides. Esters of -hydroxy acids react in the same way.
R R
O OH HO H O O H O

+
H  2 H2O H
OH HO O O O O
R R
lactide 1,4-dioxane
R = CH 3 : lactic acid (3,6-dialkyl-2,5-dioxo-1,4-dioxane) parent compound

Intramolecular cyclodehydration (intramolecular esterification) of hydroxy acids yields lactones. Lactones of - and -
hydroxy acids are readily formed due to the resulting particularly stable five- and six-membered rings, exemplified by the
formation of -butyrolactone from 4-hydroxybutanoic acid and -valerolactone from 5-hydroxypentanoic acid.

CO2H C O CO2H C
OH  H 2O O OH  H 2O O O
-hydroxybutyric acid -butyrolactone -hydroxyvaleric acid -valerolactone
(4-hydroxybutanoic acid) (5-hydroxypentanoic acid)

Similarly to hydroxy acids, amino acids including those obtained by hydrolysis of proteins (Chapter 68) undergo ambident
reactions because of their bifunctionality. Paralleling the - and -hydroxy acids, five- and six-membered lactams such as
-butyro- and -valerolactam are easily formed from - and -amino acids. Formally derived from the parent heterocycles
pyrrolidine and piperidine (Chapter 60.1), these lactams were given the common synonyms pyrrolidin-2-one and piperi-
din-2-one although they are not ketones (Chapter 48.1) but cyclic acid amides. Hydrolysis with aqueous acids opens the
lactam rings to yield the amino acids.

CO2H C O CO2H C
NH 2  H2 O N NH2  H2 O N O
H H
-aminobutyric acid -butyrolactam -aminovaleric acid -valerolactam
(4-aminobutanoic acid) (pyrrolidin-2-one) (5-aminopentanoic acid) (piperidin-2-one)

________________________________________________________________________________________________
Chapter 43 permits answers to the following:
(43.1) Draw the structural formulas of (a) 2-oxopropanoic acid, (b) 3-oxobutanoic acid ethyl ester, (c) 2,2-dichloropentanoic
acid, (d) 4-hydroxy-5-phenylpentanoic acid methyl ester, and (e) trifluoroacetic acid N,N-diethylamide.
(43.2) Formulate equations to suggest the preparations of pure (a) 2-bromobutanoic acid and (b) 3-bromobutanoic acid.
(43.3) What products are obtained by reacting chloroacetic acid with (a) potassium cyanide, (b) sodium hydroxide,
(c) ammonia, and (d) thionyl chloride. Write the equations.
(43.4) Ethyl bromoacetate reacts with zinc dust in toluene. What product is formed when benzaldehyde is added to the
mixture? Formulate the appropriate equations.
(43.5) What are (a) lactides, (b) lactones, and (c) lactams? Give examples.
(43.6) Which lactam gives -aminocaproic acid (6-aminohexanoic acid) upon acidic hydrolysis?
118 44 Absolute Configuration

44 Absolute Configuration
44.1 Stereogenic Center, Enantiomers, Chirality
2-Butanol shown in Fig. 44.1 has one carbon atom (C-2, marked with *) which is attached to four different groups (OH,
C2H5, CH3, H). This asymmetrically substituted carbon atom is said to be a stereogenic center. Because of this stereo-
genic center, two stereoisomers exist for 2-butanol which are related to each other as image and non-superimposable
mirror image (Fig. 44.1). Such stereoisomers are referred to as enantiomers.

mirror
C 2H 5 C2H5

H3C
*C
H H
C * CH3
OH HO
image mirror image
2-butanol

Fig. 44.1. Enantiomers of 2-butanol: tetrahedral projection formulas; tube and space-filling molecular models (carbon: black; oxygen:
red; hydrogen: gray)

Thus, two enantiomers with a mirrored arrangement of substituents exist for 2-butanol, differing like the right and the left
hand (Fig. 44.1). This feature is called handedness, dissymmetry, or chirality, derived from the Greek word cheir = hand.
2-Butanol is said to be a chiral molecule; its structural isomer 1-butanol without a stereogenic center is said to be achiral.
To conclude, a stereogenic center (an asymmetrically substituted C atom, traditionally called an "asymmetric carbon") is
one of the reasons for chirality. Generally, any molecule which is non-superimposable with its mirror image is chiral.

44.2 Optical Activity and Specific Rotation


Enantiomers have identical physical properties (boiling points, melting points, refractive indices, adsorption coefficients,
molecular spectra) with one exception: They interact differently with polarized light and are therefore said to be optically
active. Optical activity is the ability of a compound to rotate the plane of linearly polarized light by a certain angle .
Enantiomers of a particular compound rotate the plane of linearly polarized light in equal amounts but in opposite direc-
tions. The enantiomer that rotates the plane clockwise (to the right) is said to be dextrorotatory and designated as ()-,
while the other that rotates the plane counterclockwise (to the left) is said to be levorotatory and designated as ()-.
The amount of rotation, detected by a polarimeter, depends on concentration, wavelength of light, length of the sample
vessel, temperature (pressure), and solvent. Standardization is therefore necessary: The specific rotation [] is related to
a standard concentration c and a standard path length l; []D20 indicates that the rotation was measured with the D line of
sodium light (wavelength 589 nm) at 20 °C.

specific rotation [measured 100 / l c [degrees , °] l : path length in dm


c : concentration in g / 100 mL

44.3 Specification of the Absolute Configuration


The spatial arrangement of substituents attached to a stereogenic center is called the absolute configuration. Since they
have a mirrored arrangement of substituents, as exemplified by 2-butanol in Fig. 44.1, enantiomers differ in their absolute
configuration. Two conventions have been established to specify the absolute configuration of a stereogenic center in an
enantiomer by means of stereo descriptors (similar to cis- and trans- for configurational isomers of alkenes). These are
the generally applicable CAHN-INGOLD-PRELOG system and the traditional FISCHER convention used for protein amino acids
(Chapter 68) and carbohydrates (Chapter 71).

Efficiently Studying Organic Chemistry: Exam training for chemists, biochemists, pharmacists, life and health scientists,
Third Edition. Eberhard Breitmaier. © 2022 WILEY-VCH GmbH. Published 2022 by WILEY-VCH GmbH.
44.3 Specification of the Absolute Configuration 119

44.3.1 CAHN-INGOLD-PRELOG Convention [(R)- and (S)-Descriptors]


Following the CAHN-INGOLD-PRELOG convention ("CIP") to specify the absolute configuration by the stereo descriptors R
and S, the four groups attached to the stereogenic center are ranked using three basic rules:

(1) The ranking of the atoms directly connected to the asymmetrically substituted carbon decreases with decreasing
atomic number (atom mass) in the periodic table, H always having the lowest priority:
H CH3 D (D = 2H, deuterium)
F C I Cl C H F C H
Br I Br
I > Br > F > H I > Cl > C > H Br > F > D > H

(2) Where two or more atoms directly connected to the stereogenic center are the same, the atomic number of the
attached atom (in the second sphere) defines the ranking (CH2CH3 > CH3):
CH 3
CH 3 CH 2Cl H 3C C H
HO C H H3C CH2 C CH 3 H C Br
CH 2 CH 3 H CH 2 CH3

OH > C 2H5 > CH3 > H CH 2Cl > C 2H5 > CH 3 > H Br > (CH 3)2CH > C 2H5 > H

(3) Double and triple bonds to attached atoms are considered to be split into two and three single bonds, respectively,
and counted twice or three-fold, an aldehyde group (CH=O) thus ranking higher than a primary alcohol function
(CH2OH):
O H
C
glyceraldehyde HO C H OH > CH=O > CH2OH > H
CH 2 OH

Once the ranking of all four groups connected to the stereogenic center has been defined, the molecule is held so that
the lowest ranking group (usually H) points away from the viewer (lies behind the plane of the paper). Then, if the ranking
of the other groups decreases clockwise (to the right), the absolute configuration is specified as R (from Lat. rectus =
right); correspondingly, the ranking decreases counterclockwise (to the left) for an S configuration (from Lat. sinister =
left). The enantiomers of 2-butanol exemplify this and demonstrate that the direction ()- or ()- of optical rotation as a
physical property does not at all correlate with the descriptors R and S of absolute configuration as defined by the CIP
convention: (R)-2-Butanol is levorotatory [(R)-()-2-butanol] while the (S)-enantiomer is dextrorotatory [(S)-(+)-2-butanol].

C 2H 5 C 2H 5
ranking decreases ranking decreases
counterclockwise C C clockwise
H 3C H H CH3
OH HO
(S)-2-butanol (R)-2-butanol
[] 20
D
= + 15° [] 20 =  15°
D
 ( c = 10 g / 100 mL in methanol )
(S)-(+)-2-butanol (R)-()-2-butanol

44.3.2 FISCHER Convention (D- and L-Descriptors)


The FISCHER convention was originally established for the enantiomers of glyceraldehyde as the simplest representative
of carbohydrates (Chapter 71). In order to apply the FISCHER convention to 2-butanol, the molecule is held so that the
longest carbon chain is vertical, the larger group placed at the top. In Fig. 44.2 a, for example, ethyl of 2-butanol is drawn
at the top, methyl at the bottom. Horizontally drawn groups such as H and OH in Fig. 44.2 rise above the paper plane
pointing towards the viewer. In the tetrahedral projection (Fig. 44.2 b, Chapter 8.4.1) vertical bonds point behind and are
therefore dotted, while horizontal bonds are wedges, pointing above the paper plane. The FISCHER projection simplifies
120 44 Absolute Configuration

this (Fig. 44.2 c), presenting dotted and wedge bonds as per covalent bonding lines. Then, if the OH group points to the
right, the enantiomer is said to have a D configuration (from Lat. dextra = right); conversely, it has an L configuration (from
Lat. laevus = left) when the OH group points to the left (Fig. 44.2 c).

mirror
C 2H 5 C2H 5
tetrahedron
(a) HO H H OH (view from outside, lines
are edges, not bonds)
CH3 CH 3

C 2H 5 C2H 5
(b) HO C H H C OH tetrahedral projections
CH3 CH 3

C 2H 5 C2H 5
(c) HO C H H C OH FISCHER projections
CH3 CH 3
L-2-butanol D-2-butanol

Fig. 44.2. Enantiomers of 2-butanol: specification of the absolute configuration following the FISCHER convention

FISCHER projections may be shifted in the paper plane and rotated by 180° but they cannot be rotated by 90° or taken out
of the paper plane, as shown for L-(+)-lactic acid (occurring in dairy products and body fluids):

OH CO2H CH 3
H 3C C CO2H HO C H H C OH
H CH 3 CO2H
L-(+)-lactic acid

44.3.3 Correlation of D,L and R,S Descriptors


Fig. 44.3 exemplifies the correlation of D,L to R,S descriptors for 2-butanol: The FISCHER projection (Fig. 44.3 a) is con-
verted into the tetrahedral projection (Fig. 44.3 b), and the tetrahedron is rotated so that the lowest ranking group (H)
points away from the viewer, standing behind the paper plane (Fig. 44.3 c). Thus, the relationship L = (S) and D = (R)
applies for the enantiomers of 2-butanol, as demonstrated by Fig. 44.3; however, this is not a general rule. If a bromome-
thyl group (CH2Br) replaces methyl in 2-butanol, the resulting L-1-bromo-2-butanol is (R)-1-bromo-2-butanol because
bromomethyl ranks higher than ethyl, in keeping with the second rule of the CIP convention (Chapter 44.3.1).

L-2-butanol mirror D-2-butanol


C2H5 C2H5

(a) HO C H H C OH FISCHER projections


CH3 CH3

C2H5 C2H5
(b) HO C H H C OH tetrahedral projections
CH3 CH3

C2H5 C2H5

(c) C C tetrahedral projections


H 3C H H CH3 (H pointing away from viewer)
OH HO
(S)-2-butanol (R)-2-butanol

Fig. 44.3. Translation of D,L to R,S descriptors of absolute configuration


44.4 Stereospecificity of the Bimolecular Nucleophilic Substitution 121

44.3.4 Racemates and Their Resolution


An equimolar mixture of the (R) and (S) enantiomers of a specific compound is called a racemic mixture or a racemate.
Racemic 2-butanol is referred to as (RS)-, or (DL)-, or (±)-2-butanol. In contrast to the enantiomeric components, a race-
mate is optically inactive ([] = 0°) and its physical properties such as melting point, boiling point, and refractive index
often differ from those of the enantiomers. Chemical reactions of achiral compounds which create a stereogenic center
(chirogenic reactions), such as the bromination of pentanoic acid to yield 2-bromopentanoic acid (Chapter 43.2.1), pro-
duce racemates under normal conditions.
A general procedure to resolve a racemic mixture to the separate enantiomers involves the reaction of the racemate with
the pure enantiomer of a chiral compound to give diastereomers (Chapter 46). As an example, a racemic alcohol [(R,S)-
alcohol)] reacts with the pure (R)-enantiomer of a carboxylic acid, acid halide or anhydride to give two diastereomeric
esters , the (R,R)- and the (S,R)-ester:
hydrolysis
(R) - R 1 (R) - R 1 + Na OH
esterification C O C O (R) - R OH
 (R)-R 1 CO2 Na
+ 2 (R)- R1 CO2H (R) - R O resolution (R) - R O
(R,S) - R OH
 2 H 2O (R)- R 1 (R) - R 1 + Na OH
C O C O (S) - R OH
 (R)-R 1 CO2 Na
(S)- R O (S) - R O
racemic alcohol diastereomeric esters diastereomeric esters enantiomeric alcohols
mixture separated separated

In contrast to enantiomers, diastereomers exhibit different physical properties (solubilities, melting points, boiling points,
adsorption coefficients) which permit separation by crystallization, distillation or chromatographic methods. Once the two
diastereomeric esters have been separated, they can be individually hydrolyzed to the enantiomeric alcohols.

44.4 Stereospecificity of the Bimolecular Nucleophilic Substitution


A nucleophilic substitution proceeding by way of an SN2 mechanism involves WALDEN inversion of the configuration
(Chapter 31.1). Thus, if the substrate is an enantiomer such as (R)-()-2-bromooctane, WALDEN inversion is expected to
invert the absolute configuration of the reacting stereogenic center. Indeed, (R)-()-2-bromooctane undergoes nucleo-
philic substitution with sodium hydroxide to give (S)-()-2-octanol:

CH 3 CH 3 CH 3
+    SN2
HO + C Br HO C X HO C + Br
H H
H
C 6H 13 C 6H 13
C 6H 13
(R)-()-2-bromooctane transition state (S)-()-2-octanol
[]20
D =  44.9° []17
D =  9.9°

To conclude, SN2 reactions of chiral substrates are stereospecific, converting a stereochemically pure substrate [(R)-()-
2-bromooctane] into a stereochemically pure product [(S)-()-2-octanol]. On the other hand, the observation of WALDEN
inversion, evidenced by the specific rotations of substrate and product, for a nucleophilic substitution of an enantiomeri-
cally pure substrate provides evidence for an SN2 mechanism.
________________________________________________________________________________________________
Chapter 44 permits answers to the following:
(44.1) What is a stereogenic center? Define the term chirality. How do enantiomers of a specific compound differ?
(44.2) Draw the enantiomers of 2-(4-isobutylphenyl)propanoic acid and assign the absolute configurations.
(44.3) Which structural isomers of heptane C7H16 exist as enantiomers? Draw, name, and specify them.
(44.4) Draw the appropriate projection formulas of (R)- and (S)-2,3-dibromopropanoic acid. Is L = (S) and D = (R) a rule?
(44.5) What is a racemic mixture? Suggest a method to resolve racemic 2-butanol.
(44.6) What product is expected from reacting (S)-2-bromooctane with sodium hydroxide? Characterize this reaction.
122 45 Enantiomers without Carbon as Stereogenic Center

45 Enantiomers without Carbon as Stereogenic Center


45.1 Heteroatoms as Stereogenic Centers
All tetravalent heteroatoms with tetrahedral bonds connecting four different substituents are stereogenic centers. Exam-
ples include silanes, tri- or tetraalkylammonium salts with four, and amine N-oxides and sulfonium salts with three differ-
ent alkyl or aryl groups:
H C 6H 5 C 6H 5 non-bonding
electron pair
Si N Cl N as substituent H 3C S C H
H 3C H 3C H H 3C O 6 5
C 6H 5 CH(CH 3)2 CH(CH 3)2 CH(CH 3)2
(S)-methyl-  -naphthylphenylsilane (R)-isopropyl(methyl)phenyl- (S)-isopropyl(methyl)phenyl- (S)-isopropyl(methyl)phenyl-
ammonium chloride amine N-oxide sulfonium ion

A stereogenic center is not the only possible origin of chirality. There is no stereogenic center in hexahydroxycyclo-
hexanes, known as inositols, which are sweet tasting and occur in plant and animal organisms. Eight cis-trans-isomers
exist. One of those, named chiro-inositol, has been resolved into enantiomers since the mirror images are non-
superimposable due to lack of symmetry (dissymmetry). Because of axial symmetry, no enantiomers exist for the other
isomers differing in relative configuration, such as scyllo-inositol.

HO OH HO OH HO OH
HO OH HO OH HO OH
HO OH HO OH
OH HO OH HO
OH HO OH HO
(+)-chiro-inositol ()-chiro-inositol scyllo-inositol (image and mirror image are superimposable)

All molecules possessing neither a mirror plane, nor a center of symmetry, nor an alternating axis of symmetry are chiral
(dissymmetric).

45.2 Axial Chirality


Axial chirality is detectable for molecules which form "elongated tetrahedrons". Spirocycles such as 3,3´-disubstituted
spiro[3.3]heptanes or cumulenes with an even number of cumulated double bonds such as 1,3-dialkyl- and 1,3-
diarylpropadienes are examples. The stretched tetrahedral shape of allenes is explained by sp hybridization of the central
carbon, providing two p orbitals with perpendicular axes for sidewise overlapping to two  bonds (Chapter 17.2.3). An odd
number of cumulated double bonds, however, will result in cis-trans-isomerism.

H H H H
Br Br R R
chirality axis chirality axis C C C C C C
H H H H
Br Br R R
(aR)- (aS)- (aR)- (aS)-
3,3'-dibromospiro[3.3]heptane 1,3-dialkylallene

The absolute configuration of axially chiral compounds is specified either by (aR)/(aS) descriptors, a indicating axial
chirality, or by (P)/(M) descriptors, P and M indicating plus and minus.
In order to determine the (aR) or (aS) descriptor, the molecule is observed from either of the two ends along the chirality
axis. Then, reference is first made to the substituent facing the viewer and ranking higher according to the CIP convention
(R > H in 1,3-dialkylallene). The position of the higher ranking substituent at the other end of the molecule specifies the
descriptor. In the 1,3-dialkylallene drawn on the left, the route from the front alkyl group (left) via the front H (right) to the
back alkyl group (bottom) is clockwise, indicating an (aR) configuration. Looking at the 3,3´-dibromospiro[3.3]heptane
enantiomer depicted on the left in the same manner also indicates an (aR) configuration.

Efficiently Studying Organic Chemistry: Exam training for chemists, biochemists, pharmacists, life and health scientists,
Third Edition. Eberhard Breitmaier. © 2022 WILEY-VCH GmbH. Published 2022 by WILEY-VCH GmbH.
45.3 Planar Chirality and Helicity 123

The (P) and (M) descriptors refer to the spatial arrangement of the three bonds connecting the atoms 1-4:
3 4 3 4 4 3 4 3
1 1

2 2 1 2 1 2
(P) (M)

Clockwise rotation of bond 12 to an eclipsed arrangement relative to bond 34 specifies the (P) enantiomer. The same
occurs by counterclockwise rotation for the (M) enantiomer.
Positions 1 and 4 are given to the higher ranking substituent in allenes and spirocycles. These are alkyl groups in 1,3-
dialkylallene and the bromine atom in 3,3'-dibromospiro[3.3]heptane. A NEWMAN projection along the chirality axis for the
allene and the spirocycle shows that the descriptors (aR) and (aS) correspond inversely with (P) and (M).
H H 1
1 H H 1 H H 1
1 2(3) R 2 2 R 1 2(3) Br 2 3 3 2 Br
R C H C C C C C C Br C H
H 3 3 H H H
R R Br Br
R 4 Br 4 4
4 (M)- 4 (P)- 4 (M)- (P)-
1,3-dialkylallene 3,3'-dibromospiro[3.3]heptane

Biaryls substituted by four bulky groups at the ortho positions are also axially chiral because the aryl rings cannot freely
rotate due to steric hindrance. The aryl rings adopt perpendicular planes. If the substituents in each ring are different
there is no plane of symmetry. The molecule is chiral, and two resolvable enantiomers exist, as shown for 2,2'-dibromo-
6,6'-dimethylbiphenyl and 2,2'-dihydroxybinaphthyl (BINOL). Configurations are easily specified by (P) and (M) de-
scriptors. This case of axial chirality is known as atropisomerism.

4 4
H 3C 3 Br Br 3 CH 3 OH HO
H 3C 2
1 Br Br 1 2
CH 3 OH HO

(aR)- = (M)- (aS)- = (P)- (aR)- = (M)- (aS)- = (P)-


2,2'-dibromo-6,6'-dimethylbiphenyl [] 25 = + 34.3° [] 25 =  33.3° ( c = 1 g / 100 mL in THF)
D D
2,2'-dihydroxybinaphthyl (BINOL)

45.3 Planar Chirality and Helicity


Other kinds of enantiomers without stereogenic centers show planar chirality, such as 2,2'-dialkyl-p-cyclophanes, while
helical chirality or helicity is exemplified by hexahelicene with six right- or left-handed (helically) fused benzenoid rings.
The absolute configuration of both kinds of enantiomers can be readily specified by (P) and (M) descriptors:

1 4
3 2 1 2
4 3
R R
-helix -helix
(M)- (P)- (M)- (P)-
2,2'-dialkyl-p-cyclophane hexahelicene

________________________________________________________________________________________________
Chapter 45 permits answers to the following:
(45.1) Which kinds of chiral compounds with heteroatoms as stereogenic centers do you know? Draw example formulas.
(45.2) Which kinds of chiral compounds without stereogenic centers do you know? Draw example formulas.
(45.3) Enantiomers of 1,3-diphenylallene exist. Why? Look at the molecular orbital model in Chapter 17.2.3.
(45.4) Which one of (a) hexa-2,3-diene and (b) hexa-2,3,4-triene exists as (E)- and (Z)-isomers, and which one
as enantiomers? Specify the enantiomers.
(45.5) What are atropisomers? Draw formulas of a pair of enantiomers and specify their absolute configuration.
124 46 Diastereomers

46 Diastereomers
Diastereomers are stereoisomers with two and more stereogenic centers, such as the esters obtained from (S)-mandelic
acid (-hydroxyphenylacetic acid) and racemic 2-butanol:

C 2H 5 C 2H 5 C 2H 5 C 2H 5
SC
SC
CR
H CR
H H H 3C H CH 3
H 3C CH3
O OH + OH HO O O + O O
C C C
SC (S)-2-butanol (R)-2-butanol SC SC
H racemate H H
OH OH OH
(S)-mandelic acid (SS)- (SR)-
mandelic acid 2-butyl ester
diastereomeric esters

Unlike enantiomers, diastereomers do not behave like image and mirror image. Their relative configurations [(SS) and
(SR)], however, do differ. Thus, they possess individual physical properties which permit separation. Racemates can be
resolved by converting them into diastereomers and by separating the latter (Chapter 44.3.4).

46.1 Compounds with Two Different Stereogenic Centers


Two different stereogenic centers, such as in 3-bromo-2-butanol, give rise to four (22 = 4) stereoisomers a-d. Following
the procedure outlined in Chapter 44.3.1, the absolute configurations can be specified. For example, ranking of substitu-
ents at both asymmetrically substituted carbon atoms in formula a decreases counterclockwise, so that the stereoisomer
is assigned as (2S,3S)-3-bromo-2-butanol. Isomer b with the mirrored configurations (2R,3R) is then the enantiomer of a;
stereoisomers c and d represent another pair of enantiomers. However, isomeric pairs a,c and a,d, as well as b,c and b,d,
are diastereomeric, differing in relative configuration and not like image and mirror image.
C-1 CH 3 CH3 CH 3 CH 3
C-2 H C OH HO C H H C OH HO C H
C-3 Br C H H C Br H C Br Br C H
C-4 CH 3 CH3 CH 3 CH 3
a b c d
(2S,3S) (2R,3R) (2S,3R) (2R,3S)

To summarize, two diastereomers, each one representing a pair of enantiomers, exist for compounds with two different
stereogenic centers. Diastereomers have different relative configurations [e.g. (2S,3S)- and (2S,3R)-] while enantiomers
exhibit mirrored absolute configurations [e.g. (2S,3S)- and (2R,3R)-]. Diastereomers with different absolute configurations
of only one stereogenic center, such as (2S,3S)- and (2S,3R)-3-bromo-2-butanol, are also referred to as epimers.
Absolute configurations of amino acids (Chapter 68), carbohydrates (Chapter 71) and other natural products are usually
specified by the FISCHER convention: The absolute configuration of carbohydrates with two or more stereogenic centers is
related to the stereogenic center next to the highest numbered CH2OH in the carbon chain: OH pointing to the right
defines a D, OH pointing to the left an L configuration. The epimeric aldotetroses threose and erythrose (sugars with four
carbons in the chain, Chapter 71.1) exemplify this:

1
threose erythrose
CH=O CH=O CH=O CH=O C-1
2
HO C H H C OH H C OH HO C H C-2
3
H C OH HO C H H C OH HO C H C-3
4
CH 2OH CH 2OH CH 2OH CH 2OH C-4
a b c d
FISCHER (D,L) D L D L configuration at C-3
CIP (R,S) (2S,3R) (2R,3S) (2R,3R) (2S,3S)

The pairs a,b and c,d are enantiomers, referred to as threo- and erythro-DL pairs, each pair having mirrored configura-
tions. Each one of the four aldotetroses is optically active. Equimolar mixtures of the pairs a,b and c,d are racemic and

Efficiently Studying Organic Chemistry: Exam training for chemists, biochemists, pharmacists, life and health scientists,
Third Edition. Eberhard Breitmaier. © 2022 WILEY-VCH GmbH. Published 2022 by WILEY-VCH GmbH.
46.2 Compounds with Two Equally Substituted Stereogenic Centers 125

optically inactive. Both DL pairs (racemates) differ in relative configuration, specified by threo- and erythro-, and therefore
exhibit different melting points and other physical properties. The configurational relationship between all isomeric pairs
except the enantiomers a,b and c,d is diastereomeric since they do not differ like image and mirror image. An equimolar
mixture of two diastereomers is not a racemate.
To generalize, 2n stereoisomers and 2n/2 diastereomers as pairs of enantiomers exist for compounds with n different
stereogenic centers. As shown, 3-bromo-2-butanol and aldotetroses with two different asymmetrically substituted carbon
atoms (n = 2) form four stereoisomers, and these consist of two diastereomeric pairs of enantiomers.

46.2 Compounds with Two Equally Substituted Stereogenic Centers


Compounds that possess two stereogenic centers with identical substituents exist as two enantiomers, (R,R) and (S,S),
and a meso-isomer (meso from Greek mesos = center). meso-Isomers with the absolute configurations (R,S) and (S,R)
can be superimposed because of axial symmetry and therefore are not enantiomeric. They differ from the (R,R)- and
(S,S)-enantiomer as well as from the racemic mixture of them [(R,R)/(S,S)]. 2,3-Dihydroxybutanedioic acid, better known
as tartaric acid, is an example: the (2R,3R)-(+)-enantiomer occurs in grapes and many other fruits; following the FISCHER
convention, this is L-(+)-tartaric acid. Table 46.1 summarizes the properties of all stereoisomers of this acid.

CO2H CO2H CO2H CO2H


H C OH HO C H H C OH HO C H
HO C H H C OH H C OH HO C H
CO2H CO2H CO2H CO2H
(2R,3R)- (2S,3S)- (2R,3S)- (2S,3R)-
L-(+)-tartaric acid D-()-tartaric acid meso-tartaric acid

Table 46.1. Physical properties of the tartaric acids


specific rotation melting point density at 20 °C solubililty
[] 20 (c = 17.4 in H 2O) [°C] [ g/mL] [ g/100 g H 2O ]
D
D-tartaric acid  12.7° 171-174 1.7598 139
L-tartaric acid (natural) + 12.7° 171-174 1.7598 139
DL-tartaric acid (racemate) 0° 206 1.788 20.6
meso -tartaric acid 0° 146 1.666 125

Among the three structurally isomeric cyclohexanediols, the 1,4-isomer has no stereogenic center because of symmetry;
two configurational isomers cis- and trans- exist (Table 20.1). However, 1,3- and 1,2-cyclohexanediol also possess two
equally substituted stereogenic centers. Their cis-isomers represent meso forms [(R,S)/(S,R)] because of axial symmetry
while the trans-isomers exist as pairs of enantiomers [(R,R)/(S,S)]:
cis- trans-
(S,R) (R,S) (R,R) (S,S)
OH ring ring OH
inversion OH OH inversion
1,2-cyclohexanediol OH OH HO HO OH HO
OH HO

1,3-cyclohexanediol HO OH HO OH OH HO
HO OH
meso forms enantiomers

________________________________________________________________________________________________
Chapter 46 permits answers to the following:
(46.1) How do enantiomers and diastereomers differ?
(46.2) Which stereoisomers exist for 3-bromo-2-butanol? Specify their configurations and classify them.
(46.3) Which stereoisomers exist for 2,3,4-trihydroxybutanoic acid? Specify their configurations and classify them.
(46.4) Which stereoisomers exist for 2,3-dihydroxybutanedioic acid? Characterize a meso-isomer.
(46.5) Draw all stereoisomers of (a) 1,2-, (b) 1,3-, and (c) 1,4-diaminocyclohexane and fully name them.
126 47 Aldehydes

47 Aldehydes
47.1 Survey, Nomenclature
Aldehydes belong to the carbonyl compounds which contain the carbonyl (CO) double bond. Their functional group is
the aldehyde or formyl group, CH=O.
IUPAC names of aldehydes are derived from those of alkanes, alkenes, and alkynes with the same chain length. The
ending e of the hydrocarbon is replaced by al for an aldehyde, and by edial for a dialdehyde (Table 47.1). Common
names originate from the common names of the carboxylic acids with the same chain length, related to their natural
occurrence (Chapter 41.1), the ending aldehyde replacing ic acid or oic acid (Table 47.1). Thus, the common name of
methanal is formaldehyde derived from formic acid, and benzaldehyde comes from benzoic acid. An aldehyde group
attached to a cycloaliphatic or heterocyclic ring is also named -carbaldehyde; cyclobutanecarbaldehyde or pyridine-2-
carbaldehyde (Table 47.1) are examples.

Table 47.1. Aldehydes (alkanals, alkenals, alkynals, arenealdehydes, dialdehydes)


alkanals alkenals alkynals
O
methanal
H C (formaldehyde)
H
O
ethanal
CH 3 C (acetaldehyde)
H H
O H C O O
propanal propenal propynal
CH 3 CH 2 C C C H C C C (propargylaldehyde)
(propionaldehyde) (acrolein)
H H H H
H
O H C O O
butanal (E)-2-butenal
CH 3 CH 2 CH 2 C C C H 3C C C C 2-butynal
(butyraldehyde) (crotonaldehyde)
H H 3C H H
O H
pentanal H 3C C O
CH 3 CH 2 CH 2 CH 2 C (valeraldehyde) (Z)-2-butenal
H C C
O H H
6 5 4 3 2 1 hexanal
CH 3 CH 2 CH 2 CH 2 CH 2 C (caproaldehyde)
H
arenealdehydes (aromatic and heteroaromatic aldehydes)
O H
O O O O C O

C C C N C C
H H H H H

HO HO
OCH 3 pyridine-2-carbaldehyde - -
benzaldehyde p-hydroxybenzaldehyde vanillin (pyridine-2-aldehyde) naphthaldehyde
dialdehydes O O
O O
O O O O O O C C C C
H H H H
C C C CH 2 C C CH 2 CH 2 C
H H
H H H H H H C C
ethanedial propanedial butanedial O O
(glyoxal) (malondialdehyde) (succindialdehyde) o- m- p-
benzenedialdehyde
phthalaldehyde isophthalaldehyde terephthalaldehyde

Volatile aldehydes such as formaldehyde, acetaldehyde, acrolein, and crotonaldehyde radiate pungent odors, causing
lacrimation, coughing, bronchial spasms, and irritation of mucous membranes. Benzaldehyde, with an odor like almond
oil, and vanillin (Table 47.1) with its pleasant vanilla odor and taste, are used as flavors and fragrances.

Efficiently Studying Organic Chemistry: Exam training for chemists, biochemists, pharmacists, life and health scientists,
Third Edition. Eberhard Breitmaier. © 2022 WILEY-VCH GmbH. Published 2022 by WILEY-VCH GmbH.
47.2 Preparation 127

47.2 Preparation
47.2.1 Oxidation of Methyl and Hydroxymethyl Groups
The oxidation of primary alcohols by air oxygen in the presence of copper or by various oxidation reagents is a general
method to prepare aldehydes (Chapter 35.2). Methyl groups attached to benzenoid or heteroaromatic rings are also
oxidized to aldehyde functions by air oxygen in the presence of catalysts.
O
V2O5 , MoO3
R CH 2 OH R C + O2 O + H 2O
N CH3 N C
 2 [H ] ,  2 e0
H
primary alcohol aldehyde H
2-methylpyridine (-picoline) pyridine-2-carbaldehyde

47.2.2 Hydrolysis of 1,1-Dihaloalkanes


,-Dihaloalkyl groups are hydrolyzed to provide aldehydes in a process involving intermediate aldehyde hydrates
(Chapter 49.1.1). Benzaldehyde can be produced from toluene by photochlorination to yield ,-dichlorotoluene which
undergoes hydrolysis to the unstable benzaldehyde hydrate.
O
CH3 h , heat CHCl2 + 2 H2 O CH(OH)2 C
+ 2 Cl2 H
 2 HCl  2 HCl  H2O
toluene ,-dichlorotoluene (benzal chloride) benzaldehyde hydrate benzaldehyde

47.2.3 Reduction of Carboxylic Acid Derivatives


Acyl halides are catalytically hydrogenated in the presence of a deactivated catalyst (Pd/ BaSO4, LINDLAR catalyst) to
provide aldehydes (ROSENMUND reduction):
O O O
+ Pd (BaSO4 ) + H2
R C R C R C
 Pd,  HCl
Cl PdCl H

Complex metal hydrides (LiAlH4) reductively substitute N -alkylarylcarboxamides (anilides) to give aldehydes:
O
C CH 3 O CH 3
LiAlH 4 , tetrahydrofuran N
R N + Li H C + Li
R H

aldehyde
carboxylic acid N-methylanilide lithium N-methylanilide

47.2.4 Formylation of Arenes


A reaction which introduces the aldehyde or formyl group is known as formylation. The GATTERMANN-KOCH formylation
involves electrophilic substitution of donor-substituted benzenes by formyl chloride originating from hydrogen chloride and
carbon monoxide. Similar to FRIEDEL-CRAFTS acylation, this reaction requires a LEWIS acid (AlCl3, CuCl) to generate the
formyl cation [CH=O] as intermediate electrophile.
HCl + CO
O O
AlCl3 , CuCl
R H + Cl C R C + HCl
H H
formylchloride p-substituted benzaldehyde

N-Methylformanilide, which provides the intermediate formyl cation in the presence of phosphoryl chloride as LEWIS acid,
is a commercial reagent for electrophilic formylation (VILSMEIER formylation):
O
H C POCl3
O H
H 3CO H + N H 3CO C + N
H3C H H 3C
anisole N-methylformanilide p-methoxybenzaldehyde (+ o-isomer) N-methylaniline
128 47 Aldehydes

REIMER-TIEMANN formylation with chloroform and potassium hydroxide involves highly reactive dichlorocarbene (:CCl2 or
ICCl2) as electrophile generated by -elimination of hydrogen chloride; p-substituted phenols can be formylated at the o-
position by way of intermediate dichloromethyl derivatives.
CHCl3 + OH

 H2O ,  Cl O
O O O OH
CCl2
H H CHCl2 + H3O C
+ ICCl2 H
 2 HCl

R R R R

47.3 Molecular Shape, Resonance Formulas, Reactivity


Molecular models of ethanal (acetaldehyde) illustrate that all bonds originating from the carbonyl carbon atom are copla-
nar, enclosing bond angles of 120° (Fig. 47.1).

Fig. 47.1. Molecular models (tube, ball-spoke, space-filling) of acetaldehyde

As shown by resonance formulas, the ()-M effect of the oxygen polarizes the carbonyl carbon atom of aldehydes (and
ketones as well) such that it functions as an electrophile which attracts nucleophiles. This is the background behind all
carbonyl reactions covered in Chapter 49.

_ _ _
Nu + C OI
_ C OI
_ Nu C OI
_
carbonyl resonance

47.4 Reactions Specific for Aldehydes


47.4.1 Oxidation to Carboxylic Acids, Identification Reactions
Unlike ketones, aldehydes are reducing reagents because they are readily oxidized to carboxylic acids. Three test tube
reactions for the identification of aldehydes make use of this reduction reactivity.
The TOLLENS reaction generates a silver mirror (Ag  Ag) with silver nitrate in aqueous ammonia solution:
H
R C + 2 [Ag(NH3)2] + 2 OH R CO2 NH4 + H 2O + 3 NH 3 + 2 Ag
O

The FEHLING reaction produces a reddish-brown precipitate of cuprous oxide (Cu2O, Cu2  Cu) with alkaline cupric
salt solutions:
H
R C + 2 Cu 2 + Na + 5 OH R CO2 Na + 3 H 2O + Cu 2O
O

The NYLANDER reaction gives a black precipitate of bismuth (Bi3  Bi) with alkaline bismuth(III) salt solutions:
H
3R C + 2 Bi 3 + 9 OH 3 R CO2 + 6 H 2O + 2 Bi
O
47.4 Reactions Specific for Aldehydes 129

47.4.2 CANNIZZARO Disproportionation of Aromatic Aldehydes


Representing an intermediate oxidation state, aldehydes are oxidized to carboxylic acids and reduced to primary alco-
hols. Accordingly, benzaldehydes undergo a self-oxidation-reduction (disproportionation) in alkaline solution to give
benzyl alcohols and benzoates, known as the CANNIZZARO disproportionation.
O
2 Ar C + OH Ar CO2 + Ar CH 2 OH
H
arenealdehyde (benzaldehyde) carboxylate (benzoate) primary alcohol (benzyl alcohol)

Self-oxidation-reduction starts with nucleophilic addition of a hydroxide anion to the electrophilic carbonyl carbon. The
intermediate -hydroxyalkoxide anion loses a nucleophilic hydride anion which is transferred to the electrophilic carbonyl
carbon of another aldehyde molecule. Primarily formed carboxylic acid and alkoxide finally equilibrate in favor of carbox-
ylate and primary alcohol.
_
O I OI O
_
2 Ar C + IOH
_ Ar C OH + C Ar
H H H
-hydroxyalkoxide
anion
_
O IOI
Ar C + H C Ar Ar CO2 + HO CH 2 Ar
OH H

Self-oxidation-reduction of an alkanal, known as the TISHCHENKO reaction, requires the presence of very strong bases
such as aluminum ethoxide and produces the ester of the carboxylic acid and the primary alcohol.
O O
Al(OC2H 5) 3
2R C R C
H O CH 2 R (R = alky l)

47.4.3 Addition of Hydrogensulfite


Resonance formulas show that the sulfur atom in hydrogensulfite is nucleophilic. Nucleophilic addition of the hydrogen-
sulfite anion to the carbonyl carbon of an aldehyde produces an -hydroxy sulfonic acid as the so-called bisulfite adduct.
_
O O O Na IOI OH OH
_ _
R C + I S OH
_ I S OH
_ Na R C S O R C SO3 Na
H O _I
IO H O H
sodium hydrogensulfite bisulfite adduct

The reaction occurs with aliphatic aldehydes and sterically unhindered ketones (with small alkyl groups). It can be used
for purification (crystallization) of these carbonyl compounds which are then regenerated from the adducts by heating in
slightly acidic or alkaline aqueous solution.
________________________________________________________________________________________________
Chapter 47 permits answers to the following:
(47.1) Draw the structural formulas of (a) cis- and trans-2-hexenal, (b) 5-hexenal, (c) 2-hexynal, and (d) hexanedial.
(47.2) Draw the structural formulas of all aldehydes with the molecular formula C5H8O and name them.
(47.3) Formulate equations to describe the preparation of benzaldehyde from toluene.
(47.4) Formulate equations to describe the preparation of hexanedial from adipic acid (hexanedioic acid).
(47.5) Which reactions permit preparation of p-methoxybenzaldehyde from methyl phenyl ether (methoxybenzene)?
(47.6) One test tube contains butanal, the other butanone (C4H8O, Table 48.1). Formulate three test tube reactions to
identify the aldehyde.
(47.7) What happens when p-tolualdehyde (p-methylbenzaldehyde) is reacted with sodium hydroxide? Formulate
the mechanism.
130 48 Ketones

48 Ketones
48.1 Survey, Nomenclature
Aldehydes and ketones are carbonyl compounds containing the carbonyl double bond. Differing from aldehydes
(RCH=O), ketones contain two alkyl or aryl residues connected to the carbonyl group (R2C=O).
The names of alkanones, alkenones, and alkynones are derived from the corresponding alkanes, alkenes, and alkynes
by replacing the ending e of the hydrocarbon by one for ketones and edione for diketones (Table 48.1). For longer alkyl
chains, the position of the carbonyl group must be indicated by the lowest possible number. Alternatively, the names of
alkyl groups can be listed in alphabetical order, followed by the ending ketone. The simplest ketone propanone is dime-
thyl ketone and has the common name acetone (containing the acetyl group like acetic acid). The carbonyl group is part
of the ring in cycloalkanones (Table 48.1). Ketones with one or two aromatic residues are named phenones (e.g. aceto-
phenone); enones such as butenone and 2-cyclohexenone are (,-) unsaturated ketones.

Table 48.1. Ketones (alkanones, alkenones, alkynones, cycloalkanones, cycloalkenones, phenones, alkanediones, cycloalkanediones)
alkanones alkenones (enones) al k y n o n e s
O
propanone
H 3C C (dimethyl ketone, acetone) H O O butynone
CH 3
C C butenone H C C C (ethynyl methyl
O
butanone H C CH 3 (methyl vinyl ketone) CH 3 ketone)
CH 3 CH 2 C (ethyl methyl ketone)
CH 3 H
H O
O O
2-pentanone C C (E)-3-penten-2-one 3-pentyn-2-one
CH 3 CH 2 CH 2 C (methyl n-propyl ketone) H 3C C C C (methyl 1-
H 3C C CH 3 (trans-methyl 1-
CH 3
H
propenyl ketone) CH 3 propynyl ketone)
O
2-hexanone
CH 3 CH 2 CH 2 CH 2 C (n-butyl methyl ketone)
CH 3
O
3-pentanone
CH 3 CH 2 C
(diethyl ketone)
CH 2 CH 3
O
3-hexanone
CH 3 CH 2 CH 2 C (ethyl n-propyl ketone)
CH 2 CH 3
cycloalkanones cycloalkenones
O O O O O O
O O

cyclopropanone cyclobutanone cyclopentanone cyclohexanone cycloheptanone bicyclo[2.2.1]heptan-2-one 2- 3-


cyclohexenone
phenones (aryl ketones)
O O OH O O O
C C C C N C
CH 3 CH 3 CH 3 CH 3

HO
acetophenone p-hydroxyacetophenone o-hydroxyacetophenone benzophenone 2-acetylpyridine
(methyl phenyl ketone) (diphenyl ketone) (methyl 2-pyridyl ketone)
diketones (alkanediones) cycloalkanediones
O H H
O O O O O
C CH 3 H 3C C CH 3
H 3C C C C C CH 2 CH 2 C
O O H 3C CH 3 O O
O
butanedione 2,4-pentanedione 2,5-hexanedione O
(diacetyl, butter flavor) (acetylacetone) (acetonylacetone) 1,2- 1,3- 1,4-
cyclohexanedione

The pairs 2- and 3-pentanone, 2- and 3-hexanone, and 2- and 3-cyclohexenone are structurally isomeric (regioisomeric)
ketones differing in the position of the carbonyl group or the CC double bond (Table 48.1).

Efficiently Studying Organic Chemistry: Exam training for chemists, biochemists, pharmacists, life and health scientists,
Third Edition. Eberhard Breitmaier. © 2022 WILEY-VCH GmbH. Published 2022 by WILEY-VCH GmbH.
48.2 Preparation 131

48.2 Preparation
48.2.1 Oxidation of Secondary Alcohols
The oxidation of secondary alcohols yields ketones. Ecologically harmful reagents such as chromium trioxide (CrO 3) in
acetic acid can be replaced by organic oxidation reagents such as acetone in the presence of aluminum t-butoxide (OP-
PENAUER oxidation).

R R R R
CH 3CO2H
CH OH C O 3 R C OH + 2 CrO3 3 C O + Cr2O3 + 3 H 2O
 2 [H ] ,  2 e0
R R H R
secondary alcohol ketone R CH 3 R CH3
Al [OC(CH3) 3]3
R C OH + O C C O + HO C CH 3
H CH 3 OPPENAUER R H
oxidation
secondary alcohol acetone ketone 2-propanol

48.2.2 Catalytic Oxidation of Alkenes by Air (WACKER Process)


Catalytic oxidation of terminal alkenes (vinyl groups) by air opens access to methyl ketones (WACKER process). Acetone
(R = CH3) is produced industrially from propene in this way (alternative: Chapter 52.3.1).
H 2C H 3C
PdCl2 , CuCl2
CH + 1/2 O2 C O
R R
terminal alkene methyl ketone

48.2.3 Oxidation of Activated Methylene Groups (RILEY Oxidation)


Methylene groups adjacent to carbonyl groups are oxidized to a keto function by selenium dioxide (RILEY oxidation), so
that a 1,2-dicarbonyl compound is obtained, for example cyclohexane-1,2-dione from cyclohexanone.
O O
1,4-dioxane
+ SeO2 + Se + H 2O
H O
H
cyclohexanone cyclohexane-1,2-dione

48.2.4 Acylation of Arenes to Phenones (FRIEDEL-CRAFTS Acylation)


Electrophilic FRIEDEL-CRAFTS acylation of aromatics (Chapter 25.4) with acylation reagents such as carboxylic acid halides
or anhydrides in the presence of a LEWIS acid is a general method to prepare phenones.
CH 3
O C
AlCl 3 O
+ Cl C
H 3CO OCH 3  HCl H 3CO OCH 3
CH 3
resorcinol dimethyl ether 2,4-dimethoxyacetophenone
(1,3-dimethoxybenzene)

________________________________________________________________________________________________
Chapter 48 permits answers to the following:
(48.1) Draw the structural formulas of all unbranched heptanones and name these compounds.
(48.2) Draw the structural formulas of all isomeric ketones with the molecular formula C 5H8O and name them.
(48.3) Which secondary alcohol can be oxidized to 3,3-dimethyl-2-pentanone? Formulate the reaction.
(48.4) Write equations which suggest methods to prepare 1,2-cyclopentanedione from an appropriate (a) diol and
(b) ketone.
(48.5) Acetylation of 1,3-dimethoxybenzene gives 2,4-dimethoxyacetophenone as the major product. Why?
(48.6) Look again at Chapter 36.2 and suggest a method to prepare t-butyl methyl ketone (3,3-dimethyl-2-butanone).
132 49 Carbonyl Reactions

49 Carbonyl Reactions
Resonance formulas of the carbonyl group show that the ()-M effect of the oxygen polarizes the carbonyl carbon atom of
aldehydes and ketones. The resulting electrophilic carbonyl carbon atom attracts nucleophiles to form new covalent
bonds. This is the principle of all carbonyl reactions of aldehydes and ketones.

_ _ _
C OI
_ C OI
_ + Nu Nu C OI
_

aldehyde or ketone: carbonyl resonance

49.1 Reactions with Oxygen and Sufur Nucleophiles


49.1.1 Hydration (Water as Nucleophile)
Water as an oxygen nucleophile adds to the electrophilic carbon of aldehydes and ketones. Equilibration with hydrates
occurs in aqueous solution. Trichloroacetaldehyde hydrate (chloral hydrate R1 = CCl3 , R2 = H) crystallizes from aqueous
solution; hexafluoroacetone hydrate (R1 = R2 = CF3) is a stable liquid. Apart from these exceptions, aldehyde and ketone
hydrates cannot be isolated; they are unstable due to two hydroxy groups attached to one carbon atom (ERLENMEYER’s
rule).
H R1 H R1 R1
R2 = H : aldehyde hydrate
O + C O O
 C O HO C OH
R2 = alkyl, aryl : ketone hydrate
H R2 H R2 R2

49.1.2 Formation of Acetals and Ketals (Alcohols as Nucleophiles)


Analogously to water, alcohols as oxygen nucleophiles add to the carbonyl carbon of aldehydes or ketones, primarily
yielding a hemiacetal or hemiketal:

R1 H IOI H OH
C OI + O R1 C O R1 C OR' + [H ]
R2 R' R2 R' R2
R1 = alkyl or aryl, R2 = H: aldehyde hemi- or semiacetal
R1 = R2 = alkyl or aryl: ketone hemi- or semiketal

In the presence of anhydrous acids, the protonated hemiacetal (hemiketal) undergoes dehydration to give an intermedi-
ate oxonium-carbenium ion susceptible to addition of a second nucleophilic alcohol molecule, finally yielding an acetal or
ketal:

OH 2 R1 R1 OR' R' OR'


H 2O _ + ROH  [H ]
R1 C OR' C _ R'
O C _ R'
O R1 C O R1 C OR'
R2 R2 R2 R2 H R2
protonated hemiacetal (hemiketal) oxonium-carbenium ion protonated acetal (ketal) acetal (ketal)

49.1.3 Formation of Thioacetals (Mercaptals)


Analogously to acetals and ketals, thioacetals and thioketals (mercaptals) are formed by reacting thiols as sulfur nucleo-
philes with aldehydes or ketones.
R1 thiol (mercaptan) R1
[H ]
C O + 2 R' SH R2 C SR' + H 2O
R2 SR'
R1 = alkyl or aryl, R2 = H: aldehyde thioacetal
R1 = R2 = alkyl or aryl: ketone thioketal

Having two nucleophilic sulfur atoms, propane-1,3-dithiol undergoes cyclodehydration with aldehydes, yielding cyclic
thioacetals. In these 1,3-dithianes, the former electrophilic aldehyde carbon is CH acidic due to resonance stabilization of

Efficiently Studying Organic Chemistry: Exam training for chemists, biochemists, pharmacists, life and health scientists,
Third Edition. Eberhard Breitmaier. © 2022 WILEY-VCH GmbH. Published 2022 by WILEY-VCH GmbH.
49.2 Reactions with Nitrogen Nucleophiles 133

the carbanion arising from deprotonation and therefore behaves as a nucleophile (reversal of carbonyl polarity, called
carbonyl umpolung by COREY and SEEBACH):
O
+ RLi
C + HS SH S S S _ S S S S S Li
R H  H2O C  RH C C C

aldehyde propane-1,3-dithiol R H R R R
2-alkyl-1,3-dithiane resonance formulas of lithiated 2-alkyl-1,3-dithiane

Nucleophilic lithiated 1,3-dithianes can be alkylated by electrophilic alkyl halides. Mercuric chloride reacts with the result-
ing 2,2-dialkyl-1,3-dithianes to liberate the ketones obtained in this way from aldehydes:
(HgCl 2 + H2O) R
R'
S _ R Li + R' X S R + HgO
C
S R' O
 Li X
S S Hg
ketone

lithiated 2-alkyl-1,3-dithiane 2,2-dialkyl-1,3-dithiane S

49.2 Reactions with Nitrogen Nucleophiles


The nucleophilic amino group of a primary amine adds to the carbonyl carbon of aldehydes and ketones to yield an imine
also known as an azomethine or a SCHIFF base (Chapter 39.6), while nucleophilic addition of a secondary amine to an
aldehyde or a ketone produces an enamine (Chapter 39.7):
R
R R N R
_ _
C OI + H2N R C N C C + IN R C C
 H 2O
 H2O H O H
aldehyde primary amine imine aldehyde secondary amine enamine
or ketone (azomethine, SCHIFF base) or ketone

In this way, crystalline 2,4-dinitrophenylhydrazones and oximes, which are used to identify aldehydes and ketones by
means of melting points, are obtained by reacting the carbonyl compounds with 2,4-dinitrophenylhydrazine or hydroxyl-
amine in the presence of anhydrous acid. Configurational isomers may be produced with aldehydes or unsymmetrically
substituted ketones (e.g. syn- and anti-oximes).
aldehyde O2N
or ketone H acetaldehyde (cis- / trans-isomers) OH
O2N
R1 R1 N NO2 H3C O H3C N H3C N
C O + H2N NH NO2 C C OH and C
C N + H2N OH
 H2O H  H2O
R2 R2 H H
anti- syn-
2,4-dinitrophenylhydrazine 2,4-dinitrophenylhydrazone hydroxylamine
of an aldehyde or a ketone acetaldoxime

Nucleophilic addition of a secondary amine such as dimethylamine to formaldehyde yields an intermediate carbenium-
iminium ion resonance hybrid which acts as an electrophile in MANNICH reactions with CH acidic compounds (Chapter
50.2.3) and other carbon nucleophiles (Chapter 63.3.1):
H 3C H 3C H H 3C H H3C H H 3C H H 3C H
HCl + [H ]
H3C H H H 3C
NI + C N C H N C H N C H N C N C
H2 O
H O H IOI
_ H 3C OH H3C OH 2 H 3C H H 3C H
dimethyl- carbenium ion iminium ion
amine formaldehyde
MANNICH electrophile

49.3 Reactions with Carbon Nucleophiles


49.3.1 Alkynylation
Strong bases such as sodium amide in liquid ammonia deprotonate terminal alkynes to provide alkynylides:
R' C C H + Na NH 2 R' C CI Na + NH 3
sodium alkynylide
134 49 Carbonyl Reactions

Nucleophilic addition of alkynylides to aldehydes and ketones yields alkynolates and finally alkynols, exemplified by the
reaction of cyclohexanone with acetylene to give the tertiary alkynol 1-ethynylcyclohexanol:
R2 R2 NaNH2
H
C C
R1 R2 liquid NH 3 + NH3 liquid NH3
C + IC C R' R1 C C C R' R1 C C C R' + H C C H
 NH2  78 °C
O IOI
_ OH O OH
alkynolate alkynol cyclohexanone 1-ethynylcyclohexanol

49.3.2 Cyanohydrin and Benzoin Reaction


Hydrogen cyanide, prepared in situ from potassium cyanide and acetic acid, converts aldehydes and ketones into cyano-
hydrins (-hydroxy nitriles), involving nucleophilic addition of cyanide to the carbonyl carbon.
_ _
C OI _I
C O + H IC NI C OH cyanohydrin (-hydroxy nitrile)
CN

Such cyanohydrin reactions convert cyclohexanone into 1-cyanocyclohexanol, and benzaldehyde into racemic benzalde-
hyde cyanohydrin which can be hydrolyzed to racemic mandelic acid:
KCN , CH3CO2H H KCN , CH3CO2H H H O
+ H C N + H C N + 2 H2O
C N C C C N C C
 NH3
O OH O OH OH OH
cyclohexanone 1-cyanocyclohexanol benzaldehyde DL-benzaldehyde cyanohydrin DL-mandelic acid

In alkaline solution, however, benzaldehyde and other arenealdehydes undergo the reversible benzoin reaction with
cyanide, starting with a nucleophilic addition of the cyanide anion to the carbonyl carbon (1). The cyano group in the
intermediate cyanoalkoxide acidifies the former aldehyde proton which migrates to the former aldehyde oxygen (2). The
resulting carbanion adds to another molecule of aldehyde (3). Proton exchange in the adduct generates an alkoxide (4)
which releases the cyanide anion to produce a racemic -hydroxyketone (5), also referred to as a benzoin. It is stabilized
by an intramolecular hydrogen bridge.
H CN CN H benzaldehyde
(1) (2)
2 Ar C + IC NI Ar C H Ar CI + C Ar H + H
C C
O I OI
_ OH O
O O
arenealdehyde cyanoalkoxide KCN

NC H NC H (5) Ar H H
(3) Ar C (4)
C Ar Ar C C Ar  CN C C Ar C C
O IOI
_ _I
IO O O O O O
H H H H
a benzoin DL-benzoin

49.3.3 Addition of GRIGNARD Compounds


The nucleophilic alkyl group of alkylmagnesium halides adds to the carbonyl carbon of aldehydes and ketones (GRIGNARD
reaction). The resulting secondary and tertiary alkoxides are hydrolyzed to the corresponding alcohols, as shown in
Chapter 33.3.7. As another example, methylmagnesium bromide alkylates cyclohexanone to give the tertiary alcohol 1-
methylcyclohexanol:

CH3 + H2O CH3


+ H3C MgBr
O  Mg(OH)Br
O Mg Br OH
cyclohexanone tertiary alkoxide 1-methylcyclohexanol

49.3.4 WITTIG Alkenylation (Carbonyl Alkenylation)


Alkyl halides alkylate triphenylphosphane to provide alkyltriphenylphosphonium salts. These are deprotonated by organo-
lithium compounds to give carbon nucleophiles, formulated as resonance hybrids of ylides and ylenes:
+ Li C 6H5 _
R2CH X + IP(C6H5)3 R2CH P(C6H5)3 X R 2C P(C6H 5)3 R 2C P(C6H 5)3
 Li X ,  C6H6
X = Cl , Br , I alkyltriphenylphosphonium salt phosphorylide phosphorylene
49.4 Reductions 135

The dipolar ylide and the carbonyl compound (aldehyde or ketone) add to each other so that concerted bond formation
occurs between nucleophilic and electrophilic atoms. The intermediate heterocycle, an oxaphosphetane, undergoes
cycloreversion (a reverse [2+2]-cycloaddition), opening the ring with production of an alkene and triphenylphosphane
oxide, driven by formation of the very stable PO double bond.
R
_ R
C CR 2 C C R
+ C C + O P(C 6H5)3
O P(C6H 5)3 O P(C6H5)3
R triphenyl-
oxaphosphetane alkene phosphane oxide

WITTIG carbonyl alkenylation opens access to a large variety of alkenes, such as methylenecyclohexane:
_
+ H 2C P(C 6H 5)3 H 2C P(C 6H 5)3 + O P(C 6H 5)3
O ylide ylene CH 2
cyclohexanone triphenylphosphonium methylide methylenecyclohexane

49.4 Reductions
Nucleophilic addition of the hydride anion (provided by LiAlH4 as a complex metal hydride) to an aldehyde or a ketone
produces a primary or a secondary alkoxide, used for the preparation of alcohols (Chapter 33.3.5).
R H R1 H
_ LiAlH4 _ _ LiAlH 4 _
C OI + IH R C OI
_ C OI + IH R1 C OI
_
H H R2 R2
aldehyde primary alkoxide anion ketone secondary alkoxide anion

Other methods are available to reduce carbonyl functions to methylene groups. WOLFF-KISHNER reduction starts from the
hydrazone (Chapter 49.2) of the carbonyl compound. An intramolecular hydride shift (from NH2 to C) is the reduction
step. Base-catalyzed loss of nitrogen finally liberates the methylene group.
H
N
O H N H N NH H N NI
_
+ H 2 N NH 2 OH + OH _ + H2O
C C C C R 1 CH R2 R 1 CH 2 R 2
R1 R2  H2O R1 R2 hydride R1 R2  H2O R1 R2  N2  OH
carbonyl hydrazone shift methylene
compound compound

The CLEMMENSEN reduction of aldehydes and ketones with amalgamated zinc permits direct conversion of carbonyl
functions into methylene groups. Phenones are readily reduced to alkylbenzenes by this method:
O
Zn xHg , HCl
C CH 2 CH 2 CH 2 CH3
CH 2 CH2 CH 3 + 4 [H ] + 4 e0
 H 2O
butyrophenone n-butylbenzene
________________________________________________________________________________________________
Chapter 49 permits answers to the following:
(49.1) Formulate a general equation that accounts for the typical reactions of aldehydes and ketones.
(49.2) What products arise from the acid-catalyzed reaction of ethanol with (a) benzaldehyde and (b) propanone (acetone)?
(49.3) Formulate the reactions of (a) benzaldehyde with n-butylamine and (b) cyclopentanone with diethylamine.
(49.4) Formulate the reactions of cyclohexanone with (a) 2,4-dinitrophenylhydrazine and (b) hydroxylamine.
(49.5) Formulate the cyanohydrin reaction of butanal. Which kind of isomers are formed?
(49.6) Propyne reacts with cyclopentanone in liquid NH3 and NaNH2 as base. Write the equation and name the product.
(49.7) Formulate all steps necessary to convert cyclobutanol into methylenecyclobutane, using the WITTIG alkenylation.
(49.8) Suggest reactions for the preparation of (a) 1-methylcyclopentanol and (b) racemic 2-pentanol from appropriate
carbonyl compounds.
(49.9) Which methods are available to convert butyrophenone into n-butylbenzene?
136 50 CH Acidity of Carbonyl Compounds

50 CH Acidity of Carbonyl Compounds


50.1 CH Acidity of Carboxylic Acid Esters
All CH, CH2, or CH3 groups at the -position to a carbonyl group are CH acidic because the abstraction of a proton by a
sufficiently strong base such as an alkoxide (RO) or amide anion (H2N) leaves a resonance-stabilized nucleophile; a
carbanion and an enolate anion are the resonance formulas:
R O  R'OH
R O R O
C C + R'O IC C C C
H
H OR' H OR' H OR'
ester carbanion enolate anion

50.1.1 CLAISEN Condensation


In the presence of sodium ethoxide as base, two molecules of ethyl acetate undergo CLAISEN condensation to give ethyl
acetoacetate, a -keto ester. The nucleophilic carbanion arising from deprotonation of the first ester molecule adds to the
electrophilic carbonyl carbon of a second molecule, thus forming a new CC bond:
OC2H5 H O OC2H5
H3C C + IC C H3C C CH2 CO2C2H5 H3C C CH2 CO2C2H5
O  C2H5O
H OC2H5 IOI
_ OI
_
C electrophile C nucleophile 3-oxobutanoic acid ethyl ester
ethyl acetate (ethyl acetoacetate)

OR' R O OR'
in general: R CH 2 C + IC C R CH2 C CH CO2R' R CH 2 C CH CO2R'
 R'O
O H OR' IOI
_ R OI
_ R
C electrophile C nucleophile -keto ester
carboxylic acid ester

50.1.2 DIECKMANN Cyclocondensation


Intramolecular CLAISEN condensation of alkanedioic acid diesters, known as the DIECKMANN cyclocondensation, yields
cyclic -keto esters. Best results are achieved with 1,4- and 1,5-dioic diesters driven by the favored formation of stable
five- and six-membered rings:
CH2 CO2R  RO
CH
_ CO2R CH CO2R CH CO2R
(CH2)n (CH2)n (CH 2)n (CH2)n
OR  ROH OR n = 3, 4 OR  RO
C C C C
O O OI
I_ O
hexanedioic or heptanedioic cyclopentanone- or cyclohexanone-
acid diester (n = 3, 4) 2-carboxylic acid ester (n = 3, 4)

DIECKMANN cyclodehydration of adipic acid diesters yields 2-(ethoxycarbonyl)cyclopentanone. This cyclic -keto ester can
be hydrolyzed to the -keto acid which readily decarboxylates to yield cyclopentanone.
CO2C 2H5
CH2 CO2C 2H 5 CO2H
NaOC 2 H5  H2 O (OH )
OC2H5
C  C 2H 5 OH  C 2 H5 OH  CO2
O O O
O
adipic acid diethyl ester 2-(ethoxycarbonyl)cyclopentanone 2-oxocyclopentanecarboxylic acid cyclopentanone

50.2 CH Acidity of Aldehydes and Ketones


Aldehydes and ketones are also CH acids if they contain at least one hydrogen atom attached to the carbon  to carbon-
yl. Sufficiently strong bases such as hydroxide (HO) or alkoxide (RO) can abstract this proton, leaving an intermediate
carbon nucleophile which is resonance-stabilized and appropriately described as a hybrid of an acyl carbanion and an
enolate anion:
H R  H2 O R R
_
C C + HO C C C C
O O O
aldehyde (R = H) or ketone acyl carbanion enolate anion

Efficiently Studying Organic Chemistry: Exam training for chemists, biochemists, pharmacists, life and health scientists,
Third Edition. Eberhard Breitmaier. © 2022 WILEY-VCH GmbH. Published 2022 by WILEY-VCH GmbH.
50.2 CH Acidity of Aldehydes and Ketones 137

50.2.1 Aldol Reaction


Nucleophilic addition of an acyl carbanion generated from one molecule of an aldehyde or ketone to the electrophilic
carbon of another molecule produces a new CC bond. Water protonates the resulting alkoxide anion, giving a -hydroxy
aldehyde or ketone called an aldol (abbreviation of aldehyde-alcohol) or ketol:
H R R H R R H R R
+ H2O ,  OH
C C + IC C C C C C C C C C
O O IOI
_ O OH O
electrophile nucleophile aldol (R = H) or ketol
(aldehyde or ketone after deprotonation)

This method of CC bond formation is called the aldol reaction. Aldehydes are converted into -hydroxy aldehydes, exem-
plified by acetaldehyde which undergoes aldol reaction to give (racemic) 3-hydroxybutanal.
H H H H H H H H
OH OH
H C C + H C C H C C C C H 3C C + H 3C C H 3C C CH 2 C
O O OH O O O OH O
-hydroxy aldehyde (aldol) acetaldehyde (ethanal) 3-hydroxybutanal (acetaldol)

Subsequent acid-catalyzed dehydration of the alcohol part, which is susceptible to -elimination, introduces a double
bond conjugated to the carbonyl group. This so-called aldol condensation produces ,-unsaturated carbonyl com-
pounds, such as trans-2-butenal from 3-hydroxybutanal.
H H H H3C H
acid , -unsaturated carbonyl compound
aldol (3-hydroxybutanal) H3C C C C C C
 H2O
(trans-2-butenal, crotonaldehyde)
OH H O H C O
H

50.2.2 CLAISEN Condensation


Nucleophilic addition of the acyl carbanion originating from an aldehyde or ketone to the electrophilic carbonyl carbon of
an ester constitutes a general method to prepare 1,3-diketones. This kind of mixed CLAISEN condensation permits the
production of acetylacetone (2,4-pentanedione) from ethyl acetate and acetone:
H H
OC 2H 5 H O OC 2H 5 O
NaOC2H 5 H 3C C CH 3
H 3C C + IC C H 3C C CH 2 C C C
O CH 3  C2H 5O
H CH 3 IOI
_ O O
C electrophile C nucleophile 2,4-pentanedione
ethyl acetate from acetone (acetylacetone)

50.2.3 MANNICH Reaction


CH acidic carbonyl compounds can be aminomethylated with the electrophile obtained from dimethylamine and form-
aldehyde (Chapter 49.2). This MANNICH reaction converts acetophenone to 3-(N,N-dimethylamino)-1-phenyl-1-propanone
which eliminates dimethylamine when heated, producing phenyl vinyl ketone.
H3C H H3C H O H3C O O
90 °C
N C N C + H CH2 C N CH 2 CH 2 C H2C CH C
H H  [H ]  (H3C)2NH
H3C H3C H3C

MANNICH electrophile acetophenone 3-(N,N-dimethylamino)-1-phenyl-1-propanone phenyl vinyl ketone


from formaldehyde and dimethylamine (MANNICH base)

________________________________________________________________________________________________
Chapter 50 permits answers to the following:
(50.1) Explain why carboxylic acid derivatives, aldehydes, and ketones are CH acidic.
(50.2) Write equations for (a) a CLAISEN condensation and (b) an aldol reaction with subsequent dehydration.
(50.3) Formulate the reaction of ethyl formate and ethyl acetate in the presence of sodium ethoxide and name the product.
(50.4) Write equations to describe a synthesis of cyclopentanone from adipic acid.
(50.5) Suggest a simple synthesis of 2-methyl-2-pentenal (trans- and cis-isomers) by aldol reaction and dehydration.
(50.6) Which reaction permits production of 2,4-pentanedione? Suggest an analogous preparation of 3,5-heptanedione.
138 51 1,3-Dicarbonyl Compounds

51 1,3-Dicarbonyl Compounds
1,3-Dicarbonyl compounds include propanedioic acid diesters such as diethyl malonate, 3-oxoalkanoic acids (-keto
esters) such as ethyl acetoacetate, 1,3-diketones such as acetylacetone, 1,3-dialdehydes such as propanedial (malon-
dialdehyde), and 3-oxoaldehydes (-keto aldehydes) such as 3-oxobutanal (formylacetone).
H H H H H H H H H H H H
H 3CH 2CO C OC 2H 5 H 3C C OC 2H 5 H3C C CH 3 H 3C C H H C H R1 C R2
C C C C C C C C C C C C
O O O O O O O O O O O O
diethyl malonate ethyl acetoacetate acetylacetone formylacetone malondialdehyde general

51.1 CH Acidity
The CH2 groups between both carbonyls in all 1,3-dicarbonyl compounds are strongly CH acidic, their deprotonation
leaving particularly stable carbanions. The negative charge is delocalized in these carbanions which can be represented
as resonance hybrids of three canonical formulas including one carbanion and two enolate anions:

R1 R1 R1 _ R1
R1
C O + NaOR C OI
_ _
C OI C OI
_ O
H 2C H CI H C H C Na or H Na
 ROH
_
C O C OI
_ C OI
_ C OI
_ O
R2 R2 R2 R2 R2
anion of a 1,3-dicarbonyl compound: resonance formulas
R1 = R2 = OC2H5 (OR') : malonic acid diester
R1 = OC2H5 (OR'), R2 = H, alkyl, aryl : 3-oxoalkanoic ester (-keto ester)
R1 = R2 = H, alkyl, aryl : 1,3-diketone, 1,3-dialdehyde, 3-oxo aldehyde (-keto aldehyde)

Carbanions of 1,3-dicarbonyl compounds are carbon nucleophiles which react with appropriate electrophiles. Therefore,
1,3-dicarbonyl compounds are versatile reagents for organic syntheses.

51.2 Typical Reactions


51.2.1 Alkylation and Cycloalkylation
C-Alkylation of malonic acid diesters with alkyl halides yields alkylmalonic acid diesters. These can be hydrolyzed to the
alkylmalonic acids which readily undergo decarboxylation to provide the alkylated acetic acids.

RO
O CO2R CO2H CO2H
  + 2 H2O > 60 °C
R' X + Na H R' CH R' CH R' CH 2
 Na X  2 ROH CO2
O CO2R CO2H
alkyl halide RO alkylated
X = Cl, Br, I alkylmalonic acid diester alkylmalonic acid acetic acid

Following this concept, cyclobutanecarboxylic acid can be prepared in two steps by cycloalkylation of diethyl malonate
with 1,3-dibromopropane to give the cyclobutane 1,1-diester which is hydrolyzed and decarboxylated:
Br
CH 2 CO2C2H5 CO2C2H 5 CO2H H
+ NaOC2H5 CO2C 2H 5 + NaOC2H5 + 2 H2O heat
H2C + H2C
 NaBr CO2C 2H 5  NaBr  2 C2H5OH CO2
CH 2 COC 2H 5 H CO2C2H 5 CO2H CO2H
 C2H5OH  C2H5OH
Br Br
1,3-dibromo- cyclobutane-1,1- cyclobutane-1,1- cyclobutane-
propane dicarboxylic acid diethyl ester dicarboxylic acid carboxylic acid

Efficiently Studying Organic Chemistry: Exam training for chemists, biochemists, pharmacists, life and health scientists,
Third Edition. Eberhard Breitmaier. © 2022 WILEY-VCH GmbH. Published 2022 by WILEY-VCH GmbH.
51.2 Typical Reactions 139

Correspondingly, the acidic methylene group of -keto esters and 1,3-diketones is alkylated by alkyl halides in the pre-
sence of a base:
H H H R' H H H R'
R C OR' R' X R C OR' R C R R' X R C R
C C C C C C C C
C-alkylation C-alkylation
O O O O O O O O
3-oxoalkanoic acid ester 2-alkyl-3-oxoalkanoic 1,3-diketone 2-alkyl-1,3-diketone
acid ester 1,3-dialdehyde (R = H) 2-alkyl-1,3-dialdehyde

51.2.2 Carbonyl Alkenylation (KNOEVENAGEL Alkenylation)


Nucleophilic addition of dialkyl malonate to the electrophilic carbonyl carbon of an aldehyde or ketone formally follows the
aldol condensation (Chapter 50.2.1): Primarily formed (hydroxyalkyl)malonate is dehydrated to provide the alkenylated
dialkyl malonate (KNOEVENAGEL alkenylation), driven by the formation of a double bond in conjugation to the ester carbon-
yls. Hydrolysis of the diester and decarboxylation of the resulting alkenylated malonic acid finally yields an ,-
unsaturated carboxylic acid.
CO2R H CO2R CO2H CO2H
base + 2 H2O
C O C OI + H C H C C CO2R C C C C C C
 H 2O  2 ROH CO2
CO2R HO CO2R CO2R CO2H H
carbonyl resonance
aldehyde or ketone (hydroxyalkyl)malonate alkenylated alkenylated ,-unsaturated
malonic acid diester malonic acid carboxylic acid

Cinnamic acid (and its substituted derivatives) can be prepared by this pathway from benzaldehyde and diethyl malonate
with piperidine as the base.
H CO2C 2H 5 NH H CO2C 2H 5 H CO2H H CO2H
+ 2 H2O
C O + H 2C C C C C C C
 H2O CO2C 2H 5  2 C2H5OH CO2H CO2 H
CO2C 2H 5

benzaldehyde diethyl malonate diethyl benzylidenemalonate benzylidenemalonic acid cinnamic acid


major product: trans-

KNOEVENAGEL alkenylations of aldehydes and ketones can also be performed with the active methylene group of -keto
esters and 1,3-diketones in the presence of a base:
H H C C H H C C
R C OR' O R C OR' R C R O R C R
C C C C C C C C
KNOEVENAGEL KNOEVENAGEL
O O alkenylation O O O O alkenylation O O
3-oxoalkanoic acid ester 2-alkenylated 1,3-diketone 1,1-diacylalkene
3-oxoalkanoic acid ester

51.2.3 Nucleophilic Addition to CC Double Bonds (MICHAEL Addition)


Nucleophilic dialkyl malonates are able to add to an electrophilic carbon atom of an alkene substituted by electron-
withdrawing residues X. This kind of nucleophilic addition, known as MICHAEL addition, produces malonic acid esters
alkylated by groups with the functional group X at the -position.
CO2R CO2R
X CH CH2 X CH CH 2 + IC H H X CH 2 CH2 C H
CO2R CO2R
electrophilic alkene carbanion (nucleophile) C-alkylated dialkyl malonate

Electron-withdrawing groups X attached to the alkene include carbonyl and cyano functions. Thus, ,-unsaturated
aldehydes like propenal (acrolein), enones like methyl vinyl ketone, and acrylonitrile can be represented as hybrids of
resonance formulas displaying a positively charged (electrophilic) terminal:
R R
_ _
IO
_ C CH CH2 IO
_ C CH CH2 IN C CH CH2 IN C CH CH2
enone (R = H: propenal) acrylonitrile (cyanoethene)
140 51 1,3-Dicarbonyl Compounds

As an example, nucleophilic addition (MICHAEL addition) of diethyl malonate to acrylonitrile yields diethyl -cyanoethyl-
malonate which, upon hydrolysis and decarboxylation, is converted into 4-cyanobutanoic acid.

_ CO2C2H5  
CO2C2H5  
IN C CH CH 2 IN C CH CH2 + H2C N C CH CH2 C H N C CH CH2 CH 2 CO2H
CO2C2H5 H CO2C 2H 5 H
electrophile
acrylonitrile (cyanoethene) diethyl -cyanoethylmalonate 4-cyanobutanoic acid
[(2-cyanoethyl)malonic acid diethyl ester]

Analogously, MICHAEL addition of nucleophilic 3-oxoalkanoic acids (-keto esters) and 1,3-diketones to the electrophilic
carbon of enones such as butenone (methyl vinyl ketone) opens access to tricarbonyl compounds:
O O O O
H H H2 C CH C H CH2 CH2 C H H H2 C CH C H CH2 CH 2 C
R C OR' CH 3 R C OR' CH3 R C R CH 3 R C R CH3
C C C C C C C C
MICHAEL MICHAEL
O O addition O O O O addition O O
3-oxoalkanoic 2-acyl-5-oxohexanoic acid ester 1,3-diketone 3-acetyl-2,6-heptanedione
acid ester ( R = CH3 )

51.2.4 Oxo-Enol Tautomerism


Because of the strong CH acidity of the CH2 group between the carbonyl functions, an intramolecular migration of one
methylene proton to the carbonyl oxygen (1,3-shift) can easily occur. For this reason, -keto esters such as ethyl aceto-
acetate equilibrate with their enol tautomers. The keto tautomer predominates in this equilibrium, known as oxo-enol or
keto-enol tautomerism. -Keto esters are more strongly enolized (up to 10%, Chapter 83.3) than aldehydes and ketones
(less than 1%).
> 90 % carbanion two enolate anions < 10 %
H H H H H H
H 3C C OC 2H5 H 3C C
_ OC 2H5 H 3C C OC 2H 5 H 3C C OC 2H 5 H 3C C OC 2H 5
C C C C C C C C C C
O O O O H IO_I O O IO
_I H O O
H H
keto or oxo tautomer resonance formulas of the ethyl acetoacetate anion enol tautomer
with conjugated double bonds,
stabilized by intramolecular
hydrogen bonding

Similarly, 1,3-diketones such as acetylacetone (2,4-pentanedione) equilibrate with their cis-enol tautomers which are
stabilized by intramolecular hydrogen bonding (chelated protons). A trans-configuration would not permit this stabilizing
hydrogen bridge. In contrast to -keto esters, 1,3-diketones equilibrate in favor of their enol tautomers:
H H H H H
H 3C C CH3 H 3C C CH3 H 3C C CH3 HO C CH 3
C C C C C C C C
O O O O O O H 3C O
H H
keto tautomer (13 %) cis-enol tautomers (87 %) trans-enol tautomer

Two test tube reactions indicate the presence of an enol: A solution of bromine is decolorized by an enol due to addition
of bromine to the double bond, and a purple iron(III) chelate is formed with an enol upon addition of ferric chloride solu-
tion.
H Br Br 2 / CCl4
H FeCl3 / C2H5OH
H
H 3C
Br C C C OC2H 5 H 3C
C
C
C
OC2H 5 H 3C
C
C
C
OC 2H5

O O O O O O
H H
Fe
bromo adduct ethyl acetoacetate 3 iron(III) chelate
51.2 Typical Reactions 141

The enol tautomers of 1,3-dicarbonyl compounds are bidentate chelate ligands, providing carbonyl and hydroxy for
clawlike complexation (chelation) of metal cations to metal chelates (from Greek chele = claw of the lobster). Acetyl-
acetone reacts with the cations of transition metals (Fe3, Cu2) and lanthanoids (rare earth metals) to give metal che-
lates (acetylacetonates) which crystallize from the reaction solutions:
R R R
C O 2 C O O C
+ Cu copper(II) acetylacetonate
2 H C H H C Cu C H (R = CH3, blue crystals)
 2 [H ]
C O C O O C
R R R

51.2.5 Cyclizations
All 1,3-dicarbonyl compounds offer two electrophilic carbonyl groups in one molecule, able to undergo cyclizations with
bifunctional nitrogen nucleophiles to produce heterocycles. Thus, unlike monofunctional aldehydes and ketones (Chapter
49.2), ethyl acetoacetate does not react to give hydrazones with hydrazine and its substituted derivatives. Instead, dehy-
drating cyclizations occur to provide substituted pyrazoles (Chapter 61.1).
CH 3 CH3
3

O O + NH2 N 5 1 N
HN O N N
OC 2H5  C2H5OH ,  H2O
C 6H5 C 6H5 H
ethyl acetoacetate phenylhydrazine 3-methyl-5-oxo-1-phenyl-4,5-dihydropyrazole pyrazole

Similarly, 1,3-diketones such as acetylacetone cyclodehydrate with hydrazines to give substituted pyrazoles, and with
hydroxylamine to yield substituted isoxazoles:
R1 R1 R1 R1
+ R 3 NH NH2 + HO NH2
N R2 O R2 O N
R2 N  2 H2O  2 H2O R2 O
O OH
R 3 R3 = H, alky l, ary l oxo tautomer enol tautomer
3,5-dialkylpyrazole 3,5-dialkylisoxazole

The cyclic diamide of urea (Chapter 55.4.1), barbituric acid, is prepared by cyclization of diethyl malonate as bifunctional
electrophile with urea as bifunctional nitrogen nucleophile:
O O
H
NH2 H5C2O NaOC2H5 N N
+
 2 C2H5OH
O NH2 H5C2O O O N O N
urea diethyl malonate H
barbituric acid pyrimidine

Barbituric acid belongs to the heterocycles derived from pyrimidine (Chapters 62.1, 62.3.2). Barbituric acids alkylated at
the carbon between the carbonyls and prepared from the corresponding alkylated diethyl malonates have been used as
sedatives and sleep-inducing drugs known as barbiturates.
________________________________________________________________________________________________
Chapter 51 permits answers to the following:
(51.1) Draw the structural formulas of all kinds of 1,3-dicarbonyl compounds you know.
(51.2) Formulate and explain the CH acidity of 1,3-dicarbonyl compounds.
(51.3) Formulate a sequence of reactions that permits the synthesis of cyclopropanecarboxylic acid.
(51.4) Propose a synthesis of 3-methyl-2-pentenoic acid. Which isomers can be formed?
(51.5) What product is obtained by MICHAEL addition of acetylacetone to butenone?
(51.6) Which test tube reactions indicate the presence of enol tautomers in samples of 1,3-dicarbonyl compounds?
(51.7) What products are formed by reaction of hydrazine with (a) ethyl acetoacetate and (b) acetylacetone?
142 52 Phenols

52 Phenols
52.1 Nomenclature
Phenols contain hydroxy groups which are directly attached to an aromatic ring (ArOH). Hydroxyarenes are therefore
the IUPAC names; however, the parent hydroxybenzene is much better known as phenol, and common names related to
the natural origin are usual. Hydroxytoluenes (o-, m-, and p-isomers) are called cresols, while the isomeric -
hydroxytoluene with the common name benzyl alcohol is not a phenol. Catechol, resorcinol, and hydroquinone are diphe-
nols. Hydroxynaphthalenes are known as naphthols.
CH 2OH OH OH OH OH OH OH OH OH
CH 3 OH OH

CH 3 OH
benzyl alcohol phenol m-cresol CH 3 resorcinol OH -naphthol -naphthol
(1° alcohol) o-cresol p-cresol catechol hydroquinone

52.2 Resonance Formulas, Acidity Relative to Alcohols


Phenol has a higher resonance energy Hres (168 kJ/mol) than benzene (151 kJ/mol). The stronger resonance stabiliza-
tion is attributed to the unshared pairs of electrons of the phenolic oxygen. One of these is released to the  electron
sextet of benzene, such that phenol can be described as a resonance hybrid including resonance formulas with an in-
creased electron density at the o-, p-, and o´-positions [()-M effect, Chapter 26.1]:
_ _ _ _ _
IOH IOH OH OH OH

resonance formulas
of phenol

KEKULÉ formulas dipolar formulas

Since no separation of charges is required, the phenoxide anion is even more resonance-stabilized than phenol. Phenols
are therefore much stronger OH acids than alcohols because alkoxide anions are not stabilized.
_ _ _ _ _
OH IOI IOI OI OI OI
 [H ]

resonance formulas of the phenoxide anion

Similar to OH acids, phenols (ArOH) can be neutralized with alkali hydroxide to provide phenoxides, while alcohols
(ROH) do not react at all with hydroxides; conversion of alcohols into alkoxides requires strongly reducing metals such
as alkali and alkali earth metals.
_ + H2O
R OH + Na R OI
_ Na R OH + NaOH
 1/2 H2
_
R OH + NaOH x R OI
_ Na + H 2O
_
Ar OH + NaOH Ar OI
_ Na + H 2O

Substituents which additionally delocalize the negative charge of the phenoxide anion due to their ()-M effect increase
the acidity of phenols. Nitrophenols are thus much more acidic than phenol, indicated by their acidity contants Ka:
_ _ _ _
OH OH IOI OI OI OI
 [H ]

NO2 N _ N _ N _ _ N _
IO
_ O
_I IO
_ O
_I IO
_ O
_I IO
_ O
_I
Ka = 1.12 x 1010 Ka = 720 x 1010
resonance formulas of the p-nitrophenoxide anion

Efficiently Studying Organic Chemistry: Exam training for chemists, biochemists, pharmacists, life and health scientists,
Third Edition. Eberhard Breitmaier. © 2022 WILEY-VCH GmbH. Published 2022 by WILEY-VCH GmbH.
52.3 Preparation 143

52.3 Preparation
52.3.1 HOCK Process (Synthesis of Phenol and Acetone)
Phenol and acetone are produced industrially from cumene (isopropylbenzene) and air oxygen (Fig. 52.1). This synthesis,
known as the HOCK process, starts with the radical autoxidation of cumene to the hydroperoxide. Acid-catalyzed cleavage
of cumene hydroperoxide proceeds in five steps, beginning with protonation (1). Dehydration (2) leaves an oxenium ion
with an electron sextet at oxygen. The oxenium ion rearranges by 1,2-phenyl shift to a carbenium ion (3), involving a
resonance-stabilized spirocyclic phenonium ion. Nucleophilic addition (4) of water to the carbenium ion produces a proto-
nated hemiketal (Chapter 49.1.2) which, after deprotonation (5), liberates its components phenol and acetone.

(1)
_ _ + [H ]
+ O
_ O
_ O H OH 2
H3C C H H 3C C O H 3C C O
CH 3 CH 3 CH3
cumene cumene hydroperoxide
(2)  H 2O
resonance-stabilized phenonium ion

(3)

H 3C C OI
_ H 3C C O H 3C C O H3C C O
CH3 CH 3 CH 3 CH3
oxenium ion

(4)
H 3C
(5)  H2 O
C + O H 3C OI
_ H 3C O H 3C O
H 3C O H C H  [H ] C C
O OH2
H 3C H 3C CH3
acetone phenol hemiketal of acetone and phenol carbenium ion

Fig. 52.1. HOCK process: industrial synthesis of phenol and acetone

52.3.2 Hydrolysis of Substituted Chlorobenzenes


The synthesis of phenol by catalytic hydrolysis of chlorobenzene (RASCHIG process, Chapter 27.1) requires high tempera-
ture and pressure.
360 °C , 300 bar , Cu _
Cl + 2 NaOH Ol
_ Na + NaCl + H 2O

sodium phenoxide

()-M Substituents o- and p- to halogen moderate the conditions required for nucleophilic substitution (Chapter 27.1):
30 °C
O2N Cl + OH O2N OH + Cl

NO2 NO2
1-chloro-2,4-dinitrobenzene 2,4-dinitrophenol

52.3.3 Catalytic Oxidation of Methylarenes


Some phenols can be prepared by catalytic air oxidation of methylated arenes:
2 2
Cu , Mg
CH3 + 2 O2 OH + CO2 + H2O
144 52 Phenols

52.3.4 Melting of Arenesulfonates with Alkali Hydroxides


Arenesulfonates, obtained by electrophilic sulfonation of arenes (Chapter 25.6) and salt formation, yield phenols when
fused with alkali hydroxide. A nucleophilic substitution of the sulfite anion occurs.
_
SO3 Na IOI Na OH
300 °C + H2SO4
+ 2 NaOH
 Na2SO3 ,  H2O  NaHSO4
sodium -naphthalenesulfonate -naphtholate -naphthol

52.3.5 Hydrolysis of Arenediazonium Salts


Some substituted phenols can be prepared from the anilines by diazotization (Chapter 39.2) and subsequent hydrolysis
of the resulting diazonium salts, involving nucleophilic substitution of nitrogen by water. 3-Chlorophenol is prepared from
3-chloroaniline in this way:
Cl 1. HCl
Cl Cl
2. NaNO2 + H2O , 80 °C
NH 2 N NI Cl OH
 HCl ,  N2
3-chloroaniline 3-chlorobenzenediazonium chloride 3-chlorophenol

52.4 Typical Reactions


52.4.1 Conversion into Aryl Ethers (WILLIAMSON Synthesis)
The phenoxide anion is an oxygen nucleophile. Nucleophilic substitution of the halide anion in alkyl halides by a phen-
oxide produces an aryl ether, according to the WILLIAMSON synthesis (Chapter 37.3.2):

+ OH _ + RX R
Ar OH Ar OI
_ Ar O
 H2O phenoxide  X alkyl aryl ether
_
OI
_ Na O
CH2 CH3
+ Br CH2 CH3 + NaBr

sodium phenoxide bromoethane ethyl phenyl ether

52.4.2 Esterification and FRIES Rearrangement


Phenyl esters are readily formed by acylation of phenols with acyl halides or carboxylic acid anhydrides. If the p-position
of the resulting phenyl ester is unsubstituted, intramolecular electrophilic acylation takes place there, in the presence of a
LEWIS acid, known as the FRIES rearrangement. Phenyl acetate as an example, obtained by reacting phenol with acetic
anhydride, rearranges to p-hydroxyacetophenone:
O
C CH3
OH O CH3 AlCl3 in CS2 OH
+ O C ( + o-isomer )
C CH3  CH 3CO2H O H3C
C
O phenyl acetate O
acetic anhydride (acetic acid phenyl ester) p-hydroxyacetophenone

Esterification of salicylic acid with acetic anhydride yields acetylsalicylic acid (ASS, aspirin), the well-known aryl ester of
acetic acid (analgesic, antipyretic, antirheumatic agent, platelet aggregation inhibitor).
O
CO2H CO2H
C CH 3
OH O CH 3
+ O C + H 3C CO2H
C CH 3 O
salicylic acid O acetylsalicylic acid
52.4 Typical Reactions 145

52.4.3 Electrophilic Substitution


Representing a typical ()-M substituent, the OH group activates the benzenoid ring for electrophilic substitution, directing
the second substituent to the o- and p-positions (Chapter 26):
_ _ _ _ _
IOI OI OI OI OI OH OH
H
Y Y
+ Y and and
 [H ]

Y H Y

Thus, depending on the conditions, phenol undergoes regioselective electrophilic nitration to give o- and p-nitrophenol or
2,4,6-trinitrophenol, better known as picric acid (explosive in the dry state).

NO2 HNO3
OH fuming OH HNO3 / H 2SO4 OH OH
25 °C  10 °C
and
O2N NO2 NO2 O2N
o- p-
2,4,6-trinitrophenol (picric acid) nitrophenol

52.4.4 Oxidation
The oxidation of phenols usually produces quinones (Chapter 53.2.1). Substituents (other than OH) in the ortho and para
positions of the phenol hinder the formation of quinones. In this case, the oxidation yields peroxides formed by combina-
tion of intermediate radicals which can be stabilized by bulky alkyl groups ortho to the hydroxy function. Thus, oxidation of
2,4,6-tri-t-butylphenol with silver oxide or potassium hexacyanoferrate(III) in benzene under nitrogen atmosphere yields
the 2,4,6-tri-t-butylphenoxyl radical which crystallizes from the solution. As a radical, the blue compound is paramagnetic
and must be kept under nitrogen to prevent oxidation to the yellow peroxide by biradicalic air oxygen.
OH IOI
(H 3C)3C C(CH 3)3
_air_ C(CH 3)3
(H 3C)3C C(CH3)3 + Ag 2O (benzene , N2) (H 3C)3C C(CH 3)3 + O
_ O _ O
2 2 O O
 H 2O ,  2 Ag O
(H 3C)3C
C(CH 3)3 C(CH3)3 (H 3C)3C C(CH 3)3
2,4,6-tri- t-butylphenol 2,4,6-tri- t-butylphenoxyl radical (blue) peroxide (yellow)

These aroxyl radicals are relatively stable due to two factors. First, the bulky substituents ortho to oxygen sterically pro-
tect the unpaired electron and prevent the dimerization to peroxides; second, the unpaired electron is delocalized in the
benzenoid ring, best described by resonance formulas which distribute the unpaired electron among the ortho- and para-
positions.
IO I IO I IO I IO I
R R R R R R R R
R = C(CH 3)3

R R R R

________________________________________________________________________________________________
Chapter 52 permits answers to the following:
(52.1) Why are phenols much stronger acids than alcohols? Explain your reasoning with resonance formulas.
(52.2) Formulate all equations necessary to outline the production of phenol and acetone from cumene.
(52.3) What products are formed by reaction of acetic anhydride with (a) phenol and AlCl3, and (b) salicylic acid?
(52.4) What product is formed by reaction of phenol with 3-bromo-1-propene (allyl bromide) in sodium hydroxide?
(52.5) What products are likely to be formed by nitration of (a) -naphthol and (b) -naphthol? Look at Chapter 28.3.
(52.6) What product is expected from the oxidation of 2,4,6-tri-t-butylphenol? Account for the stability of the product.
146 53 Quinones

53 Quinones
53.1 Survey and Nomenclature
Quinones arise from the oxidation of phenols and diphenols (Chapter 52.1). Catechol (1,2-dihydroxybenzene) is oxidized
to o-benzoquinone, and hydroquinone (1,4-dihydroxybenzene) to p-benzoquinone:
O O
o-benzoquinone p-benzoquinone
O O

Quinones (o- and p-) are six-membered cyclic 1,2- and 1,4-diketones with crossed conjugation of CO and CC double
bonds. 1,3-Quinones (m-quinones) do not exist. A methylene group interrupts cyclic conjugation of double bonds in 4-
cyclohexene-1,3-dione which is therefore not a quinone but the unstable diketo tautomer of resorcinol:
O OH OH

O OH O
resorcinol
diketo tautomer (unstable) (bis-enol tautomer, stable) monoenol tautomer (unstable)

The generally accepted name of a quinone begins with the position numbers of the quinoid carbonyl groups (1,2-, 1,4- or
o-, p-) followed by the prefix derived from the aromatic parent skeleton (benzo-, naphtho-, anthra-, phenanthra-) and
ending with quinone (Table 53.1). Quinoid six-membered rings are non-aromatic but in some polycyclic representatives
such as anthraquinone they are fused with benzenoid rings. Quinones are colored compounds; quinoid partial structures
account for the color of the so-called carbonyl dyes (anthraquinone dyes, Chapter 66.3.4).

Table 53.1. Nomenclature and color of selected quinones


benzoquinones naphthoquinones
O O O O O O
O O O

O O
O O O
o- p- 1,2- 1,4- 2,6- 9,10-anthraquinone 9,10-phenanthraquinone
red yellow red yellow orange pale yellow orange-red

53.2 Preparation
53.2.1 Oxidation of Phenols and Primary Arenamines
Oxidation of diphenols yields quinones in which the location of the carbonyl groups is determined by the positions of the
hydroxy functions in the diphenol. Hydroquinone is oxidized by various oxidation reagents to p-benzoquinone, catechol to
o-benzoquinone, and 2,6-dihydroxynaphthalene to 2,6-naphthoquinone.

OH anhydrous ether, MgSO4


O OH toluene
O
+ Ag2O + PbO2
 H2O, 2 Ag  H2O, PbO
OH O HO O
catechol o-benzoquinone 2,6-dihydroxynaphthalene 2,6-naphthoquinone

1,4-Diamines such as the toxic p-phenylenediamine are oxidized to 1,4-quinones involving p-quinone diimines as inter-
mediates.
NH2 NH O
+ 2 H2O

 2 [H ] ,  2 e0  2 NH 3
H 2N HN O
p-phenylenediamine p-quinone diimine p-benzoquinone

Efficiently Studying Organic Chemistry: Exam training for chemists, biochemists, pharmacists, life and health scientists,
Third Edition. Eberhard Breitmaier. © 2022 WILEY-VCH GmbH. Published 2022 by WILEY-VCH GmbH.
53.3 Reactions 147

Industrially, p-benzoquinone is produced by oxidation of aniline with manganese dioxide and sulfuric acid. The reduction
of silver salt by p-aminophenol as the developer to (black) silver on the exposed film is used in non-digital photography.
OH O
+ 2 Ag + 2 OH + 2 Ag + H 2O + NH 3
H 2N O
p-aminophenol p-benzoquinone

53.2.2 Oxidation of Polycyclic Aromatics


The oxidation of aromatic hydrocarbons by chromic acid is a common method for the preparation of polycyclic quinones
such as naphthoquinones and 9,10-phenanthraquinone. An ecologically more desirable method involves catalytic oxida-
tion with air.
O O
O
H2 SO4
+ 2 CrO3 H2SO4
 Cr 2O3, H2O + 2 CrO3
 Cr 2O3 , H 2O
O
1,4-naphthoquinone 9,10-phenanthraquinone

53.2.3 FRIEDEL-CRAFTS Acylation of Arenes with Phthalic Anhydride


Linearly fused quinones such as 9,10-anthraquinone are synthesized by electrophilic acylation of arenes with phthalic
anhydride in the presence of a LEWIS acid:

O O O
H AlCl3 or H 2SO4
C
H2SO4
+ O
 H2O
H HO2C
O O
benzophenone-o-carboxylic acid 9,10-anthraquinone

53.3 Reactions
53.3.1 Reduction-Oxidation Quinone-Hydroquinone Equilibrium
Reduction of quinones yields diphenols. Reduction of p-benzoquinone and oxidation of hydroquinone equilibrate, the
equilibrium depending on the concentration of protons given by the pH:
O reduction OH
+ 2 [H ] + 2 e0
O oxidation HO
p-benzoquinone hydroquinone

Therefore, the redox equilibrium constant K depends on pH in aqueous solution:

c (hydroquinone) lg K = lg c(hydroquinone)  lg c(p-benzoquinone)  2 lg c (H 3O )


K = or in logarithmic form:
c (p-benzoquinone) c2 (H3O ) lg K = lg c(hydroquinone)  lg c(p-benzoquinone)  2 pH

The equilibrium gives a reproducible redox potential in an electrolytic cell. For this reason, the quinone-hydroquinone
redox system (quinhydrone electrode) was used for electrometric pH detection prior to the development of the glass-
electrode pH meter.
Reduction of p-benzoquinone (yellow) to hydroquinone (colorless) involves an intermediate known as quinhydrone.
Quinhydrone crystallizes from the reddish-brown ethanol solution of equimolar amounts of hydroquinone and p-
benzoquinone (green crystals with a metallic luster). The color due to light absorption in the visible part of the spectrum
148 53 Quinones

(Chapter 66.1) arises from an electron transfer between the hydroquinone dianion as donor to p-benzoquinone as accep-
tor. Quinhydrone represents a so-called charge-transfer (CT) or electron-donor-acceptor complex (EDA complex).

electron acceptor O O quinhydrone


2 e0 2H a charge-transfer complex
electron donor O O

The electron transfer between hydroquinone and p-benzoquinone in quinhydrone occurs in two steps, involving the
resonance-stabilized semiquinone radical anion:
O O O O
+ e0 + e0

O O O O
p-benzoquinone semiquinone radical anion hydroquinone dianion

53.3.2 Autoxidation of Anthrahydroquinone


Catalytic hydrogenation of 9,10-anthraquinone and its 2-alkyl derivatives (R = ethyl, t-butyl, pentyl) produces anthrahydro-
quinones which undergo air-oxidation to anthrahydroquinone endoperoxides. From these, anthraquinones are regenerat-
ed by elimination of hydrogen peroxide which is produced industrially in this way.

 H 2O2
O OH autoxidation OH
+ H 2 / Ni _air_
R R + O R
30 °C _ O _
O
O

O OH OH
2-alkylanthraquinone 2-alkylanthrahydroquinone 2-alkylanthrahydroquinone
endoperoxide

53.3.3 Additions
Non-aromatic quinoid rings react like enones. Thus, quinones undergo additions and cycloadditions, but not electrophilic
substitutions. Bromine, as an example, adds to p-benzoquinone to form stereoisomers of 2,3,5,6-tetrabromo-1,4-
cyclohexanedione:
O O O O
H H H Br H
Br Br Br H Br
+ Br 2 + Br 2
and
H H H Br H
Br Br Br H Br
O O O O
2,3,5,6-tetrabromo-1,4-cyclohexanedione (stereoisomers)

Having electron-deficient double bonds due to electron withdrawal by the carbonyl groups, p-benzoquinones are dieno-
philes and therefore undergo [4+2]-cycloadditions (DIELS-ALDER reactions) with 1,3-dienes. Stereospecific [4+2]-
cycloaddition of 1,3-butadiene to p-benzoquinone produces tetrahydronaphthoquinone, while tetrahydroanthraquinone is
obtained from 1,3-butadiene and 1,4-naphthoquinone. The cyclohexene ring can be aromatized by catalytic dehydro-
genation, yielding the naphtho- and anthraquinone.
O O O O O
8 H 1 1 H 9
8a 9a
+ 6 4a
+ 4a

5 4 4 10
H H
O O O O O
4a,5,8,8a-tetrahydro- 1,4,4a,9a-tetrahydro-
1,4-naphthoquinone 9,10-anthraquinone
53.3 Reactions 149

53.3.4 Electrophilic Substitutions of Benzenoid Rings


Electrophilic substitutions take place at the benzenoid rings of polycyclic quinones. Due to electron withdrawal [the ()-M
effect, Chapter 26.1] by the two quinoid carbonyl groups, electrophilic substitution of 9,10-anthraquinone may occur at the
1- and 2-positions of the benzenoid ring, because both positions are meta to one of the carbonyls. Regioselectivity can be
(empirically) controlled by the reaction conditions. Thus, fuming sulfuric acid sulfonates 9,10-anthraquinone at the 1-
position when mercuric sulfate is added, while the 2-sulfonic acid is obtained without this additive.

O SO3H O O
+ SO3 (H 2S2 O7 ) + SO3 (H 2S2O7 ) SO3H
(HgSO4) , 150 °C 150 °C

O O O
-1-sulfonic acid 9,10-anthraquinone -2-sulfonic acid

9,10-Anthraquinonesulfonic acids are raw materials for the industrial production of so-called carbonyl dyes (Chapter
66.3.4) since the sulfite anion ion present in the sulfonic acid function can be substituted by hydroxide and ammonia as
nucleophiles, opening access to hydroxy- and aminoanthraquinones. These are raw materials for the production of an-
thraquinone derivatives which provide dyes with many colors of the spectrum. Alizarin, a natural dye occurring in the
roots of the madder plant and used for coloring wool, is prepared by air-oxidation of 2-hydroxy-9,10-anthraquinone.

O O O O OH
NH 2 SO3H OH + 1/2 O2 OH
+ 3 NH 3 + 2 NaOH (NaOH)
 (NH 4) 2 SO3  Na2SO3

O O O O
2-amino-9,10-anthraquinone 9,10-anthraquinone-2-sulfonic acid 2-hydroxy-9,10-anthraquinone alizarin
red yellow red

53.3.5 Carbonyl Reactions


Quinones undergo most reactions which are characteristic of carbonyl compounds (Chapter 49), exemplified by the
nucleophilic addition of hydroxylamine to p-benzoquinone, yielding p-benzoquinone dioxime.
in ethanol
O (H2SO4)
N
OH
+ 2 H 2N OH + 2 H 2O
HO
O N
p-benzoquinone dioxime

________________________________________________________________________________________________
Chapter 53 permits answers to the following:
(53.1) Which isomeric benzo- and naphthoquinones exist? Draw the structural formulas.
(53.2) Formulate the tautomerism of resorcinol.
(53.3) Formulate the preparation of (a) o-benzoquinone, (b) p-benzoquinone, and (c) 1,4-naphthoquinone, using
appropriate phenols as starting material.
(53.4) What is an EDA or CT complex? Draw the formula of quinhydrone to explain.
(53.5) Formulate (a) the redox equilibrium between hydroquinone and p-benzoquinone and (b) the mechanism of
the electron transfer in quinhydrone.
(53.6) Which reactions can be used to prepare 9,10-anthraquinone?
(53.7) Formulate an industrial method to produce hydrogen peroxide.
(53.8) What products are expected from the electrophilic sulfonation of 9,10-anthraquinone? Outline a synthesis of
1,2-dihydroxy-9,10-anthraquinone (known as alizarin).
150 54 Organosulfur Compounds

54 Organosulfur Compounds
54.1 Sulfur in Organic Compounds
Since d orbitals are available for bonding, sulfur is more than just bivalent like oxygen. Organosulfur compounds therefore
include the analogues of oxygen with bivalent sulfur (thiols, disulfides, thioethers, sulfenic acids, and thiocarbonyl com-
pounds), as well as compounds with tetravalent and hexavalent sulfur (sulfoxides, sulfinic acids, sulfones, and sulfonic
acids), as shown in Table 54.1.

Table 54.1. Organosulfur compounds and the related oxygen compounds


O
R O R O R O R O R1 C
H O R' R' OH R2
alcohols dialkyl peroxides ethers hydroperoxides carbonyl
compounds
S O O O O
R S R S R S R S R1 C R S R S O R S R S OH
H S R' R' OH R2 R' R' OH O
thiols dialkyl disulfides thioethers sulfenic acids thiocarbonyl sulfoxides sulfones sulfinic acids sulfonic acids
(dialkyl sulfides) (unstable) compounds

54.2 Organosulfur Compounds with Bivalent Sulfur


54.2.1 Thiols, Thiophenols, Disulfides
Thiols and thiophenols (RSH, ArSH) are the sulfur analogues of alcohols and phenols (ROH, ArOH). Some thiols
emit revolting odors, such as the skunk secretion 1-butanethiol. Others shape the flavors of coffee (2-furylmethanethiol),
blackcurrant (4-methoxy-2-methyl-2-butanethiol, a hemiterpene, Chapter 76.3.1), and grapefruit [the (S)-enantiomer of p-
menth-1-ene-8-thiol, a monoterpene, Chapter 76.3.1].
CH 3 CH2 CH 2 CH2 SH
1-butanethiol
O SH SH
SH CH2SH
SH
H3CO CH 2 CH 2 C CH 3 NH 2
CH 3
(S)-p-menth-1-ene-8-thiol 2-furylmethanethiol thiophenol o-aminothiophenol
4-methoxy-2-methyl-2-butanethiol

Thiols are prepared by nucleophilic substitution of alkyl halides with the hydrogensulfide anion:
NaSH
R X + SH R SH + X

Alternatively, alkylmagnesium halides can be reacted with powdered sulfur to give thiols:
ether + S + [H ] 2
R X + Mg R MgX R S Mg X R SH + Mg + X

Thiophenols are obtained by reduction of arenesulfonyl chlorides with reducing reagents such as zinc in acid:
O
Zn (dust), HCl
Ar S Cl + 6 [H ] + 6 e0 Ar SH + HCl + 2 H 2O
O thiophenol

The SH bond is longer (133 pm) and weaker (340 kJ/mol) than the OH bond (96 pm, 460 kJ/mol). As a result, thiols are
more acidic than alcohols and can be neutralized to thiolates by aqueous sodium hydroxide, in contrast to alcohols.
R SH + OH R S + H2O
thiolate anion

Efficiently Studying Organic Chemistry: Exam training for chemists, biochemists, pharmacists, life and health scientists,
Third Edition. Eberhard Breitmaier. © 2022 WILEY-VCH GmbH. Published 2022 by WILEY-VCH GmbH.
54.2 Organosulfur Compounds with Bivalent Sulfur 151

Thiols are readily oxidized to disulfides when exposed to the (biradicalic) oxygen of air, involving intermediate thiolyl
radicals that combine to produce dialkyl disulfides. In contrast, oxidation of alcohols with oxidation reagents leads to
carbonyl compounds (Chapter 35.2) and never to dialkyl peroxides.
air
R S H + O O R S + H O O 2R S R S S R
thiolyl radical hydroperoxy dialkyl disulfide
radical

54.2.2 Thioethers
Nucleophilic substitution of alkyl halides by thiolate or thiophenolate as sulfur nucleophiles, in analogy to the WILLIAMSON
synthesis of ethers (Chapters 37.3.2, 52.4.1), yields thioethers. Methyl thioethers are also obtained from thiolates and
methylation reagents such as dimethyl sulfate.
CH3
S Na S
H3C O O Na O O
H3C S Na +  CH2 CH3 H3C S CH2 CH3 + Na I + S + S
iodoethane ethyl methyl thioether H3C O O H3C O O
(ethyl methyl sulfide) thio- dimethyl thioanisole
phenolate sulfate (methyl phenyl thioether,
methyl phenyl sulfide)

Symmetric thioethers are formed by the alkylation of potassium sulfide with alkyl halides, the sulfide dianion acting as
nucleophile.

2R X + K2S R S R + 2 KX

Exhaustive alkylation of dialkyl sulfides yields trialkylsulfonium salts. In these, three different alkyl groups generate an
asymmetrically substituted sulfur atom as stereogenic center so that resolution to enantiomers is possible.
_ _
S
_ S R' H 3C S CH CH S CH
R R + R' I I H 3C H 2C 3
R 2 3
R HO2C H 2C CH 2 CO2H
trialkylsulfonium iodide enantiomers of the carboxymethyl (ethyl)methyl sulfonium ion

54.2.3 Sulfenic Acid Derivatives


Sulfenic acids, RSOH, are the sulfur analogues of hydroperoxides, ROOH. They are unstable, but derivatives such
as sulfenyl chlorides can be prepared by radicalic substitution of alkanes with sulfur dichloride or by catalytic cleavage of
diaryl disulfides with chlorine:
NO2
NO2
O2N S Fe , FeCl3
R H + SCl2 R S Cl + HCl + Cl2 2 O2N S Cl
S NO2
alkanesulfenyl chloride
O2N 2,4-dinitrobenzenesulfenyl
2,2',4,4'-tetranitrodiphenyl disulfide chloride

Derivatization of sulfenyl chlorides with alcohols in the presence of a base yields sulfenates (sulfenic acid esters), and
sulfenamides (sulfenic acid amides) with ammonia or amines.
R' R'
R S Cl + R' OH R S OR' + HCl R S Cl + 2 H N R S N + H2NR'2 Cl
alkyl sulfenate R' R'
R' = H , alkyl , aryl sulfenamide

The addition of alkanesulfenyl chlorides to alkenes produces -chloro thioethers:

C C + R S Cl Cl C C SR -chloro thioether
152 54 Organosulfur Compounds

54.2.4 Thioaldehydes, Thioketones


Phosphorus sulfides (P4S10) and other sulfur-transferring compounds such as LAWESSON’s reagent convert some alde-
hydes and ketones such as benzophenone into the thiocarbonyl compounds.
2,4-bis(4-methoxyphenyl)-1,3,2,4-dithiadiphosphetane-2,4-disulfide
S S S
O LAWESSON's reagent S
C C S P OCH3
C or C
R H R R' P2 S5 in CS2
H 3CO P S
thioaldehyde thioketone S
LAWESSON's reagent benzophenone thiobenzophenone

54.2.5 Thiolic Acids, Thionic Acids, Dithiocarboxylic Acids


Thiolic acids, thionic acids, and dithiocarboxylic acids are the sulfur analogues of carboxylic acids. Thiolic and thionic
acids equilibrate as tautomers; the thiolacetic acid ester known as acetyl-S-coenzyme A or activated acetic acid acts as
an acetyl-transferring coenzyme in the biosynthesis of fatty acids, polyketides (Chapter 75.1), and terpenes.
SH tautomerism S O O S
R C R C H 3C C H 3C C R C
O OH SH S CoA SH
thiolic acid thionic acid thiolacetic acid acetyl-S-coenzyme A dithiocarboxylic acid

Thiolic acids and their esters are prepared by acylation of hydrogen sulfide and thiols, respectively:
O O O O
base base
R C + H 2S R C R C + HS R' R C
 HCl  HCl
Cl SH Cl S R'
thiolic acid thiolic acid ester

Corresponding to the carboxylation of GRIGNARD compounds by carbon dioxide (Chapter 41.4.2), dithiocarboxylation of
alkylmagnesium halides to yield dithiocarboxylic acids can be performed with carbon disulfide:
anhydrous acid
S S S S
+ [H ]
R MgX + C C R C R C
2
S  Mg ,  X SH
S SMgX
dithiocarboxylic acid

54.3 Compounds with Tetra- and Hexavalent Sulfur


54.3.1 Sulfoxides and Sulfones
Sulfoxides such as the solvent DMSO (dimethyl sulfoxide) and sulfones are produced by oxidation of thioethers with
hydrogen peroxide.
R _ _H R _H _ R R _ + H2O2 R O
S + O
_ O
_ S
_ O
_ + IO
_ H S
_ O
_ S_ O
_ S
 H2O  H2O R O
R H R R R
dialkyl sulfide dialkyl sulfoxide dialkyl sulfone
R = CH3 : dimethyl sulfoxide R = CH3 : dimethyl sulfone

54.3.2 Sulfinic and Sulfonic Acids


Sulfinic acids, used as chelating reagents, are the sulfur analogues of carboxylic acids and can be prepared by sulfoxyla-
tion (corresponding to carboxylation, Chapter 41.4.2) of alkylmagnesium halides by sulfur dioxide with its electrophilic S
atom.
anhydrous acid O
O O O
+ [H ]
(Ar) R MgX + S S (Ar) R S (Ar) R S
2
O  Mg ,  X OH
O OMgX
sulfinic acid
54.3 Compounds with Tetra- and Hexavalent Sulfur 153

Alkanesulfonic acids are obtained by radicalic photosulfochlorination of alkanes to sulfonyl chlorides (Chapter 13.4) and
hydrolysis of the latter.
O
h , base 100 °C
R H + SO2 + Cl2 R S Cl + HCl R SO2 Cl + H 2O R SO3H + HCl
O
alkanesulfonyl chloride alkanesulfonic acid

Arenesulfonic acids arise from electrophilic sulfonation of aromatics with fuming sulfuric acid (Chapter 25.6).
H 2S2O7 (H 2SO4 x SO3 )
Ar H + SO3 Ar SO3H

54.3.3 Sulfonic Acid Derivatives


Sulfonic acids are strong organic acids comparable with H2SO4 or HClO4. Their stable salts are the sulfonates. Chlorina-
tion of arenesulfonic acids or alkali sulfonates with inorganic halides yields sulfonyl chlorides (arenesulfonic acid chlo-
rides).
O
3 H3C S O Na + PCl3 3 H 3C SO2 Cl + Na3PO3
O
sodium p-toluenesulfonate p-toluenesulfonic acid chloride

Alcoholysis of sulfonyl halides produces sulfonic acid esters (alkyl sulfonates), such as the tosylates:
O O
base
H3C S Cl + C2H5OH H3C S OC 2H 5
 HCl
O O
p-toluenesulfonyl chloride p-toluenesulfonic acid ethyl ester (ethyl tosylate)

Aminolysis of sulfonyl chlorides or sulfonic acid esters yields sulfonamides (sulfonic acid amides); some substituted
derivatives of these are used as antibacterial agents.
O O CH 3
H 3C S Cl + 2 H 2N CH 3 H3C S N
O  Cl H3N CH3 O H
N-methyl-p-toluenesulfonamide

Because the negative charge of the sulfonamide anion is distributed by the ()-M effect of the SO double bonds in the
sulfonyl group, sulfonamides and their N-alkyl but not N,N-dialkyl derivatives contain acidic NH and dissolve in sodium
hydroxide in forming salts. This permits the separation of tertiary amines from secondary and primary amines (HINSBERG
separation).
________________________________________________________________________________________________
Chapter 54 permits answers to the following:
(54.1) Which organosulfur compounds do you know? Which ones have oxygen analogues?
(54.2) Formulate equations to suggest preparations of (a) 1-butanethiol and (b) thiophenol.
(54.3) What happens when thiols are exposed to air? Why are thiols much stronger acids than alcohols?
(54.4) Suggest a preparation of ethyl phenyl sulfide. Ethyl phenyl sulfide reacts with iodomethane. Specify the product.
(54.5) Which products are obtained by the oxidation of thioethers with hydrogen peroxide? Formulate the reactions.
(54.6) Which reactions open access to (a) sulfinic acids and (b) sulfonic acids?
(54.7) Formulate equations to outline preparations of (a) sulfochlorides, (b) alkyl sulfonates, and (c) sulfonamides.
(54.8) Sulfonamides and N-alkylsulfonamides dissolve in aqueous sodium hydroxide. Explain with resonance formulas.
(54.9) Which thiocarbonyl compounds exist? Suggest syntheses of (a) thiobenzophenone and (b) phenyldithioacetic acid.
154 55 Carbonic Acid Derivatives

55 Carbonic Acid Derivatives


55.1 Survey of Derivatives
Carbonic acid (H2CO3, HOCO2H) is unstable; in aqueous solution, loosely hydrated carbon dioxide strongly predomi-
nates, equilibrating slowly with carbonic acid and producing a slightly acidic reaction. Derivatives of carbonic acid, such
as halides, esters, amides, and analogues with other heteroatoms, are stable compounds.

carbonic acid derivatives


OH Cl OR OR OH OR NH 2 NH 2 OH NH 2
O C O C O C O C O C O C O C HN C S C S C
OH Cl Cl OR NH 2 NH 2 NH 2 NH 2 OH NH 2
carbonic acid dichloride ester chloride diester monoamide ester amide diamide iminodiamide thiocarbonic acid diamide
(unstable) phosgene dialkyl carbonate carbamic acid urethane urea guanidine (unstable) thiourea
(unstable)

55.2 Carbonic Acid Chlorides


55.2.1 Phosgene
Two chlorides are derived from carbonic acid. Carbonic acid monochloride, incorrectly named chloroformic acid, is unsta-
ble but carbonic ester chlorides exist. Carbonic acid dichloride, better known as phosgene, is produced by catalytic
chlorination of carbon monoxide, or by heating carbon tetrachloride with fuming sulfuric acid.
O O
charcoal , 100 - 150 °C 80 °C
CO + Cl2 Cl C CCl4 + H 2SO4 + SO3 Cl C + 2 Cl SO3H
Cl Cl chlorosulfonic acid
phosgene phosgene

Phosgene with its suffocating odor, smelling like moldy hay when diluted with air, has been used as a poison gas. It is
hydrolyzed in warm water to carbon dioxide and hydrogen chloride. Overexposure by inhalation liberates hydrogen
chloride in the bronchial tubes, causing peribronchial edema and pulmonary congestion.
O
Cl C + H 2O CO2 + 2 HCl
Cl

55.2.2 Carbonic Acid Ester Chlorides


Carbonic acid ester chlorides are formed by nucleophilic substitution of phosgene by an alcohol in the absence of a base.
The benzyl ester chloride, also known as benzyl chlorocarbonate, benzyloxycarbonyl chloride, or carbobenzoxy chloride
and used for the protection of amino groups in peptide synthesis (Chapter 69.4.1), is obtained by passing phosgene into
benzyl alcohol:
O O O O
Cl C + HO R Cl C Cl C + HO CH2 Cl C
 HCl  HCl
Cl O R Cl O CH2
carbonic acid ester chloride
carbobenzoxy chloride
(benzyloxycarbonyl chloride)

Carbonic acid ester halides are frequently referred to as chloroformic acid esters (chloroformates). This is misleading
because these compounds are derived from carbonic acid with four carbon-oxygen bonds, and not from formic acid, with
one CH and three carbon-oxygen bonds. This difference in the oxidation states shapes the reactivity.

Efficiently Studying Organic Chemistry: Exam training for chemists, biochemists, pharmacists, life and health scientists,
Third Edition. Eberhard Breitmaier. © 2022 WILEY-VCH GmbH. Published 2022 by WILEY-VCH GmbH.
55.3 Carbonic Acid Esters 155

55.3 Carbonic Acid Esters


55.3.1 Dialkyl Carbonates, Dialkyl Dicarbonates
Carbonic acid diesters or dialkyl carbonates arise from the alcoholysis of phosgene in the presence of a base such as
triethylamine or pyridine.
Cl O R
O C + 2 R OH + 2 NR'3 O C + 2 R'3NH Cl
Cl O R
R = C2H5 : diethyl carbonate

Polycarbonates, used for manufacturing automotive components and data storage media (CDs, DVDs), are produced by
alcoholysis of phosgene with diols and bisphenols (Chapter 58.4).
While dicarbonic acid (H2C2O5) does not exist, dicarbonic acid diesters (pyrocarbonic acid diesters) can be prepared:
Passage of carbon dioxide into a solution of potassium t-butoxide in tetrahydrofuran (THF) yields the potassium salt of t-
butyl monocarbonate which is acylated with phosgene. The resulting di-t-butyl tricarbonate undergoes decarboxylation in
the presence of a base (1,4-diazabicyclo[2.2.2]octane, DABCO), leaving di-t-butyl dicarbonate (t-butyloxycarbonyl anhy-
dride, Boc2O) as a reagent used to protect amino functions in peptide synthesis (Chapter 69.4.1).
O
O N
O THF
C O O O O O
0 °C + Cl Cl ,  15 °C N
2R O K + C 2R C R C C C R R C C R
O O K O O O O  CO2 O O O
 2 KCl
O
alkyl monocarbonate (salt) R = C(CH3)3 : di-t-butyl tricarbonate di-t-butyl dicarbonate
(Boc anhydride, Boc 2O)

55.3.2 Carbamic Acid Esters (Urethanes)


Carbamic acid is the unstable monoamide of carbonic acid. Carbon dioxide (dry ice) and liquid ammonia react to give
ammonium carbamate, containing the resonance-stabilized carbamate anion. Hydrolysis with warm water cleaves the
carbamate to carbon dioxide and ammonia.
in H2O
NH 2 + NH3 NH 2 NH 2 NH 2 60 °C
O C O + NH3 O C NH 4 O C O C O C CO2 + 2 NH 3
OH O O O
carbamic acid resonance formulas of the carbamate anion

Urethanes are the esters of carbamic acid and its N-alkyl derivatives. They are prepared by nucleophilic substitution of
carbonic acid ester halides with ammonia or amines. Pure N-alkylurethanes result from the addition of alcohols to isocya-
nates (Chapter 56.3).
Cl NH 2
O C + 2 NH 3 O C ethyl urethane
 NH 4Cl (ethyl carbamate)
O C 2H 5 O C 2H 5

55.4 Urea, Thiourea, Guanidine


55.4.1 Urea
In contrast to the monoamide carbamic acid, the diamide of carbonic acid is a stable crystalline compound known as urea
and as the end product of nitrogen metabolism in mammals, excreted in urine. Urea is weakly basic and protonated to
uronium salts. The positive charge of the uronium cation arising from protonation is delocalized, as described by a hybrid
of four resonance formulas.

NH2  H2O NH 2 NH2 NH2 NH2


O C + H3O HO C HO C HO C HO C
NH2 NH 2 NH2 NH2 NH2
156 55 Carbonic Acid Derivatives

In the presence of unbranched hydrocarbons, alkyl halides, and fatty acids (C8-C30) as guest molecules, urea crystallizes
in a hexagonal lattice with long channels enclosing the unbranched guests. The formation of such inclusion compounds
or clathrates (from Lat. clatratus = caged) permits separation of unbranched from branched isomers, because the chan-
nels are too narrow for the latter.
Urea is prepared from carbon dioxide and ammonia under pressure, and by ammonolysis of ester chlorides (alkyloxycar-
bonyl chlorides), dialkyl carbonates and phosgene which can be safely disposed of by this reaction.
Cl
pressure
phosgene O C + 4 NH 3 O C O + 2 NH3
 2 NH 4Cl  H2O
Cl
Cl NH2 O R
+ 3 NH 3 + 2 NH 3
O C O C O C
O R  NH4Cl ,  R OH  2 ROH O R
NH2
carbonic acid ester chloride urea dialkyl carbonate

Urea undergoes hydrolysis when heated in the presence of strong mineral acids or bases:
NH2
+ 2 OH + 2 H3O
CO32 + 2 NH3 O C CO2 + 2 NH4 + H2O
NH2

Like all non-aromatic compounds containing primary amino groups, urea reacts with nitrite in acidic solution to give
unstable diazonium salts which decompose to carbon dioxide, nitrogen and water (VAN SLYKE reaction, Chapter 39.2).
O C(NH 2)2 + 2 HNO2 CO2 + 2 N2 + 3 H 2O

Carboxylic acid halides, anhydrides, and esters as acylation agents react with urea, forming acylureas known as ureides.
Barbituric acid (Chapter 51.2.5) is the cyclic ureide of malonic acid.
O O O O
+ acetylurea,
H3C C C NH2 H3C C N C NH2 ureide
 HCl
Cl H2N
H

Di- and tetraalkylureas are obtained by aminolysis of phosgene.


Cl NR 2
O C + 4 HNR 2 O C + 2 H 2NR 2 Cl
Cl R = alkyl or aryl NR 2

55.4.2 Guanidine and Thiourea


Guanidine, formally arising from replacement of the carbonyl of urea by imino, is one of the strongest known organic
bases (pKa = 12.5). The guanidinium cation formed by protonation of guanidine is a hybrid of four resonance formulas
similar to the uronium ion, indicating distribution of the positive charge among all atoms of the ion (Y stabilization due to Y
shape). All CN bonds are equivalent, having equal bond lengths (118 pm).

NH 2 NH2  H 2O NH2 NH 2 NH 2 NH2


O C HN C + H3O H 2N C H2N C H2N C H2N C
NH 2 NH2 NH2 NH 2 NH 2 NH2
urea guanidine resonance formulas of the guanidinium cation

Thiourea, representing the diamide of unstable thiocarbonic acid and another hybrid of resonance formulas, equilibrates
with its isothiourea tautomer.

NH2 NH2 NH 2 NH
S C S C S C HS C
NH2 NH2 NH 2 NH 2
resonance formulas of thiourea isothiourea
55.5 Derivatives of Dithio- and Trithiocarbonic Acid 157

Isothiourea is a sulfur nucleophile, which is able to substitute the halide of alkyl halides to give S-alkylisothiuronium
halides. In the presence of picric acid, anion exchange takes places, and yellow S-alkylisothiuronium picrates crystallize
from the solution. This is a test tube reaction used to identify alkyl halides.
O2N
NH2 R NH 2 R NH2 R NH2 R NH 2
_
R X + _ C
HS X S C S C S C S C O NO2
NH NH 2 NH2 NH2 NH 2
O2N
isothiourea tautomer resonance formulas of the S-alkylisothiuronium ion S-alkylisothiuronium picrate

Guanidine and thiourea are synthesized from calcium carbide CaC2 (Chapter 19.2.2) by way of calcium cyanamide,
CaCN2, and cyanamide, the tautomer of carbodiimide (Chapter 56.4). Ammonia adds to cyanamide to form guanidine,
while the addition of hydrogen sulfide yields thiourea.
1000 °C
CaC2 + N2 CaCN2 CaCN 2 = Ca 2 IN C NI
C
calcium cyanamide

+ H2O , + CO2 ,  CaCO3


NH tautomerism NH 2 NH 2
+ H 2S + NH3
HS C S C N C NH2 HN C
NH 2 NH 2 cyanamide NH 2
isothiourea thiourea guanidine

55.5 Derivatives of Dithio- and Trithiocarbonic Acid


Dithiocarbonic acid is unstable. Carbon disulfide is the stable anhydride of this acid and reacts to provide some deriva-
tives, including the esters, known as xanthogenic acids, and their salts, known as xanthogenates.
_
SH SH SI
_ M SH SR
S C S C S S C S C S C S C
OH OR OR SH SR (or H)
dithiocarbonic acid carbon disulfide xanthogenic acid xanthogenate trithiocarbonic acid trithiocarbonic acid diester

Nucleophilic addition of alcoholates to carbon disulfide produces xanthogenates:


S Na
sodium
S C S + Na O CH2 CH3 S C ethyl xanthogenate
O CH2 CH3

Unlike thio- and dithiocarbonic acid, trithiocarbonic acid is stable below 80 °C. Nucleophilic addition of ammonium hydro-
gensulfide to carbon disulfide at 0 °C yields ammonium hydrogentrithiocarbonate. Nucleophilic addition of ethanethiol to
carbon disulfide in aqueous sodium hydroxide, producing the thiolate anion as nucleophile, leads to monoethyl trithiocar-
bonate.
S NH 4 SH
NaOH
S C S + NH4 SH S C S C S + HS CH2 CH 3 S C
SH S CH2 CH 3
ammonium hydrogentrithiocarbonate monoethyl trithiocarbonate

________________________________________________________________________________________________
Chapter 55 permits answers to the following:
(55.1) Which stable derivatives of carbonic acid do you know?
(55.2) What products are obtained from the reaction of phosgene and (a) ammonia, (b) ethanol with and without base,
and (c) potassium t-butylmonocarbonate?
(55.3) Explain why all CN bonds of the guanidinium cation have equal length.
(55.4) Formulate equations to outline the syntheses of (a) tetramethylurea, (b) guanidine, and (c) thiourea.
(55.5) What are the parent compounds of (a) urethanes and (b) ureides?
158 56 Heterocumulenes

56 Heterocumulenes
56.1 Analogues of Carbon Dioxide
Carbon dioxide CO2 occurs as a minor but essential constituent of air (0.036 % v/v), required for photosynthesis (Chap-
ters 67.2.2, 71) on the one hand. On the other hand, it absorbs the infrared radiation emitted at the surface of the earth so
that increasing carbon dioxide concentration due to production of energy by combustion of fossil sources (coal, gas, oil)
warms the atmosphere (hothouse effect).
Carbon dioxide and its analogues with heteroatoms other than oxygen, such as carbon disulfide, isocyanates, isothiocya-
nates, and carbodiimides, are referred to as heterocumulenes because they can be formally derived from cumulenes
(1,2-dienes, Chapter 17.2.3) by replacing CC by CX double bonds (X = N, O, and S).
carbon dioxide derivatives (h e t e r o c u m u l e n es)
R R R R
R
C C C H2C C O O C O O C S S C S N C O N C S N C N
R
R ketene carbon carbon carbon R
cumulene (,-diene) dioxide oxide sulfide disulfide isocyanate isothiocyanate carbodiimide
(R=H : allene = 1,2-propadiene)

56.2 Carbon Disulfide


Carbon disulfide as the sulfur analogue of carbon dioxide is a mobile, highly refractive, very flammable and toxic liquid
(b.p. 46.5 °C) which has been extensively used as a versatile solvent for nonpolar organic compounds but is now being
increasingly avoided due to its toxicity. It is produced industrially by heating vaporized sulfur with charcoal or methane in
the presence of alumina.
750 °C , Al2O3
CH 4 + 4S CS2 + 2 H2S

56.3 Isocyanates, Isothiocyanates


Isocyanates arise from rearrangements of carboxylic acid derivatives (Chapter 57.1.4). They are produced by phosgena-
tion of primary amines to N-alkylcarbamoyl chlorides followed by dehydrohalogenation. Phosgene as an acylation reagent
is being increasingly replaced by the less toxic dimethyl carbonate in the presence of catalysts.
H
Cl R N R
3 R NH 2 + C O C O + R NH 2 N C O
Cl  R NH 3 Cl Cl  R NH 3 Cl
N-alkylcarbamoyl chloride isocyanate

Isothiocyanates are prepared by addition of primary amines to carbon disulfide in the presence of ammonia, followed by
elimination of hydrogen sulfide from the intermediate thiocarbamates, driven by lead salts which form insoluble lead
sulfide (PbS).
H
R N R
NH 3 Pb salt
R NH 2 + S C S C S N C S
H 2 S
HS
thiocarbamate (ammonium salt) isothiocyanate

The CX double bonds of isocyanates and isothiocyanates are electrophilic due to the ()-M effect of the doubly bonded
heteroatoms, thus reacting with nucleophiles. Nucleophilic addition of alcohols to isocyanates or isothiocyanates yields
pure urethanes (Chapter 55.3.2) or thiourethanes.
H
_ N
X = O : phenyl isocyanate X = O : ethyl N-phenylurethane
X = S : phenyl isothiocyanate X C N + _ C2H 5
HO X C X = S : ethyl N-phenylthiourethane
O C 2H 5

Efficiently Studying Organic Chemistry: Exam training for chemists, biochemists, pharmacists, life and health scientists,
Third Edition. Eberhard Breitmaier. © 2022 WILEY-VCH GmbH. Published 2022 by WILEY-VCH GmbH.
56.4 Carbodiimides 159

Nucleophilic addition of ammonia, and primary and secondary amines to isocyanates produces alkylurea and di- or
trialkylureas. Alkylated thioureas arise analogously from isothiocyanates.
R NH R R NH R R R1 NH R
O C N + NH 3 O C O C N + R 1 NH 2 O C O C N + NH O C
NH 2 NH R 1 R2 NR 1 R 2
alkylurea dialkylurea trialkylurea

Symmetrical dialkylureas are directly obtained in this way by reacting phosgene with excess primary amine.

Cl _ NH R
R + RNH2
O C + 3 H 2N R O C N O C
R = alkyl or aryl  2 RNH3 Cl NH R
Cl
isocyanate dialkylurea

56.4 Carbodiimides
Unlike dialkyl- or diarylcarbodiimides, unsubstituted carbodiimide and cyanamide (Chapter 55.4.2) equilibrate with each
other. Molecular spectra provide evidence for this equilibrium of tautomers.
cyanamide N C NH2 HN C NH carbodiimide

Diaryl- or dialkylcarbodiimides are prepared by bimolecular decarboxylation of isocyanates in the presence of phospho-
line oxide as catalyst, probably involving two sequences of [2+2]-cycloaddition and cycloreversion.
isocyanate iminophosphorane R carbodiimide cat. =
R N R
R O R R N R
N C O cycloaddition N C cycloreversion N C N C cycloreversion N C N
+ + + P
R 3P O  O=C=O R 3P O R3P O R
R3P O (cat.) R 3P O O R

Other methods involve dehydration of N,N´-dialkylureas and derivatives or, analogously, elimination of hydrogen sulfide
from N,N´-dialkylthioureas promoted by the presence of mercuric oxide.
H H H R
tautomerism + HgO
N N N N N C N + HgS + H2O
R C R R C R
R
S SH

As the reverse of these eliminations, nucleophilic addition of water to carbodiimides gives N,N´-dialkylureas:
H H H
R + H2O N N tautomerism N N
N C N R C R R C R R= R= CH(CH 3)2
 H 2O OH O
R base dicyclohexylcarbodiimide diisopropylcarbodiimide
dialkylcarbodiimide N,N'-dialkylurea DCC DIC

Carboxylic acids add to carbodiimides to yield isourea esters with a leaving group much better than OH (active esters),
reacting much more readily with nucleophiles such as alcohols or amines. Therefore, DCC and DIC are used as dehy-
drating coupling reagents for esterifications and in peptide synthesis (Chapter 69.4).
O R O O NH R
+ HO R2
R1 C + N C N R1 C N R R1 C + O C
O H R O C O R2 NH R
NH R
carbodiimide adduct (isourea active ester) ester N,N´-dialkylurea
________________________________________________________________________________________________
Chapter 56 permits answers to the following:
(56.1) What is a heterocumulene? Give examples. How are carbodiimide and cyanamide related to each other?
(56.2) Formulate the preparation of (a) phenyl isocyanate, (b) phenyl isothiocyanate, and (c) DCC.
(56.3) Formulate the reactions of phenyl isocyanate with (a) 1-butanol, (b) ammonia, and (c) diethylamine.
(56.4) React DCC, first with a carboxylic acid, then with (a) an alcohol and (b) a primary amine. Formulate all reactions.
160 57 Rearrangements

57 Rearrangements
Most rearrangements (Chapter 11.6, symbolized by a loop on the reaction arrow) involve migration of an alkyl or aryl
group from one atom to the adjacent atom and are known as 1,2-shifts. Formally, these groups migrate as anions in
anionotropic or nucleophilic shifts, less frequently as cations in cationotropic or electrophilic shifts, and rarely as radicals
in radical shifts.

57.1 Anionotropic 1,2-Shifts


Anionotropic 1,2-shifts, also called sextet rearrangements, are initiated by the formation of a reactive intermediate having
an atom (C, N, or O) with a sextet of electrons. Carbenium ions, carbenes, nitrenium ions, nitrenes, and oxenium ions are
such electron-deficient intermediates.

57.1.1 General Mechanisms (Sextet Rearrangements)


- Electron Sextet at Carbon (Carbenium Ions, Carbenes)
Many rearrangements are driven by the initial generation of a less stable carbenium ion which is able to undergo aniono-
tropic 1,2-shift of an alkyl group to a more stable carbenium ion. The same force triggers the rearrangement of acylcar-
benes to ketenes by anionotropic 1,2-shift of the alkyl group.
R R O H
_
C C C C C CH O C C
1 2 1 2
R R
rearranged carbenium ion acylcarbene ketene

The intermediate carbenium ions and carbenes are useful for explanation but do not necessarily react as such. Experi-
mental results provide evidence that the group X leaves an ion or carbene intermediate in which disconnection of the
original bond and formation of the new bond occur simultaneously:
R X R R
C C C C C C
 XI

-Electron Sextet at Oxygen and Nitrogen (Oxenium and Nitrenium Ions, Nitrenes)
Oxenium and nitrenium ions and acylnitrenes are reactive intermediates with an electron sextet at the oxygen or nitrogen
atom. They undergo anionotropic 1,2-shifts to give carbenium ions and isocyanates, respectively.
R R R R O
_ R _ _
R C OI
_ C O
_I C N C N C NI O C N
R R R R R R
oxenium ion carbenium ion dialkylnitrenium ion carbenium ion acylnitrene isocyanate

Generally, the relative stability of the intermediate ions (tertiary > secondary > primary in the case of carbenium ions) con-
trols all 1,2-alkyl shifts, usually resulting in formation of the more stable ion:

H 3C CH3 subsequent
H H3C reactions
H 3C C C C C H
H 3C CH3 H3C CH3
secondary carbenium ion tertiary carbenium ion
more stable

Stabilization of the cyclic intermediate occurring during migration increases the tendency of a group R to migrate. To
conclude, aryl groups tend to migrate more than alkyl groups and hydrogen (hydride) due to resonance stabilization of the
cyclic intermediate, as exemplified in the HOCK process (Chapter 52.3.1).

Efficiently Studying Organic Chemistry: Exam training for chemists, biochemists, pharmacists, life and health scientists,
Third Edition. Eberhard Breitmaier. © 2022 WILEY-VCH GmbH. Published 2022 by WILEY-VCH GmbH.
57.1 Anionotropic 1,2-Shifts 161

57.1.2 1,2-Shifts from Carbon to Carbon


- WAGNER-MEERWEIN and Pinacol Rearrangement
WAGNER-MEERWEIN rearrangements (Chapter 36.1) stabilize the carbenium ions (arising from the first step of electrophilic
additions, E1, and SN1 reactions) by anionotropic 1,2-alkyl shifts so that rearranged alkenes are formed. Pinacol rear-
rangements (Chapter 36.2) of alkylated 1,2-diols to ketones follow the same path.

- WOLFF Rearrangement
WOLFF rearrangement of intermediate acylcarbenes, generated by nitrogen elimination from -diazo ketones, to ketenes
opens access to homologized carboxylic acids from acid halides and diazomethane. This sequence of reactions is known
as the ARNDT-EISTERT synthesis.
O + CH2N2
O O H O
+ H2O
R C R C R C O C C C CH2 R
HCl  N2 _
Cl CH
_ N2 CH R HO
carboxylic acid homologized
chloride -diazo ketone acylcarbene ketene carboxylic acid

57.1.3 1,2-Shifts from Carbon to Oxygen


- BAEYER-VILLIGER Oxidation
An anionotropic 1,2-alkyl shift from the carbonyl C to the electron-deficient peroxy O is the key step of the BAEYER-
VILLIGER oxidation of ketones to carboxylic acid esters with peroxycarboxylic acids as reagents.
O
ketone C _
R R IOI O
O + O R C R
1
R C
1
[H ]  R´CO2
C O C O O R
R' O H R' O 2 2

carboxylic acid ester


peroxycarboxylic acid

57.1.4 1,2-Shifts from Carbon to Nitrogen


Some anionotropic 1,2-alkyl shifts of nitrenium ions and acylnitrenes open access to special primary amines from carbox-
ylic acid derivatives (amides, hydroxamic acids and azides), as summarized in Fig. 57.1. Isocyanates are the key prod-
ucts primarily arising from these rearrangements; they undergo hydrolysis to the primary amines by way of unstable N-
alkylcarbamic acids.

O
R C
1
2 NH
2
carboxamide

HOFMANN + Br 2  2 HBr

O O O O
LOSSEN CURTIUS  NaN3
R C R C R C R C
_  NaCl
1
 H2 O 1
 N2 1
2 NH OH 2 N 2 N
_ N NI Cl
hydroxamic acid acylnitrene carboxylic acid azide carboxylic acid halide

R
R N H
1 2 + H2 O
O C N O C R NH 2
 CO2
isocyanate OH primary
N-alkylcarbamic acid amine

Fig. 57.1. Primary amines from carboxylic acid derivatives involving isocyanates (HOFMANN, LOSSEN and CURTIUS rearrangements)
162 57 Rearrangements

BECKMANN rearrangement of oximes to N-alkylcarboxamides starts with protonation of the hydroxy group. Dehydration
leaves an intermediate dialkylnitrenium ion which undergoes anionotropic 1,2-alkyl shift. Water adds to the resulting
carbenium ion to give the N-alkylcarboxamide.
R R R OH O
+ [H ] 1 1 + 2 H2O 1 tautomerism 1
R C R C R C R C R C R C
 H 2O  H 3O
N OH N OH 2 2 N 2 N R 2 N R 2N R

H
oxime carbenium ion N-alkylcarboxamide

57.2 Cationotropic 1,2-Shifts


Cationotropic 1,2-shifts usually require the presence of a base for deprotonation to an intermediate carbanion which
rearranges to a more stable anion by migration of a cation to the adjacent negatively charged carbon.
R H R R R subsequent
_ _ reactions
C1 C2 C1 C 2 C 1 C2 C1 C 2
[H ]

57.2.1 FAVORSKII Rearrangement (from Carbon to Carbon)


The FAVORSKII rearrangement converts -halo ketones into esters. Alkoxide as the base initially deprotonates at the '-
position. The resulting carbanion forces the halide ion to leave upon closure to an intermediate cyclopropanone ring in the
rearrangement step. Ring-opening nucleophilic addition of an alkoxide anion to the carbonyl carbon produces the rear-
ranged ester.

R R R R R R R R
H R R
R '  R + R'O R _ R + R'O
R C
_ C R
+ R'OH
R C C R
C C C C R R R R
C  R'OH C  Cl _  R'O
R Cl R Cl C H CO2R'
O R'O OI
_ R'O O
O O

FAVORSKII rearrangement of -halocycloalkanones results in ring contraction. This can be used for preparing cyclo-
pentane derivatives from appropriately substituted -halocyclohexanones. In the simplest case, -chlorocyclohexanone
provides cyclopentanecarboxylic acid esters:

O O O O
O OR'
C OR' CO2R'
Cl + R'O H _ Cl + R'O + R'OH

 R'OH  Cl  R'O

57.2.2 STEVENS Rearrangement (from Nitrogen to Carbon)


The STEVENS rearrangement transforms quaternary ammonium ions to tertiary amines. First, a base deprotonates  to
the nitrogen, facilitated by an electron acceptor E such as carbonyl or cyano at this position. The resulting nitrogen ylide
undergoes cationotropic 1,2-alkyl shift from nitrogen to the adjacent carbanionic carbon, producing a rearranged tertiary
amine.
R R R
2 1 + NH 2 1 2 1
E CH 2 N R E CH
_ N R E CH N
_
NH 3 2
R R R R
quaternary ammonium ion nitrogen ylide rearranged tertiary amine
O
E = R C , RO2 C  , N C R = CH2CH=CH 2 (allyl) , CH2C6H5 (benzyl) , CH(C 6H5)2 (benzhydryl)

57.2.3 WITTIG Rearrangement (from Oxygen to Carbon)


The WITTIG rearrangement converts allyl or benzyl ethers into alkoxides. Strong bases such as alkyllithium compounds
are necessary to deprotonate the methylene carbon  to the ether oxygen. The resulting carbanion abstracts the other
57.3 Rearrangements at Benzenoid Rings 163

alkyl group of the ether which formally migrates as a cation from oxygen to the adjacent carbanionic carbon atom. WITTIG
rearrangements are useful for preparing secondary and tertiary alcohols with alkenyl and aryl residues, favored by reso-
nance stabilization of the intermediate benzyl or allyl anions.

2 1 base 1 2 _1
R CH2 O R CH
_ O R CH OI
_
[H ] 2
R' R' R'
ether , R = alkenyl or aryl carbanion rearranged alkoxide

57.3 Rearrangements at Benzenoid Rings


Rearrangements of substituted benzenoid aromatic compounds include migrations of a substituent to another position of
the ring or from the side chain to the ring. Some of these are migrations of electrophiles following the pattern of intramo-
lecular electrophilic substitutions into positions predestined by resonance effects. The FRIES rearrangement (Chapter
52.4.2) of phenyl esters to o- and p-acylphenols involving acylium ions as migrating electrophiles is an example. In con-
trast, the SOMMELET-HAUSER rearrangement of quaternary benzyl- or benzhydrylammonium ions, competing with the
STEVENS rearrangement, involves intramolecular nucleophilic substitution by a terminal carbanion which is arranged in a
five-membered intermediate before forming the bond with the benzenoid ring.
H
CH2 + H2N
C
_ CH 2 CH 2 CH 3
NR 2 NR 2 NR2
CH 2 H  NH3 CH 2 H ICH2 H _ CH 2 NR 2
CH 2 NR2
dialkyl(benzyl)methyl- N,N-dialkyl-2-
ammonium ion methylbenzylamine

57.4 Sigmatropic Rearrangements


A concerted reorganization of  and  bonds in one step represents a sigmatropic rearrangement or sigmatropic shift
(Chapter 11.6). As an example, a 1,5-diene rearranges to an isomeric 1,5-diene when heated. This is the COPE rear-
rangement which involves two intermediate allyl radicals (fragment with three carbon atoms, C 3), therefore also referred
to as a [3,3]-sigmatropic shift.
2
R R R R 1
3 degenerate
4
6
5
1,5-diene allyl radicals rearranged 1,5-diene 1,5-hexadiene

Original and rearranged 1,5-diene are identical for 1,5-hexadiene with R = H because of symmetry about the 34-bond,
meaning that the rearrangement is not detected (degenerate COPE rearrangement, fluctuating diene).
The oxa-COPE rearrangement of allyl vinyl ethers produces ,-unsaturated aldehydes (R = H) or ketones (R = alkyl or
aryl). CLAISEN rearrangement of allyl phenyl ethers is a variation of the oxa-COPE rearrangement, primarily generating a 2-
allylcyclohexadienone which undergoes rearomatization to give o-allylphenol.
R oxa-COPE R
CLAISEN
heat heat
O O O O HO
H

alkenyl allyl ether -unsaturated carbonyl compound allyl phenyl ether o-allylphenol
________________________________________________________________________________________________
Chapter 57 permits answers to the following:
(57.1) Formulate the general mechanisms of (a) anionotropic and (b) cationotropic 1,2-shifts.
(57.2) Formulate a sequence of reactions which results in homologization of a carboxylic acid (RCO2H  RCH2CO2H).
(57.3) Formulate all reactions which can be used to convert a carboxylic acid into a primary amine.
(57.4) Write equations for the reactions of (a) 2-bromocyclohexanone, (b) benzyltrimethylammonium salt, and
(c) benzyl methyl ether with a base.
(57.5) Sodium phenoxide is reacted with 3-bromo-1-propene, and the product is heated (200 °C). What happens?
164 58 Polymers, Polymerization

58 Polymers, Polymerization
58.1 Monomers, Oligomers, Polymers
When heated under pressure in the presence of an initiating additive, the gas ethene (ethylene) is converted into a solid
compound called polyethene (polyethylene), involving dimeric, trimeric, and tetrameric intermediates.
n n
n H 2C CH 2 CH 2 CH 2 CH 2 CH 2 CH 2 CH 2
2 2 3 3 n
ethene (ethylene) polyethene (polyethylene)
monomer oligomomers polymer

In this process, called polymerization, the small molecule ethene repetitively reacts with its own kind to provide the mac-
romolecule polyethene containing many ethylene units (CH2CH2). Ethene is said to be the monomer, polyethene is
the polymer, and the intermediates (di-, tri-, tetra-, pentamers, etc.) are characterized by the term oligomers. Usually,
monomers (M) are the repetitive units of oligomers and polymers (M)n, as described by the general reaction equation for
a polymerization.
n n
nM MM MMM . . . MMMMMMMM . . .
2 3

Apart from this type of polymerization, polyadditions and polycondensations also yield polymers.

58.2 Vinyl and Diene Polymers


All compounds containing at least one CC double bond such as terminal alkenes, also called vinyl compounds, are able
to polymerize. Polymerizations of terminal alkenes produce saturated vinyl polymers:
H H
vinyl polymerization
vinyl compound n C C CH CH2 n vinyl polymer
R H R

Vinyl polymers (Table 58.1) such as polyethene, polypropene, polyvinyl chloride (PVC), and polyacrylonitrile (PAN) from
the monomers ethene, propene, chloroethene, and acrylonitrile, respectively, are used to produce fibers, foils, hoses,
pipes, and containers. Polytetrafluoroethene, known as Teflon, resists heat and chemicals such as sulfuric acid and
hydrogen fluoride. It is therefore used to produce gaskets, pumps, containers, and labware.
Table 58.1. Vinyl polymers
monomer polymerization polymer

ethene (ethylene) n H 2C CH 2 CH 2 CH 2 polyethene (polyethylene, PE)


n
propene (propylene) n H 2C CH CH 3 CH 2 CH polypropene (polypropylene, PP)
n
CH 3
chloroethene (vinyl chloride) n H 2C CH Cl CH 2 CH polychloroethene (polyvinyl chloride, PVC)
n
Cl
cyanoethene (acrylonitrile) n H 2C CH CN CH 2 CH polyacrylonitrile (PAN, acrylic fibers, acrylic glass)
n
CN
tetrafluoroethene (tetrafluoroethylene) n F 2C CF 2 CF 2 CF 2 polytetrafluoroethene (PTFE, Teflon)
n

Polymerization of 1,3-dienes such as 1,3-butadiene and its 2-methyl derivative, known as isoprene (R = CH3), produces
polyalkenes (unsaturated polymers) in which the diene units are connected at the 1,4-positions.
R R
1,3-diene polymerization
n CH 2 C CH CH 2
n

1,3-diene diene polymer (cis- and trans-configuration possible)


R = CH3 : isoprene (2-methyl-1,3-butadiene) R = CH3 , cis-isomer : natural and synthetic caoutchouc

Such diene polymers exist as cis- and trans-isomers and undergo addition reactions. Natural and synthetic caoutchouc,
for example, are cis-polyisoprene. Vulcanization of this sticky material to mechanically stable and elastic rubber by addi-
tion of sulfur and heating in the presence of a catalyst crosslinks the chains of polyisoprene by forming sulfide bridges.
Efficiently Studying Organic Chemistry: Exam training for chemists, biochemists, pharmacists, life and health scientists,
Third Edition. Eberhard Breitmaier. © 2022 WILEY-VCH GmbH. Published 2022 by WILEY-VCH GmbH.
58.3 Polyethers 165

Polymerizations are reaction cascades (chain reactions) involving initiation, propagation, and termination steps. Depend-
ing on the conditions, radicals, cations, anions, and metal chelates are the reactive intermediates.
Initiation of radical polymerizations occurs with initiating additives such as azobisisobutyronitrile (AIBN, Chapter 40.2)
which readily generate initiating radicals (In.) on gentle heating:
In 2 2 In

Propagation, the growth of the polymer chain, starts with the addition of an initiating radical (In.) to the double bond of
the monomer. When the alkene is unsymmetrically substituted, the radical favorably adds at the more accessible carbon
atom of the alkene, thus generating the higher alkylated and more stable alkyl radical:
R R R
unfavorable favorable
CH 2 C In In + H2C C In CH 2 C
H H H

The addition of this radical to another alkene produces a dimeric radical. Successive repetition of this step elongates the
chain from dimeric via oligomeric to active polymeric radicals (macroradicals):
R R + (n 2) R
R R R R
+ H2C CHR + H 2C CHR H 2C CHR
In CH 2 C In CH 2 C CH 2 C In CH 2 C CH 2 C In CH 2 C CH 2 C
2 n
H H H H H H H
dimeric oligomeric polymeric radical

Termination reactions prevent further propagation. Combination of active oligo- or macroradicals to an inert polymer as
final product is one of such termination reaction.
R R R R R R R R
In CH 2 CH CH2 C + C CH2 CH CH 2 In In CH 2 CH CH 2 CH CH CH 2 CH CH 2 In
x y x y
H H
active radicals inert polymer

58.3 Polyethers
In the presence of acids or bases, epoxides (oxiranes, Chapter 60.1) polymerize to yield polyethers with terminal hydroxy
groups. Acid-catalyzed polymerization involves growing carbenium ions successively adding to oxirane in each step,
while base catalysis drives the growth of alkoxide anions by successive nucleophilic additions to oxirane.

+ +n
O O
HO CH2 CH2 HO CH2 CH2 O CH2 CH2 HO CH2 CH2 O CH2 CH2 O CH2 CH2
propagation n

+ H2O  [H ] termination
acid + [H ]
catalysis HO CH2 CH2 O CH2 CH2 O CH2 CH2 OH
O O n
polyether
H

+n
O
RO CH 2 CH2 O CH2 CH 2 O RO CH 2 CH2 O CH2 CH 2 O CH 2 CH2 O
propagation n

base catalysis + + ROH  [RO ] termination


O

O + OR RO CH2 CH 2 O RO CH 2 CH2 O CH2 CH 2 O CH 2 CH2 OH


n
polyether

58.4 Polyesters
Polyesters are formed by self-dehydration (condensation) of an -hydroxyalkanoic acid,
O O O
n HO (CH 2) x C H O (CH2) x C O (CH2) x C
 (n1) H2O n1
OH OH
-hydroxyalkanoic acid polyester with one unit
166 58 Polymers, Polymerization

or by polycondensation of dioic acids with diols or biphenols (e.g. bisphenol A):


O O O O O O
C C R' C C R' C C R'
n HO R OH + n HO OH HO R O O R O OH
(2n1) H2O n1
dioic acid diol , biphenol polyester with two units
CH 3
R = (CH2 ) x ; R' = (CH 2 ) y ; C (bisphenol A)
CH 3

Like other esterifications, these polycondensations require catalytic amounts of an acid and a high-boiling solvent (tolu-
ene) to remove water continuously during the reaction by azeotropic distillation. Polycondensations of glycerol and other
triols instead of diols with the dioic acids produces crosslinked polyesters.
In the presence of catalysts, lactides of D- and L-lactic acid (Chapter 43.3.2) undergo ring-opening polymerization. The
resulting polyesters, referred to as polylactides or polylactates, are used to produce foils which can be biologically de-
graded and composted.
H O
O O H CH 3
CH 3 cat.
n O
O n
H O O H CH 3 O
H 3C
lactide (bislactone of L-lactic acid) poly-L-lactate (PLLA)

Also catalyzed by acids, dimethyl or diethyl esters of dicarboxylic acids equilibrate with diols or triols to undergo poly-
transesterification, providing polyesters and the volatile alcohols methanol or ethanol which are continuously removed by
distillation.
O O O O O O
C C R C C R C C R
n R 2O R1 OR 2 + n HO OH R2O R1 O O R1 O OH
(2n1) R2OH n1
diester diol polyester

As an example, polytransesterification of dimethyl terephthalate (R1 = p-C6H4 , R2 = CH3) with glycol (R = CH2CH2)
yields the polyglycol terephthalate, known as Terylene or Dacron and used as a textile fiber.
Polyacylation involving acylation of diols (R’ = CH2CH2) or diphenols with dioic acid dichlorides (R = p-C6H4, tere-
phthalic acid dichloride) in the presence of a base is a third method to produce polyesters.
O O O O O O
C C R' C C R' C C R'
n X R X + n HO OH X R O O R O OH
(2n1) HX n1
diacid dihalide ( X = Cl, Br ) diol polyester

The polycarbonate Makrolon used for manufacturing construction materials was produced by polyacylation of the so-
called bisphenol A with phosgene. Polytransesterification of dimethyl carbonate with bisphenol A in a melt process is an
ecologically less harmful alternative.
O O
CH3 CH3
C + 2n NaOH C
n HO C OH + n Cl Cl H O C O OH
(2n1) NaCl
CH3 (2n1) H2O CH3 n
2,2-bis(4-hydroxyphenyl)propane (bisphenol A) Makrolon (a polycarbonate)

58.5 Polyamides
Polyamides are formed by polycondensation of dicarboxylic acids and diamines. Nylon-6,6, a synthetic polyamide fiber, is
produced from adipic acid (hexanedioic acid) and 1,6-diaminohexane (two C6 reactants):
O O O
H
n HO C NH 2 HO C N C NH 2
C OH + n H 2N C N C N
(n1) H2O H H
O O O n2
Nylon-6,6
58.6 Polyurethanes, Polyureas 167

Another polyamide known as Perlon or Nylon-6 is industrially produced by self-polycondensation of 6-aminohexanoic acid
(-aminocaproic acid as one C6 reactant). This -amino acid arises from BECKMANN rearrangement (Chapter 57.1.4) of
cyclohexanone oxime to -caprolactam.
N H2SO4 O
OH
n n -caprolactam
BECKMANN N
H
cyclohexanone rearrangement
oxime + H2O (H2 SO4)
O
H
OH polycondensation N C
n H2N C H 2N C N CO2 H
(n1) H2O H
O O n 2
6-aminohexanoic acid = -aminocaproic acid Nylon 6 (Perlon)

Polyaminolysis of diesters with diamines or polyacylations of diamines with dioic acid dichlorides are alternative ways to
prepare polyamides.

58.6 Polyurethanes, Polyureas


Nucleophilic addition of alcohols or phenols to isocyanates yields urethanes (Chapters 55.3.2, 56.3).
H
R1 _ N R1
O C N + HO
_ R2 O C urethane
O R2

Polyurethanes are produced industrially by polyaddition of diols or diphenols to diisocyanates without byproducts. Having
reactive end groups, polyurethanes can be crosslinked with other diols. The resulting mechanically resistant material is
used to produce brakepads and the soles of shoes.
O O O
R R' R C R' C R C R'
n O C N N C O + n HO OH O C N N O O N N O OH
diisocyanate diol H H H n1

polyurethane (with reactive end groups)


R = (CH 2) x ; ; CH2

R' = (CH 2) x ;

Polyureas are produced analogously by polyaddition of diamines to diisocyanates.


O O O
R R' R C R' C R C R'
n O C N N C O + n H 2N NH 2 O C N N N N N N N NH2
H H H H H H
diisocyanate diamine n1
polyurea (with reactive end groups)

________________________________________________________________________________________________
Chapter 58 permits answers to the following:
(58.1) What is a polymer?
(58.2) What are (a) vinyl polymers and (b) diene polymers? How do they differ? Formulate examples.
(58.3) What kinds of reactions are (a) polymerizations, (b) polycondensations, and (c) polyadditions? Formulate examples.
(58.4) Formulate equations to explain the mechanism of a radical polymerization.
(58.5) Formulate equations to outline the formation of polyethers from oxiranes.
(58.6) Draw the structure of a polylactide. Which reactions permit the production of polyesters? Formulate examples.
(58.7) Which types of polymers are (a) polyamides and (b) polyurethanes? What kinds of reactions permit their production?
(58.8) Which types of polymers are formed from the reaction of 1,4-diisocyanatobenzene with (a) 1,3-propanediol
and (b) 1,4-diaminobenzene (p-phenylenediamine)?
168 59 Syntheses with Organosilicon Compounds

59 Syntheses with Organosilicon Compounds


59.1 Comparison of Organosilicon and Organic Compounds
Some organosilicon compounds display properties similiar to those of organic compounds, such as the NMR reference
tetramethylsilane (TMS) which has the tetrahedral shape and the inert reactivity of the comparable hydrocarbon neopen-
tane (Table 59.1). On the other hand, silicon is not able to form multiple bonds by sidewise overlapping of p orbitals
because the  bonds are too long; thus silacarbonyl compounds are unknown.

Table 59.1. Silicon analogues of some organic compounds


CH 3 CH3 CH 3 CH3 CH 3 H 3C H3C OH
CH4 H3C C CH3 H3C C Cl H 3C C OH H3C C OCH3 H3C C NH C(CH3)3 C O C
CH 3 CH3 CH 3 CH3 CH 3 H 3C H3C OH
methane neopentane t-butyl chloride t-butyl alcohol t-butyl methyl ether di-t-butylamine acetone acetone hydrate
(unstable)
CH 3 CH3 CH 3 CH3 CH 3 H 3C H3C OH
SiH4 H3C Si CH3 H3C Si Cl H 3C Si OH H3C Si OCH3 H3C Si NH Si(CH 3)3 Si O Si
CH 3 CH3 CH 3 CH3 CH 3 H 3C H3C OH
silane tetramethylsilane (TMS) chlorotrimethylsilane trimethylsilanol methyl trimethylsilyl ether hexamethyldisilazane silaacetone dimethylsilanediol
(tetramethylsiloxane) (unknown) (dehydrates to polysiloxanes)

59.2 Halosilanes
Halosilanes as starting chemicals for the preparation of other organosilicon compounds are obtained by nucleophilic
alkylation of silicon tetrachoride with an alkyl- or arylmagnesium halide:
SiCl4 + MgCl SiCl3 + MgCl2 C 6H 5SiCl3 + C6H 5 Mg Cl (C 6H5)2SiCl2 + MgCl2
trichlorophenylsilane (C 6H5)2SiCl2 + C6H 5 Mg Cl (C 6H5)3SiCl + MgCl2
chlorotriphenylsilane

Chloro(methyl)silanes are produced industrially by passing chloromethane over heated copper-activated silicon (MÜLLER-
ROCHOW process). Primarily formed dichlorodimethyldisilane undergoes disproportionation to trichloro(methyl)silane and
chlorotrimethylsilane (TMSCl), the latter as a reagent for trimethylsilylation.
(Cu powder , 300 - 400 °C)
2 CH3Cl + Si (H3C)2SiCl2 2 (H3C)2SiCl2 (H 3C)3SiCl + H 3CSiCl3
dichlorodimethylsilane chlorotrimethylsilane

59.3 Preparative Significance of Trimethylsilyl Compounds


59.3.1 Trimethylsilylation
Following WILLIAMSON’s ether synthesis (Chapter 37.3.2), nucleophilic substitution of chlorotrimethylsilane by alcohols in
the presence of a base yields trimethylsilyl ethers. Trimethylsilylation protects hydroxy groups from unwanted reactions.
Aqueous acids remove the trimethylsilyl protective group.
trimethylsilylation removal
N(C 2 H5 ) 3 + H 2 O (H 3O )
R OH + (H3C)3Si Cl R O Si(CH 3)3 R OH
 HCl  (H3 C) 3SiOH
trimethylsilyl ether

59.3.2 Syntheses with Silyl Enol Ethers


- -Alkylation of Enolizable Carbonyl Compounds
Enols are converted into silyl enol ethers in the presence of basic catalysts such as triethylamine and nonafluorobu-
tanesulfonate (nonaflate). Silyl enol ethers are C-nucleophilic similar to enamines (Chapter 39.7).
_ _ + R3Si Cl _
C O C OI C OI
_ C4 F9SO3 K , N(C2H5) 3
C OSiR
_ 3 C OSiR
_ 3
C CI C C C
H  HCl
carbonyl compound carbanion enolate anion trialkyl silyl enolether (resonance formulas)

Efficiently Studying Organic Chemistry: Exam training for chemists, biochemists, pharmacists, life and health scientists,
Third Edition. Eberhard Breitmaier. © 2022 WILEY-VCH GmbH. Published 2022 by WILEY-VCH GmbH.
59.4 Silicones 169

1-(Trimethylsilyloxy)cyclohexene, prepared from cyclohexanone, substitutes the halide in alkyl halides (e.g. allyl bromide)
to provide the silyl oxonium salt which is hydrolyzed to racemic 2-allylcyclohexanone.

+ (H 3 C) 3 Si Cl
C 4 F9 SO3 K , N(C2 H 5 ) 3 + H2O
O OSi(CH 3)3 + Br OSi(CH 3)3 Br O
 HCl  HBr,  (H 3 C) 3 SiOH
1-(trimethylsilyloxy)cyclohexene allyl bromide (R,S)-2-allylcyclohexanone

- MUKAIYAMA Aldol Reaction


Nucleophilic addition of silyl enol ethers to aldehydes in the presence of titanium tetrachloride as a coordinating LEWIS acid
yields racemic -hydroxy ketones after hydrolysis of the primarily formed silyl oxonium aldolate.
_ R2 (H3C)3Si Cl
O IOSi(CH 3)3 H O OSi(CH 3)3 OH O
+ TiCl4 , CH2Cl2 TiCl 3 + H2O
+ O
R1 H R2  78 °C  TiCl4 R1 R2  TMSOH R1 R2
O
aldehyde silyl enol ether R1 silyl oxonium aldolate (RS)--hydroxy ketone

- MUKAIYAMA-MICHAEL Addition
Nucleophilic addition (MICHAEL addition) of silyl enol ethers to electrophilic alkenes such as ,-unsaturated carbonyl
compounds opens access to 1,5-diketones. Titanium tetrachloride acts as a catalyst.
R1 R2 TiCl 4 , CH2Cl2 R1 R2 + H 2O R1 R2
+
 78 °C  TMSOH
O IOSi(CH
_ 3)3 O OSi(CH3)3 O O
enone silyl enol ether 1,5-diketone

- IRELAND-CLAISEN Rearrangement
Enolized allyl carboxylates are trimethylsilylated with lithium diisopropylamide (LDA) as the base. The resulting ketene
allyl trimethylsilyl ketals readily undergo oxa-COPE rearrangement (Chapter 57.4) to provide ,-unsaturated trimethylsilyl
carboxylates. Hydrolysis liberates the ,-unsaturated carboxylic acids.
O OH OSi(CH 3)3 OSi(CH 3)3 OH
78 °C, LDA
O O + (H3C) 3Si Cl O O + H2O O
 HCl  (H3 C) 3 Si OH

oxo tautomer enol tautomer ketene allyl trimethylsilyl ,-unsaturated ,-unsaturated carboxylic acid
allyl carboxylate ketal trimethylsilyl carboxylate (4-pentenoic acid)

59.4 Silicones
Dialkyl-, alkylaryl- and diaryldichlorosilanes are hydrolyzed to silanediols which, unlike the hydrates of aldehydes and
ketones (Chapter 49.1.1), do not dehydrate to give the monomeric carbonyl analogues containing a silicon-oxygen double
bond but undergo polydehydration to provide polysiloxanes referred to as silicones:
R R R
+ 2n H2O
n Cl Si Cl n HO Si OH Si O
 2n HCl  n H2 O
R R R n
dihalosilane (R = alkyl, aryl) silanediol polysiloxane (a silicone)

Because of their thermal stability, hydrophobism, chemical inertness, and resistance to weathering and oxidation, silicones
are used in thermoresistant, anticorrosive, and water-repellant lubricants and paints. Crosslinked silicones derived from
alkylsilanetriols are elastomers used as rubber substitutes and in reconstructive surgery.
________________________________________________________________________________________________
Chapter 59 permits answers to the following:
(59.1) Which organosilicon compounds do you know? Why don’t silacarbonyl compounds exist? What are silicones?
(59.2) Formulate all reactions you know to describe the preparative significance of chlorotrimethylsilane.
(59.3) Design a synthesis of 4-pentenoic acid from 2-propen-1-ol and acetic anhydride.
170 60 Heteroalicycles

60 Heteroalicycles
60.1 Nomenclature
Heteroalicycles are cyclic ethers, enol ethers, thioethers, amines and enamines, also including cyclic esters and amides
(lactones and lactams, Chapter 43.3.2). Table 60.1 summarizes the nomenclature of the rings with the most frequently
occurring heteroatoms. The ending for the ring size (e.g. irane for a saturated three-membered ring) is added to the prefix
of the heteroatom (az, ox, thi for N, O, and S).

Table 60.1. Endings for heteroalicyle nomenclature (HANTZSCH-WIDMAN system; endings in brackets apply for nitrogen only)
ring size fully unsaturated saturated r ing size fully unsaturated saturated
3 irene (irine) irane (iridine) 7 epine epane
4 ete etane (etidine) 8 ocine ocane
5 ole olane (olidine) 9 onine onane
6 ine ane (inane) 10 ecine ecane

Table 60.2. Nomenclature of unsubstituted heteroalicycles


heteroatom
ring size O S N
fully unsaturated saturated fully unsaturated saturated fully unsaturated saturated
H H
3 O O S S N N N

oxirene oxirane thiirene thiirane 2H-azirine 1H-azirine azirane


(aziridine)

H
O O S S N N
4
oxete oxetane thiete thietane azete azetane
(azetidine)

H H
O O S S N N
5
furan tetrahydrofuran thiophene tetrahydrothiophene pyrrole(azole) pyrrolidine
H H
O O S S N N
O S
O S N NH
1,2- 1,3- 1,2- 1,3- imidazole imidazolidine
dioxolane dithiolane (1,3-diazole) (1,3-diazolidine)
H H
O O O S S S N N N
6

2H-pyran 4H-pyran tetrahydro- 2H-thiopyran 4H-thiopyran tetrahydro- pyridine 1,4-dihydro- piperidine


pyran thiopyran pyridine
H
O O O S S S N N

O S
O O S S N N
H
1,4-dioxine 1,3-dioxane 1,4-dioxane 1,4-dithiine 1,3-dithiane 1,4-dithiane pyrazine piperazine
H H
O O S S N N N
7

oxepine oxepane thiepine thiepane 1H-azepine 2H-azepine azepane

Hence, azirane (aziridine) and azetane, oxirane and oxetane, thiirane and thietane are the names of the saturated three-
and four-membered heterocycles with nitrogen, oxygen, and sulfur as heteroatom (Table 60.2). Prefixes di- and tri-
indicate two or three identical heteroatoms (e.g. dioxane). Frequently, the heteroalicycle is named as the hydrogenated
derivative of the unsaturated heterocycle or heteroaromatic ring [e.g. tetrahydrofuran (THF), tetrahydrothiophene, Table
60.2], or in relation to its natural origin such as piperidine which occurs as partial structure in the alkaloid piperine from

Efficiently Studying Organic Chemistry: Exam training for chemists, biochemists, pharmacists, life and health scientists,
Third Edition. Eberhard Breitmaier. © 2022 WILEY-VCH GmbH. Published 2022 by WILEY-VCH GmbH.
60.2 Preparation 171

black pepper (Lat. Piper nigrum = black pepper). 1,3-Dioxolanes, 1,3-dioxanes, 1,3-dithiolanes, and 1,3-dithianes are
cyclic acetals and thioacetals, respectively.
Benzo-fused heteroalicycles are named similarly to fused aromatic compounds: fusing bonds are indicated by a, b, c, ...,
bond a originating from the heteroatom as illustrated for TCDD, the highly toxic dibenzo-fused 1,4-dioxine ("Seveso
dioxine") produced by combustion of waste containing multiply chlorinated phenols.

piperidine O piperic acid


fOa O Cl O Cl
O e b e b e b
N
d Oc O Cl O Cl
O
1,4-dioxine dibenzo[b,e][1,4]dioxine 2,3,7,8-tetrachlorodibenzo[b,e][1,4]dioxine,
piperine (alkaloid causing pungency of pepper) TCDD, "Seveso dioxine"

60.2 Preparation
60.2.1 Intramolecular Cyclizations
Many cyclic ethers are prepared by intramolecular dehydrohalogenation of -halo alcohols (halohydrins) in the presence
of a base according to the WILLIAMSON synthesis of ethers (Chapter 37.3.2). The efficiency of such cyclizations depends
on the ring strain and the probability of having the reacting nucleophilic and electrophilic centers in close proximity. Thus,
ring-forming ability decreases with the chain length.
yields
n = 2 (oxirane) : good
NaOH
n = 3 (oxetane) : moderate
Br (CH 2)n OH (CH 2)n O
 HBr n = 4 (tetrahydrofuran) : good
n = 5 and more : moderate to poor

Analogously, cyclic thioethers are obtained by intramolecular dehydrohalogenation of -halo thiols:


Cl SH S
NaHCO3 + H2S Ba(OH) 2
S CH 2Cl CH2 Cl
 HCl O HO  HCl
SH HO
6-thiabicyclo[3.1.0]hexane 3-hydroxythietane

Kinetically controlled intramolecular dehydrohalogenation of -halo amines gives the best results for pyrrolidines because
of minimal strain of five-membered rings (n = 4) and high ring-forming probability as reflected in a particularly large rate
constant.
n relative rate constant
2 70
NaOH 3 1
Br (CH2)n NH2 (CH2)n NH 4 60 000
 HBr
5 1 000
6 2

60.2.2 Cycloadditions
- [2+1]-Cycloadditions
[2+1]-Cycloadditions produce three-membered rings, exemplified by the PRILEZHAEV epoxidation of alkenes to oxiranes
(Chapter 16.7) with peroxy acids (RCO3H), involving a concerted "butterfly mechanism". Analogously, some imines are
converted into oxaziranes.
H 3C H 3C
H3C H H3C H
O O
+ O O + O O
C  RCO2H N C  RCO2H N
H3C O R H 3C (H3C)3C O R (H3C)3C
(Z)-2-butene peroxy- cis-2,3-dimethyl- acetaldehyde cis-2-t-butyl-3-methyl-
carboxylic acid oxirane N-t-butylimine oxazirane
172 60 Heteroalicycles

- [2+2]-Cycloadditions
[2+2]-Photocycloaddition of ketones to alkenes is the obvious way to prepare oxetanes (PATERNO-BÜCHI reaction):
H3C C 6H 5
H 3C CH3 H5C 6 C6H5
h C C h H3C C6H5
+ +
O O C O H3C O
H 3C CH3
H3C
2,3-dimethylbutene benzophenone 2,2,3,3-tetramethyl-
4,4-diphenyloxetane

- [3+2]-Cycloadditions
1,3-Dipolar cycloaddition of a 1,3-dipolar molecule (C3) to an alkene (C2) is one way to synthesize five-membered nitro-
gen heterocycles. Cycloaddition of diazoalkanes and azides as 1,3-dipoles to alkenes yields pyrazolines and triazolines.
These eliminate nitrogen when heated (), leaving three-membered rings.
diazomethane phenylazide
N H5C6 N C6H5
H2C N N  N N H5C6 N N N 
N N
 N2  N2
R R H5C6
R R H5C6
R R cis- cis- H5 C6
(Z)-alkene 3,4-dialkylpyrazoline dialkylcyclopropane styrene 1,5-diphenyltriazoline 1,2-diphenylazirane

- [4+2]-Cycloadditions
[4+2]-Cycloadditions (DIELS-ALDER reactions, Chapters 18.2.1, 21.4) of heterodienes such as acrolein to alkenes substi-
tuted by carbonyl groups or other electron-pair acceptors yields six-membered heteroalicycles. Following this concept, 2-
formyl-3,4-dihydro-2H-pyran is produced by the dimerization of acrolein.
H H

+ 6
H H H
H O O O O H O
H
O O O
2H-pyran 4H-pyran
acrolein 2-formyl-3,4-dihydro-2H-pyran

60.2.3 Catalytic Hydrogenation of Aromatic Heterocycles


Catalytic hydrogenation of readily available aromatic heterocycles opens access to heteroalicycles. Substituted piperi-
dines are obtained by catalytic hydrogenation of suitably substituted pyridines.
N
cat.
R + 3 H2 R
NH

60.3 Reactions
60.3.1 Heteroatom as Nucleophile
Apart from oxiranes, most saturated heteroalicycles undergo the typical reactions of acyclic ethers, thioethers, and
amines. Thietane, as an example, is oxidized to the sulfoxide and sulfone with hydrogen peroxide and alkylated by iodo-
methane to yield the sulfonium iodide.
LiAlH 4
O
CH 3 O
S + CH3 I S + H2O2 S + H 2 O2 S O
I
 H 2O  H2 O
S-methylsulfonium iodide thietane sulfoxide sulfone
60.3 Reactions 173

As a nitrogen nucleophile, azirane (aziridine) is not only alkylated by alkyl halides but also adds to electron-deficient CC
double bonds (MICHAEL addition):

base
NH + R Br N R NH + H 2C CH C N N CH 2 CH2 C N
 HBr
1-alkylazirane 1-(2-cyanoethyl)azirane

60.3.2 Ring Opening


Driven by their ring strain, three-membered heteroalicycles are likely to undergo ring opening with nucleophiles. When
bond breaking slowly generates a stabilized carbenium ion which "pulls" a nucleophile in the fast step, ring opening
follows an SN1 pull mechanism. In the alternative SN2 push mechanism, the nucleophile attacks at the side opposite to
the heteroatom "pushing" it to cleave the bond as leaving group.
Nu Nu
S N2
Nu +
X X X
transition state

Steric hindrance in asymmetrically substituted strained rings such as methyloxirane causes the nucleophile to attack at
the less substituted position. Thus, phenylmagnesium bromide and methyloxirane yield 1-phenyl-2-propanol and the
absolute configuration of the stereogenic center does not change.
O OH
H + H2 O
MgBr + CH2 C
H
CH3  Mg 2 ,  OH ,  Br
CH3
(S)-methyloxirane (S)-1-phenyl-2-propanol

60.3.3 Ring Expansion


Similar to the thermal rearrangement of vinylcyclopropane to cyclopentene (Chapter 21.3), vinylaziranes rearrange to 3-
pyrrolines upon heating, driven by the relief of ring strain.
R R
 N  N

vinylcyclopropane cyclopentene 1-alkyl-2-vinylazirane 1-alkyl-3-pyrroline

As an analogue of 1,2-divinylcyclopropane (Chapter 21.5), 2,3-divinyloxirane undergoes C OPE rearrangement to yield


4,5-dihydrooxepine. Another type of reaction takes place in an industrially applied ring expansion: Catalytic dehydration of
2-(hydroxymethyl)tetrahydrofuran to provide 3,4-dihydro-2H-pyran involves an intermediate carbenium ion which rear-
ranges to the expanded ring.
H Al2O3 H
 H + [H ] H 35 °C
O O CH2
O CH 2OH  H2O O CH2  [H ]
O O H
2,3-divinyl- 4,5-dihydro- 2-(hydroxymethyl)- 3,4-dihydro-
oxirane oxepine tetrahydrofuran 2H-pyran
________________________________________________________________________________________________
Chapter 60 permits answers to the following:
(60.1) Which heteroalicycles in Table 60.2 are (a) peroxides, (b) disulfides, (c) acetals, (d) thioacetals, (e) secondary
amines, (f) enol ethers, and (g) enamines?
(60.2) Write equations to prepare (a) tetrahydrofuran, (b) tetrahydrothiophene, and (c) 7-thiabicyclo[4.1.0]heptane.
(60.3) What product is expected from the reaction of cyclohexene and benzophenone under UV light? Write the equation?
(60.4) What products are obtained from the reaction of azirane with (a) iodomethane, (b) acrylonitrile, and (c) butenone?
(60.5) Ethylmagnesium bromide is reacted with (R)-phenyloxirane. Draw the equation and name the product.
(60.6) 5,6-Epoxy-1,3-cyclohexadiene and oxepine equilibrate at 25 °C. Formulate and provide an explanation.
(60.7) What happens when 2-hydroxymethyl)tetrahydrofuran reacts with an acid? Write the mechanism.
174 61 Five-Membered Aromatic Heterocycles

61 Five-Membered Aromatic Heterocycles


61.1 Survey and Nomenclature
Monocyclic five-membered aromatic heterocycles can be formally derived from the cyclopentadienide anion (Chapter
29.1.2) by replacing one ring CH fragment by a heteroatom (N, O, S):
H O
N O S O C
6 6 6 6 H
cyclopentadienide pyrrole furan thiophene furan-2-carbaldehyde
anion furfural
furfurol

Many aromatic heterocycles were first obtained from or found in natural products. Non-systematic common names re-
flecting the natural origin of the heterocycle are therefore generally accepted. As an example, heating bran (Lat. furfur)
with fuming sulfuric acid (oleum) yields furan-2-carbaldehyde (furan-2-aldehyde) which was originally called furfurol
because of this preparation and later changed to furfural due to the aldehyde function. Thus, the name furan was adopted
for the unsubstituted heterocycle.
Five-membered aromatic heterocycles with two and more heteroatoms are referred to as azoles (Table 61.1). Numbering
round the ring is necessary when substituents are present, starting at the highest ranking heteroatom (Table 61.1) such
that substituent positions are indicated by the lowest possible numbers.

Table 61.1. Nomenclature of monocyclic five-membered aromatic heterocycles


H H
N1 O S P Se
5 2

4 3
pyrrole furan thiophene phosphole selenophene
H 1 1 H 1 1
N1 O S N1 O S
N2 N2 N2
N3 N3 N3
pyrazole isoxazole isothiazole imidazole oxazole thiazole
H 1 1 H H
N1 O S N O S N
N2 N2 N2 N N N N N
4N N N N N N N N N N N
1,2,4-triazole 1,2,4-oxadiazole 1,2,4-thiadiazole tetrazole oxatriazole thiatriazole pentazole

61.2 -Excessive Aromatic Heterocycles


The heteroatom (O, NH, S) in furan, pyrrole, and thiophene provides a non-bonding electron pair, releasing it to the
carbons of the five-membered ring. As detected by a dipole moment, directed from the heteroatom to the ring, and as
shown by four dipolar resonance formulas, this electron pair contributes to the  electron sextet accounting for aromatici-
ty. Six  electrons are delocalized over five ring atoms, paralleling the situation in the cyclopentadienide anion (Chapter
29.1.2).

_X X X X X X
\ / or 6 dipole moment 
\ /

Thus, with more than one  electron (6/5) for each atom of their rings, furan, pyrrole, thiophene and the azoles (Table
61.1) are classified as -excessive aromatic heterocycles, attractive for electrophilic reagents.

Efficiently Studying Organic Chemistry: Exam training for chemists, biochemists, pharmacists, life and health scientists,
Third Edition. Eberhard Breitmaier. © 2022 WILEY-VCH GmbH. Published 2022 by WILEY-VCH GmbH.
61.3 Typical Syntheses 175

61.3 Typical Syntheses


61.3.1 Furan, Pyrrole, Thiophene
1,4-Dicarbonyl compounds undergo cyclodehydration to provide alkyl-substituted furans (PAAL-KNORR synthesis), proba-
bly involving dienediols as intermediates. Cyclodehydration of 2,5-hexanedione (acetonylacetone, R1 = R4 = CH3; R2 = R3 =
H) yields 2,5-dimethylfuran.
R3 R2 R3 R2 R3 R2 H2SO4
H H [H ]
H2C CH2
heat
R4 R1 R4 R1 C C CH
 H 2O R4 R1 H 3C 3  H 2O H3C CH3
O O OH HO O O O O
diketo tautomer dienediol tautomer alkylated furan 2,5-hexanedione 2,5-dimethylfuran

Variations of the PAAL-KNORR synthesis open access to other five-membered aromatic heterocycles. Pyrroles are ob-
tained by reacting 1,4-dicarbonyl compounds with ammonia or primary amines, and thiophenes are the cyclization prod-
ucts with phosphorus sulfides. Cyclizations of -acylamino ketones to oxazoles, with ammonia to imidazoles, and with
phosphorus sulfides to thiazoles, follow the same concept.

1,4-dicarbonyl compound -acylamino ketone

P4 S10 R NH 2 N NH 3 NH P4 S10 N
R
S N N R S R
O O O O
[H ] R H [H ]
thiophene pyrrole imidazole thiazole
N
furan oxazole
O O R

Cyclodehydration of -keto esters with -amino carbonyl compounds is the key step of the KNORR synthesis of pyrroles.
The -amino carbonyl compound is obtained in situ by nitrosation of a -keto ester, followed by reduction of the resulting
nitroso-oximino tautomers. KNORR pyrrole synthesis with ethyl acetoacetate (Chapter 50.1.1) produces 2,4-dimethyl-
pyrrole-3,5-dicarboxylic acid diethyl ester (R1 = CH3; R2 = C2H5); hydrolysis of the diester to the diacid followed by decar-
boxylation yields 2,4-dimethylpyrrole.

-keto ester oximino ester nitroso ester


R1 R1 R1
O + HNO2 O tautomerism O
CH 2  H2O OH O
R2O2C R2O2C N R2 O2C N
H
3,5-dialkoxycarbonyl-
Zn , CH 3CO2H + 4 [H ] , + 4 e0 , H2O
 2,4-dialkylpyrrole

R1 CO2R2 R1 CO2R2 R1 CO2R2


O + H2C
-amino carbonyl H H
intermediate R1 N R1 R2O2C N R1
NH 2 O  2 H2O R 2O2C
R2O2C H
-keto ester

61.3.2 Azoles
1,3-Dipolar cycloadditions of diazoalkanes or azides as 1,3-dipoles and alkynes or nitriles as dipolarophiles open access
to azoles. Appropriate reactants can be deduced from the target heterocycles. Pyrazoles are obtained from diazoalkanes
and alkynes, tetrazoles from nitriles and azides:
alkynone nitrile
R1 C C COR2 R1 COR 2 R1 COR 2 R1 C N R1
tautomerism N
+ +
CH N R3 N R3 NH N N N N
R3 N H N N R2 N R2 N
diazoalkane 5-acyl-3,4-dialkylpyrazole alkyl azide 1,5-dialkyltetrazole
176 61 Five-Membered Aromatic Heterocycles

61.4 Typical Reactions


61.4.1 Basicity and Acidity of Pyrrole
Pyrrole is very weakly basic when compared with typical secondary amines because it would lose aromaticity upon
protonation due to disruption of the  electron sextet:
H H H
N + [H ] N
6

Pyrrole and the azoles, however, are weak acids, the acidity of pyrrole corresponding to that of methanol. Salts called
pyrrolides or pyrryl metals are therefore formed with alkali metals or metal hydrides. Similar to alkoxides, pyrrolides are
readily hydrolyzed with water.
+ K (LiH) + H 2O
+ KOH (+ LiOH)
N  1/2 H2 ( H2) N N
K (Li )
H H

Formally derived from the cyclopentadienide anion (Chapter 29.1.2), the pyrrolide anion is a hybrid of five resonance
formulas. In the presence of an alkali metal for deprotonation, pyrrole and cyclopentadiene react with ferrous chloride,
forming the red sandwich complex azaferrocene.
+ 2 K , + FeCl2
6 + Fe
N  2 KCl ,  H2
N N N N N N N
H H H
azaferrocene

61.4.2 Electrophilic Substitutions


Because they are -excessive aromatic heterocycles (Chapter 61.2), thiophene, pyrrole, and furan are activated for
electrophilic substitution, comparable to donor-substituted benzenes. Electrophilic substitution at the 2-position takes
preference. This regioselectivity is reasonable because the positive charge in the  complex is better distributed for 2-
substitution (three resonance formulas) than for 3-substitution (only two formulas):
H H Y
Y Y

X X  [H ] X
3- or -substitution
+ [Y ]
X
H H H
X X X  [H ] X Y
Y Y Y
2- or -substitution

Thus, acetic anhydride acetylates pyrrole without a catalyst to give 2-acetylpyrrole, and sodium nitrate nitrates furan in
anhydrous acetic acid to yield 2-nitrofuran involving in situ generated nitronium acetate as electrophile.

+ (CH 3 CO) 2 O + CH 3 CO2 NO2


CH 3
X C  CH 3 CO2 H X 10 °C,  CH3CO2H O NO2
O 2-nitrofuran (X = O)
2-acetylpyrrole (X = NH)

61.4.3 1,3-Diene Reactions


Among all five-membered aromatic heterocycles, thiophene exhibits an aromaticity similar to that of benzene. Furan,
particularly, also reacts like a 1,3-diene, susceptible to 1,4-additions and cycloadditions, exemplified by its reaction with
bromine in methanol to produce 2,5-dimethoxy-2,5-dihydrofuran.
+ Br 2 , + CH3 OH

O  2 HBr H 3CO O OCH 3


61.4 Typical Reactions 177

Clearly, furan undergoes additions more likely than pyrrole and thiophene, and the ease of nitration by nitronium acetate
as electrophile may also be explained by an addition-elimination mechanism:

+ CH 3CO2  NO2 H H
O  CH 3CO2H
O O O NO2
C O NO2
H 3C

Similar to 1,3-dienes, furan and some N-substituted pyrroles react with butynedioic acid diester as dienophile to form 7-
oxabicyclo[2.2.1]hepta-2,5-diene derivatives by [4+2]-cycloaddition (DIELS-ALDER reaction). Driven by the formation of a
benzenoid ring, the bicycle aromatizes to a 3-hydroxyphthalic acid diester in the presence of acid.
O OH
CO2R
CO2R
[4+2]-cycloaddition [H ]
O + CO2R
CO2R
CO2R CO2R
butynedioic acid diester 7-oxabicyclo[2.2.1]hepta-2,5-diene- 3-hydroxyphthalic
(acetylenedicarboxylic acid diester) 2,3-dicarboxylic acid diester acid diester

61.4.4 Nucleophilic Substitutions


Nucleophilic substitution of halide at thiophene is as difficult as for benzene; however, an electron-withdrawing substituent
at a position comparable to ortho or para in benzene facilitates an SN reaction. Thus, 2-bromo-5-nitrothiophene is substi-
tuted by potassium methoxide to provide the 2-methoxy-derivative, while 2-bromothiophene does not react under similar
conditions.
+ KOCH3 + KOCH3
corresponding to: O2N Br O2N OCH 3
O2N S Br  KBr O2N S OCH 3  KBr
2-methoxy-5-nitrothiophene p-nitroanisole

61.4.5 Ring Opening


Relative to pyrrole and thiophene, furan most easily undergoes ring opening. In a reversal of the PAAL-KNORR synthesis,
acid-catalyzed hydrolysis of furan yields butanedial (succindialdehyde) as the 1,4-dicarbonyl compound:
H cyclohemiacetal butanedial
H
tautomerism
+ H3O + H 2O H H H H
O O  [H ] O OH
OH O O O

Ring opening of pyrrole occurs with hydroxylamine in acidic ethanol, yielding the dioxime of butanedial. Thiophenes resist
hydrolytic ring cleavage but they do undergo ring-opening catalytic hydrogenation and desulfurization to yield substituted
butanes:
RANEY-Ni
C 6H 5 + H 2S
H 5C 6 C 6H 5 + 3 H2 H 5C 6
S
2,5-diphenylthiophene 1,4-diphenylbutane

________________________________________________________________________________________________
Chapter 61 permits answers to the following:
(61.1) Explain the term -excessive heteroaromatic compounds. Which representatives do you know?
(61.2) Account for the acidity of pyrrole and cyclopentadiene in terms of resonance stabilization of the anions.
(61.3) Which aromatic heterocycles can be prepared by the PAAL-KNORR synthesis? Write the equations.
(61.4) Devise syntheses of (a) 2,4-dimethylpyrrole, (b) 5-acetylpyrazole, and (c) 1,5-diphenyltetrazole.
(61.5) What products are obtained from (a) acetylation and (b) nitration of N-methylpyrrole?
(61.6) Formulate two mechanisms for the nitration of furan in anhydrous acetic acid. Which one is more likely?
(61.7) What product arises from reaction of furan with (a) bromine in methanol and (b) butynedioic acid dimethyl ester?
(61.8) Formulate the hydrolysis of furan in aqueous acid. What is the name of the reverse reaction?
178 62 Six-Membered Aromatic Heterocycles

62 Six-Membered Aromatic Heterocycles


62.1 Survey and Nomenclature
Monocyclic six-membered aromatic heterocycles can be formally derived from benzene (Chapter 23) by replacing one
ring CH fragment by a heteroatom (N, P, O, S):

N P O S
6 6 6 6 6

benzene pyridine phosphabenzene pyrylium ion thiopyrylium ion

Non-systematic common names reflecting the natural origin of the heterocycle are accepted for many aromatic heterocy-
cles, as mentioned in Chapter 61.1. The story concerning azabenzene, also called azine, deals with the distillate of the
tar resulting from heating bones on a fire (Greek pyr); thus it was called pyridine, and this has remained the common
name. Phosphabenzene is the phosphorus analogue of pyridine. Oxygen and sulfur as heteroatoms contribute one of
their non-bonding pairs of electrons to the  electron sextet, so that these aromatic heterocycles are ionic, existing as
pyrylium and thiopyrylium salts. Aromatic heterocycles with two and more nitrogen atoms in the ring are named di-, tri-,
and tetrazines as shown in Table 62.1, with the heteroatoms defining the numbering necessary to indicate the position of
substituents. 1,3-Diazines, better known as pyrimidines are the core structures of nucleobases in nucleic acids (Chapter
73).

Table 62.1. Nomenclature of monocyclic six-membered aromatic heterocycles


1 1 1 1
N P O S N N N
6 2 N2
5 3 N3
4
N
4
pyridine phosphabenzene pyrylium ion thiopyrylium ion pyridazine pyrimidine pyrazine
azine (azabenzene) 1,2-diazine 1,3-diazine 1,4-diazine
1 1 1 1 1 1
N N N N N N
N2 N2 N2 N2 N2
N3 3 5N N3 N3 5N N3 5N
N N N
4 4
1,2,3-triazine 1,2,4-triazine 1,3,5-triazine 1,2,3,4-tetrazine 1,2,3,5-tetrazine 1,2,4,5-tetrazine

62.2 -Deficient Aromatic Heterocycles


The trivalent nitrogen atom of pyridine, formally replacing a CH fragment of the benzenoid ring, preserves the sextet of 
electrons, and two canonical KEKULÉ formulas (Chapter 23.1.3) can be drawn. However, a dipole moment directed to
nitrogen in the molecule is detectable, and molecular orbital calculations give reduced  electron densities at the 2-, 4-,
and 6-positions. To conclude, the imino nitrogen atom as an acceptor in the ring disturbs the equal distribution of  elec-
trons, withdrawing a pair of  electrons and leaving positively charged ring carbon atoms at the 2-, 4-, and 6-positions as
depicted by three dipolar resonance formulas:
_ _ _ _ _
N N N
_ N
_ N
_ N
6 dipole moment 

Thus, with -electron-deficient ring carbon atoms due to electron withdrawal by the imino nitrogen, pyridine as the proto-
type of six-membered aromatic heterocycles is classified as a -deficient aromatic heterocycle, attractive for nucleophilic
reagents, in contrast to pyrrole (Chapter 61.2).

62.3 Typical Syntheses


62.3.1 Pyridines
Corresponding to the cyclotrimerization of alkynes to benzenes (Chapters 19.3.7, 24.3.2), alkynes and nitriles react to
give pyridines in the presence of cobalt  complexes which optimally arrange the reactants for cyclization. As an exam-

Efficiently Studying Organic Chemistry: Exam training for chemists, biochemists, pharmacists, life and health scientists,
Third Edition. Eberhard Breitmaier. © 2022 WILEY-VCH GmbH. Published 2022 by WILEY-VCH GmbH.
62.3 Typical Syntheses 179

ple, 2,2'-bipyridine, as a chelating reagent for ferrous ions, is prepared by catalytic cyclization of 2-cyanopyridine and
acetylene:

cat. cat.
+ + cat. = Co
R
N 95 % N
N N R N
N
2-cyanopyridine 2,2'-bipyridine

Symmetrically substituted pyridines are prepared by cyclization of a -oxo ester (two equivalents) with an aldehyde and
ammonia (HANTZSCH synthesis). One equivalent of the -oxo ester reacts with nucleophilic ammonia to give a -enamino
ester, the other undergoes KNOEVENAGEL alkenylation with the aldehyde to yield an enone. The two intermediates cyclo-
dehydrate to provide a 1,4-dihydropyridine which is oxidized to the pyridine by nitric acid.
-enamino ester enone R3
R3 R3
R 2O2C C CO2R 2 R 2O2C H C CO2R 2 2,4,6-trialkyl-
CH H C C H C pyridine
R1 N R1
C C  H2 O C C
R1 NH 2 + O R1 R1 R1 decarboxylation
N
 2 H2O hydrolysis

R3 H R3 R3

C R2O2C CO2R 2 R2O2C CO2R 2


R 2O2C CO2R2 HNO3
CH + H O + H 2C
 2 [H ] ,  2 e0
C C R1 N R1 R1 N R1
R1 OH + NH3 + O R1
H 2,4,6-trialkylpyridine-
enol tautomer keto tautomer 3,5-dicarboxylic acid diester

62.3.2 Pyrimidines
Cyclodehydration of 1,3-dicarbonyl compounds (1,3-diketones, 1,3-dialdehydes and their acetals) with binucleophilic
amidines produces 2-alkylpyrimidines. The preparation of 2,4,6-trimethylpyrimidine (R1 = R2 = R3 = CH3) from acetylacetone
(R1 = R2 = CH3) and acetamidine (R3 = CH3, Chapter 42.6.4) is an example.
R2 R2 R2
oxo-enol NH 2
O tautomerism O N
+ C
H HN R3  2 H2 O
R1 O OR1 R1 N R3
1,3-dicarbonyl compound amidine 2,4,6-trialkylpyrimidine

Guanidine, thiourea, and urea are also amidines and undergo cyclodehydration with 1,3-dicarbonyls to yield pyrimidines.
2-Aminopyrimidines arise from guanidine; 2-thiopyrimidones and 2-pyrimidones as stable tautomers of pyrimidine-2-thiols
and pyrimidin-2-ols are obtained from thiourea and urea, respectively:
R2 R2 R2 R2
NH2 NH2
O N O N
+ C + C
R1 O HN NH2  2 H2O R1 N NH2 R1 O H2N X  2 H2O R1 N X
H
guanidine 2-aminopyrimidine X = O: urea X = O: 2-pyrimidone
X = S : thiourea X = S : 2-thiopyrimidone

62.3.3 Pyrylium Salts


Cyclodehydration of 1,3-diketones such as acetylacetone and acetophenone in the presence of perchloric acid yields
stable 2-phenylpyrylium perchlorates, involving intermediate cyclohemiketals:
R R R

CH3 O + HClO4
+
C H 2O H 2O Ph =
Ph O HO R Ph O R Ph O R ClO4
HO
acetophenone 1,3-diketone (enol) cyclochemiketal 4,6-dialkyl-2-phenylpyrylium perchlorate
180 62 Six-Membered Aromatic Heterocycles

62.4 Typical Reactions


62.4.1 Reactions at the Imino Nitrogen
Pyridine is a weaker base (pKa = 5.25) than aliphatic amines (pKa  9). The non-bonding electron pair at the N atom,
however, does not contribute to the  electron sextet as it does in pyrrole; therefore, pyridine is a stronger base than
pyrrole. Electron-releasing substituents [e.g. NR2, ()-M effect] at the 2- and 4-positions increase the basicity while
electron-withdrawing groups [e.g. CHO, ()-M effect] act oppositely. 4-N,N-Dimethylaminopyridine (DMAP) is a strong
base (pKa = 9.7) and therefore used as an acylation catalyst.
NH2 NH 2

H H
N N
N N O O
4-aminopyridine : pKa = 9.11 pyridine-2-carbaldehyde : pKa = 3.80

Pyridine with its nucleophilic nitrogen substitutes alkyl halides and dialkyl sulfates, yielding N-alkylpyridinium salts. Dia-
zines are monoalkylated; steric influences of alkyl groups control the regioselectivity.
CH 3 CH 3 CH3
+ CH 3 I
N N I + CH 3 I not
N N N
CH 3 N N N CH 3
I I
N-methylpyridinium iodide 4-methylpyridazine CH 3

N-Oxidation of pyridines and diazines occurs with peroxides and peroxy acids. Resonance formulas of pyridine N-oxide
imply a reversal of polarity at positions 2, 4, and 6 of the ring from electron deficiency (Chapter 62.2) to nucleophilicity:

O
+ (C6H5) 3P
+ R C
 R CO2H N N N N  (C6H5) 3P=O
N O OH N
O O O O
resonance formulas of pyridine N-oxide

This kind of "umpolung" activates the 2- and 4-positions of the pyridine ring for electrophilic substitution. Oxygen-
attracting reducing reagents such as triphenylphosphane, (C6H5)3P, can regenerate pyridine from its N-oxide.

62.4.2 Nucleophilic Substitutions


Nucleophiles are attracted to the electron-deficient carbon atoms in 2-, 4-, and 6-positions of the pyridine ring (Chapter
62.2). Pyridine regioselectively undergoes nucleophilic substitutions at these positions, exemplified by the CHICHIBABIN
amination with sodium amide. The leaving hydride anion combines with a proton of the amino group to give hydrogen H2.
Excess amide substitutes first to give 2,6-di- and then 2,4,6-triaminopyridine.
 H2 O
+ Na I NH2
H Na  H2  NaOH
N N N NH Na N NH2
sodium amide NH2
sodium 2-pyridylamide 2-aminopyridine

Nucleophilic alkylations and arylations of pyridine by alkyllithium and phenyllithium, substituting a hydride anion under a
protecting gas, are also possible:

+ Li 2-phenylpyridine
H Li LiH
N N N C 6H5
phenyllithium C6H5
62.4 Typical Reactions 181

While phenyl halides and 3-halopyridines with no other substituents resist nucleophilic substitutions under moderate
conditions, 2-halopyridines as nitrogen analogues of acid halides (imidoyl halides) readily undergo such SN reactions, for
example with alkoxides:
25 °C
+ OCH 3 Cl  Cl
N Cl N N OCH 3
OCH 3
2-chloropyridine 2-methoxypyridine

62.4.3 Electrophilic Substitutions


Looking at the resonance formulas (Chapter 62.2), only the  carbons of pyridine do not bear a positive charge. It is
reasonable therefore, that electrophilic substitutions, when they occur, take place at a -position, as confirmed by nitra-
tion and sulfonation of pyridine:
SO3 / H 2 SO4 KNO3 / H 2 SO4
SO3H NO2
(H 2 S2 O7 ) (370 °C)

N N N
pyridine-3-sulfonic acid 3-nitropyridine

N-Oxidation, however, activates the 2-, 4-, and 6-positions of the pyridine ring for electrophilic substitution (Chapter
62.4.1). Substitution at the 4-position takes preference because electrostatic repulsion of the electrophile by the equally
charged N atom prevents an attack at the 2-position. Strong donor substituents such as the dimethylamino group direct
the electrophile to a "para" position.
NO2
 [NO2 ]  [NO2 ] O2N
(HNO3/H2SO4) (HNO3/H2SO4)

 [H ]  [H ]
N CH 3 N CH 3 N N(CH 3)2 N N(CH 3)2
O O O O
2-methyl-4-nitropyridine N-oxide 2-(N,N-dimethylamino)-5-nitropyridine N-oxide

62.4.4 CH Acidity of Methyl Groups


Methyl groups at the 2- and 4-positions of the pyridine ring are CH acidic because deprotonation leaves a carbanion
which is resonance-stabilized, similar to the carbanions arising from the deprotonation of methyl or methylene ketones
(Chapter 50.2). Therefore, 2-methylpyridines undergo KNOEVENAGEL alkenylation with aldehydes and ketones.

+ IB H ZnCl2 H
 HB C
N CH 2 N CH 2 N CH 2 N CH 3 + O C 6H 5 N C 6H 5
H H
2-methylpyridine trans-2-styrylpyridine

________________________________________________________________________________________________
Chapter 62 permits answers to the following:
(62.1) Explain the term -deficient heteroaromatic compound, using pyridine and the pyrylium ion as examples.
(62.2) Formulate equations for the preparations of (a) 2,2´-bipyridine and (b) 2,4,6-trimethylpyridine.
(62.3) Devise syntheses of (a) 2,4,6-trimethylpyrimidine, (b) unsubstituted pyrimidine, and
(c) 2-amino-4,6-dimethylpyrimidine.
(62.4) Outline a synthesis of 2-phenyl-4,6-dimethylpyrylium perchlorate.
(62.5) 4-N,N-Dimethylaminopyridine (DMAP) is much more basic than pyridine. Give an explanation.
(62.6) Formulate the reaction of pyridine with (a) sodium amide in liquid NH3 and (b) butyllithium under nitrogen.
(62.7) Draw the resonance formulas of pyridine N-oxide. What can be concluded concerning reactivity?
(62.8) Suggest reactions to prepare (a) 3-nitropyridine and (b) 4-nitropyridine.
(62.9) Draw resonance formulas of the intermediates resulting from deprotonation of (a) acetone and (b) 2-methylpyridine.
(62.10) 2-Methylpyrimidine is reacted with benzaldehyde in the presence of ZnCl2. Which (E)-isomer is the major product?
182 63 Benzo-Fused Five-Membered Heteroaromatics

63 Benzo-Fused Five-Membered Heteroaromatics


63.1 Survey and Nomenclature
Benzenoid rings can be fused with aromatic heterocycles at different positions. For an unambiguous nomenclature, the
fusing bond is indicated by italic lower case letters in square brackets, where the a bond originates from the highest
ranking heteroatom (O > S > N), exemplified by two isomeric benzofurans and benzisoxazoles with [b] and [c] fusion.
When enamino NH groups are present in the heterocycle, the position of the NH must be additionally specified as shown
for 1H- and 2H-benzo[d]pyrazole. Specification of the fusing bond is not necessary when only one fusion option exists, for
example in dibenzofuran and dibenzothiophene (Table 63.1).
H
7a O1 7a O1 N 7a N1 N
a ba a ba a a
b 2 c O d
c
N2 c O d N2 d NH
3a 3a 3a 3
4 3 4 3 4
benzo[ b]furan benzo[ c]furan benzo[ d]isoxazole benzo[ c]isoxazole 1H-benzo[ d]pyrazole 2H-benzo[ d]pyrazole

Common names related to the natural origin are also preferred for benzo-fused five-membered heterocycles (Table 63.1).
Thermal degradation of the blue natural dye indigo, obtained from the plant Indigofera (Leguminosae) growing in India,
gave the so-called indigo oil which was called indole (Table 63.1) and turned out to be benzo[b]pyrrole.

Table 63.1. Selected benzo-fused five-membered aromatic heterocycles


H
H 8a N9 1 O S
7a N1 O S
2 2
4b 4a
3a 3 3
4 4
indole coumarone thionaphthene carbazole dibenzofuran dibenzothiophene
benzo[b]pyrrole benzo[b]furan benzo[b]thiophene (dibenzopyrrole)
H H
7a N1 O S 7a N1 O S
N2 N N 2
3a 3 3a N3 N N
4 4
benzo[d]pyrazole benzo[d]isoxazole benzo[d]isothiazole benzimidazole benzoxazole benzothiazole
(1H-indazole) (indoxazole)

63.2 Typical Syntheses


63.2.1 Benzo[b]furan, Benzo[b]thiophene
Benzo[b]furans (coumarones) are prepared by intramolecular cyclodehydration of phenoxymethyl ketones which are
obtained from phenoxide and -halo ketones following WILLIAMSON’s ether synthesis (Chapter 37.3.2). An analogous
procedure is used to convert thiophenoxides (thiophenolates) into benzo[b]thiophenes (thionaphthenes).
HO R
O R R
O R C
C
 NaCl CH2 H2O
X Na + H 2C X X X
Cl
X = O : phenoxide -halomethyl X = O : coumarone
X = S : thiophenoxide ketone X = S : thionaphthene

63.2.2 Benzo[b]pyrrole (Indole)


The FISCHER synthesis of indole begins with the formation of a phenylhydrazone from an -methylene carbonyl com-
pound and phenylhydrazine (Chapter 49.2). In the presence of a LEWIS acid, the tautomer 1 of the phenylhydrazone
undergoes a diaza-COPE rearrangement to give the 1,4-diimine 2 which regenerates the benzenoid ring in forming -(o-

Efficiently Studying Organic Chemistry: Exam training for chemists, biochemists, pharmacists, life and health scientists,
Third Edition. Eberhard Breitmaier. © 2022 WILEY-VCH GmbH. Published 2022 by WILEY-VCH GmbH.
63.3 Typical Reactions 183

aminophenyl)iminoethane 3. Nucleophilic addition of the amino group to the electrophilic imino carbon closes the ring to
yield 2-amino-2,3-dihydroindole 4 which finally eliminates ammonia, leaving the target indole.
R3 R3 R3 H R3
H H R3
H 2C R2 R2 R2 R2
C R2
N NH NH NH NH 2
N N NH NH 2 N
R1 H R1 H R1 R1 R1 H
phenylhydrazone 1 2 3 4

 H2O cat. cat. = ZnCl 2 , H2SO4 or BF3  NH 3

R3 R3
H2C R2
NH 2 + C R2
N
R1 H O N
R1 H
substituted -methylene
phenylhydrazine carbonyl compound indole

63.2.3 Benzo-1,3-azoles
Benzo-1,3-azoles such as benzimidazoles, benzoxazoles, and benzothiazoles are readily obtained by cyclization of o-
phenylenediamine, o-aminophenol, and o-aminothiophenol, respectively, with a carboxylic acid or an ester.
NH 2 O N
+ C R R
 H 2O ,  R'OH
XH R'O X
X = NH, O , S R = H , alkyl , aryl X = NH : substituted benzimidazole
R' = H or alkyl X = O : substituted benzoxazole
X = S : substituted benzothiazole

63.2.4 Carbazole
Reductive cyclization of o-nitrobiphenyl produces carbazole (dibenzopyrrole). The reducing reagent triethyl phosphite is
oxidized to triethyl phosphate

+ 2 P(OC 2H 5) 3
o-nitrobiphenyl carbazole
 2 O=P(OC 2H 5) 3
N
NO2 H

63.3 Typical Reactions


Benzenoid benzo[b]-fused furans, pyrroles, and thiophenes occur more frequently than the benzo[c]-fused isomers with
their quinoid structure:
benzo[b] derivatives benzo[c] derivatives
X=O : coumarone X X=O : isocoumarone
X = NH, NR : indole X = NH, NR : isoindole X
X=S : thionaphthene X=S : isothionaphthene

Non-aromatic but o-quinoid benzo[c]-fused heterocycles represent s-cis-1,3-dienes and are expected to undergo [4+2]-
cycloadditions (Chapter 63.3.2). Only one dipolar resonance formula with an undisturbed benzenoid ring can be drawn for
the aromatic benzo[b]-fused heterocycles indicating nucleophilicity at the 3-position, so that electrophilic substitutions
most likely occur at that position.

X X
184 63 Benzo-Fused Five-Membered Heteroaromatics

63.3.1 Electrophilic Substitutions


Electrophilic substitutions of indole (and benzo[b]thiophene) such as sulfonation (Chapter 25.6), formylation (Chapter
47.2.4), and MANNICH aminomethylation (Chapter 50.2.3) as well as MICHAEL addition (Chapter 51.2.3) regioselectively
take place at the nucleophilic 3-position as expected and summarized in Fig. 63.1.

SO3H

indole-3-sulfonic acid
N
H

+ N SO3  N
H
MANNICH
C O aminomethylation CH2 N(CH3)2
+ O=CHN(CH3) 2
(POCl3) + H2CO , + HN(CH3) 2

 HN(CH3) 2  H2O
N N N
H VILSMEIER H H
indole-3-carbaldehyde formylation O 3-(N,N-dimethylaminomethyl)indole
MICHAEL + H2C CH C
(gramine)
addition
CH3

C O
H 3C 4-(3-indolyl)-2-butanone
N
H

Fig. 63.1. Selected reactions of indole

Alkylmagnesium halides deprotonate indole, leaving a resonance-stabilized anion nucleophilic at the 3-position so that
carboxylation by carbon dioxide occurs there:
CO2H
+ R Mg X 1. + CO2 , 2. + [H ]

N  RH ,  Mg 2 ,  X N N N
H H
indole-3-carboxylic acid

Unlike thionaphthene, benzo[b]furan (coumarone) is substituted at the 2-position due to a weakening of electron release
caused by the inductive effect of the oxygen atom. Indole decomposes when treated with nitrating reagents.
CH3CO2 NO2 NO2 CH3CO2 NO2
+ [NO2 ] + [NO2 ]
NO2
S  [H ] S O  [H ] O
3-nitrobenzo[b]thiophene 2-nitrobenzo[b]furan

63.3.2 Cycloadditions
Driven by the formation of a stable benzenoid  electron sextet in the adduct, o-quinoid benzo[c]thiophene undergoes
[4+2]-cycloaddition (DIELS-ALDER reaction) with typical dienophiles such as maleic anhydride.
O O
S
S + O S O

O O
benzo[c]thiophene maleic acid 7-thiabenzobicyclo[2.2.1]- O
anhydride heptane-2,3-dicarboxylic acid anhydride O
O
63.3 Typical Reactions 185

[2+1]-Cycloaddition of dichlorocarbene, generated from chloroform with potassium hydroxide, to 2,3-dimethylindole


results in a tricyclic dichlorocyclopropane. Subsequent formation of 3-dichloromethyl-2,3-dimethylindole competes with
ring-opening ring expansion, producing 3-chloro-2,4-dimethylquinoline:

CH3 H3C Cl CH3


CHCl3 , KOH
+ [ICCl2] Cl Cl
CH3 CH3
N N  HCl N CH3
H
H 3-chloro-2,4-dimethylquinoline
2,3-dimethylindole

H3C H3C
CCl2 H CHCl2

CH3 CH3
N N
3-dichloromethyl-2,3-dimethylindole

63.3.3 Reactions of 2- and 3-Hydroxy Derivatives


Paralleling oxo-enol tautomerism, 2- and 3-hydroxybenzo[b]pyrroles, -furans, and -thiophenes are able to equilibrate with
their oxo tautomers:
OH O

OH O
X X X X
2-hydroxy tautomer (enol) 2-oxo tautomer 3-hydroxy tautomer (enol) 3-oxo tautomer

3-Hydroxyindole, better known as indoxyl, exists as the oxo tautomer. Its methylene group is readily oxidized. This is the
key step to produce the blue dye indigo. In the dyeing process, indigo is reduced with sodium dithionite to water-soluble
leucoindigo ("vatting"). After soaking in this solution, the fabric is hung in the air in order to re-oxidize the dye on the fiber.
Forming hydrogen bridges with the OH groups of cotton (cellulose, Chapter 72.2), the dye sticks on the fabric.
vatting (reduction)
O O H Na2S2O4 / NaOH O H
N + 2 e0 N
2
N 4 [H ] ,  4 e0 N  2 e0 N
H H O O2 H O
re-forming the dye on the fiber
indoxyl indigo (oxidation) leucoindigo dianion

Phenol-like reactions of hydroxyindoles are also known, exemplified by the conversion of indoxyl into the methyl ether by
dimethyl sulfate.
O OH OCH3
+ (CH3O) 2 SO2 (NaOH)
 H3COSO3H
N N N
H H H
indoxyl tautomers 3-methoxyindole

________________________________________________________________________________________________
Chapter 63 permits answers to the following:
(63.1) Why are benzo[c]furan and its analogues not aromatic? Write the equation for their reaction with maleic anhydride.
(63.2) Which reactants are required to synthesize 5-methoxyindole? Write the mechanism of the reaction.
(63.3) What products are obtained by nitration of (a) benzo[b]furan and (b) benzo[b]thiophene? Explain your answer.
(63.4) Formulate reactions to prepare (a) indole-3-carboxylic acid and (b) 3-(N,N-dimethylaminomethyl)indole from indole.
(63.5) Propose a reaction to convert a substituted indole into a substituted quinoline.
(63.6) Formulate (a) the tautomerism and oxidation of 3-hydroxyindole and (b) the dyeing of fabrics with indigo.
186 64 Benzo-Fused Six-Membered Heteroaromatics

64 Benzo-Fused Six-Membered Heteroaromatics


64.1 Survey and Nomenclature
Common names related to the natural origin are used for many benzo-fused six-membered aromatic heterocycles (Table
64.1). As an example, Javanese Cinchona bark contains an antimalarial agent which was given the name quinine; the
heterocycle obtained from chemical degradation of this alkaloid was named quinoline (Table 64.1). The 3-
phenylbenzopyrylium ion (flavylium ion, Table 64.1) is the core structure of the anthocyanidins which give color to many
blossoms (cornflower, pansy) and fruits (cherry, blueberry).
Unambiguous nomenclature indicates the fusing bond by italic lower case letters in square brackets, where the a bond
originates from the highest ranking heteroatom (O > S > N). The largest possible heterocycle is selected as parent struc-
ture, exemplified by benzo[g]isoquinoline which should not be called naphtho[c]pyridine, and by benzo[g]quinazoline
which should not be called naphtho[d]pyrimidine (Table 64.1).

Table 64.1. Selected benzo-fused six-membered aromatic heterocycles


8 1 1 1 9 10 1 9 10 1
8a N O S 9a 10a 9a 10a
2 N2 2 a N
g b
3 3 e c a
4a C6H5 5a 4a 5a
N 4a 4
5 4 6 5 4 6 5
quinoline isoquinoline benzopyrylium ion thiobenzopyrylium benzo[g]isoquinoline acridine
(benzo[b]pyridine (benzo[c]pyridine) (3-phenylbenzopyrylium ion ion (benzo[b]quinoline,
= flavylium ion) dibenzo[b,e]pyridine)
1 9 10 1 9 10
1
8a N N N 9a 10a N 9a N 10a
N2 N a
g b
e c
4a
3 N N 5a 4a
N 5a N 4a
4 N 6 4 6 4
5 5
cinnoline phthalazine quinazoline quinoxaline benzo[g]quinazoline phenazine
(benzo[b]quinoxaline,
(dibenzo[b,e]pyrazine)

64.2 Typical Syntheses


64.2.1 Quinolines
Quinolines are prepared by MICHAEL addition of aniline to the double bond of an ,-unsaturated aldehyde or ketone,
followed by electrophilic cyclodehydration catalyzed by a LEWIS acid and, finally, oxidation of the intermediate 1,2-dihydro
derivative to the target quinoline (SKRAUP synthesis). Unsubstituted quinoline is synthesized from aniline and glycerol
which is oxidized to acrolein (propenal) in the reaction mixture.
R3 R3 HO R3 R3
O H
R2 R2
O H ZnCl2 / FeCl 3 R2
+
R2  H2O
NH 2 R1 N N R1 N R1
H R1 H H H
R1 , R2 , R3 = H , alkyl , aryl
 2 [H ] ,  2 e0 FeCl3 or C6H5NO2
R3
R2
substituted quinoline
N R1

Cyclodehydration of 2-aminophenones or 2-aminobenzaldehydes and -methylene carbonyl compounds to yield quino-


lines involves imine formation and KNOEVENAGEL alkenylation (FRIEDLÄNDER synthesis).
R1 R1
R2 R2
O H 2C [H ]
+
2 H 2O
NH2 O R3 N R3
2-aminoacetophenone butanone R1 = R2 = R3 = CH3 2,3,4-trimethylquinoline

Efficiently Studying Organic Chemistry: Exam training for chemists, biochemists, pharmacists, life and health scientists,
Third Edition. Eberhard Breitmaier. © 2022 WILEY-VCH GmbH. Published 2022 by WILEY-VCH GmbH.
64.3 Typical Reactions 187

64.2.2 Isoquinolines
In the presence of a LEWIS acid, N-acyl-2-phenylethylamines are cyclodehydrated to give 3,4-dihydroisoquinolines which
are oxidized or dehydrogenated to provide isoquinolines (BISCHLER-NAPIERALSKI synthesis).
O
KMnO4 (oxidation)
+ R C or catalytic (Pd)
Cl PCl5 dehydrogenation
NH
NH2  HCl  H2O N  2 [H ] ,  2 e0 N
O
R R R
2-phenylethylamine N-acyl-2-phenylethylamine 3,4-dihydroisoquinoline 1-alkylisoquinoline

Intramolecular electrophilic cyclization of the imine obtained from 2-phenylethylamine and an aldehyde follows the same
strategy of ring formation (PICTET-SPENGLER synthesis):
O
+ R C
H [H ]
NH2 H2O N NH  4 [H ] ,  4 e0 N

R R R
2-phenylethylamine N-(2-phenylethyl)imine 1-alkyltetrahydroisoquinoline 1-alkylisoquinoline

The imines arising from benzaldehydes and aminoacetaldehyde diethyl acetal undergo electrophilic cyclization involving
an intermediate carbenium ion and acid-catalyzed elimination of ethanol (POMERANZ-FRITSCH synthesis).
aminoacetaldehyde OC2H 5
diethyl acetal C 2H 5
OC2H 5 OC2H5 OC2H5
H H O OC 2H5  H2O
H2N H
+ [H ] H (H2SO4)
+
O H2O  C2H5OH  [H ] N  C2H5OH N
RO RO N RO N RO RO
H
3-methoxybenzaldehyde 7-methoxyisoquinoline
(R = CH3) (R = CH 3)

64.2.3 Benzopyrylium Salts


Salicylaldehydes cyclodehydrate with acetophenones in acetic anhydride and perchloric acid to provide the flavylium
perchlorates (ROBINSON synthesis), involving KNOEVENAGEL alkenylation and a cyclohemiketal.
H

O H 3C HClO4 + HClO4
+ ClO4
C H 2O C 6H 5 H 2O
OH O C 6H 5 OHO OC 6H 5 O C 6H 5
OH
salicylaldehyde acetophenone cyclohemiketal flavylium perchlorate

64.3 Typical Reactions


64.3.1 Basicity and Reactions at the Imino Nitrogen
In quinazoline, which is more basic than the parent heterocycle, the fused benzene ring clearly compensates electron
withdrawal by the second nitrogen of the pyrimidine ring. The slightly stronger basicity of isoquinoline and the slightly
weaker basicity of quinoline relative to the parent heterocycle reflect the interaction of the -nitrogen with the benzenoid 
electron sextet. Apart from quinazoline, the basicities of benzo-fused six-membered heteroaromatics do not significantly
differ from those of the corresponding monocycles.
N N N N
N N
quinoline isoquinoline cinnoline quinazoline quinoxaline
N
N
pKa = 4.90 (5.25) 5.42 (5.25) 2.37 (2.24) 3.43 (1.30) 0.56 (0.65)
(pKa values of parent heterocycles in brackets)
188 64 Benzo-Fused Six-Membered Heteroaromatics

Iminium salts and N-oxides can be formed as described for the monocycles (Chapter 62.4.1).

64.3.2 Catalytic Hydrogenation and Oxidative Ring Opening


The benzenoid rings of quinoline and isoquinoline resist catalytic hydrogenation so that 1,2,3,4-tetrahydro derivatives are
obtained:
Pd Pd
+ 2 H2 + 2 H2
N NH
N N
H
1,2,3,4-tetrahydroquinoline 1,2,3,4-tetrahydroisoquinoline

Oxidation reagents attack the benzenoid ring which provides a higher electron density than the electron-deficient pyridine
ring. Thus, oxidation of quinoline and isoquinoline yields the pyridinedicarboxylic acids:
CO2H
CO2H CO2H
KMnO4 KMnO4

N N CO2H N N
quinoline pyridine-2,3-dicarboxylic acid isoquinoline pyridine-3,4-dicarboxylic acid

64.3.3 Nucleophilic Additions


Electron deficiency of the six-membered heterocycle facilitates nucleophilic additions. Hydride (LiAlH4) adds to quinoline,
yielding the 1,2-dihydro derivative, while 1-butyl-1,2-dihydroisoquinoline arises from nucleophilic addition of butyllithium to
isoquinoline.

+ :H + H2O + Li + H 2O

N N H  OH N H N N  Li OH N
H H H Li H
H 1-butyl- H
1,2-dihydroquinoline 1,2-dihydroisoquinoline

Nucleophilic ethoxide adds to flavylium salts, yielding 4-ethoxy-4H-benzopyrans. Quaternization of the pyridine N-atom in
quinoline additionally activates for nucleophilic additions:
flavylium ion (salt) H OC 2H 5 N-methylquinolinium ion (salt) H CN
+ OC2H5 + CN

O C 6H 5 O C 6H 5 N N
4-ethoxy-2-phenyl-4H-benzopyran CH 3 CH 3
4-cyano-1-methyl-1,4-dihydroquinoline

64.3.4 Nucleophilic Substitutions


Nucleophilic substitutions of benzopyridines take place as described for pyridine (Chapter 62.4.2). Typical examples
include the CHICHIBABIN amination of quinoline and isoquinoline by sodium amide in liquid ammonia to yield 2-amino-
quinoline and 1-aminoisoquinoline, respectively.

+ Na NH2 + Na NH2
NaH N NaH N
N N NH2
2-aminoquinoline 1-aminoisoquinoline NH2

As nitrogen analogues of acid halides, 2-haloquinolines readily undergo nucleophilic substitution with alkoxides to give
the corresponding ethers and with water to give 2-quinolone which is the stable lactam of the lactim tautomer 2-
hydroxyquinoline.
+ NaOC2H5 + H2O tautomerism
 NaCl  HCl
N OC2H5 N Cl N OH N O
H
2-ethoxyquinoline 2-chloroquinoline 2-hydroxyquinoline (unstable) 2-quinolone
64.3 Typical Reactions 189

64.3.5 Electrophilic Substitutions


Electrophilic substitutions of benzo-fused pyridines and diazines predominantly occur at the more nucleophilic benzenoid
ring. Quinoline, isoquinoline and benzodiazines are nitrated and sulfonated at the benzenoid 5- and 8-positions. 3-
Nitroquinoline is obtained in low yield by nitrating quinoline in hot acetic anhydride.
NO2
reflux
NO2
CH 3 CO2 NO2 HNO3 / H 2SO4
and
N N N N
3-nitroquinoline 5-nitroquinoline NO2
SO3 / H 2 SO4 8-nitroquinoline

quinoline-
8-sulfonic acid
N
SO3H

N-Oxidation activates for electrophilic substitution at the 4-position of the pyridine ring (Chapter 62.4.3). Elevated temper-
atures are required. Isoquinoline N-oxides are nitrated and sulfonated only at the benzenoid ring.
NO2
NO2
 [NO2 ]  [NO2 ]  [NO2 ]
(HNO3 /H2 SO4 ) (HNO3 /H2 SO4 ) (HNO3/H2 SO4)

 [H ]  [H ]  [H ]
N N N N N
< 10°C > 60 °C > 60 °C
NO2 O O O O O
8-nitroquinoline 4-nitroquinoline 5-nitroisoquinoline
N-oxide N-oxide N-oxide

64.3.6 CH Acidity of Methyl Groups


As described for pyridine (Chapter 62.4.4), methyl and methylene groups  (and ) to nitrogen in azines are CH acidic,
comparable to methyl and methylene groups  to carbonyl. Therefore, they can be deprotonated by bases, leaving
nucleophilic carbanions which are able to undergo KNOEVENAGEL alkenylations, such as that of 1-methylisoquinoline with
benzaldehyde, and CLAISEN ester condensations, such as that of methylquinoline with diethyl oxalate:

N H N
ZnCl2
CH 3 + O
H 2O

1-methylisoquinoline benzaldehyde (E)-1-styrylisoquinoline (major isomer)

O NaOC 2 H 5 O
C OC 2H 5  C 2H 5 OH
N CH 3 + H 5C 2O C N CO2C 2H 5
2-methylquinoline O diethyl oxalate 2-quinolylpyruvic acid ethyl ester

________________________________________________________________________________________________
Chapter 64 permits answers to the following:
(64.1) Draw the structural formulas of all benzodiazines with the molecular formula C8H6N2 and name them.
(64.2) What compound arises from reacting aniline and butenone in nitrobenzene with ZnCl2? Write the mechanism.
(64.3) Write equations to suggest two syntheses of the alkaloid parent compound 1-benzylisoquinoline (Chapter 70.1).
(64.4) Recalling Chapter 44, two products are obtained by the catalytic hydrogenation of 1-benzylisoquinoline? Draw them.
(64.5) Isoquinoline reacts with (a) NaNH2 in liquid NH3 and (b) n-butyllithium under nitrogen. What are the products?
(64.6) 2-Chloroquinoline reacts with (a) water and (b) sodium ethoxide. Formulate the reactions and name the products.
(64.7) What products arise from the nitration of (a) quinoline and (b) isoquinoline?
(64.8) Write an equation to propose a synthesis of 2-quinolylacetone.
190 65 Fused Aromatic Heterocycles

65 Fused Aromatic Heterocycles


Two and more heteroaromatic rings are connected by joint bonds in fused aromatic heterocycles. Common names exist
for some representatives (Tables 65.1, 65.2, 65.3). Otherwise, systematic names are derived from a parent heterocycle
with the larger ring and the larger number of heteroatoms, following the rules outlined in Chapter 63.1, assigned the
lowest possible lettering and numbering in the same direction. 1H-Pyrazolo[4,3-d]pyrimidine and pyrido[3,4-d]pyrimidine
are examples.
parent heterocycle fused heterocycle parent heterocycle fused heterocycle
the larger ring fusing heterocycle with more heteroatoms fusing heterocycle
H
H
N 7a N
N 3 N6 1 N 4 N
a d 1N N a d
4 4 3 N1 N
b N c N 3a b N c N
pyrimidine pyrazole 1H-pyrazolo[4,3-d]pyrimidine pyrimidine pyridine pyrido[3,4-d]pyrimidine

Some fused aromatic heterocycles occur as core structures in natural products such as alkaloids and nucleobases, and
in metabolites, as described in the following.

65.1 Heterobicycles with Nitrogen as Bridgehead


Dehydropyrrolizinium- and dehydroquinolizinium ion are heterobicycles with nitrogen as bridgehead atom and cyclically
conjugated double bonds (Table 65.1). Their partially and fully hydrogenated derivatives include the so-called "izines"
such as indolizine and "izidines" such as quinolizidine (Table 65.1). Pyrrolizidine, indolizidine, and quinolizidine are the
heterocyclic core structures of some alkaloids (Chapter 70).

Table 65.1. Heterobicycles with nitrogen as bridgehead


7 1 8 9
7a 8a 9a
8 2
6 9a
4 2 2
N 3 N 6 N 5N N
5 3 4
6 4
3H-pyrrolizine dehydropyrrolizinium ion indolizine dehydroquinolizinium ion 9aH-quinolizine

N N N
pyrrolizidine indolizidine quinolizidine

A synthesis of 2-alkylindolizines uses the nucleophilicity of nitrogen and the CH acidity of an  methyl group at the pyri-
dine ring: Nucleophilic substitution of bromine in an -bromomethyl ketone yields the picolinium bromide which under-
goes intramolecular KNOEVENAGEL alkenylation to yield the 3H-indolizinium bromide. Dehydrobromination with sodium
hydrogencarbonate produces the target 2-alkylindolizine.
CH 3 O CH 3
NaHCO3
+ C O R R
NI R N H 2O N HBr N
Br Br R Br
-picoline -bromomethyl ketone picolinium bromide 2-alkyl-3H-indolizinium bromide 2-alkylindolizine

65.2 Purines
65.2.1 Survey
Imidazo[4,5-d]pyrimidines, commonly known as purines (from Lat. purum urinae = pure compound from urine), occur as
nucleobases in nucleic acids (DNA and RNA, Chapter 73), as purine stimulants, such as theophylline, caffeine, and
theobromine, and as metabolites, such as uric acid and xanthine (Table 65.2).

Efficiently Studying Organic Chemistry: Exam training for chemists, biochemists, pharmacists, life and health scientists,
Third Edition. Eberhard Breitmaier. © 2022 WILEY-VCH GmbH. Published 2022 by WILEY-VCH GmbH.
65.2 Purines 191

Purine with its traditional ring numbering is usually written as the 9H tautomer, but spectroscopy has also revealed the
existence of the 7H tautomer:
H 6 7 H
N N1 5 N N
5 1N N 7
a d 8
b c 4 9
N N3 N 4 N N N
9
3 H
imidazo[4,5-d]pyrimidine = purine 9H-purine 7H-purine

Table 65.2. Purine derivatives of natural origin


NH2 O O

nucleobases N N HN N HN N
(in nucleic acids)
N N H2N N N N N
H H H
adenine guanine hypoxanthine
H 2N O
O O
H H H
O N
HN N HN N HN
metabolites O NH
N N HN NN
O N O N H
H H H H OH
xanthine uric acid OH
(animal organs) (urinary bladder stones, saxitoxin (perhydropurine derivative,
gout deposits, neurotoxin produced by marine bacteria,
excrements of birds and reptiles) enriched in mussels and oysters)

O O CH 3 O CH3
H 3C N N H3C N
N HN N
purine stimulants
O N N O N N O N N
H
CH 3 CH3 CH 3
theophylline theobromine caffeine
(tea) (tea , cocoa , cola nut) (tea , coffee , maté)

65.2.2 Syntheses of Purines


Analogously to the formation of benzimidazoles (Chapter 63.2.3), the last step of purine synthesis involves cyclodehydra-
tion of 4,5-diaminopyrimidines with formic acid or a derivative as a C1 electrophile:
R R R H R
H
NH 2 O N H N N
N N C N H N
+ C H
 H2O O N OH  H2O
N
N NH 2 HO N NH 2 N N
H
H

Cyclization of guanidine and ethyl cyanoacetate to give 2,4-diamino-6-oxo-1,6-dihydropyrimidine is the first step in
TRAUBE’s synthesis of guanine. Electrophilic nitrosation and reduction of the nitroso function gives 2,4,5-triamino-6-oxo-
1,6-dihydropyrimidine. Cyclodehydration with formic acid in acetic anhydride yields guanine.
ethyl cyanoacetate O
O O
C
H5C2O CH2 NaOC2H5 NO
HN NaNO2 HN
+
NH C  C2H5OH [NO ] ,  H ]
guanidine H2N N NH2 H2N N NH2
C N
H2N NH2 2,4-diamino-6-oxo-1,6-dihydropyrimidine
[H ] , 4 e0
NH4SH or Na2S2O4
 H2O

O O
+ HCO2H
N (CH3CO) 2O NH2
HN HN

N  2 H2 O
H2N N H2N N NH2
H
guanine 2,4,5-triamino-6-oxo-1,6-dihydropyrimidine
192 65 Fused Aromatic Heterocycles

Cyclization of formamidine and (phenylazo)malodinitrile, prepared by the coupling of nucleophilic malodinitrile with elec-
trophilic benzenediazonium chloride, initiates TODD’s synthesis of adenine by forming 4,6-diamino-5-(phenylazo)-
pyrimidine. Catalytic hydrogenation produces 4,5,6-triaminopyrimidine which undergoes cyclization with dithioformate as
C1 electrophile to yield adenine.
(phenylazo)malodinitrile
H NH2
N
C N C6H5 N C 6H 5
C N C4 H9 O N N
NH + 4,6-diamino-5-(phenylazo)pyrimidine
C
C N NH2
H NH 2 N
formamidine + H2 / Ni  C6 H5 NH 2

S
NH2 NH 2 NH 2
+H C H
NH2 N H N
N S Na N C N
NaSH S  H 2S
N NH2 N NH 2 N N
H
4,5,6-triaminopyrimidine adenine

65.2.3 Oxidative Cleavage of Purines (Uric Acid)


Excrements of birds and reptiles (guano) are an industrial source of uric acid which is poorly soluble in water. Oxidation of
uric acid with nitric acid cleaves the imidazole ring, leaving water-soluble 2,4,5,6-tetraoxohexahydropyrimidine. It is better
known as alloxan and equilibrates with its lactam and lactim tautomers.
uric acid alloxan tautomers
O O OH OH OH
H
N O O O O
HN + 2 HNO3 HN N N N
O
 N2O
O N N O N O O N O HO N O O N OH
H  2 H2O
H  CO2 H H
bislactam monolactim bislactim ( o-quinoid) bislactim ( p-quinoid)

65.3 Pteridines
65.3.1 Survey
Derivatives of pteridine (pyrazino[2,3-d]pyrimidine) were first found in the wings of butterflies. Pteridines are widely
spread in insects and as partial structures of essential active substances in mammals (Table 65.3).

5
O
4a N N
3N HN
pteridine pterine
N 8a N H 2N N N
1 8

65.3.2 Syntheses of Pteridines


The cyclodehydration of o-phenylenediamines with 1,2-dicarbonyl compounds (e.g. ethanedial or butanedione) to provide
quinoxalines can be transferred to a synthesis of pteridines by using 2,4,5-triaminopyrimidine as the binucleophilic reac-
tant:
NH 2 O R1 N R1
+
2 H2 O
NH 2 O R2 N R2
o-phenylenediamine quinoxaline

NH 2 O R1 N R1
N N
+
2 H2 O
H2N N NH 2 O R2 H 2N N N R2
2,4,5-triaminopyrimidine 2-aminopteridine
65.3 Pteridines 193

Table 65.3. Pteridine derivatives of natural origin


O O
H H
pigments in the wings xanthopterin N O leucopterin N O
HN HN
of butterflies in yellow wings in white wings
H 2N N N H 2N N N O
H
O CO2H
active substances of 6-methylpterine C
the vitamin B group O N
folic acid N H
(spinach , liver, kidney, HN N CO2H
bacterial growth factor) H
H 2N N N
O CO2H
H O C
folinic acid O C N
N-formyltetrahydrofolic acid , N H
citrovorum factor HN N CO2H
(growth factor , coenzyme F, H
"F" for formylation, H 2N N N p-amino-
C1 transferring coenzyme) H benzoic acid L-glutamic acid
5-formyl-6-methyl-
5,6,7,8-tetrahydropterine

R CH 2OH
vitamin B 2 , H 3C N N O HO C H
riboflavin
R = HO C H
(nutritional factor found in NH
leafy vegetables, H 3C N HO C H
malted barley, O CH 2
milk, eggs, 7,8-dimethyl-10-ribitylisoalloxazine
liver, kidney, heart) (7,8-dimethyl-2,4-dioxo-10-ribityl- (ribityl-)
2,3,4,10-tetrahydrobenzo[ g]pteridine)

Following the same strategy, the pteridine ring of 10-alkylisoalloxazines can be formed by reacting appropriately substi-
tuted o-phenylenediamines with alloxan prepared by oxidation of uric acid (Chapter 65.2.3).
CH3 CH3
H3C NH HO N O H 3C N N O
+
NH  2 H2 O NH
H3C NH2 O H 3C N
O O
N,4,5-trimethyl-o-phenylenediamine alloxan 7,8,10-trimethylisoalloxazine
(monolactim tautomer)

7,8,10-Trimethylisoalloxazin (lumiflavine, orange crystals) is alternatively obtained by UV irradiation of riboflavin (Table


65.3). This photoreaction contributed to the elucidation of the riboflavin structure.
________________________________________________________________________________________________
Chapter 65 permits answers to the following:
(65.1) Draw the structural formulas of (a) 3H-pyrrolo[1,2-a]pyrrole, (b) pyrrolo[1,2-a]pyridine, and (c) pyrido[2,3-d]pyrimidine.
(65.2) Which of the heterobicycles in Table 65.1 are aromatic?
(65.3) Design a synthesis of 2-methylindolizine.
(65.4) Draw structural formulas of two purine isomers that contain a fused pyrazole instead of an imidazole ring,
and name them.
(65.5) Which tautomers exist for (a) purine and (b) 2,4,5,6-tetraoxohexahydropyrimidine (alloxan)?
(65.6) Which (a) purine nucleobases and (b) purine stimulants do you know?
(65.7) Formulate equations to describe TRAUBE’s guanine synthesis.
(65.8) Formulate an equation to suggest the preparation of pterine from an intermediate in TRAUBE’s guanine synthesis.
(65.9) Formulate equations to describe preparations of (a) adenine and (b) 6,7-dimethyl-2-aminopteridine.
(65.10) Formulate an equation to synthesize the heterocycle of riboflavin (Table 65.3).
194 66 Absorption of Light, Color, Dyes

66 Absorption of Light, Color, Dyes


66.1 Absorption of Light, Color
A colored compound absorbs in the visible range of radiation (wavelength 380-770 nm, 1 nm = 109 m, Fig. 66.1). The
human eye perceives the non-absorbed, scattered and reflected part of the spectrum as the complementary color (Table
66.1).

Table 66.1. Absorbed and scattered light of colored compounds (complementary colors)
absorbed light scattered and reflected light
wavelength (range) [nm] color perceived (complementary) color
380 - 430 violet yellowish green
430 - 480 blue yellow
480 - 490 greenish blue orange
490 - 500 bluish green red
500 - 560 green purple
560 - 580 yellowish green violet
580 - 595 yellow blue
595 - 605 orange greenish blue
605 - 750 red bluish green
750 - 770 purple green

Ultraviolet and visible light absorption spectra (UV-vis spectra, electronic spectra) record the absorptivity  (ordinate)
versus the wavelength  (abscissa, Fig. 66.1). Every compound has an individual spectrum because of its specific ab-
sorptivity. The maximum absorption of -carotene is observed at max = 450 nm; this is the greenish-blue range of the
visible spectrum (Fig. 66.1); thus, -carotene is seen as having the complementary color orange (Table 66.1). Color
perception of a compound does not require the maximum absorption to be in the visible range; a maximum in the UV with
significant absorption in the visible range is sufficient.

140000

120000
-carotene
100000

80000

60000

40000

20000

200 300 400 500 600 700 800 nm


wavelength 

UV IR
Fig. 66.1. UV-vis spectrum (electronic spectrum) of -carotene

Electronic excitation of chromophores induces light absorption. Chromophores (Greek chroma = color, phero = to carry)
are groups in a molecule which are able to undergo specific kinds of electronic transitions giving rise to absorption in the
UV or visible range. In -carotene, as an example, eleven conjugated CC double bonds are the chromophores which
cause color due to absorption in the visible range (Fig. 66.1).
_ _ _
chromophores C C C C C O
_ C S
_ _
C N _
N N

* transitions n* and * transitions

Efficiently Studying Organic Chemistry: Exam training for chemists, biochemists, pharmacists, life and health scientists,
Third Edition. Eberhard Breitmaier. © 2022 WILEY-VCH GmbH. Published 2022 by WILEY-VCH GmbH.
66.2 Dyes and Pigments 195

In organic molecules, absorption in the UV and visible range predominantly arises from two kinds of electronic transitions:
n* transitions excite non-bonding (n) electrons of carbonyl, imino or azo groups to excited  states (*, antibonding 
orbitals); * transitions promote  electrons of CC and CX multiple bonds to * energy levels. Visible light absorption of
-carotene (Fig. 66.1) arises from a series of * transitions.
Electronic transitions in molecules are coupled with vibrational and rotational excitations; the fine structure due to these
transitions cannot be well-resolved, so that broad absorption bands are usually observed in the electronic spectra (Fig.
66.1).
One double bond or another chromophore in the molecule does not ensure that the compound is colored since n* and
* transitions require the higher energy of UV excitation (max < 300 nm, Table 66.2). An isolated arrangement of several
double bonds increases the intensity of absorption (max) with the number of these chromophores while max remains the
same, exemplified by ethene and 1,5-hexadiene in Table 66.2.
Conjugation of x chromophores, however, increases the absorptivity (max) more than x times as much as that of a com-
parable compound with one chromophore (hyperchromic effect). Moreover, conjugation shifts the maximum absorption
(max) to longer wavelengths (bathochromic or red shift), exemplified by the series ethene, 1,3-butadiene, 1,3,5-
hexatriene, 2,4,6,8-decatetraene, and -carotene in Table 66.2.

Table 66.2. Electronic spectroscopic data of polyenes (in ethanol solution): dependence on the number of conjugated CC double bonds
compound n max [nm] max color
ethene H2C CH2 1 185 10 000 colorless
1,5-hexadiene H2C CH CH2 CH2 CH CH2 or 2 185 20 000 colorless

1,3-butadiene 2 217 21 000 colorless


1,3,5-hexatriene (trans-) 3 258 35 000 colorless
2,4,6,8-decatetraene (all trans-) 4 310 76 000 colorless
2,4,6,8,10-dodecapentaene (all trans-) 5 342 122 000 colorless

 -carotene (all trans-) 11 450 140 000 orange

To conclude, the excitation energy of polyenes decreases with an increasing number of conjugated double bonds as
chromophores. Clearly, conjugation stabilizes the excited state more strongly than the ground state, and this stabilization
increases with the number of conjugated double bonds.
All influences disturbing coplanarity of conjugated chromophores (e.g. bulky groups, steric repulsion) will act oppositely,
causing the absorption maximum to move to shorter wavelengths (hypsochromic or blue shift). For example, photoiso-
merization converts trans-azobenzene (max = 330 nm) into the cis-isomer. Steric repulsion of the hydrogen atoms at the
o-positions of the phenyl rings in the cis-isomer prevents coplanarity required for undisturbed conjugation of the chromo-
phores, and the smaller max (255 nm) reflects this.

trans-azobenzene cis-azobenzene
max = 330 nm N h H max = 255 nm
conjugation N H no coplanarity
N N disturbed conjugation

66.2 Dyes and Pigments


A colored compound such as the hydrocarbon -carotene is not necessarily a dye. A dye (colorant) is a colored compound
such as indigo (Chapter 63.3.3) which, due to the presence of functional groups, interacts or reacts with a carrier (sub-
strate). Non-bonding interactions such as dipole-dipole or interionic interactions, hydrogen bridges or, even better, cova-
lent bonding of the fiber with reactive dyes providing reactive functional groups, will cause the dye to stick on the sub-
strate. The dye tranfers its color to the carrier, such as a fiber (dyeing).
Pigments such as some phthalocyanine metal chelates (Chapter 67.1) are insoluble in the media in which they are ap-
plied. They do not interact or react with the carrier. Their particles are attached to the carrier (paper, wooden and plastic
materials, steel sheet) by an additive such as in the melt of a polymer colored by the pigment particles, or in a paint,
forming a colored layer printed, painted or sprayed on the surface.
196 66 Absorption of Light, Color, Dyes

66.3 Basic Types of Dyes


66.3.1 Structural Properties of Dyes: Azo Dyes
Azobenzene, the parent compound of the azo dyes, is orange-red due to a * transition giving rise to a very strong
absorption and an n* transition resulting in a much weaker absorption (Table 66.3). However, azobenzene is not a dye
since no interactive functional group is present. Introduction of a donor group such as N,N-dimethylamino into one of the
p-positions converts azobenzene into an azo dye known as butter yellow. The dimethylamino group of this toxic and
carcinogenic 4-(N,N-dimethylamino)azobenzene undergoes salt formation with acidic groups of a fiber, thus dyeing the
fiber yellow. Additionally, the dimethylamino group shifts the * absorption band to longer wavelengths, which then
overlaps with the weaker band due to the n* transition (Table 66.3). Introduction of an acceptor group such as nitro at
the other p-position induces an additional bathochromic shift (Table 66.3).

Table 66.3. Light absorption data of azobenzene derivatives


compound solvent max max transition
[nm] [mol1 cm1]

N 330 17000 *


azobenzene n-hexane
(trans-isomer) N 450 465 n*

4-(N,N-dimethylamino)- N N(CH3)2
ethanol 408 27500 *
azobenzene N

N N(CH3)2
4-(N,N-dimethylamino)- ethanol 478 33100 *
4'-nitroazobenzene O2N N

The azobenzene derivatives and their spectroscopic data in Table 66.3 indicate the typical structure of a dye: The azo
group extended by both conjugated phenyl rings is the chromophore. A basic donor group such as dimethylamino en-
ables azobenzene to interact with a fiber, qualifying the substituted azobenzene as a dye and is therefore called the
auxochrome. The donor additionally contributes to the resonance stabilization of the chromophore, reflected in the batho-
chromic shift and increased intensity of the absorption band due to the * transition. An acceptor substituent such as the
nitro group, also called the antiauxochrome, induces an additional red shift and hyperchromic effect. To conclude, a
typical dye is an electronic push-pull system with the structural elements donor-chromophore-acceptor, or ()-M substitu-
ent-chromophore-()-M substituent, exemplified by 4-(N,N-dimethylamino)-4´-nitroazobenzene:

O O
N N
O O
N N
N N

N(CH 3)2 N(CH 3)2

66.3.2 Polymethine Dyes


Polymethine dyes are conjugated polyenes (chromophore) with an odd number of methine groups linking a donor D of
electron pairs (push) with an acceptor A (pull), and thus containing the structural elements of a dye. As shown by the
resonance formulas, donor D and acceptor A interchange their roles. Usually, the double bonds and the substituents D
and A are parts of a heterocyclic ring such as quinoline, for example in blue-violet pinacyanol:

A nD A nD N N N N I

polymethine dye : resonance formulas C 2H 5 C 2H 5 pinacyanol C 2H 5 C 2H 5


66.3 Basic Types of Dyes 197

66.3.3 Triarylmethine Dyes


Triphenylmethane is the parent skeleton of triarylmethine dyes which can be considered as polymethine dyes incor-
porating donor-substituted phenyl rings in their chromophore system. Crystal violet, used in inks and as a dye, biological
stain, and topical anti-infective, is a typical representative. It is prepared by electrophilic acylation of N,N-dimethylaniline in
three steps to provide MICHLER’s hydrol which finally dehydrates with hydrogen chloride to give the salt of the dye. Solu-
tions in water and ethanol are deep violet. All three dimethylamino nitrogen atoms share the positive charge due to reso-
nance, interchanging their roles as donors and acceptors.
H

(H3C) 2N N(CH3) 2 N(CH3 )2 N(CH3 )2 N(CH3 ) 2 IN(CH3 )2

+ O=CCl2  HCl + +
Cl
H H  HCl
C C C Cl C
O  HCl O H2O
OH
(H3C) 2N (H3C) 2N (H3C) 2N N(CH3) 2 (H3 C) 2 N N(CH3) 2

p-(N,N-dimethylamino)- MICHLER's ketone MICHLER's hydrol crystal (gentian) violet


benzoyl chloride

Triphenylmethine dyes of the phthalein type, such as the indicator phenolphthalein, are obtained by electrophilic substitu-
tion of phenols with phthalic anhydride. The colorless lactone undergoes ring opening in alkaline solution (pH > 8) to give
the deep purple quinoid triphenylmethine dye, where carbonyl and phenoxide oxygen interchange their roles as donor
and acceptor in the chromophore system.

HO OH O O O O
HO OH
 2 [H ]
H + H C C
C
O  H2 O O  2 [H ] CO2 CO2

O O
phenolphthalein
O colorless lactone in acidic and neutral solutions purple quinoid triphenylmethine in alkaline solutions

66.3.4 Carbonyl Dyes


Carbonyl dyes are dicarbonyl compounds (acceptors A) linked by one conjugated double bond or benzenoid rings, both
substituted by donor groups D. Indigo (Chapter 63.3.3) and donor-substituted 9,10-anthraquinones (Chapter 53.3.4) are
typical examples. Colors vary with changing substituents and heteroatoms, and many dyes of both classes are produced
industrially for the vat dyeing of fibers.
O O DI O
D O
D DI
C C
C C n frequently n = 1 D
O D O O O
indigo dye anthraquinone dyes
D = NH , O , S , Se D = NH2 , NHR, NR 2, OH, OR

________________________________________________________________________________________________
Chapter 66 permits answers to the following:
(66.1) Which structural features cause the color of a compound?
(66.2) List the chromophores in organic compounds giving rise to UV and visible light absorption and classify them with
respect to the electronic transitions they may undergo upon excitation.
(66.3) Explain why trans-stilbene (1,2-diphenylethene) has a longer wavelength UV absorption than the cis-isomer.
(66.4) What are the structural elements of a dye? Draw the structural formulas of some azobenzene derivatives to explain.
(66.5) Draw resonance formulas of (a) pinacyanol, (b) indigo, (c) crystal violet (= gentian violet), and (d) phenolphthalein
at pH > 8. Classify these dyes.
(66.6) How do dyes differ from pigments?
198 67 Porphyrinoids

67 Porphyrinoids
67.1 Porphyrins and Phthalocyanines as Polyaza[18]annulenes
Porphyrinoids comprise porphyrins (from Greek porphyrous = purple) of natural origin and synthetically obtained phthalo-
cyanines including their metal chelates. They represent polyaza[18]annulenes.
Four pyrrole rings linked with methine groups at the , ´-positions make up the parent macrocyclic porphyrin ring system
called porphine. Following the red bonds in the tautomers given, porphyrins (X = CH) and their metal chelates turn out to
be diaza[18]annulenes.
Four imino nitrogen atoms replace the methine fragments in the benzo-fused phthalocyanine. Once more tracking the red
bonds in the formulas, phthalocyanines (X = N) and their metal chelates can be considered as hexaaza[18]annulenes.
To conclude, planar porphyrins and phthalocyanines have 18 electrons and obey HÜCKEL’s (4N + 2) rule with N = 4 as a
criterion for aromaticity (Chapters 23.3, 29.1). Metal-free porphyrins exist as tautomers in which diagonal ring nitrogen
atoms exchange their hydrogen atoms.

R R R R
X X N
R R R R
N N N N N N N N N N
4
H X H X X H X
H H H N Cu N
3-aminoacrolein imine
partial structure N N N N N N N N
R R R R
X X N
porphine R R R R
tautomers of porphyrin (X = CH) and phthalocyanine ( X = N) copper(II) phthalocyanine

The outstandingly intense SORET band ( max 410 nm) in the electronic spectra (Chapter 66.1) of porphyrins and longer-
wavelenth maxima ( max > 500 nm) are attributed to the tetrameric push-pull system of 3-aminoacrolein imine present in
the porphyrin ring.
As tetradentate ligands with four nitrogen donors (N4 ligands), porphyrins and phthalocyanines chelate metal cations with
ionic radii of about 70 pm (1 pm = 10 12 m). Substituted phthalocyanine metal chelates, industrially prepared from phthalic
acid derivatives such as phthalimide and phthalodinitrile in the presence of metal salts, are widely used as colorants,
dyes, and pigments. Substituents at the benzenoid rings ( Cl, SH, C6H5) change the color from blue to green. Red
phthalocyanines contain 1,4-dithiane rings instead of benzene rings.

67.2 Porphyrinoids in Blood and Chloroplasts


67.2.1 Heme
In the protein hemoglobin (structure: Fig. 69.2 c and d) of red blood corpuscles, the ferrous chelate (chelated ion: Fe2 ) of
a substituted porphyrin called heme (from Greek haima = blood) is the non-protein organic function. It is referred to as the
prosthetic group of hemoglobin, shown in Fig. 67.1 c, not only giving the red color to the protein but, most importantly,
also enabling it to carry oxygen in the breathing process. The ferrous ion is coordinated (chelated) to the four pyrrole
nitrogens of the porphyrin ring and to an imidazole nitrogen of the amino acid histidine (Chapter 68.1) in the protein
sequence. This fifth coordination to the so-called proximal histidine ties heme to the protein (Fig. 67.1 c). Oxygen takes
the sixth coordination position of the ferrous ion of heme protected in a hydrophobic pocket of hemoglobin during
transport in the blood flowing from the lungs to the muscles and other tissues where oxygen is needed. Carbon dioxide
produced by "metabolic burning" of carbohydrates in the tissues reacts with N-terminal amino groups of hemoglobin to
give carbamate (Chapter 55.3.2); in this state it is carried away by the protein in the blood back to the lungs where it is set
free and exhaled. Stronger binding carbon monoxide (IC OI ) and the isoelectronic cyanide anion (IC NI) of hydro-
gen cyanide displace oxygen in oxygenated hemoglobin, disrupting oxygen transport, causing tachypnea, cyanosis,
coma, and death.

Efficiently Studying Organic Chemistry: Exam training for chemists, biochemists, pharmacists, life and health scientists,
Third Edition. Eberhard Breitmaier. © 2022 WILEY-VCH GmbH. Published 2022 by WILEY-VCH GmbH.
67.2 Porphyrinoids in Blood and Chloroplasts 199

Hot acetic acid cleaves hemoglobin. Hemin is the ferric chelate (chelated ion: Fe3 ) which crystallizes from the solution
obtained by pouring blood into hot acetic acid containing some sodium chloride (Fig. 67.1 b). Protoporphyrin is the name
of the metal-free ligand (Fig. 67.1 a).

CH3 CH 3

H 3C H 3C
N N N N
H
H Fe
N N N N
H 3C CH 3 H 3C CH 3

Cl

HO2C CO2H HO2C CO2H


protoporphyrin hemin
red bonds : diaza[18]annulene

a b c
Fig. 67.1. Protoporphyrin (a), hemin (b), and tube model of heme in hemoglobin (c, without hydrogen atoms), attached to the helical
parts (green) of the protein by coordination of the ferrous ion to the imidazole nitrogen atom of the proximal histidine (left side); oxygen
takes the sixth coordination position of the ferrous ion in oxygenated hemoglobin, controlled by the distal histidine (right side)

67.2.2 Chlorophyll
Chlorophyll a (methyl) and b (aldehyde) are found in the chloroplasts of plants. They can be extracted from leaves with
methanol or acetone, separated by chromatography on silica gel, and isolated as waxy blue-black microcrystals; the
waxyness arises from the diterpenoid phytyl ester residue (Chapter 76.3.3). Both chlorophylls are magnesium chelates of
a substituted chlorin. The parent chlorin derived from porphine contains one partially hydrogenated pyrrole ring; thus, the
aromatic diaza[18]annulene system is not disrupted.
R = CH=O : chlorophyll b R
R = CH3 : chlorophyll a
H 3C C 2H 5
N N N N
H
H Mg
N N N N
H 3C
CH 3
H
chlorin H
parent skeleton of chlorophyll a and b phytyl residue (diterpene) H O
red bonds : diaza[18]annulene CO2CH 3
C O
O

The light reaction of photosynthesis occurs in the membrane of chloroplasts where chlorophyll-protein complexes collect
photons. These excite other chlorophyll molecules, inducing photolysis of water to oxygen by the nicotinamide coenzyme
NADP which undergoes reduction to 1,4-dihydronicotinamide NADPH + H . The dark reaction of photosynthesis in the
stoma of chloroplasts involves reductive conversion of carbon dioxide into D-glucose via D-glyceraldehyde (Chapter 71.1)
by the reducing coenzyme NADPH + H . Without photosynthetically produced oxygen and D-glucose, animal and human
life on earth would be impossible.
H O H H O 6 CO2 + 12 H + 12 NADPH
C chloroplasts C
NH 2 h NH 2 dark reaction
H 2O + + H + 1/2 O2
N light reaction N C6H12O6 + 6 H2O + 12 NADP
R NADP R NADPH D-glucose

________________________________________________________________________________________________
Chapter 67 permits answers to the following:
(67.1) How do you account for the outstanding stability of porphyrins and phthalocyanines?
(67.2) What is the difference between heme and hemin? How is hemin obtained?
(67.3) What is the difference between porphine and chlorin? Draw formulas.
(67.4) Briefly outline the biological function of (a) heme and (b) chlorophyll.
200 68 Amino Acids

68 Amino Acids
68.1 General Survey
Protein amino acids are the twenty -amino carboxylic acids obtained by hydrolysis of proteins (Table 68.1). Micro-
organisms and plants biosynthesize all protein amino acids themselves. Mammals are only able to produce the so-called
non-essential amino acids themselves and have to rely on their protein food as a source of the ten essential amino acids
(the red ones in Table 68.1).
With the exception of glycine, all protein amino acids are optically active because their  carbon atoms are stereogenic
centers. The absolute configuration is specified traditionally as L related to glyceraldehyde (FISCHER convention) and S in
keeping with the CAHN-INGOLD-PRELOG convention, apart from L cysteine which has an R configuration due to the higher
ranking sulfur atom in the side chain (R = CH2SH) .
Amino acids have extremely high melting points, much higher than those of other substituted carboxylic acids such as
hydroxy or halo acids. This and molecular spectra revealing the presence of a protonated amino function (H3N) and a
carboxylate anion (CO2) provide evidence that amino acids are dipolar ions ("zwitterions"), resulting from acid-base
interaction between the amino and carboxy functions:
CO2H CO2 CO2
S configuration
L-amino acid H2N C H H3N C H H3N C H (for R = alkyl)
R R R
FISCHER projection formula "zwitterion"

Table 68.1. FISCHER projection formulas of protein amino acids (essential amino acids are red), and three- and one-letter abbreviations
CO2 CO2 CO2
H3N C H glycine Gly G H3N C H asparagine Asn N H3N C H cysteine Cys C
(seleno-
H CH2 CH2 Sec
cysteine)
CO2 CONH 2 SH (SeH)
H3N C H alanine Ala A
CO2
CO2
CH3 H3N C H glutamic Glu E
acid H3N C H methionine Met M
CO2 CH2
CH2
H3N C H valine Val V CH2
CH2
CH(CH 3)2 CO2H
S
CO2 CO2 CH3
H3N C H leucine Leu L H3N C H glutamine Gln Q
CH2 CH2 CO2
CH(CH 3)2 CH2 H3N C H phenylalanine Phe F

CO2 CONH 2 CH2


H3N C H isoleucine Ile I CO2
CH CH3 H3N C H lysine Lys K
C2H5 CH2
CH2
CO2
CO2 proline Pro P CH2
N H3N C H tyrosine Tyr Y
H2 H3N CH2
CH2
CO2
CO2
H3N C H serine Ser S
H3N C H arginine Arg R
CH2 OH
CH2 OH
CO2 H2N CH2
H3N C H threonine Thr T C NH CH 2
CH OH H2N
CO2
CH3 CO2
H3N C H tryptophane Trp W
H3N C H histidine His H
CO2 CH2
H3N C H aspartic Asp D CH2
acid
CH2 NH NH
CO2 N

Efficiently Studying Organic Chemistry: Exam training for chemists, biochemists, pharmacists, life and health scientists,
Third Edition. Eberhard Breitmaier. © 2022 WILEY-VCH GmbH. Published 2022 by WILEY-VCH GmbH.
68.2 Preparation 201

The exclusive natural occurrence of L-amino acids, referred to as homochirality, is assumed to be an accident in favor of
the L configuration. Chiral recognition by enzymes in biosynthesis, not only of proteins, requires one configuration.

68.2 Preparation
L-Amino acids are produced industrially by hydrolysis of proteins. One of numerous chemical syntheses of racemic amino
acids involves nucleophilic substitution of alkyl halides by dialkyl (N-acetylamino)malonate as a synthetic reagent to
attach the amino acid residue to an alkyl group R:
hydrolysis
decarboxylation
alkyl halide CO2C 2H 5 CO2C 2H5 CO2
+ C 2H 5ONa + 3 H2 O (H 3O )
R X + H C NH COCH3 R C NH COCH3 R CH NH 3
 NaX  C2H 5OH
CO2C 2H5  C 2H5 OH CO2C 2H5  CH3 CO2 H
diethyl (N-acetylamino)malonate  CO2 DL-amino acid

Almost pure L- or D-amino acids are obtained by HARTWIG-SCHÖLLKOPF’s diastereoselective synthesis, starting from the
bislactam (dioxopiperazine) of L-valine and glycine which is converted into the bislactim ether by trimethyloxonium tetra-
fluoroborate (MEERWEIN’s salt). The carbanion arising from deprotonation of the CH acidic methylene group of the glycine
substitutes an alkyl halide. The bulky isopropyl group from (S)-valine forces the electrophilic alkyl group to attach from the
opposite side, inducing diastereoselectivity in favor of the (S,R)-product. Hydrolysis of the alkylated (S,R)-bislactim ether
liberates the (R)-amino acid.
O OCH 3 OCH 3
1.) + base
NH (H3C) 3 OBF4 N 2.) + RX ,  HX N + 3 H2 O (H3O ) NH3
HN CH2 N CH2 N  CH3 OH
R  L-valine O2C R
O H
OCH 3 OCH 3
dioxopiperazine of L-valine
and glycine (bislactam) (S)-bislactim ether C-alkylated ( S,R)-bislactim ether (R)-amino acid

68.3 Identification
Amino acids undergo almost all reactions described for primary amines and carboxylic acids, such as the VAN SLYKE
reaction with nitrous acid characteristic of primary amino groups (Chapter 39.2). The ninhydrin reaction permits the
analysis of protein hydrolysates. Upon heating with 1,2,3-indanetrione hydrate (ninhydrin), amino acids react to give the
tautomers of a blue imine with specific light absorption maxima in the electronic spectrum (Chapter 66). This is the basis
for the detection of amino acids in amino acid analyzers.
R
O O O
R CH CO2H
OH H
+ H 3N CH CO2 N
OH  2 H2O  CO2 N CH R
O O O
ninhydrin
(indanetrione hydrate) + H 2O  R CH O
O
OH
+ O
O O OH O OH
O H
 2 H 2O NH2
N N
H O
O O O O
imine (tautomers) 2-aminoindanedione

________________________________________________________________________________________________
Chapter 68 permits answers to the following:
(68.1) How do you account for the high melting points of amino acids?
(68.2) What are essential amino acids and which ones do you know?
(68.3) Specify the absolute configuration of (a) L-serine and (b) L-cysteine following the CIP convention.
(68.4) Write equations to suggest a synthesis of racemic phenylalanine from a derivative of diethyl malonate.
(68.5) Write equations to suggest a synthesis of (R)-phenylalanine from benzyl bromide (-bromotoluene).
202 69 Peptides, Proteins

69 Peptides, Proteins
69.1 Amino Acid Sequence
Polyamides of the naturally occurring amino acids (Table 68.1), held together by amide = peptide bonds, are called pep-
tides. The peptide bond is a partial CN double bond, as indicated by the two resonance formulas.
H H

amide resonance formulas N N


C C
O O

All atoms linked by the peptide bond lie in one plane, the amide N H and amide C=O favorably adopting a trans-
configuration, as shown in Fig. 69.1. Conventionally, any peptide or protein chain begins with an N-terminal amino acid
and ends with a C-terminal amino acid (Fig. 69.1). The order of amino acids from the N-terminal to the C-terminal end is
called the sequence or primary structure of the peptide or protein. A peptide is named by indicating its sequence begin-
ning with the N-terminal amino acid and by using the abbreviations of the amino acids as listed in Table 68.1. Thus, a
tetrapeptide fragment of the pancreas peptide hormone insulin, shown in Fig. 69.1, has the sequence Ala-Ser-Glu-Cys or
ASEC.

side chains CO2


CH 2
CH 3 H O CH 2 H
N terminal C terminal
CH N C CH N CO2 (carboxylate group)
(protonated H 3N C CH N C CH
amino group)
O CH 2 H O CH 2
OH SH

peptide bonds
Ala Ser Glu Cys
Fig. 69.1. ASEC partial sequence of insulin

Without sharp limits, peptides are divided into three classes: oligopeptides contain up to 9, polypeptides more than 10,
and proteins more than 100 amino acids. Proteins are biopolymers.

69.2 Biological Function


Peptides and proteins perform many biological functions. Peptide hormones secreted by glands and transported to their
area of activity by blood circulation regulate metabolic processes (Greek hormaino = to drive, to regulate); insulin pro-
duced by pancreatic beta cells, as an example, controls carbohydrate homeostasis and lipid metabolism. Enzymes
catalyze such processes and all biosyntheses including those of proteins. Structure proteins build up animal tissues such
as keratin in hair, nails, and nerves or collagen in skin, connective tissue, and in the organic substance of bones and
teeth, comprising one-third of the total protein content in mammals. The globular transport protein hemoglobin in red
blood cells carries oxygen from the lungs to body tissues and facilitates the return transfer of carbon dioxide in the breath-
ing process (Chapter 67.2.1). Antibody proteins contribute to the protection from infections.

69.3 Structure of Proteins


The secondary structure describes the conformation of a protein, essentially controlled by hydrogen bonding. Intermole-
cular hydrogen bonding between their NH and carbonyl groups can join two or more polypeptide chains, arising in a
sheet-like arrangement which is therefore called the sheet structure. Intramolecular amide-amide hydrogen bonding within
a chain generates a right-handed arrangement like a screw or a spiral staircase of the polypeptide chain called an -helix
with three or four amino acids per turn (Fig. 69.2 b). More or less disordered arrangements are said to be random coil
secondary structures.

Efficiently Studying Organic Chemistry: Exam training for chemists, biochemists, pharmacists, life and health scientists,
Third Edition. Eberhard Breitmaier. © 2022 WILEY-VCH GmbH. Published 2022 by WILEY-VCH GmbH.
69.4 Peptide Synthesis 203

A sequence of more than 150 amino acids constitutes the primary structure of hemoglobin in red blood cells, as an ex-
ample. The actual shape of hemoglobin is neither one unique sheet structure nor one -helix. Structure elucidation has
revealed eight -helices which are folded by more flexible random coil parts of the sequence to the approximately spheri-
cal shape of this globular protein, as shown in Fig. 69.2 c. This folding of the helices to the actual shape of the molecule is
called the tertiary structure of the protein. Finally, several protein molecules may aggregate to oligomeric clusters; four
molecules aggregate to a tetramer in the case of hemoglobin (Fig. 69.2 d). Such aggregates are said to be the quaternary
structure of a protein.
Hemoglobin contains a coordinated non-protein organic function called the prosthetic group in general and heme in this
protein (Fig. 69.2 c). Heme is a ferrous chelate of a porphyrin (Chapter 67.2.1) which can be seen in the model (Figs. 67.1
c, 69.2 c). The ferrous ion is coordinated to the pyrrole nitrogens of the porphyrin ring and to an imidazole nitrogen of
histidine in the protein sequence. Oxygen takes the sixth coordination position in oxygenated hemoglobin in a pocket of
the protein during the transport in flowing blood (Chapter 67.2.1).

86 92 95

A T L S E L H C D K NH2
CH2
- Ala - Thr - Leu - Ser - Glu - Leu - His - Cys - Asp - Lys -
CH2
CH3 OH CH(CH3)2 SH CH2
H O CH OH H O CH2 H O CH2 H O CH2 H O CH2
N C CH N C CH N C CH N C CH N C CH
CH N C CH N C CH N C CH N C CH N C
CH3 H O CH2 H O CH 2 H O CH2 H O CH2 H O

CH(CH3)2 CH 2 CO2H
NH
CO2H N

a b

c d
Fig. 69.2. Primary, secondary, tertiary, and quaternary structure of hemoglobin: (a) section of the primary structure (amino acid se-
quence 86-95); (b) secondary structure ( -helix segment of the amino acid sequence 86-95); (c) tertiary structure (folding of eight
helices to the globular structure of the protein with the prosthetic group heme in the pocket for oxygenation; (d) quaternary structure
(tetramer with different colors for the monomeric proteins)

69.4 Peptide Synthesis


69.4.1 Protective Groups
Dipolar amino acids ("zwitterions", Chapter 68.1) do not react to peptides. Thus, they must be converted to reactive
derivatives. The reaction of such activated amino acids to a dipeptide involves selective formation of one peptide
bond between the carboxy function of one amino acid with the amino function of the other. It is necessary to prevent
reaction of the amino and carboxy functions not concerned with the peptide bond to be formed. For this reason,
amino and carboxy functions which are not required for the desired reaction must be protected. In order to synthesize the
204 69 Peptides, Proteins

dipeptide Ala-Val (AV), the NH2 group of the N-terminal alanine and the CO2H group of valine (blue in Fig. 69.3) have to
be derivatized by the use of protective groups. Reactive groups of side chains must also be protected.
The amino group of an amino acid is mostly derivatized to a urethane for protection by N-acylation of the amino acid with
di-t-butyl dicarbonate (Chapter 55.3.1). This reaction smoothly introduces the t-butyloxycarbonyl or Boc group. The Boc
group can be removed (deprotection) after peptide coupling by acidolysis, affording gases as byproducts (carbon dioxide
and methylpropene, Fig. 69.3).
CH 3 O O CH 3
pH = 8 - 10
CH + (H 3C)3C C C C(CH 3)3 CH
H 3N CO2 O O O CO2 (H 3C)3C O CO N CO2H
(CH3)3COH
alanine di-t-butyl dicarbonate H
N-t-butyloxycarbonyl-alanine ( Boc-alanine)

In order to protect its carboxy function, the amino acid is esterified to give a methyl or benzyl ester. These protective
groups can be readily removed by hydrolysis (methyl esters) or catalytic hydrogenation (benzyl esters are hydrogenated
to toluene and acid).
anhydrous HCl
H3N CO2 + CH3OH + HCl Cl H3N CO2CH3
CH H2O CH
CH(CH3)2 CH(CH3)2
valine valine methyl ester hydrochloride

69.4.2 Carboxy Activation


An amino acid with a protected carboxy function (ester) and another amino acid with a protected amino group (Boc) do
not react to form a dipeptide. Instead, the salt (an alkylammonium carboxylate of both reactants crystallizes from the
solution:
O CH 3 O CH 3
(CH 3)3C O C NH CH 2 CO2H + H 2N CH CO2CH 3 (CH 3)3C O C NH CH 2 CO2 H 3N CH CO2CH 3
Boc-glycine alanine methyl ester salt (ammonium carboxylate)

Peptide coupling requires nucleophilic attack of the free amino function at the free electrophilic carboxy C atom. There-
fore, the electrophilicity of the free carboxy function must be activated by esterification to an active ester unable to under-
go salt formation and having an electron-withdrawing, easily leaving alkoxy residue. Strongly electron-withdrawing groups
at the carboxy C atom destabilize the CO acyl single bond.
Classical active esters include carbodiimide adducts (esters of isourea, Chapter 56.4) and o-nitrophenyl esters. The latter
activate electrophilicity of the carboxy carbonyl carbon of an amino acid with RA by electron withrawal due to the reso-
nance effect [( )-M] of the nitro group at the o-position, facilitating nucleophilic substitution of the leaving phenoxy by the
amino function of an amino acid with RB.
aminolysis
of the active ester O O
O RB O RB
(peptide coupling)
RA C + N H RA C N H C RB C RB
RA N RA N
O OH
H O H H H
O2N
O2N peptide bond
O N o-nitrophenyl ester
O of amino acid with RA

N-Hydroxysuccinimidyl esters prepared by esterification of a Boc-amino acid with N-hydroxysuccinimide and diisopropyl-
carbodiimide (DIC, Chapter 56.4) are now frequently used as active esters:
CH 3 O CH 3
N O
CH CH NH CH(CH 3)2
O
(H 3C)3C O CO N CO2H + HO N + C (H 3C)3C O CO N C N + O C
H N H NH CH(CH 3)2
O
O O
Boc-alanine N-hydroxysuccinimide DIC Boc-alanine N-hydroxysuccinimidyl ester diisopropylurea
69.4 Peptide Synthesis 205

69.4.3 Peptide Coupling


A typical synthesis of a dipepetide such as Ala-Val (AV) consists of four steps (Fig. 69.3): First, protective groups (Boc
and ester) are introduced. Second, the carboxy group is activated. Third, peptide coupling is achieved by nucleophilic
substitution of the active ester leaving group by the free amino function. Finally, removal of protective groups liberates the
dipeptide.
CH 3
H3N CO2
N -terminal amino acid CH CH C -terminal amino acid
H 3N CO2
CH(CH3)2
O O
introduction of
protective groups + (H3 C) 3 C C C C(CH3 ) 3 pH = 8 - 10 + CH3 OH, + HCl H2O
O O O
CO2, (CH3)3OH
CH3 Cl H3N CO2CH3
CH CH
(H 3C)3C O CO N CO2H CH(CH 3)2
H
O

activation HO N , DIC DIPEA HCl

O
CH3
O
CH O H2N CO2CH 3
(H 3C)3C O CO N C N + CH
H O CH(CH3)2
O
N-hydroxysuccinimidyl active ester O

coupling HO N

O
CH 3 H
CH N CO2CH 3
protected dipeptide
(H3C)3C O CO N C CH
H O CH(CH3)2
removal of a) hydrolysis [ OH + H2 O , CH3 OH
protective groups b) acidolysis [H ] CO2 , (H3 C) 2 C=CH2

CH 3 H
CH N CO2 Ala-Val (AV)
H 3N C CH
O CH(CH3)2

Fig. 69.3. Synthesis of the dipeptide Ala-Val (AV), using t-butyloxycarbonyl and methyl ester protection and N-hydroxysuccinimide active
ester coupling (DIC = diisopropylcarbodiimide; DIPEA = diisopropylethylamine as base)

MERRIFIELD’s solid-phase peptide synthesis involves esterification of the C-terminal amino acid with side-chain R1 to a
support polymer (pol), repetitive coupling-deprotection steps with the next amino acids (R2, R3, …), and cleavage of the
target peptide from the polymer with hydrogen bromide in trifluoroacetic acid.
R2 O
HO R2 H R2 NHBoc
O NHBoc O H 1.) CF3CO2H O HO
NH 2 DIC O N 2.) N(C2H5)3 N DIC R3
pol O pol O NHBoc pol O NH 2
coupling deprotection next coupling
R1 R1 O R1 O
________________________________________________________________________________________________
Chapter 69 permits answers to the following:
(69.1) The C N bond in amides and peptides is shorter (132.5 pm) than in amines (148.7 pm). Why?
(69.2) What is an -helix? Explain the terms primary, secondary, tertiary, and quaternary structure of a protein.
(69.3) Look at Table 68.1 to draw the structural formula of the peptide with the sequence LATE.
(69.4) Which protective groups are used for (a) amino and (b) carboxy functions? Outline their introduction and removal.
(69.5) Formulate the reaction of a Boc-amino acid with an amino acid ester. Explain the purpose of ester activation.
(69.6) Write the equations for all steps necessary to synthesize the dipeptide Phe-Leu (FL).
206 70 Alkaloids

70 Alkaloids
70.1 Origin, Significance, Nomenclature
Alkaloids are biologically active metabolites mainly produced by plants and fungi to protect themselves from diseases
(antibacterials) and from being eaten (antifeedants, narcotics). Because of these activities, some alkaloids are used as
prescripted medicines (analgesics, anesthetics, antibacterials), as illegal drugs (addictive hallucinogens), or have been
adopted as inspiring model scaffolds to develop new pharmaceuticals. A heterocycle is the core structure of most alka-
loids, defining their classification (indole and isoquinoline alkaloids). Four amino acids (ornithine, lysine, tyrosine, and
tryptophane) have been found to be the biochemical precursors (Table 70.1).

Table 70.1. Amino acids as alkaloid precursors, heterocyclic alkaloid skeletons, and typical representatives
amino acid precursor alkaloid parent skeletons representative

H3C N
CO2H H 3C N
Orn NH2 N
O CH CH 2OH
H2N pyrrolizidine tropane C
non-essential amino acid
O
(vertebrate tissues, bird excreta) atropine from Atropa belladonna

CO2H H CH 2OH
Lys
NH 2 N
N
NH 2 quinolizidine lupinine from Lupinus species
CO2H H 3CO
Tyr
NH 2 NH 2 N N
HO HO HO H3CO
p-hydroxyphenylethylamine H 3CO

HO H3CO
benzylisoquinoline papaverine from Papaver somniferum
HO2C CH 3
NH 2 NH 2 N
H N
HO2C
N
Trp
H 3CO2C CH 3
N N N red : terpene OH
H H H partial structure
tryptamine (indole) lysergic acid vincamine from Vinca minor

The term "alkaloid" arises from alkali-like (Arabian al qualja = plant ash and Greek eidos = similarity, shape); nevertheless,
not all alkaloids are alkaline. Their nomenclature is frequently derived from the Latin name of the plant from which they
were first isolated. Examples are atropine from the deadly nightshade Atropa belladonna, lupinine from Lupinus species,
papaverine from the opium poppy Papaver somniferum, and vincamine from the evergreen Vinca minor (Table 70.1).

70.2 Biologically Active Alkaloids


About 20 000 alkaloids have been documented. This large variety dictates a selection (Table 70.2) limited to those alka-
loids of particular importance due to their pharmacological activity.
The tropane alkaloid ()-cocaine is the main component of the alkaloid mixture extracted from the leaves of the coca
bush Erythroxylum coca. Its stable hydrochloride is a controlled substance with a high potential to cause addiction, used
as a topical anesthetic and injected, snuffed, and smoked illegally (cocaine doping).
The pyrrolidine alkaloid ()-nicotine is the active substance of dried tobacco leaves (contents up to 8 %) from Nicotiana
tabacum. It stimulates the nervous system and, as a vasoconstrictor, increases blood pressure when tobacco is smoked.
Produced from tobacco waste, it is applied as an insecticide (human lethal dose 100 mg).

Efficiently Studying Organic Chemistry: Exam training for chemists, biochemists, pharmacists, life and health scientists,
Third Edition. Eberhard Breitmaier. © 2022 WILEY-VCH GmbH. Published 2022 by WILEY-VCH GmbH.
70.2 Biologically Active Alkaloids 207

()-Lysergic acid, obtained by alkaline hydrolysis of the ergot alkaloids from dried sclerotia of the fungus Claviceps pur-
purea, parasitically developing on spurred rye, represents a tryptamine alkaloid with a hemiterpene subunit (C5, Chapter
76.3.1). ()-Lysergic acid N,N-diethylamide, known as LSD (from Lysergsäure-N,N-diethylamid) and prepared by ami-
dation of the acid with diethylamine, is one of the strongest illegal hallucinogens known (dose 0.05 mg), confusing senses
and feelings and causing psychotic mental disorders.
Polycyclic indole alkaloids derived from tryptamine contain a monoterpene subunit (C10, Chapter 76.3.1). Representatives
are vincamine (Table 70.1), bitter-tasting ()-strychnine, formerly prescribed as a central stimulant but causing tetanic con-
vulsions, and its dimethoxy derivative ()-brucine, used for resolution of racemic acids by formation of diastereomeric salts,
the latter two obtained from the seeds of Strychnos nux-vomica.
The blue fluorescent quinoline alkaloid ()-quinine from Javanese Cinchona bark (contents up to 8 %) acts as an antima-
larial and muscle relaxant. Tasting pleasantly bitter, it is used as a flavor in carbonated beverages.
Opium from the poppy Papaver somniferum contains up to 20 % ()-morphine and smaller amounts of its methyl ether ()-
codeine. Acetylation of ()-morphine yields the non-natural diacetyl derivative known as heroin, another illegal hallucino-
gen potentially causing addiction. Immortalizing in its name MORPHEUS, the god of dreams in Greek mythology as narrat-
ed by OVID, ()-morphine is the most prescribed analgesic of plant origin worldwide.

Table 70.2. Selection of pharmacologically active alkaloids (only the listed enantiomers have the activities described in the text)
tropane alkaloid pyrrolidine alkaloid N indol e alkaloids H
R N
CO2CH 3 CH 3
H3C N O N
H H
N R
O N H
CH 3 red: hemiterpene red: monoterpene H
H N
O subunit subunit O
O H
()-cocaine ()-nicotine N
H R = H : () - strychnine
coca bush tobacco (+)-lysergic acid N,N-diethylamide (LSD) R = OCH3 : () - brucine
Erythroxylum coca Nicotiana tabacum ergot (spurred rye, Claviceps purpurea) Strychnos nux-vomica
quinoline alkaloid benzyltetrahydroisoquinoline alkaloids
H morphine derivatives
H RO
2
OR
HO 1
3
RS N
11 4
H 3CO
10
R = R' = H : () - morphine O H H 12 O
16 15 13
N R = OCH3 ; R' = H : () - codeine N H3C 5
N 9 7
() - quinine R = R' = COCH3 : () - heroin CH 3 17 14 8
R'O OR' 6
cinchona bark opium poppy
Cinchona officinalis Papaver somniferum ROBINSON formula stereoformula with ring numbering

Mescaline, a psychotomimetic phenylethylamine alkaloid from mescal buttons of the Mexican peyote cactus, is the model
scaffold for some illegal designer drugs known as "amphetamine", "speed" and "ecstasy".
H3CO NH 2 S NH 2 S NHCH3 O S NHCH3

CH 3 CH3 CH 3
H 3CO O
OCH3 (S)-amphetamine (S)-N-methylamphetamine (S)-3,4-methylendioxy-N-methylamphetamine
mescaline (RS)-"speed" ("meth") (RS)-"ecstasy"

________________________________________________________________________________________________
Chapter 70 permits answers to the following:
(70.1) What are alkaloids and which biological functions do they perform?
(70.2) Which amino acids are alkaloid precursors and which alkaloid classes are derived from them?
(70.3) Draw the structural formulas of (a) ()-cocaine, (b) ()-nicotine, and (c) ()-LSD. Classify these alkaloids.
(70.4) Which alkaloid subunit do you recognize in the illegal drugs "speed" and "ecstasy"?
(70.5) Formulate the reaction of ()-morphine with acetic anhydride.
(70.6) Count the number of stereogenic centers in morphine. How many stereoisomers are possible? Look at Chapter 46.1.
208 71 Carbohydrates: Aldoses and Ketoses

71 Carbohydrates: Aldoses and Ketoses


Word-for-word and formally, carbohydrates represent "hydrates of carbon" (CH2O)n. They comprise one of the most
abundant classes of organic compounds found in living organisms as a main source of energy. Glucose (Fig. 71.1) with
molecular formula C6H12O6, also referred to as grape sugar due to one of its origins, is the parent representative. It oc-
curs freely and as a subunit of biopolymers in fruits and other parts of plants, formed there in the chloroplasts from carbon
dioxide and water by photosynthesis, catalyzed by green chlorophyll (Chapter 67.2.2) and enzyme systems.

71.1 Aldoses
Glucose as a pentahydroxyhexanal represents one of the aldohexoses. Due to four stereogenic centers, 24 = 16 stereo-
isomers exist; these include the eight diastereomers depicted in Fig. 71.1, each one of which occurring as a pair of enan-
tiomers. All eight naturally abundant aldohexoses possess the absolute configuration of D-(+)-glyceraldehyde at the
carbon adjacent to the hydroxymethyl (most distant from carbonyl) and therefore have been assigned as D-aldohexoses
(Fig. 71.1). It would be inconvenient to use the CIP convention and IUPAC names, exemplified by D-glucose which would
then be called (2R,3S,4R,5R)-2,3,4,5,6-pentahydroxyhexanal. The family of natural aldoses consists of eight D-aldo-
hexoses, four D-aldopentoses, two D-aldotetroses and D-glyceraldehyde (Fig. 71.1). Their enantiomers are the L-aldoses,
very rarely occurring in nature.

H 1 O H O H O H O H O H O H O H O
C C C C C C C C
2
H C OH HO C H H C OH HO C H H C OH HO C H H C OH HO C H
3
H C OH H C OH HO C H HO C H H C OH H C OH HO C H HO C H
4
H C OH H C OH H C OH H C OH HO C H HO C H HO C H HO C H
5
H C OH H C OH H C OH H C OH H C OH H C OH H C OH H C OH
6
CH 2OH CH2OH CH2OH CH 2OH CH2OH CH2OH CH 2OH CH2OH
(+)-D-allose (+)-D-altrose (+)-D-glucose (+)-D-mannose ()-D-gulose ()-D-idose (+)-D-galactose (+)-D-talose 8 aldohexoses
(eight epimers)

H O H O H O H O
C C C C
H C OH HO C H H C OH HO C H
H C OH H C OH HO C H HO C H
H C OH H C OH H C OH H C OH
CH 2OH CH2OH CH 2OH CH2OH
()-D-ribose ()-D-arabinose (+)-D-xylose ()-D-lyxose 4 aldopentoses
(four epimers)

H O H O
C C
H C OH HO C H
H C OH H C OH
CH2OH CH2OH
()-D-erythrose (+)-D-threose 2 aldotetroses
(two epimers)

H O
C
H C OH
CH2OH
(+)-D-glyceraldehyde (glycerose) 1 aldotriose

Fig. 71.1. Family of diastereomeric D-aldoses, possessing the absolute configuration of D-glyceraldehyde at the stereogenic center most
distant from the aldehyde group (FISCHER convention); FISCHER projection formulas; (+)- or ()- indicates the sign of the optical rotation

71.2 Ketoses
Pentahydroxy-2-hexanones are functional isomers of aldohexoses (pentahydroxyhexanals). They are called ketohexoses
or hexuloses (Fig. 71.2). Due to three stereogenic centers, 23 = 8 stereoisomeric ketohexoses exist. These include four
diastereomers (Fig. 71.2), each one of which occurring as a pair of enantiomers. Naturally abundant diastereomers have

Efficiently Studying Organic Chemistry: Exam training for chemists, biochemists, pharmacists, life and health scientists,
Third Edition. Eberhard Breitmaier. © 2022 WILEY-VCH GmbH. Published 2022 by WILEY-VCH GmbH.
71.3 Cyclohemiacetals, Cyclohemiketals: Pyranoses, Furanoses 209

the D configuration, such as D-fructose (fruit sugar) which represents a functional isomer of D-glucose with the same
absolute configuration of stereogenic centers (C-3, C-4, C-5). The family of natural ketoses consists of four D-keto-
hexoses, two D-ketopentoses, and one D-ketotetrose (Fig. 71.2).
1
CH2OH CH 2OH CH2OH CH2OH
2
C O C O C O C O
3
H C OH HO C H H C OH HO C H
4
H C OH H C OH HO C H HO C H
5
H C OH H C OH H C OH H C OH
6
CH2OH CH 2OH CH2OH CH2OH
(+)-D-psicose (D-allulose) ()-D-fructose (levulose) (+)-D-sorbose ()-D-tagatose 4 hexuloses
(four epimers)

CH2OH CH 2OH
C O C O
H C OH HO C H
H C OH H C OH
CH2OH CH 2OH
()-D-ribulose (D-erythro-pentulose, adonose) ()-D-xylulose (D-threo-pentulose, D-lyxulose) 2 pentuloses
(two epimers)

CH2OH
C O
H C OH
CH2OH
()-D-glycero-tetrulose (D-erythrulose, D-threulose) 1 tetrulose

Fig. 71.2. Family of D-ketoses possessing the configuration of D-glyceraldehyde at the stereogenic center most distant from carbonyl

71.3 Cyclohemiacetals, Cyclohemiketals: Pyranoses, Furanoses


Having aldehyde and hydroxy in one molecule, D-glucose does not exist freely but forms a six-membered cyclohemiace-
tal which is called glucopyranose because it is derived from tetrahydropyran (Table 60.2). Cyclohemiacetal formation
involves nucleophilic attack of the C-5-OH group at the aldehyde carbonyl carbon, creating a new stereogenic center at
C-1, called the anomeric carbon. Two stereoisomers called anomers arise. These are -D-glucopyranose with axial OH at
C-1 and -D-glucopyranose with equatorial OH at C-1.
H 1 O D-glucose, stereoisomeric cyclohemiacetals (anomers)
C rotation about cyclohemi-
H C-4C-5 bond H acetalization H H
H C OH H CH 2OH CH 2OH CH 2OH
4 H 6 H H H
HO C H R CH 2OH R OH O O
HO HO HO HO
HO 5 2 1 O HO 5 1 O 5 5
H C OH HO 1 H HO 1 OH
3
C C
5 OH OH H OH H HO H HO
H C OH
H H H H H OH H H
CH2OH
-D-glucopyranose  -D-glucopyranose
D-glucose (acyclic) cis-configuration trans-configuration
of OH groups at C-1 and C-2

The same happens to D-ribose and 2-deoxy-D-ribose which are abundant aldopentoses (Fig. 71.1) as their phosphoric
acid diesters form the backbone of nucleic acids RNA (D-ribose) and DNA (D-2-deoxyribose, Chapter 73). Nucleophilic
addition of the C-4 OH group to the aldehyde carbonyl carbon atom of D-ribose leads to a five-membered cyclohemiace-
tal called ribofuranose because it is derived from tetrahydrofuran (Table 60.2). Two diastereomeric anomers are formed,
called - and -D-ribofuranose.
H 1 O stereoisomeric cyclohemiacetals (anomers)
C
H C OH HO H 2C H H H HO H 2C H H H HO HO H2C H H OH HO OH
4 OH C1 4 O 1 4 O 1 4 O 1 4 O 1
H C OH
4
H C OH H O H OH OH H H
HO OH HO OH HO OH HO OH HO OH
CH 2OH -D-ribofuranose  -D-ribofuranose
D-ribose (acyclic) cis-configuration of OH groups at C-1 and C-2 trans-configuration of OH groups at C-1 and C-2
210 71 Carbohydrates: Aldoses and Ketoses

In the same manner, cyclohemiacetalization of 2-deoxy-D-ribose provides 2-deoxy-- and -D-ribofuranose:


H 1 O stereoisomeric cyclohemiacetals (anomers)
C
CH 2 HO H2C H H H HO H 2C H H H HO HO H 2C H H OH HO OH
4 OH C1 4 O 1 4 O 1 4 O 1 4 O 1
H C OH
4
H C OH H O H OH OH H H
HO H HO H HO HO H HO
CH 2OH 2-deoxy--D-ribofuranose 2-deoxy- -D-ribofuranose
2-deoxy-D-ribose (acyclic) cis-configuration of OH groups at C-1 and C-3 trans-configuration of OH groups at C-1 and C-3

Ketoses, as well, do not exist as such and form cyclohemiketals. Paralleling the aldohexose D-glucose, the ketohexose D-
fructose predominantly occurs as a mixture of - and -D-fructopyranose (Fig. 71.3), the -anomer crystallizing from an
aqueous solution. Fructofuranoses are also present in aqueous solution (Fig. 71.3); the -D-fructofuranose is a structural
unit of sucrose (Chapter 72.1).
H H
5 OH OH
6 1
HO O 1 CH OH
2 HO O
H CH2OH H OH
4 3 2 2C O
H H
OH OH 3 OH CH2OH
HO C H
-D-fructopyranose 4  -D-fructopyranose
H C OH (predominant, crystallizes from water)
5
H C OH
HOH2C 6 1 CH 2 OH HOH2C OH
O 6
CH 2 OH O
5 HO 2 HO
D-fructose CH2OH
4 3 OH
(pentahydroxy ketone,
OH only traces detectable in water) OH
-D-fructofuranose  -D-fructofuranose

Fig. 71.3. Equilibrating pyranoses and furanoses of the ketohexose ()-D-fructose in aqueous solution

71.4 Mutarotation
When a freshly prepared aqueous solution of -D-glucopyranose is placed in a polarimeter, an initial specific rotation of
+113° decreases with time to a final value of +52.5°; following the same procedure with -D-glucopyranose, an initial
specific rotation of +19° approches the same final value, +52.5°. This change of optical rotation with time is known as
mutarotation (from Lat. mutatio = change), indicating an equilibration of the two anomers by way of the open-chain pen-
tahydroxy aldehyde, which reaches a final composition of the mixture specific for D-glucose, 63 % - and 37 %-D-
glucopyranose.
H 6 H 6 H 6
HO CH2 HO CH2 HO CH2
H H H
HO O HO OH HO O
HO
5
H HO
5 O HO
5
OH
1 C1 1
H OH H OH H OH
H OH H H H H
20 20
-D-glucopyranose , []D = + 113° D-glucose (-al form)  -D-glucopyranose , []D = + 19°

71.5 Typical Reactions


71.5.1 Glycosides, Glycosidation
O-Glycosides (O for oxygen) are full acetals of furanoses and pyranoses, in which an alkyloxy or aryloxy group substi-
tutes the OH of the hemiacetal. The non-sugar unit of the glycoside, such as salicyl alcohol in the analgesic glucopyra-
noside salicin from the willow (Lat. Salix), is called the aglycon, the sugar unit is the glycon. An amino function substitutes
the hydroxy groups of hemiacetals in N-glycosides. N-Glycosides are the subunits of nucleic acids (RNA and DNA,
Chapter 73).
CH 2OH salicin HOH 2C N nucleobase
O CH 2OH N-glycoside O
HO 2-(hydroxymethyl)phenyl
HO O (ribonucleoside from RNA)
 -D-glucopyranoside
OH from leaves and bark of willow ( Salix) OH OH
glycon aglycon
71.5 Typical Reactions 211

Glycosides are hydrolyzed upon heating in aqueous acids or enzymatically to provide the sugar and the aglycon. Glyco-
sidation by reacting pyranoses with alcohols in the presence of anhydrous acid (FISCHER synthesis) yields a mixture of
anomeric glycosides, such as methyl - and -D-mannopyranoside from D-mannose:

HOH2C HO HOH2C HO HOH2C HO


[H ] , heat
HO O HO O HO O
2 +
HO OH + 2 CH3OH HO HO OCH3

- and -D-mannopyranose OCH3


(anomeric cyclohemiacetals) methyl - and  -D-mannopyranoside (anomeric acetals)

Nucleophilic substitution of peracetylated -glycosyl halides such as acetobromo--D-glucose by alcohols involves configu-
rational inversion at C-1, stereospecifically yielding the -glycosides (KÖNIGS-KNORR synthesis):
CH2OAc CH2OAc
R'
O AcO O
2 AcO + 2O + Ag2CO3 2 + 2 AgBr + CO2 + H2O
AcO 1
AcO 1 OR'
H
O Br OAc
H3C 2,3,4,6-tetra-O-acetyl- -D-glucopyranoside
Ac = COCH3 O acetobromo--D-glucose

71.5.2 O-Alkylation, O-Acylation


The non-anomeric alcohol OH functions (attached to C-2, C-3, C-4, and C-6) of aldoses and ketoses can be O-alkylated,
for example with methyl iodide to yield tetramethyl ethers in the presence of a base (WILLIAMSON synthesis) or O-acylated,
for example with acetic anhydride to give tetra-O-acetyl derivatives (esters):
O
H 3COH 2C HOH 2C AcOH 2C C
OCH 3 OH O CH 3
H 3CO O CH3I , Ag 2O HO O (CH3CO) 2 O , pyridine AcO O
H 3CO OCH 3 HO OCH 3 AcO OCH 3
2,3,4,6-tetramethyl ether methyl  -D-mannopyranoside 2,3,4,6-tetra- O-acetyl derivative

71.5.3 Reduction and Oxidation


Aldoses and ketoses can be reduced to polyols (alditols, sugar alcohols) electrochemically, by catalytic hydrogenation,
and by reducing reagents such as sodium amalgam or borohydrides. Reduction of D-glucose with amalgamated sodium
produces the laxative sweetener glucitol (sorbitol).
Depending on the oxidation reagent, glucose and other aldoses are oxidized to aldonic acids (polyhydroxy acids), ald-
uronic acids (aldehyde acids), and aldaric acids (dicarboxylic acids). Weaker oxidation reagents such as diluted nitric acid
oxidizes the masked aldehyde function of glucopyranose, yielding D-gluconic acid. Fuming nitric acid oxidizes D-
glucopyranose to D-glucaric acid. Oxidation of a glucoside, however, yields the lactone of D-glucuronic acid.
CH 2OH CO2H CHO CO2H
H C OH H C OH HOH 2C H C OH H C OH
Na / Hg , OH CH 2OH
HO C H HO C H  H2O
HO O HO C H HO C H
in H2 O HO O HNO3
OH O
H C OH HO H C OH H C OH H C OH
OH
H C OH OH H C OH OH H C OH H C OH
CH 2OH CH2OH CO2H CO2H
D-glucitol - and  -D-glucopyranose D-gluconic acid D-glucono--lactone D-glucuronic acid D-glucaric acid
(= sorbitol) (an aldonic acid) (an alduronic acid) (an aldaric acid)

________________________________________________________________________________________________
Chapter 71 permits answers to the following:
(71.1) Which compound is named ()-(2R,3S,4R,5R)-2,3,4,5,6-pentahydroxyhexanal? How many stereoisomers exist?
(71.2) Write FISCHER projections of all (a) aldohexoses, (b) aldopentoses, and (c) hexuloses, all with the D configuration.
(71.3) Draw the formulas of (a) - and -D-mannopyranose and (b) - and -D-ribofuranose. Define the term mutarotation.
(71.4) What is a glucoside? Draw the structural formula of 2-nitrophenyl -D-glucopyranoside and design a synthesis.
(71.5) D-Glucose is exposed to ethanol containing some anhydrous hydrogen chloride. What products are formed?
(71.6) What products are obtained from glucose by (a) reduction and (b) oxidation?
212 72 Carbohydrates: Oligo- and Polysaccharides

72 Carbohydrates: Oligo- and Polysaccharides


Depending on the number of their aldose or ketose units, carbohydrates also named saccharides (from Greek saccharon
= sugar) are subdivided into three classes. Monosaccharides are (monomeric) aldoses and ketoses (Chapter 71). Oligo-
saccharides comprise di-, tri-, tetra-…-saccharides with 2-15 monosaccharides as units which can be liberated by acidic
or enzymatic hydrolysis. Polysaccharides also referred to as glycans are biopolymers with up to thousands of monosac-
charide units. Glycosidic bonds connect the monosaccharides in oligomers and polymers.

72.1 Oligosaccharides
Disaccharides are the most abundant natural oligosaccharides, containing two monosaccharide units connected by a
glycosidic bond (full acetals) and sharing typical properties of monosaccharides: They taste sweet, dissolve in water, and
crystallize from these solutions. Two types of disaccharides exist.
Reducing disaccharides belong to the maltose type: The glycosidic bond connects the anomeric carbon (C-1) of one
sugar with any carbon except C-1 of another, exemplified by maltose where two D-glucopyranose units are connected by
an -glycosidic bond from C-1 of the first unit to C-4 of the second, referred to as a 1-O-4- or -(14)-glycosidic bond
(Fig. 72.1). The anomeric carbon of the second sugar unit is still a hemiacetal masking the aldehyde function so that the
disaccharide is able to undergo mutarotation (Chapter 71.4) and aldehyde reactions, detectable by the FEHLING, TOLLENS
and NYLANDER tests (Chapter 47.4.1). Cellobiose and lactose are also reducing disaccharides, but with a 1-O-4- or -
(14)-glycosidic bond (Fig. 72.1).

OH
HO
O O HO OH OH
1
HO HO4 HO O HO OH O HO OH
HO 1 O 4 HO O
HO OH OHO OHOH O O
HO OH HO OH
1
HO
4

maltose cellobiose lactose


4-O-(-D-glucopyranosyl)-D-glucopyranose 4-O-(-D-glucopyranosyl)-D-glucopyranose 4-O-(-D-galactopyranosyl)-D-glucopyranose
C-1-O-C-4-glycosidic bond C-1-O-C-4-glycosidic bond C-1-O-C-4-glycosidic bond
a glycosyl-glycose, reduces and mutarotates a glycosyl-glycose, reduces and mutarotates a glycosyl-glycose, reduces and mutarotates

HO 1 HO OH
OH O O OH
HO HO
O OH O
HO 2 O
HO 1 OH 1 OH 1
6
OH O HO HO OH O O
OH
O
1 HO OH HO 1
HO HO OH O 2
trehalose sucrose (saccharose)
-D-glucopyranosyl--D-glucopyranoside -D-glucopyranosyl- -D-fructofuranoside = HO OH
C-1-O-C-1-glycosidic bond -D-fructofuranosyl- -D-glucopyranoside raffinose
a glycosyl-glycoside a glycosyl-glycoside [6-O-(-D-galactopyranosyl)- O
-D-glucopyranosyl]- OH
-D-fructofuranoside
HO

HO OH
HO O
HO O O O OH
O OH
O O OH
O OH HO OH OH O
O OH HO HO O HO O
HO
O
HO
OH HO
OH O OH OH
HO
O OH O
O HO
O O
OH OH OH
HO
HO O OH O
O
OH OH HO
O
HO OH
O O
OH O HO HO OH
OH
O HO O HO HO
O OH OH
HO OH OH
OH O O HO O O
O OH
HO O HO
O OH O O HO
HO OH OH O
O
HO O OH
O
- - HO - OH
cyclodextrin

Fig. 72.1. Oligosaccharides: disaccharides, trisaccharides, cyclodextrins

Non-reducing disaccharides belong to the trehalose type: The glycosidic bond connects the anomeric C-1 carbons of
both monosaccharides so that neither carbonyl reactions nor mutarotations are possible. Trehalose with two glucopyra-

Efficiently Studying Organic Chemistry: Exam training for chemists, biochemists, pharmacists, life and health scientists,
Third Edition. Eberhard Breitmaier. © 2022 WILEY-VCH GmbH. Published 2022 by WILEY-VCH GmbH.
72.2 Polysaccharides 213

nose units connected by a 1-O-1-glycosidic bond and sucrose (saccharose) with glucopyranosyl and fructofuransyl
linked by a 1-O-1-glycosidic bond are examples (Fig. 72.1).
()-Sucrose (saccharose) produced from sugar cane and sugar beet is widely used for sweetening food. ()-Maltose (malt
sugar), the disaccharide unit of starch, is another nutrient and sweetener produced by degradation of starch by the en-
zyme diastase. ()-Lactose occurs in the milk of mammals (cow’s 4.5 %). Not freely occurring, ()-cellobiose is the disac-
charide unit of cellulose. ()-Trehalose is found in bacteria, yeasts, fungi, and insects. ()-Raffinose from sugar beet is a
trisaccharide connecting sucrose and galactose (Fig. 72.1).
Cyclodextrins, prepared by enzymatic degradation (Bacillus macerans amylase) of starch, are macrocyclic oligosaccha-
rides built up from 1-O-4- or simply -(14)-linked glucopyranose units. They form chiral cavities (diameter 500-850
pm), enabling them to enclose organic compounds. Chiral cyclodextrin phases for the resolution of racemates by chroma-
tographic methods are obtained by derivatization of cyclodextrin OH groups.

72.2 Polysaccharides
Starches are polysaccharides stored as energy reservoirs in seeds and tubers of plants (cereals, potatoes). Granules of
starch consist of water-insoluble amylopectin outside (up to 80 %), covering water-soluble amylose inside (at least 20 %).
Amylose forms unbranched -(14)-glycosidically connected, helically arranged chains of glucopyranose units (molecu-
lar mass 17000-225000 built from 100-1400 units). These chains are branched in amylopectin by -(16)-connected
side chains (4 %) of 20-25 glucopyranose units. The helical channels of amylose enclose the I3 ions in aqueous solu-
tions of iodine and potassium iodide (KI + I2 KI3), yielding a deep violet solution, thus providing a sensitive test for
amylose and iodine.
O O O O O O
HO 1 HO HO HO 1 HO
HO
4 4
O OH OHO OHO HO OH O HO OH OHO OHO HO OH O
HO
part of a m y l o s e (water-soluble starch) with -(14)-connected D-glucopyranose units

Cellulose and the biopolymer lignin form the cell walls of plants and woods: cotton contains more than 90 %, woods about
50 % and straw about 30 %. Unlike amylose, cellulose forms -(14)-glycosidically connected glucopyranose units. The
chain length varies from several 100 to several thousand units.
HO OH HO OH
HO O HO O OHO OH
HO O O HO O O
1 4
OH HO OH n HO
part of c e l l u l o s e with -(14)-connected D-glucopyranose units

After cellulose, chitin (from Greek chiton = shell) is the largest renewable source of polysaccharide raw material, making
up the chief component of the exoskeletons of invertebrates and insects, and the cell walls of yeasts and fungi. Like
cellulose, chitin forms predominantly unbranched chains of -(14)-glycosidically connected 2-acetamido-2-deoxy-D-
glucopyranosyl units. The monomer is called N-acetyl-D-glucosamine.
CH 3 HO HO
O HO O HO O
HO NH HO HO HO OH
HO O HO O O NH NH
chitin OH
HO O O HO OH O O
NH HO n NH CH 3 CH 3
O O -anomer  -anomer
CH 3 CH 3 N-acetyl-D-glucosamine
________________________________________________________________________________________________
Chapter 72 permits answers to the following:
(72.1) What compound is named -D-fructofuranosyl--D-glucopyranoside? Draw the structural formula.
(72.2) What is the difference between maltose and cellobiose? Draw the structural formulas to explain.
(72.3) What is the difference between maltose and trehalose? Suggest a test tube reaction to identify one of them.
(72.4) What products are obtained by enzymatic degradation (hydrolysis) of starch?
(72.5) Draw the structures of (a) amylose, (b) cellulose, and (c) chitin. What are the sources of these polysaccharides?
214 73 Nucleic Acids: DNA and RNA

73 Nucleic Acids: DNA and RNA


73.1 Nucleotides, Nucleosides, Nucleobases
Nucleic acids are biopolymers occurring in the cells of all living organisms. Their stepwise hydrolysis yields first nucleo-
tides, then nucleosides and phosphoric acid, and finally heterocyclic nucleobases and pentoses such as ribose from a
ribonucleic acid (RNA) and 2-deoxyribose from a deoxyribonucleic acid (DNA).
NH 2 NH 2

O OH N N N N NH 2
P
5´ N N
nucleic acid O O N HO N HO N N
O O O OH +
(polynucleotide) 4´ 1´
3´ 2´ N N
HO OH HO OH HO OH H
adenosine 5´-monophosphate (AMP) adenosine D-ribofuranose adenine
nucleotide (nucleoside phosphate) nucleoside (N-ribofuranoside) pentose nucleobase

Nucleobases comprise three pyrimidines and two purines. Pyrimidine nucleobases include cytosine C, thymine T in
DNA, and uracil U instead of thymine in RNA (Fig. 73.1). Adenine A and guanine G (Fig. 73.1) constitute purine nucleo-
bases. Nucleosides are N-glycosides, such as 9--D-ribofuranosyladenine called adenosine. Nucleotides are phos-
phoric acid esters of nucleosides, exemplified by adenosine 5´-monophosphate (AMP), in which the primary alcohol
group at ribose carbon C-5´ is esterified with phosphoric acid.
Nucleic acids are polynucleotides in which phosphoric acid connects two nucleosides from C-5´ of one pentofuranose
to C-3´ of the next. Thus, as shown in Fig. 73.1, D-ribofuranose and 2-deoxy-D-ribofuranose 3´,5´-polyphosphates are the
backbones of RNA and DNA, arranging the nucleobases in a specific sequence.

NH2 NH2

O OH N N O OH N N
P P adenine (A) P P adenine (A)
O O N N O O N N
rib O drib O
O NH2

O O OH N NH O O
guanine (G) N cytosine (C)
P P P P
5´ N 5´
O O N NH2 O O N
O O O
rib NH2 drib O
3´ 3´ H3C
O O OH O O
N cytosine (C) NH thymine (T)
P P P P
5´ N 5´
O O O O N
O O O O
rib O drib O
3´ 3´
O O OH O O N
NH NH
P P uracil (U) P P guanine (G)
O O N O O N N NH2
O O O
rib drib

HO OH HO

A G C U A C T G
abbreviated 3' abbreviated 3'
formula P P P P OH formula P P P P OH
a 5'
b 5'

Fig. 73.1. Nucleic acid fragments: (a) RNA fragment, tetraribonucleotide, abbreviation p-5'-AGCU-3'-OH; (b) DNA fragment, tetradeoxy-
ribonucleotide, abbreviation p-5'-dACTG-3'-OH or simply ACTG

The sequence of the four nucleobases A, C, T, and G in DNA encodes the sequence of amino acids in all proteins speci-
fic for any living organism, defining the genetic code. Different kinds of RNA act as carriers of information and amino
acids in protein biosynthesis.

Efficiently Studying Organic Chemistry: Exam training for chemists, biochemists, pharmacists, life and health scientists,
Third Edition. Eberhard Breitmaier. © 2022 WILEY-VCH GmbH. Published 2022 by WILEY-VCH GmbH.
73.2 Base Pairing and Double Helix of DNA 215

73.2 Base Pairing and Double Helix of DNA


The pairs of nucleobases adenine-thymine (AT) and guanine-cytosine (GC) associate by two and three hydrogen bridges,
respectively, as shown in Fig. 73.2. The unique matching of these two pairs of nucleobases and the particular stability of
their multiple hydrogen bridges results in an exclusive base pairing of the so-called complementary bases A=T and GC.
H H
284 282
N O H N N N H O CH 3

N 284 N 291
N H N N H N
R N N R N N
284
N H O R O R
H
1085 pm 1085 pm

G C A T

Fig. 73.2. top: Base pairs GC and AT in DNA with multiple hydrogen bonds, lengths 282-291 pm; bottom: tube models including deoxy
riboside units with inserted hydrogen bonds (green)

These base pairs A=T and GC enforce two strands of DNA, complementary with respect to their sequences of nucleo-
bases, to adopt a unique and particularly stable screwlike conformation called the double helix, robustly held together by
the base pairs (Fig. 73.2), as portrayed in Fig. 73.3.

Fig. 73.3. Calculated tube model of the DNA sequence (ATGC)n with inserted ribbons to indicate the double helix

Thus, base pairing compresses the genetic information of the DNA to the nanometer (nm) dimension. The double helix
has a constant inner diameter of 2.2 nm because each purine nucleobase faces a pyrimidine base (Figs. 73.2, 73.3). Ten
bases complete one turn of each coil, corresponding to a length of 3.4 nm. Both DNA strands of the double helix are
right-handed in all living organisms on earth and run along oppositely (Fig. 73.3) due to their complementary sequence of
nucleobases.
________________________________________________________________________________________________
Chapter 73 permits answers to the following:
(73.1) Disconnect a nucleic acid stepwise from larger fragments to the monomeric components. Formulate and name them.
(73.2) Adenosine 5´-triphosphate (ATP) is an energy source, phosphorylation reagent, and coenzyme in biosyntheses.
Draw the structural formula and classify this compound.
(73.3) Draw the DNA sequence ACGT in detail and abbreviated.
(73.4) What are base pairs and how are they held together?
(73.5) An open-chain DNA strand would have a length of about 2 m. In fact, the length is in the nm range. Why?
216 74 Lipids

74 Lipids
74.1 Classification
Lipids (Greek lipos = fat, bacon, oil) comprise long-chain carboxylic acids called fatty acids, their esters and other deriva-
tives. All lipids dissolve poorly or not at all in water, and readily to unlimited in organic solvents (chloroform, ethanol).
Hydrophobism and lipophilism of fatty acids and compounds sharing these properties (lipoids) arise because of their long
alkyl chains. Fig. 74.1 provides a survey of typical examples.
Lipids

fatty acids waxes fats phosphatides glycolipids aminolipids

H 3C (CH 2)n CO2H R1 CO OR2 CH 2 O CO R 1 CH2 O CO R 1 CH 2 O CO R 1 R 1 CO NH


acids, esters, amides wax ester from CH O CO R2 CH O CO R 2 CH O CO R2 H3N (CH2)3 CH
saturated, unsaturated fatty acid and O
CH 2 O CO R 3 CH 2 O CH2 O CO
unbranched, branched long-chain alcohol CH2 O P O
triglycerides
hydroxylated, predominantly galactose CH2 O CO R2
(H 3C) 3N CH2 CH 2 O
with an even number of C atoms
choline glycerol phosphatides glycerol glycolipid ornithine lipid
(-lecithin) (galactosyl diglyceride)

Fig. 74.1. Classes of lipids (R1, R2, and R3 represent long alkyl and alkenyl chains)

Fats, also referred to as triglycerides, are the glycerol esters of some saturated and unsaturated fatty acids (Tables 74.1,
74.2); they are the most abundant, physiologically and commercially important lipids. Phosphatides such as the lecithins
are dipolar triglycerides in which a phosphoric diester connects an ethanolamine with glycerol instead of one fatty acid
acyl residue; they contribute to the formation of biomembranes and are isolated from nerve tissue, brain, heart, liver, egg
yolk, and soy beans. Glycolipids and aminolipids are involved in synaptic transfers of stimuli and cell-virus interactions.

74.2 Fatty Acids, Fats, Soaps


74.2.1 Saturated and Unsaturated Fatty Acids
More than 300 naturally abundant, unbranched saturated and unsaturated fatty acids are known. Most of them possess
an even number of carbon atoms (C14 , C16 , C18 , Table 74.1) as a result of their biogenesis from acetic acid C2 units
(Chapter 75.1).

Table 74.1. Selection of unbranched, saturated fatty acids


IUPAC name common name melting point
formula abbreviation
acid acid [°C]
even number H3C (CH 2)10 CO2H dodecanoic lauric C12:0 45
of C atoms: H3C (CH 2)12 CO2H tetradecanoic myristic C14:0 54
H3C (CH 2)14 CO2H hexadecanoic palmitic C16:0 62
H3C (CH 2)16 CO2H octadecanoic stearic C18:0 70
H3C (CH 2)18 CO2H eicosanoic arachidic C20:0 76
H3C (CH 2)20 CO2H docosanoic behenic C22:0 80
H3C (CH 2)22 CO2H tetracosanoic lignoceric C24:0 84
odd number H3C (CH 2)7 CO2H nonanoic pelargonic C9:0 12
of C atoms: H3C (CH 2)15 CO2H heptadecanoic margaric C17:0 61

Additionally, many naturally occurring triglycerides contain unsaturated fatty acids, such as oleic acid prepared by hydro-
lysis of olive oil. Their CC double bonds start at the seventh or ninth carbon atom (7- and 9-positions) relative to the
terminal methyl group (1, Table 74.2). Double bonds in multiply unsaturated fatty acids are predominantly isolated (non-
conjugated, Table 74.2) with cis-configuration (c).
The human organism is unable to biosynthesize the so-called essential fatty acids and has to rely on food as a source of
these acids for the prevention of deficiency diseases. Arachidonic acid required for the biosyntheses of tissue hormones
is found in glandular organs, liver, brain, and animal depot fats. Linoleic acid is an essential nutrient abundant in oils from

Efficiently Studying Organic Chemistry: Exam training for chemists, biochemists, pharmacists, life and health scientists,
Third Edition. Eberhard Breitmaier. © 2022 WILEY-VCH GmbH. Published 2022 by WILEY-VCH GmbH.
74.2 Fatty Acids, Fats, Soaps 217

linseed and other seeds (cotton, poppy, sunflower), corn, peanut, and soy bean. Linolenic acid occurs in most vegetable
oils (major component of linseed oil); as an 3 fatty acid it stabilizes the biomembranes in cells and down-regulates the
level of cholesterol and triglycerides in the blood, decreasing the risk of sclerotic diseases.
Erucic acid (C22:1(13c), c = cis) from cabbage, mustard and some rape species harms the heart muscles of mammals. Rape
oils containing more than 2% erucic acid are only used industrially (production of biofuels).

Table 74.2. Selection of unsaturated fatty acids (red names for essential acids)
formula common name abbreviation family m.p. [°C]
acid
18 9 1
CO2H oleic C18:1(9c) 9 12
1 9
1
9
1 CO2H elaidic C18:1(9t) 9 46
18 9 1
22 13
CO2H erucic C22:1(13c) 9 33.5
1 9
16 9 1
CO2H palmitoleic C16:1(9c) 7 0.5
1 7
12 9 1
1 CO2H linoleic C18:2(9c,12c) 6 5
6
18
14 11 8 5 1
1 CO2H arachidonic C20:4(5c,8c,11c,14c) 6 49
6
20
18 3 15 12 9 1
CO2H -linolenic C18:3(9c,12c,15c) 3 14
1

74.2.2 Waxes, Soaps, Biofuel


Esters of fatty acids (C16 - C36) with long-chain alkanols (C16 - C36) are major components of naturally abundant waxes in
several animal and plant products, exemplified by beeswax.
O myricyl palmitate (palmitic acid myricyl ester) from beeswax

Soaps are the sodium or potassium salts (hard or soft soaps) of fatty acids, produced by hydrolysis (saponification) of
trigylcerides with aqueous sodium or potassium hydroxide and used as tensides for cleaning. Soap anions have a lipo-
philic (hydrophobic) hydrocarbon end and a hydrophilic (lipophobic) carboxylate anion end; they "dip" their lipophilic
hydrocarbon ends into an oily dirt particle so that the anionic ends jutting out at the surface make the dirt particle hydro-
philic as a whole. As a result, the dirt can be washed off with water.
glycerol tristearate (component of fats) O
C hydrophilic (lipophobic) end
O O CH2
hydrophobic (lipophilic) end O HO CH2
C + 3 NaOH C
3 O Na + HO CH
O O CH

C HO CH2
O CH2
sodium stearate (component of hard soap) glycerol

Acid-catalyzed transesterification of the triglycerides of rape oil with methanol to yield the methyl esters of fatty acids is
industrially applied to produce the so-called rape oil methyl esters (RMEs) as substitutes of or additives to diesel oil
(biofuel as renewable energy source).
H3C (CH2)16 CO O CH2 [H ]
HO CH2
H3C (CH2)14 CO O CH + 3 CH3OH HO CH + 2 H3C (CH2)14 CO2CH3 + H 3C (CH2)16 CO2CH3
H3C (CH2)14 CO O CH2 HO CH2
triglyceride of palmitic and stearic acid (component of industrial rape oil) glycerol palmitic and stearic acid methyl esters (components of rape oil methyl esters)

________________________________________________________________________________________________
Chapter 74 permits answers to the following:
(74.1) (a) Explain the term lipid. (b) What are fats and which kinds of other lipids do you know? (c) What are RMEs?
(74.2) Which compound is described by the abbreviation C18:2(9c,12c)? Draw the formula. What is an 6 fatty acid?
(74.3) Suggest a test tube reaction to distinguish a saturated from an unsaturated fatty acid. Look at Chapter 16.2.
(74.4) What is the difference between a fat and a wax? How are soaps related to fats and how does a soap clean?
218 75 Polyketides

75 Polyketides
Polyketides are biosynthesized by microorganisms (bacteria), fungi and higher plants from reactive acetate units (C2).
Therefore, they are also referred to as acetogenins. Depending on the number n of acetate units, they are subdivided into
di-, tri-, tetra-, penta- or n-ketides. The term polyketides followed from the erroneous hypothesis that these compounds
arise from oligomerization of ketene (CH2=C=O).

75.1 Polyketide Pathway


In fact, biosynthesis of polyketides, referred to as the polyketide pathway (Fig. 75.1), starts with activated acetic acid and
activated malonic acid. Activated acetic acid (abbreviated acetyl-S-CoA) and activated malonic acid (abbreviated malo-
nyl-S-CoA) are thiolic acid esters (Chapter 54.5.2) of a coenzyme. The coenzyme is attached to a protein complex which
provides all enzymes required for the biosynthesis, called the acyl carrier protein ACP.
Enzymatic CLAISEN ester condensation of activated malonic acid (malonyl-S-CoA) as nucleophile to the electrophilic
carbonyl of activated acetic acid (acetyl-S-CoA) on the carrier protein ACP is the first step 1 (Fig. 75.1). Activated aceto-
acetate is formed by elimination of the carrier protein from the activated acetic acid unit (2), followed by decarboxylation
(3). Subsequent CLAISEN condensations with activated acetic acid successively lengthen the chain to yield an activated
polyketo acid (4, Fig. 75.1).
When the keto function of the polyketo acids is reduced by specific enzymes before each prolongation step, a fatty acid is
formed, beginning with butyric acid.
In the presence of other enzymes, the activated polyketo acids cyclize to give macrocyclic lactones (macrolides) or
undergo intramolecular carbonyl-methylene reactions (KNOEVENAGEL carbonyl alkenylations) to provide hydroxylated
mono- and polycyclic aromatic compounds such as phenols, and naphtho- and anthraquinones (Table 75.1).

O O
acetyl-S-
CoA H H 2C SCoA H 3C SACP
ACPS O O O
+ HCO3  H 2O H 3C SACP H 3C SACP
ACP 1 O  ACPS
SCoA SACP
O O 2 O O
malonyl-S- O O
CoA 3 + H3O  H2O,  CO2
O O O O
oxo-enol
O O O O O tautomerism OH O
+ AcSACP
H 3C n SACP  HSACP H 3C SACP H 3C SACP
n
activated polyketo acid (polyketide) 4 acetoacetyl-ACP

Fig. 75.1. Polyketide pathway (steps 1-3 are identical with the first steps of fatty acid biosynthesis )

Enzymatic polyketide synthesis cannot be readily imitated in vitro because aliphatic polyketo acids and their esters with
more than three unprotected carbonyl groups are too reactive.

75.2 Selected Polyketides


Acetoacetic acid represents a diketide. It is the parent compound of the mycotoxin moniliformin produced by some
species of mold (e.g. Fusarium moniliforme) which develop on fruits and corn. It occurs as the potassium salt of
semisquaric acid with a remarkably strong acidity (pKa = 0.88).
OH O OH
O HO O
O CH 3
O
HO CH 3 K O O HO O HO OH
O SACP
acetoacetic acid moniliformin squaric acid (synthetic) phloroglucinol

Efficiently Studying Organic Chemistry: Exam training for chemists, biochemists, pharmacists, life and health scientists,
Third Edition. Eberhard Breitmaier. © 2022 WILEY-VCH GmbH. Published 2022 by WILEY-VCH GmbH.
75.2 Selected Polyketides 219

Phloroglucinol (1,3,5-trihydroxybenzene) is a triketide produced by Streptomyces cultures. It occurs freely in Acacia and
Eucalyptus species and as a subunit of several natural products used for tanning.
The toxic piperidine alkaloid ()-coniine of poison hemlock (Conium maculatum) turned out to arise from a tetraketide by
enzymatic reduction, reductive amination by the amino acid alanine, and cyclization (Table 75.1). 4-Hydroxy-6-n-
pentylsalicyclic acid, known as olivetolic acid, is another tetraketide built up from activated caproic acid and three acetate
units (Table 75.1). Olivetolic acid and the monoterpene (Z)-geranyl diphosphate (Chapter 76.3.1) are the biochemical
precursos of the tetrahydrocannabinolic acids abundant in the drugs prepared from Indian hemp Cannabis sativa var.
indica. Cannabis drugs include marihuana, fermented dried parts of the plant, and hashish, the resin secreted by the
glands in the flowering tops of female plants. Tetrahydrocannabinols (THCs) differing in the position of their double bond
(8- and 9-) are the hallucinogens, formed by decarboxylation of the genuine acids upon ageing or smoking the drugs.

OH OH 8
9
10 OH
H 3 H 7
H 10a
CO2H
1
6a 2
H H H
8 6 5 4 3
 CO2
O O O
 - 9 - tetrahydrocannabinolic
  acid - 9 - THC
  - 8 - THC
 

monoterpene subunit is shown in red tetrahydrocannabinols (THC)

5-Hydroxy-1,4-naphthoquinone, named juglone because of its natural occurrence (Juglans regia) and isolated from green
walnut shells, is a pentaketide (Table 75.1) which inhibits the growth of other plants near the tree.
2,4,6-Trihydroxy-9,10-anthraquinones are octaketides kown as emodins (Table 75.1). The 8-methyl derivative occurs in
alder buckthorn and rhubarb root (rhizome). Senna leaves and Aloe species are the sources of the 8-hydroxymethyl
derivative. Emodins are used as cathartic agents.

Table 75.1. Selection of polyketides (subunits not originating from the polyketide path are shown in red)
ACPS O H
O O O
O O N H N
O O H2N

-coniceine ()-coniine
tetraketides
O SACP OH OH
OPP
H
O CO2H CO2H
O (Z)-geranyl diphosphate H
O HO O
from caproic acid (hexanoic acid) olivetolic acid 9-tetrahydrocannabinolic acid
pentaketides O O
O SCoA HO OH
O O juglone

O O OH O OH O
octaketides O
O CH3 HO R R = CH3 : emodin from alder buckthorn
and rhubarb root
O O O
SCoA R = CH2OH : emodin from Aloe species
and Senna leaves
O O O O OH O OH

________________________________________________________________________________________________
Chapter 75 permits answers to the following:
(75.1) Polyketide is a misleading term. Why? Explain your reasoning by briefly formulating the polyketide pathway.
(75.2) Explain the biogenetic relationship of polyketides and fatty acids.
(75.3) What is the smallest polyketide? Which biologically active tetraketides do you know?
(75.4) How are the benzenoid rings of naturally occurring aromatic compounds and quinones formed?
220 76 Terpenes

76 Terpenes
76.1 Survey, Isoprene Rule
The term terpenes comes from turpentine (Lat. balsamum terebinthinae), the sticky, viscous balm which flows upon
cutting the bark and wood of pine trees, and which radiates a pleasantly smelling blend of hydrocarbons.
Terpenes comprise hydrocarbons, alcohols, ethers, aldehydes, ketones, carboxylic acids, esters, and amines, predomi-
nantly biosynthesized by plants, built up from methylbutane or isoprene units (C5, isoprene rule) and therefore referred to
as isoprenoids. Depending on the number n of C5 units they contain, the about 30 000 terpenes documented so far are
subdivided into hemi- (C5), mono- (C10), sesqui- (C15), di- (C20), sester- (C25), tri- (C30), tetra- (C40), and polyterpenes
(C5)n > 8 , as shown in Table 76.1.

Table 76.1. Parent hydrocarbons of terpenes (2-methylbutane units are black, connecting bonds are red and blue)

2-methyl-1,3-butadiene
C5 hemi-
head
tail methylbutane (unit) (isoprene)

C10 mono- 2,6-dimethyloctane

2,6,10-trimethyldodecane
C15 sesqui- (farnesane)

C20 di- 2,6,10,14-tetramethylhexadecane


(phytane)

tail
C25 sester- 2,6,10,14,18-pentamethyleicosane
head

tail
2,6,10,15,19,23-hexamethyltetracosane
C30 tri- (squalane)
tail

tail
C40 tetra- lycopene
tail

all-trans-polyisoprene
(C5)n polyterpene
gutta-percha)
n

The isopropyl part of methylbutane is defined as the head, and the ethyl residue as the tail. Then, looking at Table 76.1,
tri- and tetraterpenes are formed with one tail-to-tail bond (blue) in the center of the molecules. Other bonds linking the
isoprene units in these and in all other terpenes are head-to-tail connections (red). Rearrangements and degradations
(lost carbon fragments) may conceal the isoprene rule at first glance.
One, two, three, and more bonds may connect two isoprene units, exemplified by acyclic monoterpenes linking two
isoprene units by one bond (red) to give 2,6-dimethyloctane; a second bond (green) closes a ring in monocyclic mono-
terpenes such as p-menthane and eucarvane; a third bond (green) generates bicyclic monoterpenes such as thujane and
camphane.
acyclic monocyclic red: head-to-tail linkage to bicyclic
give the monoterpene
green: cyclization

camphane
2,6-dimethyloctane p-menthane eucarvane thujane (bornane)

The nomenclature of terpenes and their hydrocarbon parent skeletons is related to their natural origin. Camphane, the
parent hydrocarbon of camphor, is derived from the Latin name Cinnamomum camphora (Lauraceae) of the camphor
tree; p-menthane, the parent hydrocarbon of the impact compounds shaping the refreshing odor of peppermint oil, relates
to the Latin name Mentha piperita (Labiatae) of peppermint.

Efficiently Studying Organic Chemistry: Exam training for chemists, biochemists, pharmacists, life and health scientists,
Third Edition. Eberhard Breitmaier. © 2022 WILEY-VCH GmbH. Published 2022 by WILEY-VCH GmbH.
76.2 Occurrence, Significance 221

76.2 Occurrence, Significance


Many plants or parts of them (roots, stems, leaves, blossoms, fruits, seeds) are known to have pleasant odors, to taste
bitter or spicy, or to exhibit attractive pharmacological activities. Terpenes predominantly shape these properties. In order
to enrich the terpenes, plants are wounded, for example for the production of turpentine from pines, frankincense and
myrrh from balm trees of the species Boswellia and Commiphora. Mostly, terpenes are extracted or steam-distilled from
the plant material. Extracts and distillates, known as ethereal or essential oils, are used as fragrances to create perfumes,
to refine flavors of food and beverages, or to produce medicines of plant origin (phytopharmaceuticals) and substances
active in plant protection.
Plants biosynthesize volatile terpenes to attract insects for pollination, bitter-tasting or numbing terpenes to repel animals
from eating those plants (antifeedants), and others as growth regulators (phytohormones). Many insects metabolize
terpenes they receive in plant food to produce pheromones (Greek phero = to carry) for communication in order to warn
(alarm pheromones), to mark the location of food resources or for assembly (trace pheromones), and to attract mates for
copulation (sexual pheromones). Harmless to the environment in small amounts, some of these terpenoid pheromones
are used to trap harmful and damaging insects such as the bark beetle.

76.3 Selected Terpenes (Flavors, Fragrances, Active Substances)


76.3.1 Hemi- and Monoterpenes
Apart from hemiterpenoid subunits found in lysergic acid and derived alkaloids (Chapter 70), about 30 hemiterpenes are
known. Some of these are acids (isovaleric, 3-methylbutenoic, tiglic, angelic acid) which are esterified in natural products.
2-Methyl-3-buten-2-ol belongs to the sexual pheromones of the bark beetle, and the isomeric (S)-3-methyl-3-buten-2-ol is
found in the ethereal oils of oranges and hops. A hemiterpenoid thiol is responsible for the flavor of blackcurrant (Chapter
54.2.1).

CO2H CO2H
HO2C HO2C
HO H OH
isovaleric acid 3-methyl-2-butenoic acid tiglic acid angelic acid 2-methyl-3-buten-2-ol (S)-3-methyl-3-buten-2-ol

Acyclic, monocyclic, bi- and polycylic monoterpenes (C10) shape the flavor of ethereal oils. Monoterpenoid subunits
contribute to the structures of Cannabis hallucinogens (Chapter 75.2) and polycyclic indole alkaloids (Chapter 70.2).
The acyclic monoterpenoid triene -myrcene, but not the -isomer, is found in the oils of bay leaf, hop, and citrus. The
isomeric monoterpenols linalool, geraniol (trans), and nerol (cis) are flowery smelling components of several ethereal oils
(lavender, linaloe, rose). Citral is a mixture of the monoterpenoid aldehydes geranial (trans) and neral (cis) shaping the
flavor of citrus and lemongrass and used in perfumery (citrus effect). Geranial is also a trace and alarm pheromone of the
honeybee.
9 OH
3 1
4 2 OH O
5
6 OH O
7
10 8
- -
myrcene rac. linalool geraniol (E) geraniol (Z) = nerol geranial (E) neral (Z)

Monocyclic terpenes are predominantly derived from p-menthane; the trans-isomer is found in turpentine. Limonene,
arising from [4+2]-cycloaddition of isoprene (Chapter 21.4), is a terpenoid cyclohexene occurring in various ethereal oils;
(R)-(+)-limonene with an odor like mandarines and oranges is the dominant component of mandarin peel oil; the (S)-()-
enantiomer, concentrated in the oil of fir cones, also smells like oranges but with a strong turpentine touch.
With three stereogenic centers, p-menthan-3-ol has 23 = 8 stereoisomers which are four pairs of enantiomers. The levoro-
tatory (1R,3R,4S)-enantiomer named ()-menthol is the major component of the peppermint oil obtained from the fresh-
flowering plant Mentha piperita (Labiatae). It smells and tastes sweetish-minty, and is fresh and strongly cooling, while
222 76 Terpenes

the ()-enantiomer smells and tastes herby-minty and is weakly cooling. Therefore, ()-menthol is widely used as a
fragrance in perfumery and for flavoring in confectionery. It has mildly anesthetic, antipruritic, antiseptic, cooling, carmina-
tive, and gastric sedative actions, and therefore is used in various phytopharmaceuticals, for example in nasal inhalators.

7 red: isoprenoid head-to-tail connection


green: cyclization OH
1 R
6 2
5 3 R
4 R S
OH
8
9 10 trans-p-menthane (R)-() - limonene () - menthol

Pinane, a bicyclo[3.1.1]heptane, and the isomeric camphane, a bicyclo[2.2.1]heptane also known as bornane, are the
parent hydrocarbons of naturally abundant bicyclic monoterpenes. Regioisomeric- and -pinene occur in conifers and
are the dominant components of turpentine oil industrially obtained from wood as the raw material. Dextrorotatory 2-
camphanone, known as ()-camphor and originally produced by steam-distilling the crushed wood of adult camphor trees
Cinnamomum camphora (Lauraceae), is the most significant camphane derivative. It also occurs in other plants, including
rosemary and sage, giving off the typical camphoric-like odor of spherical molecules, and has found versatile application
due to its analeptic, topical analgesic and antipruritic, antiseptic, and counterirritant properties. Cellulose nitrate, the nitric
acid ester of cellulose (Chapter 72.2), is plasticized by camphor to yield a semisynthetic polymer known as celluloid.

10 10
9 1
1 3
7
9 O O
7
5 3
5 (+)- ()-
pinane (+)--pinene (+)- -pinene camphane (bornane) camphor

76.3.2 Sesquiterpenes
Farnesane, naturally occurring in slate oil, is the parent hydrocarbon of about 10 000 sesquiterpenes (C15). The tetraene
-farnesene with two trans-configurated double bonds occurs as a component of flavors and natural coatings of apples
and other fruits. Flowery smelling terpenols such as (S)-(+)-nerolidol in the oil of neroli from orange flowers and farnesol
from other ethereal oils (bergamot, jasmine, lily of the valley, rose) are used in perfumery. ()-Zingiberene from ginger
rhizomes is only one of numerous mono- and polycyclic sesquiterpenes.
13 15 red: isoprenoid head-to-tail connections H
11 9 7 5 3 1
green: cyclization
farnesane
HO

OH
(E,E)--farnesene (S)-(+)-nerolidol farnesol ()-zingiberene

76.3.3 Diterpenes
About 5 000 diterpenes known to date are derived from phytane; its (3R,7R,11R)-enantiomer has been found in meteo-
rites, sediments like slate oil, and in the human liver. Chlorophyll in chloroplasts (Chapter 67.2.2) is an ester of (+)-
(2E,7R,11R)-2-phyten-1-ol, known as phytol. Vitamins of the A series are derived from 10,15-cyclophytane, including
retinol and retinal (vitamin A alcohol and aldehyde). trans-cis-Photoisomerization of the aldehyde attached to a protein in
the rods of the eyes induces the nerve impulse during the visual process.

17 18 19 20

15 13 11 9 5 3
OH
7 1 11 7 1 OH
(3R,7R,11R)-phytane (+)-(2E,7R,11R)-2-phyten-1-ol (phytol) retinol (vitamin A alcohol)
76.3 Selected Terpenes (Flavors, Fragrances, Active Substances) 223

76.3.4 Triterpenes
Approximately 5 000 naturally abundant triterpenes documented so far are derived from squalene. Arising from two far-
nesane units linked in the tail-to-tail manner (Chapter 76.1), squalene occurs in cod-liver oil and in several vegetable oils
such as those of rape- and cottonseed.
21
20
27
23
11 13 17
19 30
26
1
10 18
HO 3 5 7
2
O HO 3 -hydroxydammara-
H 20,24-diene
2,3-epoxysqualene (protonated) 29 28

25 26 27
tail 15 19 23
2 6 10
tail 28 29 30
squalene
(2,6,10,15,19,23-hexamethyl-2,6,10,14,18,22-tetracosahexaene)
HO
gonane H lanosterol

Protonated 2,3-epoxysqualene is the biosynthetic precursor of polycyclic triterpenes. Enzymatic tetracyclization gene-
rates 3-hydroxydammara-20,24-diene incorporating the tetracyclic cyclopentanoperhydrophenanthrene subunit known
as gonane. This core structure is also present in steroids (Chapter 77) which all arise from biological modification and
degradation of lanosterol, abundant in the wool fat of sheep.

76.3.5 Tetraterpenes (Carotenoids)


About 200 naturally occurring tetraterpenes are referred to as carotenoids because all of them are structural variants of -
carotene from carrots. Carotenoids occur in bacteria, in fungi, in all leaves, and in some roots, flowers or fruits of plants,
for example the acyclic red tetraterpene lycopene from rosehips and tomatoes Lycopersicon esculentum (Solanaceae).
They are precursors of vitamin A, and also used for coloring food and as antioxidants.

lycopene (red) -carotene (orange-red)

76.3.6 Polyterpenes
Polyterpenes contain more than eight isoprene units. Natural caoutchouc from the Brazilian rubber tree Hevea brasi-
liensis (Euphorbiaceae) turned out to be cis-polyisoprene and was used for production of rubber (Chapter 58.2) before
synthetic isoprene became available on a large scale. trans-Polyisoprene is the major constituent of gutta-percha, the
dried exudate of Southeast Asian Palaquium trees (Sapotaceae), which can be formed to give containers resisting ag-
gressive chemicals such as hydrogen fluoride.
cis-polyisoprene trans-polyisoprene
(natural caoutchouc) (gutta-percha)
n n

________________________________________________________________________________________________
Chapter 76 permits answers to the following:
(76.1) What is a terpenoid structure? Which classes of terpenes do you know?
(76.2) Suggest a synthesis of limonene (Chapter 21.4). Specify which stereoisomers are formed.
(76.3) How many diastereomers and enantiomers of menthol exist? Draw the structures and specify configurations.
(76.4) What is squalene? How is it related to tetracyclic triterpenes and to steroids? Try to formulate.
(76.5) What is a carotenoid? What causes the color of carotenoids? Look again at Chapter 66.1.
224 77 Steroids

77 Steroids
77.1 Survey, Fusion of the Rings
Biogenetically arising from the triterpene lanosterol and also containing the tetracyclic gonane as the core structure
(Chapter 76.3.4), steroids are modified isoprenoids. A cis- or a trans-fusion of each of the three pairs (AB, BC, and CD) of
the six- and five-membered rings (A-D) in gonane is possible so that 23 = 8 options of fusion exist. Only two of those are
realized in naturally abundant steroids: All rings are trans-fused in the 5-series (trans-trans-trans), while rings A and B
are cis-fused in the 5-series (cis-trans-trans), as shown in Fig. 77.1. Bile acids are representatives of the 5-series; all
other steroids belong to the 5-series.

H3C18 R H 3C R
2 gonane 2
H CH3 H CH 3
R1 H R2 C R1 H R2
11 13 17
R1 9H
C D R1 H D
15
1 H
10
3 5 H 7 H H H A B A
5
H H H H
B
H
H () 5-series (trans-trans-trans) H () 5-series (cis-trans-trans)
parent skeleton R1 R2 active compounds
5-estrane H H estrogens (female sexual hormones)
5-androstane CH3 H androgens (male sexual hormones)
5-pregnane CH3 C2H5 pregnanes (corpus luteum and adrenal cortex hormones)
5-cholane CH3 CH(CH3) CH2 CH 2 CH3 cholanes (bile acids)
5-cholestane CH3 CH(CH3) CH2 CH 2 CH2 CH(CH 3)2 cholestanes (sterols)

Fig. 77.1. Parent skeletons of naturally abundant steroids and fusion of their rings

77.2 Cholesterol
Degradation by enzymatic demethylation (1), involving oxidation of methyl groups to carboxylic acids and decarboxyla-
tion, enzymatic dehydrogenation (2), and enzymatic hydrogenation (3) are the biosynthetic steps for the conversion of the
triterpene lanosterol (C30) into the steroid cholesterol (C27).

(1) (2) (3)


HO HO HO HO
H (+)-lanosterol H ()-cholesterol

Discovered as the principal constituent of gallstones in 1769, the compound was named cholesterol and classified as a
steroid (from Greek chole = bile, stereos = solid, eidos = shape). Cholesterol contributes (with other lipids, Chapter 74.1) to
the formation of biological membranes in all cells and is therefore found not only in butter and egg yolk but also in body
tissues, particularly in brain and spinal cord, and as deposits arising from arteriosclerosis in blood vessels. The human
body contains about 250 g of cholesterol.

77.3 Bile Acids


Bile acids are carboxylic acids derived from the C24 parent steroid 5-cholane. They occur in conjunction (as amides) with
the amino acids glycine and taurine (2-aminoethanesulfonic acid) in the bile of vertebrates.
bile acids R1 = R2 = OH : ()-cholic acid
R2 (3,7,12-trihydroxy-5-cholan-24-oic acid)
CO2H
12 R1 = H , R2 = OH : (+)-deoxycholic acid
H (3,12-dihydroxy-5-cholan-24-oic acid)
R1 = OH , R2 = H : (+)-chenodeoxycholic acid (chenodiol)
3 5
H 7
H (37-dihydroxy-5-cholan-24-oic acid)
HO R1 R1 = R2 = H : (+)-lithocholic acid
H ()
(3-hydroxy-5-cholan-24-oic acid)

Efficiently Studying Organic Chemistry: Exam training for chemists, biochemists, pharmacists, life and health scientists,
Third Edition. Eberhard Breitmaier. © 2022 WILEY-VCH GmbH. Published 2022 by WILEY-VCH GmbH.
77.4 Steroid Hormones 225

Human bile (gall) contains cholic, deoxycholic, and chenodeoxycholic acid as principal components in the ratio 1:2:2 and
minor amounts of lithocholic acid. These acids are produced by degradation and modification of cholesterol in the liver,
stored in the gallbladder, and discharged into the duodenum, acting as emulsifiers of triglycerides (fats) in the process of
digestion.

77.4 Steroid Hormones


Pregnanes (C21), biogenetically arising from oxidative degradation of the side chain at ring D of cholesterol (C27, Chapter
77.2), comprise the hormones secreted by the corpus luteum and adrenal cortex. An enone subunit in ring A and a car-
bonyl group in the side chain at ring D characterize this group of steroids. Progesterone formed via pregnenolone is the
gestagenic (Lat. gestare = to carry) female sexual hormone secreted by the corpus luteum during the second half of the
menstrual cycle. Pregnancy results in continued secretion of the antiovulatory (contraceptive) progesterone. Corti-
costerone and cortisone, biosynthesized from cholesterol in the adrenal cortex and referred to as corticoids (Lat. cortex =
bark), up-regulate glucose concentration in the blood and suppress allergic and inflammatory reactions. Therefore, vari-
ous semisynthetic cortisone derivatives are produced industrially as antiallergic, anti-inflammatory, and antirheumatic
medicines.
O O O O

HO OH O OH OH
H biosynthetic H biosynthetic H H

H H H H H H H H
HO O O O
3-hydroxy-5-pregnen-20-one 4-pregnene-3,20-dione 11,21-dihydroxy-4-pregnene-3,20-dione 17,21-dihydroxy-4-pregnene-3,11,20-trione
(+)-pregnenolone (+)-progesterone (+)-corticosterone (+)-cortisone

Apart from gestagenic progesterone (C21), estrogenic (Greek oistráo = to stimulate) female sexual hormones derived from
the estrane skeleton (C18) with a benzenoid ring A containing a phenolic OH group at position 3, and male androgenic
hormones with the 5-androstane skeleton (C19) have been isolated from urine. Among the estranes, estradiol is the
strongest estrogenic female hormone, secreted by the ovaries to induce growth of the uterine mucous membrane and
ovulation during the first half of the menstrual cycle. Among the androstanes, the enone testosterone, biosynthesized
from progesterone in the testes, is the principal male androgenic sexual hormone.
estrogens O OH androgens O OH
18 18

H H R 19 H H

H H H H H H H H
HO HO HO O
H
(+)-estrone R = H : (+)-estradiol (+)-androsterone (+)-testosterone
3-hydroxy-1,3,5(10)-estratrien-17-one R = OH: (+)-estriol 3-hydroxy-5-androstan-17-one 17-hydroxy-4-androsten-3-one

Chemically modified estrogenic and gestagenic hormones such as 17-ethynylestradiol and norgestrel are combined to
oral contraceptives. Esters of 19-nortestosterone (without C-19 = CH3) are used as anabolic steroids.
17-ethynylestradiol norgestrel 19-nortestosterone O
(chemically modified H (chemically modified H3C H decanoate (nandrolone
OH C gestagen) OH C decanoate, anabolic) O
estrogen) C C
H D H H H H

A H H H H H H
HO O O

________________________________________________________________________________________________
Chapter 77 permits answers to the following:
(77.1) (Almost) all steroids possess a hydroxy or carbonyl function at position 3. Why?
(77.2) Briefly formulate the genesis of cholesterol in mammals.
(77.3) Which steroid hormones do you know and what are their biological functions?
(77.4) Look at Chapter 49.3.1 to suggest a partial synthesis of 17-ethynylestradiol, a component of contraceptives.
226 78 Selectivity and Specificity of Organic Reactions

78 Selectivity and Specificity of Organic Reactions


78.1 Chemoselectivity
An organic reaction is said to be chemoselective when the reacting compound contains two or more functional groups,
and only one of these reacts (predominantly).
Arylamines (anilines) are stronger nucleophiles than phenols. Thus, p-aminophenol chemoselectively reacts with acetic
anhydride to produce the analgesic p-acetylaminophenol (paracetamol) as the major product, along with only small
amounts of p-aminophenyl acetate.
H
O O
NH 2 NH 2 N
O
O O O O
O
O  CH3CO2H HO  CH3CO2H HO
p-aminophenyl acetate (minor) p-aminophenol p-acetylaminophenol (major)

Triphenylmethyl chloride (trityl chloride) chemoselectively alkylates the sterically less hindered primary alcohol function
(C-5) and not the secondary alcohol (C-3) of the nucleotide 2-deoxycytidine. The non-conjugated carbonyl function (C-7)
of 6-methylbicyclo[4.4.0]dec-1-ene-3,7-dione is chemoselectively ketalized by glycol because resonance of the enone
carbonyl function (C-3) decreases its electrophilicity.

2-deoxycytidine NH2 5-O-trityl-2-deoxycytidine NH2


O O O
N + (C6H5)3CCl N + HO OH
5 base 7 p-TosOH
HO N (C6H5)3CO N
O O 3
O 1  HCl O  H2O
O O
3 2
HO HO 6-methylbicyclo[4.4.0]dec-1-ene-3,7-dione

Only one of the two acid chloride functions of phosgene reacts with benzyl alcohol to give benzyl chlorocarbonate as the
monoester, and not dibenzyl carbonate, because resonance stabilizes the carbonchlorine bond of the chlorocarbonate
and weakens the electrophilicity of the acid chloride function in the ester chloride.
O O O O

OH + Cl Cl O Cl O Cl X O O
 HCl  HCl
benzyl alcohol phosgene benzyl chlorocarbonate dibenzyl carbonate

In general, there is almost no chance to selectively react only one of two identical functions such as only one of the
carboxy groups in dicarboxylic acids to give a monoester. Provided a cyclic anhydride of the dicarboxylic acid anhydride
is available such as succinic anhydride, however, the pure monoester of succinic acid can be readily prepared.
O
+ C2H5OH CO2C2H 5 + C2H5OH CO2H C2H5OH mixture of
succinic anhydride O X mono-
 H2O and diester
CO2H CO2H
O monoethyl succinate succinic acid

78.2 Regioselectivity
An organic reaction is said to be regioselective when it produces predominantly one of several possible regioisomers.
Examples include bromination of alkanes (Chapter 13.3), eliminations (SAYTZEFF and HOFMANN rules, Chapters 15.1.1,
39.5), electrophilic additions to CC double bonds (MARKOVNIKOV rule, Chapter 16.3), and electrophilic substitutions of
donor- or acceptor-substituted benzenoid rings (Chapter 26.2).

Efficiently Studying Organic Chemistry: Exam training for chemists, biochemists, pharmacists, life and health scientists,
Third Edition. Eberhard Breitmaier. © 2022 WILEY-VCH GmbH. Published 2022 by WILEY-VCH GmbH.
78.3 Stereoselectivity 227

Alkylations of unsymmetrically substituted ketones by alkyl halides are an additional example as strong bases deproto-
nate such ketones to give regioisomeric carbanion-enolate anion systems (Chapter 50.2).
base base
O O O O O
 [H ]  [H ]
R R R R R

Lithium diisopropylamide, LDA, Li N[CH(CH3)2]2, is a bulky base which deprotonates ketones at the less hindered alkyl side
chain, meaning that LDA deprotonates methyl groups more readily than all other alkyl groups (CH3 > CH2R > CHR2). For
example, 2-heptanone is regioselectively alkylated to 1-phenyl-3-octanone when reacted with LDA as base and benzyl
bromide as electrophile.
1. LDA ,  [H ] 1. LDA ,  [H ]
O O O
2. + Br Ph ,  Br 2. + Br Ph ,  Br
Ph

Ph 2-heptanone Ph ==
3-benzyl-2-heptanone (minor) 1-phenyl-3-octanone (major, 87%)

DIELS-ALDER reactions (Chapter 18.2.1) are regioselective when both reactants, 1,3-dienes and dienophiles, are unsym-
metric. These cycloadditions involve a cyclic transition state with a delocalized -electron sextet similar to that of ben-
zenoid rings. Usually, the dienophile is electrophilic (substituted by an acceptor group A such as alkoxycarbonyl), and the
diene is nucleophilic (substituted by an alkyl residue as a weak donor group D) in a "normal" DIELS-ALDER reaction. In
such cases, an "ortho" product will be obtained from a 1-substituted 1,3-diene while a 2-substituted 1,3-diene will react to
yield the "para" product, paralleling the ortho and para directing by donor substituents in electrophilic substitutions of
benzenoid rings (Chapter 26.2).

+ 6 and not
A A A
D D D
1-substituted "ortho" "meta"
transition state
1,3-diene electrons

A A A
+ 6 and not
D D D D A
2-substituted "para" "meta"
1,3-diene

Thus, 2-methyl-1,3-butadiene dimerizes to (R,S)-limonene (Chapter 21.4, "para" adduct); only traces of the regioisomeric
1-methyl-5-(1-methylethenyl)cyclohexene are detectable. Stereoisomers of the analgesic tilidine (an "ortho" adduct) are
produced by [4+2]-cycloaddition of 1-dimethylamino-1,3-butadiene (prepared from crotonaldehyde and dimethylamine) as
nucleophilic diene and ethyl atropate as electrophilic dienophile.

+ and Ph ==
OC 2H 5 CO2C 2H 5 Ph
Ph
Ph CO2C 2H 5
IN(CH 3)2 O (H 3C)2N (H 3C)2N
1-dimethylamino- ethyl atropate cis- trans-tilidine
1,3-butadiene removed by chelation with ZnCl 2 analgesic

78.3 Stereoselectivity
An organic reaction is said to be stereoselective when a reagent selects the more favorable of two stereochemically
different pathways with the same mechanism.
Axial attack of a hydride ion at the carbonyl of 4-t-butylcyclohexanone, for example, produces trans-4-t-butylcyclohexanol
while the cis-isomer arises from an equatorial approach. Thus, reduction of 4-t-butylcyclohexanone with lithium aluminum
hydride (LiAlH4) as a compact complex hydride produces a mixture containing the thermodynamically more stable trans-
228 78 Selectivity and Specificity of Organic Reactions

isomer as major product (70%), while bulky reducing agents such as LiAlH[OC(CH3)3]3 predominantly produce the cis-
isomer (up to 90%) due to steric hindrance of the axial approach by the two axial hydrogen atoms at positions 3 and 5.
equatorial attack + H:
Li AlH4 Li AlH[OC(CH3) 3]3
OH H
H
H H
O OH
trans-4-t-butylcyclohexanol (major) + H: axial attack cis-4-t-butylcyclohexanol (major)
4-t-butylcyclohexanone

Stereoselectivity of DIELS-ALDER cycloadditions with substituted dienophiles is exemplified by the reaction of cyclopenta-
diene with maleic anhydride to give bicyclo[2.2.1]hept-2-ene-5,6-dicarboxylic acid anhydride. The dienophile may ap-
proach from the "inside" (endo pathway) or from the "outside" (exo pathway) of the diene. The activation barrier of the
endo cycloaddition is lower due to stabilizing charge-transfer interaction (Chapter 53.3.1) between diene (donor) and
dienophile (electrophilic acceptor) so that the endo stereoisomer is the (kinetically favored) major product at lower tem-
perature, while the exo product is more stable (thermodynamically favored) and predominates when the mixture is mode-
rately heated. Cycloreversion (retro-DIELS-ALDER reaction, Chapter 21.4) to the reactants ensues at higher temperatures.
O O O
H H
H cyclopentadiene O
25°C 90°C
O H O H O O
O O
H maleic anhydride H H O
H O O O H H
O O
O H O
endo-adduct (major) endo cycloaddition exo cycloaddition exo-adduct (major)
kinetically favored more stable

Nucleophilic additions to carbonyl carbon atoms are stereoselective when the aldehydes or ketones contain a stereogenic
center  to carbonyl. In such cases, CRAM’s rule predicts which diastereomer favorably arises from the addition: The less
bulky substituents at the -carbon flank the carbonyl in the most stable conformer, and the nucleophile favorably adds at
the sterically less hindered side:
O O
small medium small medium
favored
Nu + C C
R Nu R
large large

As an example, the cyanohydrin reaction of (2S)-2-methylbutanal produces (2R,3S)-2-hydroxy-3-methylpentanenitrile as


the major product. Because one diastereomer is favorably formed in this case, the reaction is more precisely specified as
being diastereoselective (Chapter 68.2).
OH O OH
H CH 3 H CH 3 H CH 3
C H IC NI + C C
H CN H NC H
C 2H 5 C 2H 5 C 2H 5
minor major
(2S,3S)- (2S)-2-methylbutanal (2R,3S )-2-hydroxy-3-methylpentanenitrile

78.4 Stereospecificity
An organic reaction is said to be stereospecific when it produces solely one stereoisomer, as dictated by the reaction
mechanism.
Examples of stereospecific reactions include catalytic hydrogenations of alkynes and alkenes (cis specificity, Chapters
15.2.2, 22.3.2), electrophilic bromination of alkenes (trans specificity, Chapters 16.2, 22.3.1), dihydroxylations of alkenes
(cis specificity with osmium tetroxide or permanganate via cyclic esters, trans specificity via epoxides, Chapters 16.7,
22.3.3), and all bimolecular nucleophilic substitutions following an SN2 mechanism (WALDEN inversion, Chapters 31.1,
44.4).
78.4 Stereospecificity 229

Bimolecular dehydrohalogenations (E2, Chapter 15.1.1) of multiply substituted alkyl halides are stereospecific because
the base abstracting the  proton and the halide X as leaving group favor an anti configuration with a minimum of electro-
static repulsion:


X 
X 


...
HO H
HO H


Consequently, base-catalyzed dehydrobromination of 1-bromo-1,2-diphenylpropane with the phenyl groups in the anti
configuration produces trans-1,2-diphenyl-1-propene while the cis-isomer is obtained from the diastereomer with syn
configuration of the phenyl groups.
H H
Ph H H Ph
H Ph E2 , + OH
(1R,2S)-diastereomer
H 3C Ph Ph CH 3  H2 O ,  Br
Ph CH 3
Br Br
(E)-1,2-diphenyl-1-propene
Ph ==
H H
H Ph Ph H
Ph H E2 , + OH
(1S,2S)-diastereomer
H 3C Ph Ph CH 3  H2 O ,  Br
Ph CH 3
Br Br
1-bromo-1,2-diphenylpropane (Z)-1,2-diphenyl-1-propene

Cycloadditions such as cyclopropanations (Chapter 21.1) and DIELS-ALDER reactions (Chapter 21.4) are stereospecific
because their mechanism preserves the relative configuration of the reactants in the product, exemplified by simple [4+2]-
cycloadditions of 1,3-butadiene: Maleic anhydride as dienophile with cis configuration gives cis-cyclohexene-4,5-
dicarboxylic acid anhydride while dimethyl fumarate (dimethyl trans-2-butenedioate) yields trans-cyclohexene-4,5-
dicarboxylic acid dimethyl ester.

1,3-butadiene H O 1,3-butadiene CO2 CH 3


O H H
O
O
O H H3CO2C CO2CH 3
CO2CH 3
H H H H
O
maleic anhydride cis-cyclohexene-4,5-dicarboxylic dimethyl fumarate trans-cyclohexene-4,5-dicarboxylic
acid anhydride acid dimethyl ester

Similarly, 1,4-diphenyl-1,3-butadiene, with two trans double bonds, undergoes [4+2]-cycloaddition with maleic anhydride
to produce cis-3,6-diphenyl-cis-cyclohexene-4,5-dicarboxylic acid anhydride because the hydrogen atoms of the diene
and the dienophile are on the same side in the endo transition state.

Ph Ph
O
Ph H H O
H
H
H endo pathway H
+ O Ph O Ph ==
H O
H O
O O H H O
Ph Ph H
trans-trans- H cis-3,6-diphenyl-cis-cyclohexene
1,4-diphenyl-1,3-butadiene 4,5-dicarboxylic acid anhydride

________________________________________________________________________________________________
Chapter 78 permits answers to the following:
(78.1) Explain (a) chemoselectivity, (b) regioselectivity, (c) stereoselectivity, and (d) stereospecificity. Provide examples.
(78.2) 2-Pentanone and allyl bromide yield 1-octen-5-one when reacted with LDA. Formulate and characterize this reaction.
(78.3) Formulate examples for the (a) regioselectivity, (b) stereoselectivity, and (c) stereospecificity of [4+2]-cycloadditions.
(78.4) (S)-2-Bromobutane yields (R)-2-butanol when reacted with hydroxide. Formulate and characterize this reaction.
230 79 Prochirality, Enantioselectivity

79 Prochirality, Enantioselectivity
79.1 Prochirality of Tetrahedral Carbon Atoms
A tetrahedral C atom with two identical and two different substituents such as one of the methylene C atoms of butane
attached to two H atoms, one methyl and one ethyl group is said to be prochiral. The substitution of one of the two identi-
cal substituents H exemplified by the radical bromination of butane producing 2-brombutane converts the prochiral meth-
ylene C atom into a stereogenic center.
prochiral stereogenic center prochiral stereogenic center
HRe CH3 CH 3 CH 3
+ Br2 (h) + Br2 (h)
H 3C enantiotopic C C C
C methylene H atoms Br  HBr HSi (pro-S)  HBr H Si
H 3C CH 2 H 2C H 2C H2C
of butane H Re H Re (pro-R) Br
CH3 CH 3 CH 3
HSi C2H5 > CH3 > H
butane (S)-2-bromobutane (50%) butane (R)-2-bromobutane (50%)

The two H atoms at the prochiral C atom of butane differ in their spatial environment in the molecule. The environment of
one H atom is the mirror image of the other. Therefore, the identical substituents of a prochiral tetrahedral C atom are
said to be enantiotopic. The specification of two identical groups such as the two H atoms at one of the prochiral C atoms
of butane follows the CAHN-INGOLD-PRELOG convention (Chapter 44.3.1): If the ranking of substituents decreases clock-
wise to one of these H atoms this is specified as Re (HRe) or pro-R; if this happens counterclockwise, the H atom is said
to be pro-S or HSi. The substitution of one of both enantiotopic H atoms [e.g. HSi] by bromine produces one enantiomer
[e.g. (S)-2-bromobutane], the substitution of the other one [HRe] the mirror image [(R)-2-bromobutane].
The reaction of a compound containing a prochiral C atom with a chiral reagent converts the enantiotopic groups into
diastereotopic ones which display different chemical shifts (NMR, Chapter 84). Peptide coupling of N-acetylglycine con-
taining enantiotopic methylene H atoms with L-alanine methyl ester [methyl (S)-2-aminopropanoate] to the dipeptide in
which these H atoms are diastereotopic exemplifies this.

enantiotopic HRe O diastereotopic H Re O


HSi C C H HSi C C H
CH 3 CH3
H3C C N OH + H 2N C H3C C N N C
 H2 O
O H CO2CH3 O H H CO2CH 3
N-acetylglycine L-alanine methyl ester N-acetylglycinyl-L-alanine methyl ester

79.2 Prochirality of Trigonal Carbon Atoms


Trigonal C atoms in aldehydes, unsymmetrically substituted ketones and alkenes (e.g. with different alkyl groups) are
also prochiral. The planes of these alkene and carbonyl groups display two enantiotopic faces which are said to be enan-
tiofacial. These faces are specified by following the CAHN-INGOLD-PRELOG convention: Ranking of substituents decreases
clockwise at the Re face and counterclockwise at the Si face as shown for 2-acetylaminoacrylic acid methyl ester and
butanone.

CH3 CH3 OH 1.) + H: O 1.) + H: OH


+ H2 (kat.) + H2 (kat.) 2.) + H3O 2.) + H3O
C Re C Si C C Re C Si C
H Y Y H H M H2O H2O M H
X Y X E M E
(S)-N-acetyl- X (R)-N-acetyl-
(S)-2-butanol E (R)-2-butanol
alanine methyl ester alanine methyl ester
50 % X = NHCOCH3 >Y = CO2CH3 50 % 50 % E = C 2H5 > M = CH3 50 %
2-acetylaminoacrylic acid methyl ester butanone

79.3 Enantioselectivity
An organic reaction is said to be enantioselective, when it produces predominantly one enantiomer. A chirogenic reaction
which converts a prochiral C atom of the starting reagent into a stereogenic center is not enantioselective under usual
reaction conditions, producing a racemate (Chapter 44.3.4). Radical bromination of butane affords racemic 2-

Efficiently Studying Organic Chemistry: Exam training for chemists, biochemists, pharmacists, life and health scientists,
Third Edition. Eberhard Breitmaier. © 2022 WILEY-VCH GmbH. Published 2022 by WILEY-VCH GmbH.
79.3 Enantioselectivity 231

bromobutane (Chapter 79.1). The bromine radical substitutes the pro-S- and the pro-R-H atom with the same probability.
Catalytic hydrogenation of 2-acetylaminoacrylic acid methyl ester produces racemic N-acetylalanine methyl ester, and
lithiumaluminiumhydride reduces 2-butanone to yield racemic 2-butanol (Chapter 79.2). The hydrogen molecule and the
hydride anion add with the same probability to the Re and the Si face of the double bond and the carbonyl group, respec-
tively.
An enantiomeric starting reagent containing a bulky substituent which prevents substitution of one of the enantiotopic
methylene H atoms permits diastereoselective substitution of a tetrahedral prochiral methylene group, exemplified by the
HARTWIG-SCHÖLLKOPF synthesis of amino acid enantiomers (Chapter 68.2).
Enantioselective hydrogenations of substituted alkenes with prochiral trigonal C atoms in homogeneous phase are
achieved with ruthenium chelates of axially chiral triarylphosphane ligands such as the (aR)- and (aS)-enantiomers of the
BINAP-ruthenium(II)chloride complexes (NOYORI catalysts). The axial chirality (Chapter 45.2) of these ligands shapes the
coordination space of the central ion as depicted by the formula of the acetoacetic acid ester chelate. The substrates form
enantiomeric chelates in which hydrogenation at one of the enantiotopic faces (Re or Si) is energetically favored. As a
result, catalytic hydrogenation is now enantioselective, predominantly producing one enantiomer.

(1,1'-Binaphthalen)-2,2'-diyl-bis(diphenylphosphan) (BINAP) OCH3 H3 CO


+ H2 + H2
O O
Ar2 Ar2
O Ru P P Ru O
Ar2P PAr2 O P P
Ar = O
Ar2P PAr2 O Ar2 Ar2 O
+ H2 + H2
OCH3 H3CO
(aS)-()-BINAP (aR)-(+)-BINAP (aS)-(+)-BINAP-Ru chelate (aR)-(+)-BINAP-Ru chelate
with acetoacetic acid methyl ester as ligand (substrate)

Examples include enantioselective catalytic hydrogenations of acetoacetic acid methyl ester to (R)-(+)-3-hydroxybutanoic
acid methyl ester and of 2-(6-methoxy2-naphthyl)propenoic acid with NOYORI catalysts to yield the analgesic, antiinflam-
matory and antipyretic drug (S)-()-naproxen.

O O

OCH3
CH2 CH3
+ H2 + H2
OH O OH O
(aR)-(+)-BINAP-RuCl2 CO2H (aS)-()-BINAP-RuX2 CO2H
H
OCH3 OCH3 X = Acetat
H3CO H3CO
enol-tautomer of (R)-3-hydroxybutanoic acid
methyl ester (ee > 98 %) 2-(6-methoxy-2-naphthyl) propenoic acid (S)-()-2-(6-methoxynaphth2-naphthyl)-
acetoacetic acid methyl ester propanoic acid (naproxen)

Complex metal hydrides (e.g. LiAlH4) chelated with chiral ligands reduce unsymmetrically substituted ketones (R1R2C=O)
enantioselectively. For example, enantioselective reduction of 2-butanone to (R)-() or (S)-(+)-2-butanol succeeds with
lithiumaluminium hydride chelated with a chiral ligand such as (aR)-(+)- or (aS)-()-BINOL (Chapter 45.2).
The enantiomeric excess (e.e.) specifies the quality of an enantioselective reaction:
[R]  [S]
enantiomeric excess e.e. = % R  % S = 100 [%]
[R]  [S]

Enantiomeric excesses of enantioselective catalytic hydrogenations with chiral catalysts are up to 99 %.


________________________________________________________________________________________________
Chapter 79 permits answers to the following:
(79.1) Explain the term prochirality at a tetrahedral and a trigonal carbon atom.
(79.2) Draw the tetrahedral projection formula of the methylene C of ethanol and assign the enantiotopic H atoms.
(79.3) Formulate a reaction which converts enantiotopic groups into diastereotopic ones.
(79.4) Chirogenic reactions produce racemic mixtures under usual conditions. Why? Formulate examples.
(79.5) Formulate examples of enantioselective catalytic hydrogenations.
232 80 Planning Organic Syntheses

80 Planning Organic Syntheses


80.1 Retrosynthetic Disconnections
Syntheses are generally planned backwards, from the target compound to commercially available reagents. This imagi-
nary procedure is referred to as retrosynthesis. It involves retrosynthetic disconnections of strategic bonds. A retrosyn-
thetic disconnection, symbolized by  , breaks a bond in a target compound in order to derive the starting reagents.
retrosynthetic
disconnections
target compound starting reagents

A retrosynthetic disconnection of a strategic bond is the reverse of a known chemical reaction. Frequently, strategic
bonds originate from carbon atoms which are attached to functional groups. Such disconnections remove or substitute
these functional groups and are referred to as functional group interconversions (FGI), exemplified by the disconnection
of benzyl alcohol to toluene in three steps:

retrosynthetic
CH2 OH
FGI
CH 2 Br
.
CH2
FGI
CH 3
disconnection

benzyl alcohol benzyl bromide benzyl radical intermediate toluene


SN

synthesis CH2 OH
+ OH
CH 2 Br
SR .
CH2 + Br. CH 3 + Br2
 Br  HBr

80.2 Retrons and Synthons


Any carbon-carbon bond can be disconnected to radicals or ions. The homolytic disconnection of a CC single bond to
radicals is the reverse of the KOLBE synthesis of symmetric alkanes (Chapter 5.2.4). The heterolytic disconnection of a
CC single bond to a carbenium ion and a carbanion is the reverse of the alkylation of a GRIGNARD reagent or another
organometal compound by an alkyl or aryl halide (Chapter 32.3).
synthons reagents
homolytic heterolytic     
C C C . + .C C C C + C C Br + Br Mg C C C
 MgBr2

Generalized fragments obtained by retrosynthetic disconnections are referred to as synthons. Synthons may be reagents
or reactive intermediates generated from starting reagents. The reaction behind a disconnection is decoded from a sub-
unit in the target structure which is called the retron according to COREY. Thus, the red substructure of 3-hydroxybutanal
is disconnected to a carbenium ion and a carbanion. These two synthons are resonance-stabilized and arise from two
equivalents of acetaldehyde (ethanal) as the reagent, also referred to as synthetic equivalents, thus decoding the aldol
reaction (Chapter 50.2.1, Table 80.1).

OH H H OH OH H H H H O H
aldol
H3C C C C H 3C C H3C C + C C C C H3C C + H3C C
H H O H H H O H O H O
retron synthons synthetic equivalents (reagents)

Frequently, synthons and synthetic equivalents are the same, exemplified by 1-methylcyclohexene-4-carboxylic acid methyl
ester with the cyclohexene subunit as the retron. Retrosynthetic disconnection directly generates 2-methyl-1,3-butadiene
(isoprene) and methyl propenoate (acrylic acid methyl ester) as the reagents (1,3-diene and dienophile) of a DIELS-ALDER
reaction ([4+2]-cycloaddition, Chapters 18.2.1, 21.4, Table 80.2).
CO2CH3 DIELS-ALDER CO2CH 3
+
H 3C H3C
retron s y n t h o n s ( = reagents)

Efficiently Studying Organic Chemistry: Exam training for chemists, biochemists, pharmacists, life and health scientists,
Third Edition. Eberhard Breitmaier. © 2022 WILEY-VCH GmbH. Published 2022 by WILEY-VCH GmbH.
80.2 Retrons and Synthons 233

Table 80.1 reviews a small selection of retrosynthetic disconnections of different types of CC bonds and the typical reac-
tions generating these bonds.

Table 80.1. Retrosynthetic disconnections of selected CC bonds


type of bond retron synthons reagents r e a c t i o n (s)
CO2R C alkylation of CH
sp3 sp3 C C C + C C X + H C CO2R acidic compounds
(Chapter 51.2.1)
C alkylation of
C X + M C organometallic compounds
(Chapter 32.3)

addition of organometallic
C C C + C C + M C compounds to carbonyl
compounds
OH OH O (Chapters 32.3, 33.3.7)
OH OH O
aldol reaction
C C C C C C + C C C C + H C C (Chapter 50.2.1)
O O O
C alkenylation of
sp3 sp2 C C C + C C M + X C organometallic compounds
(Chapter 32.3)

C arylation of
C C + C M + X organometallic compounds
(Chapter 32.3)

O O OR O CLAISEN
C C C C C C C C C C + H C C ester condensation
(Chapter 50.1.1)
O OR O OR O OR
C alkylation of
sp3 sp C C C C + IC C C X + H C C terminal alkynes
(Chapter 19.2.4)

carbonyl alkynylation
C C C C + IC C C + H C C (Chapter 49.3.1)
OH OH O

cyanohydrin reaction
C C N C + IC N C + H C N (Chapter 49.3.2)
OH OH O
H
-elimination of HX
sp2 sp2 C C C C C C (Chapter 15.1.1)
X
KNOEVENAGEL,
WITTIG and other
C C C + IC C O + H 2C carbonyl alkenylations
(Chapters 49.3.4, 51.2.2)

alkenylation of
C C C + IC C X + M C alkenylmetal compounds
(Chapter 32.3)

O O O FRIEDEL-CRAFTS
C + C H + X C acylation of arenes
(Chapter 25.4)

C alkenylation of
sp2 sp C C C C + IC C C X + M IC C metal alkynylates
(Chapter 32.3)

SANDMEYER reaction
C N + IC N N2 + IC N (Chapter 40.1)

oxidative GLASER coupling


sp sp C C C C C C + IC C C C H + H C C of terminal alkynes
(Chapter 19.3.6)

[2+1]-, [2+2]- and [4+2]-cycloadditions are used to create three-, four- and six-membered rings, respectively. Five-mem-
bered rings arise from cyclodehydrations of hexanedioic acid esters. Ring-closing metatheses produce five-membered
and larger rings. Seven- and eight-membered rings may be the result of COPE rearrangements (Chapter 21.5). Benzenoid
rings are not synthesized but introduced or substituted. Table 80.2 reviews some typical disconnections of rings.
234 80 Planning Organic Syntheses

Table 80.2. Important retrosynthetic disconnections of selected rings

retron synthon(s) reaction(s)


[2+1]-cycloaddition
_
and PRILEZHAEV
CH R + :CH R and O + :O
_ (R CO3H) epoxidation
(Chapters 16.7, 21.1, 60.2.2)

[2+2]-cycloaddition
and PATERNO-BÜCHI
+ and +
O O reaction to give oxetanes
(Chapters 21.2, 60.2.2)

CO2R
CH CO2R DIECKMANN
O CO2R cyclocondensation
(Chapter 50.1.2)

ring-closing
X X X metathesis
(Chapter 21.6)
X = (CH 2)n f or n > 0 or heteroatom

[4+2]-cycloaddition
+ (DIELS-ALDER reaction)
(Chapter 18.2.1)

80.3 Designing Selected Syntheses


The strategic bonds of each synthetic step are selected so that each retrosynthetic disconnection
- reverts a known reaction involving a reasonable mechanism;
- generates synthons corresponding to readily available reagents (synthetic equivalents);
- finally leads to a synthesis which is as short, as simple and as sustainable as possible.

80.3.1 2-Ethyl-2-hexenal
The double bond of 2-ethyl-2-hexenal (1) is the obvious strategic bond around which to design a synthesis. One discon-
nection generates the synthons 3 (phosphorane) and 4 (acetal of 2-oxobutanal to protect the aldehyde function from
reaction with the phosphorane) of a WITTIG carbonyl alkenylation (Chapter 49.3.4). An alternative disconnection gives 2-
ethyl-3-hydroxyhexanal (5), utilizing the concept of dehydration (-elimination) of an aldol to produce an ,-unsaturated
aldehyde. The next disconnection consequently follows the reverse of an aldol reaction, resulting in the resonance-
stabilized ion pair 6 as the synthons. The aldol reaction of butanal followed by dehydration turns out to be the simpler and
more sustainable synthesis, avoiding the expensive preparation of starting compounds 3 and 4 and the disposal of tri-
phenylphosphane oxide.
OR OR
H H
O
OR P(C6H 5)3 + OR
2 3 4

H OH H OH H OH H

O O H + O H + O
 H2O base
1 dehydration 5 aldol 6
reaction

80.3.2 Ethyl 2,4-Dioxoheptanoate


A retrosynthetic disconnection of the C-2C-3 bond in ethyl 2,4-dioxoheptanoate (1) as the target compound generates
the nucleophilic carbanion 2a of 2-pentanone and the electrophilic cation 3a as the synthons. Thus, the synthesis turns
out to be a CLAISEN ester condensation (Chapter 50.2.2, Table 80.1) of 2-pentanone (2) and diethyl oxalate (3) as the
synthetic equivalents corresponding to the synthons.
80.3 Designing Selected Syntheses 235

retrosynthetic disconnection synthesis


O O O NaOC2 H5 O
3 C2 H5 OH
4 2
OC2H 5 + OC2H 5 + O O OC 2H 5
O O O O O O O O

1 2a 3a 2 3 1

80.3.3 2-(4-Isobutylphenyl)propanoic Acid


Obvious precursors of the analgesic and antirheumatic 2-(4-isobutylphenyl)propanoic acid [ibuprofen, (1)] are the magne-
sium halide 2 [carboxylation], the halide 3 [metalation], the secondary alcohol 4 [nucleophilic substitution], the acetophe-
none 5 [hydrogenation] and commercially available isobutylbenzene (6) [electrophilic FRIEDEL-CRAFTS acetylation]. In fact,
a three-step synthesis involves catalytic carbonylation of the alcohol 4 to yield ibuprofen (1).
retrosynthetic disconnection
O
C MgX X OH O
OH

FGI FGI FGI

1 2 3 4 5 6
O
H2 , RANEY-Ni , HF, 80°C
synthesis CO , PdCl2 , P(C6H5)3 O O
(HF recycling)

80.3.4 9-Tetrahydrocannabinol
9-Tetrahydrocannabinol (1, Chapter 75.2) is an analgesic and antiemetic ingredient of marihuana and hashish. Retro-
synthesis begins with a retro elimination (dehydration) 12 to give the precursor 2 for formation of the 9-double bond.
Subsequent disconnections 23 of the C-10aC-10b bond, in a reversion of an electrophilic substitution of the doubly
donor-substituted benzenoid ring, and of the ether bond C-5C-6, in a reversion of a nucleophilic addition of olivetol (4) to
the stable carbenium ion 5a as the synthon, reveal p-mentha-2,8-dien-1-ol (5) as the reagent. In a one-step synthesis,
olivetol (4) and terpenol (5) react in the presence of a LEWIS acid to give the target compound 1.
retrosynthetic disconnection
OH OH OH

8
9 OH OH OH OH
7 H 10a
H
12
H H
6 5 43
O O C5H 11 O C 5H 11 + HO C 5H11
1 2 3 5a 4
OH

BF 3 , O(C2H5)2 OH
synthesis H

+ HO C 5H11
5 4
________________________________________________________________________________________________
Chapter 80 permits designs for the syntheses of the following target compounds:
(80.1) (a) (E)-Butenal and (b) (E)-2-ethyl-2-hexenal;
(80.2) (a) 1-methylcyclohexene-4-carboxylic acid methyl ester and (b) bicyclo[2.2.1]heptane-2-carboxylic acid ethyl ester;
(80.3) (a) ethyl 2,4-dioxoheptanoate, (b) 2,4-pentanedione, and (c) 2-(4-isobutylphenyl)propanoic acid;
(80.4) 9-tetrahydrocannabinol from (R)-p-mentha-2,8-dien-1-ol.
236 81 Aspects of Molecular Structure

81 Aspects of Molecular Structure


81.1 Molecular Formula and Double Bond Equivalents
The molecular formula provides the elemental composition of a compound and is determined by elemental analysis or
mass spectrometry (Chapter 82). It gives additional information concerning the degree of unsaturation reflecting the
presence of multiple bonds or rings. For example, the molecular formula C3H6 contains two hydrogens (H2) less than that
of the comparable alkane (propane, C3H8); this indicates a double bond or, more precisely, one double bond equivalent,
because the molecular formula C3H6, for example, containing this double bond equivalent may belong to propene (double
bond) or to the isomeric cyclopropane (ring). The benzene molecule with C6H6 has eight H atoms less than the hexane
molecule C6H14, corresponding to four double bond equivalents, three for the benzenoid CC double bonds and one for
the ring.
For the commonest heteroatoms in organic molecules (nitrogen, oxygen, sulfur, halogens), the number of double bond
equivalents can be derived from the molecular formula by assuming that nitrogen may be replaced by CH, a halogen may
be replaced by H, and oxygen and bivalent sulfur may be omitted. The resulting empirical formula CnHx is then compared
with the molecular formula of an alkane also having n C atoms, CnH2n+2. The number of double bond equivalents is equal
to half the hydrogen deficit,
number of double bond equivalents = (2n  2  x) / 2.

The molecular formula of 2-chloro-4-nitroacetophenone, as an example, is C8H6ClNO3. Replacing Cl by H, N by CH, and


deletion of O gives C9H8 as the empirical formula CnHx, containing 20  8 = 12 hydrogen atoms less than the alkane C9H20
and thus six double bond equivalents. Four of those belong to the benzenoid ring, while carbonyl and nitro groups ac-
count for the other two.
O O
2-chloro-4-nitroacetophenone N C
O CH 3
Cl

81.2 Skeletal Structure: Atom Connectivities


The skeletal structure of a compound (Chapter 6) specifies the atom connectivities, the linkage between all atoms and the
types of bonds connecting these atoms. Skeletal isomers have the same molecular formula but different atom connectivi-
ties, such as butane and methylpropane (C4H10, Chapter 6); they also comprise regioisomers and functional isomers.
Regioisomers are skeletal isomers which belong to the same class of compounds; they have multiple bonds at different
positions, such as 1- and 2-butene (C4H8, Chapter 14.1), or differently positioned substituents such as the pairs 1- and 2-
bromopropane (C3H7Br, Chapter 13.3) or o- and p-nitrophenol (C6H5NO3, Chapter 26.2). Functional isomers differ in their
functional groups, such as the pairs ethanol and dimethyl ether (C2H6O) or propanal and propanone (C3H6O). Skeletal
isomers have individual physical properties (boiling points, melting points, refractive indices, adsorption coefficients).
Their different atom connectivities can be deduced from their molecular spectra (Chapters 82-86).

81.3 Conformation
The term conformation describes the total of all spatial arrangements of atoms and groups arising from rotation about CC
single bonds; a conformer is one specific arrangement (Chapter 9). Unlike skeletal or configurational isomers, conformers
usually cannot be isolated or stored as such because thermal energy at room temperature induces "free rotation". Strong
steric interactions, however, may stabilize an individual conformer of rings, exemplified by - and -D-glucopyranose
(Chapter 71.3). Spectroscopy (NMR) reveals that they really exist as 4C1 chair conformers (C-4 top, C-1 bottom, C from
chair): upon ring inversion to the 1C4 conformer, most or all substituents would be coaxial, strongly sterically repelling
each other.
4C conformer 1C conformer 4C conformer 1C conformer
1 4 1 4
HO HO OH
HO OH HO OH
4 1 4
O O 1
HO O OH HO O
1 OH
HO 1 4 HO 4
OH OH
OH -D-glucopyranose OH OH -D-glucopyranose OH OH

Efficiently Studying Organic Chemistry: Exam training for chemists, biochemists, pharmacists, life and health scientists,
Third Edition. Eberhard Breitmaier. © 2022 WILEY-VCH GmbH. Published 2022 by WILEY-VCH GmbH.
81.4 Relative Configuration 237

81.4 Relative Configuration


The arrangement of atoms, alkyl groups and other substituents relating to a double bond in alkenes, to the plane of a ring
in cycloalkanes, or to other stereogenic centers in diastereomers is referred to as the relative configuration, specified by
descriptors (cis- = Z- and trans- = E-, RR- and RS-), illustrated by 2-butene, 4-t-butylcyclohexanol, and the diastereomeric
tetroses (Chapters 46.1, 71.1).
2-butene 4-t-butylcyclohexanol D-threose D-erythrose
CH3 H
(H3C) 3C OH
CH=O CH=O
(H3C) 3C OH
H3C H3 C HO C H H C OH
H CH3
H C OH H C OH
H H OH
CH2OH CH2OH
OH (2S,3R) (2R,3R)
cis- = Z- trans- = E- cis- trans-
threo- erythro-

Stereoisomers differing in relative configuration have different physical properties (boiling points, melting points, refractive
indices, adsorption coefficients, molecular spectra).

81.5 Absolute Configuration


The spatial arrangement and ranking of substituents attached to a stereogenic center is called the absolute configuration
and is specified by descriptors [(R)- and (S)- or D- and L-, Chapter 44.3]. Apart from chiroptic characteristics such as the
direction of their specific rotation (dextro- or levorotatory, Chapter 44.2), stereoisomers differing in their absolute configu-
ration (enantiomers) have identical physical properties and molecular spectra (in non-chiral environment).
However, the absolute configuration plays a decisive role in biochemical processes. Enzymes recognize the appropriate
absolute configuration of a substrate (chiral recognition). Thus, only amino acids of the L series are selected by the
enzymes for protein biosynthesis. Photosynthesis by plants (Chapter 67.2.2) produces D glucose and not the L enantio-
mer (enantiodiscrimination). Only ()-LSD (Table 70.2) and not its enantiomer is the strong hallucinogen. Enantiomers of
synthetic drugs act differently or with different strengths, exemplified by (S)-()-ibuprofen which provides much better pain
relief than the (R)-enantiomer [(S):(R) = 150:1]. L-Amino acids taste more or less sweet in contrast to the bitter-tasting D-
enantiomers. Aspartame, the methyl ester of the dipeptide from L-aspartic acid and L-phenylalanine (Asp-Phe) is about
160 times sweeter than sucrose (in water) and used as a non-nutritional sweetener; its enantiomer tastes bitter. The
monoterpenol (S)-citronellol smells like geranium while the (R)-enantiomer radiates the flowery odor of some roses.
ibuprofen aspartame citronellol
O O
S R HO2C CO2H
HO OH
H2N S R NH 2 S R
O O
H N OCH3 H3CO N H HO OH
S R
O O
stronger analgesic sweet bitter geranium fragrant roses

________________________________________________________________________________________________
Chapter 81 permits answers to the following:
(81.1) How many double bond equivalents hide behind the molecular formulas (a) C5H5N, (b) C6H12O6, and (c) C6H7NO2?
(81.2) Give examples of (a) all kinds of skeletal isomers and (b) all kinds of stereoisomers.
(81.3) Which (a) skeletal isomers and (b) stereoisomers exist for C4H8O? Draw and classify all of them.
(81.4) What significance does the absolute configuration have in biochemical processes? Give examples.

Subsequent chapters covering some spectroscopic methods of structure elucidation do not deal with the physical back-
ground. Rather, they provide a very brief strategic introduction to the "decoding" of structures from molecular spectra.
238 82 Mass Spectrometry

82 Mass Spectrometry
82.1 Mass Spectrum
In a mass spectrometer, a small sample of a compound (< 0.1 mg) is vaporized under high vacuum and ionized to frag-
ment ions by electron bombardment or other methods. Electric fields accelerate the fragment ions and magnetic fields
segregate the beam of ions into groups of equal mass to charge ratio (m/z) where z is usually 1 because the ions are
predominantly singly charged. After separation, the ion beams reach a counter which registers the number of ions for
each m/z group. Electronic data processing generates the mass spectrum which plots the relative intensity of each m/z
group due to the abundance of ions in % as a function of m/z, as shown in Fig. 82.1.

100 44 molecular mass (high resolution): 135.1031


molecular formula : C9H13N
80

60
120
%
40

20 91 103
65
136
0
40 50 60 70 80 90 100 110 120 130 140
m/z Fig. 82.1. Mass spectrum to decode the structural formula

82.2 Base Ion, Molecular Ion


Peak is the term for the maximum of an ion beam in the mass spectrum. The highest peak arises from the most abundant
ion called the base ion; it is referred to as the base peak and set equal to 100 %. The intensities of all other peaks are
given in % relative to the base peak, for example with m/z = 44 in Fig. 82.1.
Removal of one electron from a molecule by electron bombardment generates the molecular ion which is a radical cation
formulated as M+. where the superscript period symbolizes the unpaired electron. The m/z value of the molecular peak in
the mass spectrum is the relative molecular mass when the molecular ion is singly charged (the usual case). Thus, when
identifiable in the mass spectrum, the molecular peak gives the accurate molecular weight of the compound.
With the exception of 12C as reference atom (12.000 g/mol), atomic masses are not integer because the elements occur
as mixtures of isotopes. Thus, high-resolution mass spectrometry permits the determination of the elemental composition
of the molecular ion and all other ions detectable in the spectrum. For example, molecules with formulas C6H10O (e.g. he-
xenal) and C7H14 (e.g. heptene) have the same nominal mass of 98. High-resolution measurement of the mass of the mo-
lecular ion, however, gives 98.0732 for C6H10O and 98.1095 for C7H14.
In the same manner, the accurate mass (m/z = 135.1031, relative intensity only 2 %) of the molecular ion in the mass
spectrum of Fig. 82.1 permits the determination of the molecular formula C9H13N of the sample molecule by comparative
computer search of a molecular mass / molecular formula data table. As per Chapter 81.1, the sample molecule contains
four double bond equivalents.
The odd molecular mass (135) reveals an odd number (1,3,5…) of nitrogen atoms in the molecule, because nitrogen
(14N) is the only element with even atomic mass (14) and odd number of bonds (3, trivalent nitrogen, nitrogen rule).
Atomic mass and valence are both even or odd for the predominant isotopes of all other elements.

Efficiently Studying Organic Chemistry: Exam training for chemists, biochemists, pharmacists, life and health scientists,
Third Edition. Eberhard Breitmaier. © 2022 WILEY-VCH GmbH. Published 2022 by WILEY-VCH GmbH.
82.3 Fragment Ions and Atom Connectivities 239

82.3 Fragment Ions and Atom Connectivities


The molecular ion (life span < 106 s) decomposes rapidly to leave fragment ions (Fig. 82.1) from which subunits of the
sample molecule are deducible. Electron bombardment favorably ionizes heteroatoms when present in the sample mole-
cule, knocking out electrons from non-bonding electron pairs, and generating molecular ions with their positive charge
and the unpaired electron at the heteroatom. Such radical cations undergo  fragmentations involving homolytic cleavage
of the  bond, leaving a radical and a cation with positive charge at the heteroatom.

 ionization  +  fragmentation + +
R C X R C X R + C X C X
 e0
molecular ion fragment ion

When the mass spectrum detects all three  fragmentations originating from the  carbon, atom connectivities of the
sample molecule can be reconstructed by recombining the  carbon with the radicals R. abstracted from the molecular
ion.
Masses of alkyl radicals are odd. When the molecular mass is even, masses of ions arising from -fragmentations are
odd; when the molecular mass is odd, exemplified by the nitrogen compound in Fig. 82.1, ions arising from -
fragmentations are recognized by their even masses. In Fig. 82.1, these are the fragment ions with masses 134 (M 1,
loss of an H atom, R = . H), 120 (M 15, loss of a methyl radical, R = . CH3), and 44 (M  91, loss of a benzyl radical, R = .

CH2C6H5). The presence of a benzyl group in the sample molecule is also indicated by the benzyl fragmentation to the
aromatic tropylium ion C7H7+ (m/z = 91), arising from rearrangement of the benzyl cation, and subsequent elimination of
ethyne (C2H2) to give the ion C5H5+ (m/z = 65).
+
 C N CH 2
 C2 H2
+
CH 2 C N + + C 5H 5

m / z = 91 m / z = 65
benzyl cation (C 7H7+) tropylium ion (C 7H7+)

Except for two hydrogens, all atoms of the molecular formula C9H13N are assigned. Reconnection of an H atom, a methyl,
and a benzyl group to the carbon  to nitrogen provides the skeletal structure of 2-amino-1-phenylpropane = -
methylphenylethylamine = amphetamine:
H
amphetamine CH 2 C NH 2
CH 3

Amines are proton acceptors, forming ammonium ions; therefore, the (M  1) ion with m/z = 136 occurs in the mass spec-

trum (Fig. 82.1). The ion arising from  fragmentation of methyl (m/z = 120) eliminates ammonia ( 17), leaving the frag- 

ment C8H7+ with m/z = 103, and  fragmentation of the benzyl radical generates the most abundant ion giving rise to the
base peak with m/z = 44:
H H H
+ C6H5 CH2
+ +
CH 2 C NH 2 C NH 2 C NH 2
CH 3 H3C H 3C m/z = 44

Documented compounds are most frequently identified by computer-assisted comparison of the digitized mass spectrum
with comprehensive data files of reference spectra.
________________________________________________________________________________________________
Chapter 82 permits answers to the following:
(82.1) The molecular formula of a compound can be determined by mass spectrometry. Explain how.
(82.2) Explain how you recognize a nitrogen compound with an odd number of nitrogen atoms in a molecule.
(82.3) Formulate (a) the generation of the molecular ion, (b) an  fragmentation, and (c) a benzyl fragmentation.
(82.4) Comprehend the elucidation of the skeletal structure from Fig. 82.1. Which structural aspect remains unresolved?
240 83 Infrared Spectroscopy

83 Infrared Spectroscopy
83.1 IR Spectrum
Infrared (IR) light, emitted by electrically heated filaments, covers the wavelength range of 800 nm to 1 mm. The energy
of IR light is enough to excite molecular vibrations and rotations so that any compound containing covalent bonds ab-
sorbs in the infrared region. Since glass is opaque to IR, alkali halide optics must be used in infrared spectrometers to
record the infrared absorption (the transmittance) of a sample compound as a function of the wavenumber (Fig. 83.1).
The wavenumber in cm1 is the number of waves per cm. Absorption bands (not lines) are observed in the IR or vibra-
tional spectrum due to unresolved rotational transitions.

Fig. 83.1. IR Spectrum of sample C9H13N

83.2 Molecular Vibrations


Vibrating dipoles are able to interact with electromagnetic radiation. Therefore, molecular vibrations give rise to strong IR
absorptions when they change dipole moments with time (IR-active vibrations).
Two kinds of molecular vibrations exist (Fig. 83.2). Stretching vibrations (symbol ) stretch and contract bonds without
changing bond angles. As shown for a methylene group in Fig. 83.2, symmetric and antisymmetric stretchings with differ-
ent frequencies may occur. Bending vibrations involve deformations of bonds without changing bond lengths; this may
happen in plane (scissoring or rocking, symbol ) or out of plane (twisting or wagging, symbol ).
s t r e t c h i n g vibrations b e n d i n g vibrations
in plane out of plane
H H H H H+ H+
C C C C C C
H H H H H H+

  
symmetric antisymmetric scissoring rocking twisting wagging

Fig. 83.2. Molecular vibrations of a methylene group with symbols  for stretching, and  and  for bending (in plane and out of plane)

 (stretching vibrations) OH
NH
CH
SH
C C , C X, X X
C C C , C C X , X C X (X = O, N)
C X (X = O, N)
C C
X Y (X,Y = O, N, S)
C X (X = O, N)
C X (X = F, Cl, Br, I)
NH
 in plane CH
(bending vibrations) OH
 out of plane NH
CH
wavenumber cm 1 3 500 3 000 2 500 2 000 1 500 1 000 500

Fig. 83.3. Characteristic IR group absorptions due to stretching and bending vibrations in organic molecules

Efficiently Studying Organic Chemistry: Exam training for chemists, biochemists, pharmacists, life and health scientists,
Third Edition. Eberhard Breitmaier. © 2022 WILEY-VCH GmbH. Published 2022 by WILEY-VCH GmbH.
83.3 Identification of Functional Groups and Structural Units 241

83.3 Identification of Functional Groups and Structural Units


Fig. 83.3 presents group absorptions in the IR spectrum due to stretching and bending vibrations, and shows that these
vibrations characterize functional groups and other structural units in molecules. Thus, the two bands at 3370 and 3280
cm1 in Fig. 83.1 arise from antisymmetric and symmetric NH vibrations, indicating the presence of a primary amino group
in the molecule; a secondary amino group would only permit one NH stretching vibration, absorbing at about 3300 cm1.
Another strong band at 1610 cm1 due to scissoring vibration of NH2 (NH2) confirms the primary amino group. No NH
absorption at all is expected from tertiary amines.
In Fig. 83.1, absorption bands between 2850 and 2950 cm1 due to antisymmetric and symmetric CH stretching vibra-
tions indicate CH2 and CH3 groups. Several bands appearing above 3000 cm1 reveal the presence of shorter CH bonds
of alkenyl or aryl residues, which vibrate faster than the longer CH bonds of alkyl groups. The CH bending vibrations
(wagging, CH) appearing between 700 and 900 cm1 can be used to decode the substitution pattern of benzenoid rings,
as summarized in Table 83.1. Due to four double bond equivalents derived from the molecular formula C9H13N (Chapter
81.1, Fig. 83.1), a benzenoid ring is likely to be present, and it is monosubstituted, as indicated by the IR bands at 705
and 740 cm1, according to Table 83.1.

Table 83.1. Substitution pattern of benzenoid rings and CH bending vibrations (CH) beween 700 and 900 cm1
CH [cm 1] substitution pattern
700 + 20 and 750 + H
  20 monosubstituted
750 +
 20 o-disubstituted CH 2 C NH 2
amphetamine
690 +
 30 and 780 +
 20 m-di- and 1,2,3-trisubstituted
830 +
 30 p-disubstituted CH 3
880 +
 20 and 830 +
 30 1,2,4-trisubstituted

Primary amino, methyl, methylene, and a monosubstituted benzene ring are structural units matching to the skeletal
structure of amphetamine which was also found by evaluating the mass spectrum (Chapter 82.3).
Fig. 83.4 verifies the potential of IR spectroscopy for the detection of functional groups: Strong bands at 1740 cm1 (CO
carbonyl bond), 1250 cm1 (CO acyl bond), and 1175 cm1 (CO alkyl bond) identify an ester; more precisely, it is a t-butyl
ester accounting for strong absorption bands below 3000 cm1 due to antisymmetric and symmetric CH stretching vibra-
tions, and at 1020 cm1, arising from a group vibration characterizing the t-butoxy residue. The CO acyl bond is a partial 
bond (Fig. 83.4, resonance formulas); thus, this bond vibrates faster (1250 cm1) than the CO alkyl single bond (1175
cm1).

O
CH3 C
O C(CH3)3

O
CH3 C
O C(CH3)3

Fig. 83.4. IR Spectrum of t-butyl acetate (acetic acid t-butyl ester)

In general, the IR spectrum represents a fingerprint of a specific compound, so that documented compounds can be iden-
tified by computer-assisted comparison of the digitized IR spectrum with comprehensive data files of reference spectra.
________________________________________________________________________________________________
Chapter 83 permits answers to the following:
(83.1) Which molecular vibrations do you know and what property is required for them to be IR-active?
(83.2) IR absorptions of alkynes (CC 2200 cm1) are very weak; those of nitriles (CN 2250 cm1) are very strong. Why?
(83.3) Which IR absorption bands can be used to identify a primary amino group?
(83.4) Which IR absorption bands characterize a carboxylic acid ester?
242 84 Nuclear Magnetic Resonance: Proton NMR

84 Nuclear Magnetic Resonance: Proton NMR


84.1 Nuclear Magnetic Resonance
Nuclei of atoms which have a nuclear spin, such as the proton 1H, give rise to a nuclear magnetic moment. When placed
in a magnetic field, nuclear magnetic moment vectors precess with a certain frequency 0 in the direction of the magnetic
field or in the opposite direction. These are two energy levels; nuclear precession in the direction of the magnetic field is
the lower one. Electromagnetic radiation with radio frequency RF in resonance with nuclear precession (RF = 0) excites
the nuclear spins to the higher energy level. This is the physical background behind the term nuclear magnetic reso-
nance (NMR).
Cryomagnets are used in modern NMR spectrometers, generating the strong magnetic field from the high current
strength due to superconduction in a coil of a metal alloy immersed in liquid helium (b.p. 4.2 °K).

84.2 Chemical Shift


Protons 1H with nuclear spin quantum number I = ½ are the most frequently used probes (1H NMR) Their precession
frequencies are in the order of several MHz (1 MHz = 106 Hz, 1 Hz (Hertz) = 1 cycle per second) and proportional to the
flux density (strength) of the magnetic field; a field of 1.4 Tesla causes protons to precess with 60 MHz, and 14 Tesla
accelerates precession to 600 MHz. Chemically non-equivalent protons of organic molecules, such as those of the methyl
and methoxy groups in methyl acetoacetate, precess at different rates in a magnetic field (Figs. 84.1, 84.2). This property
is referred to as the chemical shift of precession frequencies. The differences are in the order of some Hz or kHz, also
proportional to the flux density of the magnetic field, and their amounts have to be standardized to a  value independent
of the magnetic field.
Proton (and carbon-13) chemical shifts are usually referenced to tetramethylsilane [TMS, (CH3)4Si] as a standard. Then,
the  value of the chemical shift is obtained by subtracting the precession frequency i of the sample protons (e.g. those
of a methyl group in Fig. 84.1) from the precession frequency TMS of the TMS protons and dividing the difference i =
i TMS by the reference value TMS. Numerator and denominator are proportional to the field strength so that the quo-
tient  is independent of the field.
i [Hz]
 =
TMS [MHz]

As the quotient of two frequencies,  has no physical dimension but, accounting for the relationship of the two (Hz:MHz =
1:106),  values are usually given in parts per million (ppm).
Geometry of bonds (tetrahedral, linear, or trigonal) and charge densities predominantly influence the amount of the
chemical shift values  of nuclei. Proton chemical shifts H increase in the order sp3 < sp < sp2 and proportional to the
electronegativity of heteroatoms attached to carbon (Figs. 84.1, 84.2). Alkenyl protons exhibit larger shifts (H > 5) than
protons of alkyl and alkynyl groups (H < 4, Fig. 84.1). Methyl groups attached to electronegative O have much larger shifts
(H  3.5) than those connected to C (H  2, Figs. 84.1, 84.2).
sp2 sp sp3
O
OH CO2 H H H H C H C H TMS

acidic OH (carboxylic, enol)


aldehyde CH
heteroaromatic -electron deficient -electron excessive
aromatic ()-M-substituted ()-M-substituted
alkene CH ()-M-substituted ()-M-substituted
alkyne CH
acetal CH
R2CHO, RCH2O R2CHO RCH2O
CH3O
CH3N
CH3S
CH3 at C=C or C=X
cyclopropane
CH3 attached to metal

H 15 14 13 12 11 10 9 8 7 6 5 4 3 2 1 0 1

Fig. 84.1. Proton chemical shift ranges of organic compounds (reference: tetramethylsilane, H (TMS) = 0)

Efficiently Studying Organic Chemistry: Exam training for chemists, biochemists, pharmacists, life and health scientists,
Third Edition. Eberhard Breitmaier. © 2022 WILEY-VCH GmbH. Published 2022 by WILEY-VCH GmbH.
84.3 NMR Spectrum and Integration 243

Reflecting aromaticity, hydrogen atoms at benzenoid rings display larger shifts (H > 7) than alkenyl protons. Donors
decrease, acceptors increase H values of protons o and p to the substituent in benzene rings or  to the substituent in
alkenes due to resonance effects. Aldehyde protons affected by carbonyl resonance (H > 9) and acidic protons of carbox-
ylic acids, phenols, and enols (H > 11, Fig. 84.2) show particularly large shift values. Computer software permits prediction
of spectra by means of reference data and substituent increments.

84.3 NMR Spectrum and Integration


An NMR spectrum plots the intensity of the "peaks" called NMR signals of nuclei of all kinds of a particular atom such as
protons (1H) present in a molecule versus the  value of their chemical shift, as shown for neat methyl acetoacetate in
Fig. 84.2. Signals of this 1H NMR spectrum can be assigned according to Fig. 84.1.

acetoacetic acid methyl ester


OCH3 CH3
H H H
H 3C C OCH 3 H 3C C OCH 3
C C C C
O O O O
H
oxo tautomer enol tautomer CH2
91% 9%
H
OH
C

11.97 H (ppm) 4.94 3.66 3.37 2.19 1.97

Fig. 84.2. 1H NMR Spectrum of methyl acetoacetate (neat liquid, 80 MHz)

The area enclosed by a 1H NMR signal is proportional to the number of protons giving rise to that signal. The computer
which controls the spectrometer records the rising stepwise integral curves, as shown for methyl acetoacetate in Fig.
84.2. Step heights are proportional to the signal areas. For example, the ratio of step heights of methoxy and methylene
with  = 3.66 and 3.37 in Fig. 84.2 is CH3:CH2 = 3:2, confirming the signal assignment. Moreover, quantitative analysis of
mixtures is possible by integration of the NMR spectra. Evaluating the integrals of methyl proton signals at  = 2.19 (oxo)
and  = 1.97 (enol) in the expanded output of Fig. 84.2 reveals that 9 % of the enol and 91 % of the oxo tautomer are
present in the equilibrium mixture of methyl acetoacetate.

84.4 Signal Multiplets and Coupling Constants


84.4.1 Signal Multiplets
Indirect or scalar spin-spin coupling of nuclei, involving magnetic interaction of nuclear spins through covalent bonds,
causes the splitting of NMR signals into multiplets when liquids or solutions of solids are used as samples. For example,
the 1H NMR spectrum of compound C5H8O2 in Fig. 84.3 shows a triplet (t) at  = 1.3 and a quartet (q) at  = 4.2 with the
ratio of integral step heights of 3:2 for CH3 and CH2 attached to each other, forming an ethyl group CH3CH2. The
frequency distance (7 Hz) separating the lines within each of the two multiplets is called the coupling constant J.
The signal multiplicity is the number of peaks to which an NMR signal is split as a result of spin-spin coupling. Signals
without splitting are denoted as singlets (s). Those with two-, three-, four-, five-, six-, and seven-fold splitting are referred
to as doublets (d), triplets (t), quartets (q), quintets (qui), sextets (sext), and septets (sep), respectively, provided the
individual multiplet peaks are of equal distance apart due to one coupling constant J. Where two or three coupling con-
stants produce a multiplet, this is referred to as a two- or three-fold multiplet, for example a doublet of doublets (dd, Figs.
84.3, 84.4) or a three-fold doublet (ddd). Signals at  = 5.76,  = 6.09 and  = 6.39 in the 1H NMR spectrum of sample
C5H8O2 in Fig. 84.3 are two-fold doublets (dd) because two coupling constants contribute to the splitting of each signal.
The shift values ( = 5.7 - 6.4, Fig. 84.1) and the ratio of their integral step heights (1:1:1, Fig. 84.3) reveal three protons
belonging to a vinyl group –CH=CH2.
244 84 Nuclear Magnetic Resonance: Proton NMR

Thus, the sample molecule contains an ethyl and a vinyl group, C2H5 + C2H3 = C4H8, adding up to a partial structure includ-
ing all protons of the molecular formula C5H8O2 and one double bond equivalent. The difference between the molecular
formula and this substructure (C5H8O2 – C4H8 = CO2) clearly belongs to a carboxy function, identifying the sample mole-
cule as acrylic acid ethyl ester (Fig. 84.3).
d 13 d 13 d8
d 2 d 8 d 2 Hz

multiplicities and coupling constants (Hz)

dd H B
t 7 Hz dd H A O
C B A q7
q 7 Hz dd HC O
CH 2 CH3
t 7
3J = 8 (cis)
AB
3J = 13 (trans)
BC
2J = 2 (geminal)
AC

6.39 6.09 5.76 4.20 H (ppm) 1.30

Fig. 84.3. H NMR spectrum of sample C5H8O2 (in CDCl3, 80 MHz) to derive the structural formula; solution: acrylic acid ethyl ester
1

When A3 and X2 symbolize CH3 and CH2, respectively, the ethyl group represents an A3X2 spin system, and a vinyl group
with three kinds of protons is an AMX or ABC system, A symbolizing the nucleus with the smallest chemical shift. Spin
systems AmXn follow the (n  1) multiplicity rule: When coupled, n nuclei X with nuclear spin quantum number I = ½ induce
a splitting of the A signal into (n  1) multiplet lines. Relative intensities of the multiplet peaks follow the coefficients of
PASCAL’s triangle (Fig. 84.4). In general, n nuclei X with a spin quantum number IX will cause the A signal to split into
(2nIX  1) multiplet lines.

(n  1) multiplets PASCAL triangle J1 > J2


n=0 singlet (s) 1
1 doublet (d) 1 : 1 J1
2 triplet (t) 1 : 2 : 1
3 quartet (q) 1 : 3 : 3 : 1 J2 J2
4 quintet (qui) 1 : 4 : 6 : 4 : 1 J J J two coupling constants
one coupling constant doublet of doublets
5 sextet (sext) 1 : 5 : 10 : 10 : 5 : 1 quartet (q) two-fold doublet (dd)
6 septet (sep) 1 : 6 : 15 : 20 : 15 : 6 : 1

Fig. 84.4. (n  1) Multiplets, relative multiplet intensities (PASCAL’s triangle), and a quartet and doublet of doublets for comparison
 

84.4.2 Coupling Constants and Relative Configuration


Unlike chemical shifts, the frequency value of a coupling constant J does not depend on the strength of the magnetic field
but on the number of bonds separating the coupling nuclei from one another in the molecule, as shown in Table 84.1.
Couplings through one bond (1J or simply J, one-bond coupling) which are observed in 13C NMR (JCH = 125-250 Hz) differ
from the much smaller couplings through two bonds (2J, geminal), three bonds (3J, vicinal), and more (4J and 5J, long-range
couplings), resolvable in proton NMR (< 20 Hz, Table 84.1).

Table 84.1. Kinds of spin-spin coupling of protons in organic molecules and typical values of coupling constants
kind of coupling arrangement of nuclei (partial structure) separating bonds coupling constant [Hz]
H H
geminal 2 2J
HH = 0 - 20
H
H
H H H 3J
HH = 3 - 18
vicinal 3 cis- 3J
HH = 8 - 12
H H H H trans- 3J = 13 - 18
H cis- trans- HH

H H H
long range H H H 4 4J
HH = 1 - 3
and more
84.4 Signal Multiplets and Coupling Constants 245

Vicinal coupling constants (3J) reflect the relative configuration of the coupling protons because such 3J values depend on
the dihedral angle enclosed by the CH bonds in the NEWMAN projection (Chapter 9.2, Fig. 84.5). Vicinal syn-protons of
alkyl groups show smaller couplings (3Jsyn  3 Hz) than anti-protons (3Janti  16 Hz). Electronegative substituents such as
OH and OCH3 reduce these values.
If there is an equal population of the three conformers shown in Fig. 84.5 due to rotation around the CC single bond in
the alkyl group, the coupling protons pass through the syn-configuration twice and the anti-configuration once. An aver-
age coupling constant of about 7 Hz arises (Fig. 84.5); this value characterizes coupling protons of alkyl groups with
unimpeded rotation, such as those of the ethyl group occurring as a triplet and a quartet (Fig. 84.3). Cyclohexanes and
pyranosides, however, adopt stable chair conformations (Chapter 81.3), and their relative configuration is indicated by the
vicinal coupling constant 3J as shown for the - and -D-glucopyranosides. No rotation around the CC double bond of
alkenes occurs, so that trans-protons (anti) in alkenes display larger 3J couplings than cis-protons (eclipsed), as shown in
Figs. 84.3 and 84.5.
H H H methyl D-glucopyranoside alkenes
H H
HO H HO H
O O H
HO HO
HO H HO OCH 3
OH OH H H H
H OCH 3 H
syn anti syn - -
Jsyn = 3 Hz Janti = 16 Hz Jsyn = 3 Hz Jea = 4 Hz Jaa = 9 Hz cis- : 8-10 Hz trans- : 12-16 Hz
average coupling constant for equal population of conformers: equatorial-axial diaxial eclipsed anti
Javerage = ( 2 Jsyn  Janti ) / 3 = 7.3 Hz

Fig. 84.5. Vicinal coupling constant 3JHH and relative configuration (alkyl groups, glycosides, alkenes)

The proton NMR spectrum in Fig. 84.6 exemplifies the decoding of relative configuration from vicinal coupling. Shift
values and coupling constants confirm three double bond equivalents as derived from the molecular formula: The signal
at H = 3.05 reveals the terminal proton HA of an ethynyl group; an MX spin system of alkene protons with M = 4.50 and X
= 6.30 having the coupling constant 3JMX = 8 Hz identifies an ethenyl group (CHM=CHX) with protons in the cis-
configuration. A singlet at H = 3.80 belongs to three methoxy protons; the electronegative methoxy group increases X
while its donor effect reduces M. Long-range couplings 4JAM = 3 and 5JAX = 1 Hz (Table 84.1) confirm the connection of
ethenyl and ethynyl (Fig. 84.6, formulas).

1-methoxy-1-buten-3-yne
OCH3 H
H H 3CO
H H
H H
cis- or trans- ? X M A

multiplicities and coupling constants (Hz)


OCH 3 s 8 8 3
3J = 8 (cis)
dd H X MX 1 3 1 Hz
4J = 3
H A dd AM
5J = 1
dd H M AX

6.30 H (ppm) 4.50 3.80 3.05

Fig. 84.6. Decoding the relative configuration of C5H6O (1H NMR spectrum in CDCl3, 200 MHz)
________________________________________________________________________________________________
Chapter 84 permits answers to the following:
(84.1) Measure the integral steps (mm) in Figs. 84.2, 84.3, and 84.6 and evaluate the results.
(84.2) Try to predict the proton NMR spectra of 2- and 3-pentanone. All coupling constants are 7 Hz (why?).
(84.3) Specify the spin systems in Figs. 84.3 and 84.6. Assign all coupling constants.
(84.4) Which kind of NMR data reflects the relative configuration? Provide some examples.
246 85 Nuclear Magnetic Resonance: Carbon-13 NMR

85 Nuclear Magnetic Resonance: Carbon-13 NMR


85.1 Carbon-13 as NMR Probe
Two stable carbon isotopes exist; these are 12C and 13C. Only the less abundant 13C (1.1% or about 102) has a nuclear
spin (spin quantum number I = ½) and is suitable as an NMR probe. Due to the low natural abundance of 13C and its
weaker magnetic moment, carbon-13 NMR requires larger sample quantities than proton NMR, and the probability of
13C12C and 13C13C bonds occurring in organic molecules is only 102 and 104, respectively. The resulting splitting due

to 13C13C coupling in satellites (0.5 % intensity) at a distance of half the 13C13C coupling constant on either side of the
13C12C main signals (99 % intensity) usually remains concealed within the noise level of the baseline. Thus, 13C NMR

spectra are not complicated by 13C13C spin-spin coupling, and splitting of signals due to carbon-proton coupling can be
eliminated by proton decoupling so that one singlet signal is observed for each kind of carbon atom in the sample mole-
cule (Fig. 85.2 a).

85.2 Carbon-13 Chemical Shifts


As shown in Fig. 85.1, carbon-13 chemical shifts C of organic compounds essentially follow the order outlined for proton
shifts (Chapter 84.2). However, shift ranges and influences of substituents are considerably larger, meaning that 13C
chemical shifts indicate chemical nonequivalence more clearly than proton shifts. Carbon-containing functions, such as
different carbonyl groups and hetero analogues, can be directly identified by typical shift values in 13C NMR spectra (Fig.
85.1) but not in 1H NMR. Unlike 1H shifts, 13C shifts of benzenoid and heteroaromatic compounds do not significantly differ
from those of alkenes (Fig. 85.1).
sp2 sp sp3
O
C C C C C

C 200 150 100 50 0 (TMS)

carbenium ions
ketones conjugated
aldehydes conjugated
acetals, ketals
quinones
carboxylic acids and derivatives conjugated
thioureas
ureas, carbonates
imines (hydrazones, oximes)
isocyanides (isonitriles)
cyanides (nitriles)
isothiocyanates
thiocyanates
isocyanates
cyanates
carbodiimides
heteroaromatics -electron deficient -electron rich
aromatics ()-M-substituted ()-M-substituted
(cyclo-) alkenes ()-M-substituted ()-M-substituted
(cyclo-) alkynes cyclopropanes
(cyclo-) alkanes
alkyl bonded to metal

C 200 150 100 50 0 (TMS)

Fig. 85.1. Carbon-13 chemical shift ranges of organic compounds (reference: tetramethylsilane, C (TMS) = 0)

A substituent increment (C = C(RX) C(RH)) is defined as the change in carbon-13 shift of a particular C atom when an H
atom in an alkyl chain RH is replaced by a substituent X . In the -position to the substituent X,C increases proportio-
nally to the substituent electronegativity EX (C ~ EX).
H X
R H C  C  C C  C  C R X
 C  C   C  C 

substituent increments C 0 0 <0 >0 >0

Efficiently Studying Organic Chemistry: Exam training for chemists, biochemists, pharmacists, life and health scientists,
Third Edition. Eberhard Breitmaier. © 2022 WILEY-VCH GmbH. Published 2022 by WILEY-VCH GmbH.
85.3 Carbon-Proton Coupling and Detection of CH Multiplets 247

In the -position, C also increases (C > 0), while it decreases at the C atom  to the substituent due to sterically induced
polarization (C < 0,  effect). More remote carbon atoms remain almost uninfluenced (C  0). Substituent increment
tables, available for all classes of compounds, are used to predict 13C NMR spectra.

85.3 Carbon-Proton Coupling and Detection of CH Multiplets


One-bond carbon-proton coupling constants JCH (1JCH) are proportional to the s character of the hybrid bonding orbitals of
the coupled carbon atom, according to JCH = 500 s where s = 0.25, 0.33, and 0.5 for sp3, sp2, and sp hybridized C
atoms, respectively. Thus, it is possible to distinguish between alkyl (JCH 125 Hz), alkenyl or aryl (JCH  165 Hz), and
alkynyl carbon atoms (JCH  250 Hz). Further, coupling constants JCH increase proportionally to the electronegativity of
the substituent attached to the coupled carbon atom.
Like vicinal proton-proton couplings 3JHH (Chapter 84.4.2), 3JCH coupling constants reflect the relative configuration of the
coupling nuclei 13C and 1H, as shown for substituted cyclohexanes and propene:
cis- 7.5 Hz H CH 3
R X R H 8 Hz
C C
cis- H 2 Hz trans- X trans- 12.6 Hz H H

Without proton decoupling, 13C NMR spectroscopy detects doublets, triplets, and quartets for CH, CH2, and CH3, respec-
tively. Evaluation of these multiplets may be hampered by signal overcrowding in larger molecules. In this case, pulse
sequences are used to transform the information of the CH signal multiplicity due to carbon-proton coupling into a phase
relationship (positive and negative amplitudes) of the 13C signals in the proton-decoupled 13C NMR spectrum, as shown
in Fig. 85.2. Subspectra of CH carbon atoms only (Fig. 85.2 b) and of all CH, CH2, and CH3 carbons (Fig. 85.2 c) with
positive amplitudes for CH and CH3 carbon atoms and negative amplitudes for CH2 carbon atoms are the result. Quater-
nary carbon atoms appear as additional signals in the proton-decoupled 13C NMR spectrum, exemplified by the signal at
C = 38.5 in Fig. 85.2 a.

c : CH and CH3 positive


CH2 negative

C10 H18 O

27.9 23.8
H3C CH3
38.5 b : CH subspectrum
42.1 39.1 70.7
20.9 OH
CH3
34.2 48.2
47.4

isopinocampheol

a : all carbon atoms

Fig. 85.2. (a) Proton-decoupled 13C NMR spectrum of isopinocampheol [in hexadeuterioacetone (CD3)2CO, 50 MHz], detecting 10 non-
equivalent C atoms; (b) CH subspectrum; (c) subspectrum of all C atoms which are bonded to H (CH and CH3 positive, CH2 negative)

The 13C NMR spectra of isopinocampheol C10H18O in Fig. 84.2 show, for example, one quaternary C atom (C), four CH
units (C4H4), two CH2 units (C2H4) and three CH3 groups (C3H9), adding up to C10H17. When compared with the molecular
formula, C10H18O, one H is missing; thus, the molecule contains one OH group.
________________________________________________________________________________________________
Chapter 85 permits answers to the following:
(85.1) Homonuclear 13C13C couplings are usually not observed in 13C NMR spectra. Why?
(85.2) How do aldehydes and ketones differ in carbon-13 NMR spectroscopy?
(85.3) How do ethyl, ethenyl and ethynyl groups differ with respect to (a) 13C chemical shifts and (b) CH couplings?
(85.4) Summate all C, CH, CH2, and CH3 in Fig. 85.2. Why is the result different to the molecular formula?
248 86 Nuclear Magnetic Resonance: Two-Dimensional NMR

86 Nuclear Magnetic Resonance: Two-Dimensional NMR


Radio frequency pulse sequences with timings adjusted to coupling constants and other parameters permit correlations of
NMR data such as chemical shifts of one kind of nuclei (homonuclear) or of different kinds of nuclei (heteronuclear) in two
frequency dimensions. Series of spectra are the result which can be neatly shown in perspective as a stacked plot.
Usually, a "map" of this signal panorama, known as the contour plot, is recorded for more convenient evaluation in order
to decode molecular structures (Figs. 86.1, 86.2, 86.3).

86.1 Homonuclear Shift Correlation


86.1.1 Proton-Proton Shift Correlation (HH COSY)
Proton-proton shift correlation, referred to as HH COSY (COSY from correlation spectroscopy), detects the HH connec-
tivities, comprising all coupling relationships (geminal, vicinal, longer-range, Table 84.1) of the H atoms, in a molecule.
Fig. 86.1 a demonstrates this for an AB two-spin system. Due to equal shift scales in both dimensions, HH COSY plots
display square symmetry. Only diagonal signals (projections of the spectrum to the diagonal) are detected without any
coupling, while cross signals with mixed coordinates (AB and BA) reveal the connectivity of the nuclei A and B due to
spin-spin coupling (JAB, Fig. 86.1 a). The HH COSY plot shown in Fig. 86.1 b, for example, indicates that the protons at H
= 5.24, H = 1.97, and H = 5.42 are coupled with each other. Evaluation of all cross signals gives a set of vicinal connec-
tivities of protons (H = 0.83  1.97  5.42  5.24  2.21  3.50) which decodes the skeletal structure of 3-hexenol (leaf
alcohol). Its cis-configuration follows from the coupling constant of the alkene protons (3Jcis = 10.5 Hz, Chapter 84.4.2).

A
without coupling JAB
only diagonal signals
AA BB
B
0.83 1.97 2.21 3.50
H3C H2 C CH2 CH2 OH
B A C C
JAB JAB 5.42 H H 5.24
cis-3-hexenol

with coupling JAB A


diagonal signals
AA BB 1.97
and cross signals H2 C
AB BA B C C
5.42 H H 5.24
B A

a b
Fig. 86.1. HH COSY contour plot (a) of an AB system, schematic, (b) of cis-3-hexenol (400 MHz) to evaluate proton-proton connectivities

86.1.2 Carbon-Carbon Shift Correlation


A two-dimensional carbon-carbon shift correlation suppresses the intense signals of the 13C12C bonds, segregates the
two-spin systems AX of all 13C13C bonds by the sum of the shifts (A + X) of each CACX bond and symmetrizes the data
file to the quadratic COSY format as shown in Fig. 86.2. Cross signals which split into doublets due to 13C13C coupling
can be evaluated as described in Chapter 86.1.1. Fig. 86.2 shows that the carbon with C = 79.2 is bonded to the C atoms
with C = 41.5 and C = 33.2; the other CC bonds decoded in this manner establish the carbon skeleton of a methylcyclo-
pentane with one unassigned bond originating from the carbon at C = 79.2, which is a CH unit according to subspectrum
c in Fig. 86.2.

Efficiently Studying Organic Chemistry: Exam training for chemists, biochemists, pharmacists, life and health scientists,
Third Edition. Eberhard Breitmaier. © 2022 WILEY-VCH GmbH. Published 2022 by WILEY-VCH GmbH.
86.2 Heteronuclear Shift Correlation (Carbon-Proton Correlation) 249

H H
H 17.9
31.0 C
CH3
H C 41.5
20.9 C
H C 79.2
33.2 C
?
H H
H

H H H 17.9
31.0 C CH3
C 41.5

C 79.2
33.2 C ?
H H
H

H
CH3
C41.5

C 79.2
33.2 C ?
H H
H

Fig. 86.2. Carbon-carbon shift correlation to establish the carbon skeleton (a) with proton-decoupled 13C NMR spectrum (b), subspec-
trum (c) of two CH carbons, and subspectrum (d), additionally assigning three CH2 and one CH3 (in CDCl3, 100 MHz for 13C NMR)

86.2 Heteronuclear Shift Correlation (Carbon-Proton Correlation)


A two-dimensional carbon-proton shift correlation (CH COSY) adjusted to one-bond carbon-proton coupling (JCH) maps
the carbon-13 shift on to the abscissa and the proton shift on to the ordinate (or vice versa). Thus, the shifts of linked 13C
and 1H nuclei are read as coordinates of the cross signals in the CH COSY plot, as shown in Fig. 86.3 for the sample
used in Fig. 86.2. All CH bonds can be unequivocally assigned. It turns out that methylene protons of the cyclopentane
ring form (overlapping) AB systems in the 1H NMR spectrum because they are not equivalent. Moreover, one of the
protons (H = 4.33) does not give a cross signal (Fig. 86.3). Thus, this proton is attached to a heteroatom (O), and the
sample molecule is 2-methylcyclopentanol. Other NMR methods reveal the relative configuration (trans-). – CH shift
correlations adjusted to longer-range CH couplings (2JCH, 3JCH) permit localization of C atoms at a two- or three-bond
distance from a proton.

0.91 AB 1.68
H H 1.58 17.9
H
H 31.0 C
CH 3 0.74
H C41.5
1.35 AB 1.48 C 20.9
H C79.2 H 4.33
33.2 C O
H H
1.35 AB 1.66
H 3.43

proton chemical shifts in italics


C

Fig. 86.3. Carbon-proton shift correlation (CH COSY contour plot, CDCl , C: 100 MHz / 1H: 400 MHz) to assign carbonhydrogen bonds
3 13
________________________________________________________________________________________________
Chapter 86 permits answers to the following:
(86.1) Briefly describe the concept of two-dimensional shift correlations and their use in molecular structure elucidation.
(86.2) Evaluate the HH COSY plot in Fig. 86.1 including the integral steps (on top) to decode the skeletal structure.
(86.3) Evaluate the CC shift correlations in Fig. 86.2 in order to establish the carbocyclic molecular structure.
(86.4) Assign all CH bonds of the sample molecule in Fig. 86.3. One proton does not give a cross signal. Why?
250 Working on Questions

Working on Questions
Working on Questions
All questions are very closely related to the chapter´s contents so that the text answers most of them in words, by means
of structural formulas or reaction equations. Chapter references are provided in these cases. More detailed solutions are
given in the following when answers require analogous conclusions or the reader´s own reflections.

(1.1) What is an atomic orbital?


Chapter 1.2
(1.2) How do p orbitals differ from s orbitals?
Chapter 1.2
(1.3) How do p orbitals differ among themselves?
Chapter 1.2
(1.4) Write the orbital occupancy for the atoms in the first two rows of the periodic table.
Chapter 1.4, Table 1.1
(1.5) Is the tetravalency of carbon in accordance with the electronic configuration of carbon in Table 1.1?
The electronic configuration 1s2 2s2 2p2 of the carbon atom does not explain its tetravalence because only two
singly occupied p orbitals (e.g. px and py) would be available for overlapping (see also Chapter 3.1).

(2.1) What is a covalent bond?


Chapter 2.1
(2.2) How is a molecular orbital (MO) generated?
Chapter 2.2
(2.3) How can the covalent bond of the hydrogen molecule be explained?
Chapter 2.2
(2.4) Calculate the particularly high electron density between covalently bonded atomic nuclei in terms of the MO
model.
Chapter 2.2, Table 1.1
(2.5) Which options of overlapping exist for p orbitals? Which types of bonding are the result?
Chapter 2.3, Fig. 2.3

(3.1) Describe and draw the geometry of the methane molecule.


Chapter 3.1
(3.2) The electronic configuration of the carbon atom does not explain the shape of the methane molecule. Why?
Chapter 3.1
(3.3) What is hybridization of atomic orbitals? Hybrid orbitals offer better chances for overlapping. Why?
Chapter 3.2
(3.4) What is the model explanation of the tetrahedral shape of the methane molecule and the genesis of its CH bonds?
Chapter 3.3, Fig. 3.4

(4.1) What are the molecular shapes of (a) ethane, (b) ethene, and (c) ethyne?
Chapter 4.1, Fig. 4.1: answering (a); Chapter 4.2, Fig. 4.3, answering (b); Chapter 4.3, Fig. 4.5: answering (c)
(4.2) Explain the genesis of a CC single, a CC double and a CC triple bond in terms of the MO model.
Chapter 4.1, Fig. 4.2, Chapter 4.2, Fig. 4.4, Chapter 4.3, Fig. 4.6
(4.3) Draw the areas around a CC double bond where the electrons are most likely to be found.
Chapter 4.2, Fig. 4.4
(4.4) Explain why CC multiple bonds are shorter than a CC single bond.
Chapter 4.3, last section

Efficiently Studying Organic Chemistry: Exam training for chemists, biochemists, pharmacists, life and health scientists,
Third Edition. Eberhard Breitmaier. © 2022 WILEY-VCH GmbH. Published 2022 by WILEY-VCH GmbH.
Working on Questions 251

(5.1) What is a homologous series of compounds?


Chapter 5.1
(5.2) What natural sources of alkanes exist and how are alkanes produced industrially?
Chapter 5.2.1
(5.3) What reactions permit the preparation of specific alkanes?
Chapters 5.2.3, 5.2.4 (symmetric alkanes RR), Chapter 5.2.2 (depending on availability of alkenes)
(5.4) Alkanes are still an important source of energy. Why? Write the complete equation for an appropriate reaction.
Chapter 5.3

(6.1) What are structural (skeletal) isomers?


Chapter 6.1
(6.2) Which structural isomers exist for (a) butane, (b) pentane, and (c) hexane?
Chapters 6.1, 6.2
(6.3) Which properties are individual (specific) for structural isomers?
Chapter 6.2
(6.4) Which one of these properties is the basis of a method for the separation and purification of structural isomers?
Chapter 6.2, Table 5.2
(6.5) Draw the structures of the skeletal isomers of heptane with (a) one methyl group and (b) two methyl groups in
the side chain.
(a) Two singly and (b) four doubly methyl branched skeletal isomers of heptane C7H16 exist.
CH 3 CH 3 CH 3
H 3C CH CH 2 CH 2 CH 2 CH 3 H 3C CH CH CH 2 CH 3 H 3C CH 2 C CH 2 CH 3
2-methylhexane CH 3 CH 3
(a) (b) 2,3-dimethylpentane 3,3-dimethylpentane
CH 3 CH 3 CH 3
CH 3 CH 2 CH CH 2 CH 2 CH 3 H 3C CH CH 2 CH CH 3 H 3C C CH 2 CH 2 CH 3
3-methylhexane CH 3 CH 3
2,4-dimethylpentane 2,2-dimethylpentane

(6.6) Which of the following compounds (1-6) are structural isomers? Which ones are identical?

CH3
H3C CH2 CH CH 2 CH2 CH CH 3 H3C CH CH 2 CH2 CH CH3
CH3 CH 3 CH2 CH3
1 2

CH3 CH3
H3C CH2 CH CH 2 CH CH2 CH 3 H3C CH CH 2 CH2 C CH3
CH3 CH 3 CH3
3 4

CH3 CH3 C2H5


H3C CH2 C CH2 CH CH3 (H3C)2CH CH2 C CH3
CH3 CH3
5 6

1 and 2 as well as 5 und 6 are identical; 3 displays two and 4 three methyl branchings.

(7.1) What are the IUPAC names of all singly and doubly methyl branched isomers of heptane C7H16? #
IUPAC names are assigned to the structural formulas in answer (6.5).
252 Working on Questions

(7.2) Draw the structural formulas of (a) 2,2,4-trimethylhexane, and (b) 5-t-butyl-3,3-diethyloctane.
Structural formulas of 2,2,4-trimethylhexane (a) and 5-t-butyl-3,3-diethyloctane (b) are the following:
CH 3 C2H5 C(CH 3)3
(a) H3C C CH2 CH CH2 CH3 (b) H3C CH 2 C CH2 CH CH2 CH 2 CH 3
CH 3 CH3 C2H5

(7.3) Give the IUPAC names of the alkanes with structures drawn in question 6.6.
1 = 2: 2,5-dimethylheptane; 3: 3,5-dimethylheptane; 4: 2,2,5-trimethylhexane; 5 = 6: 2,4,4-trimethylhexane are the
IUPAC names.
(7.4) What is the correct IUPAC name of the following alkane?
CH(CH 3)2 C(CH 3)3 CH 3
H 3C CH 2 CH 2 CH CH CH 2 CH CH CH CH 2 CH 3
CH 3 CH 2 CH 3

5-t-Butyl-4-ethyl-8-isopropyl-3,7-dimethylundecane (not 7-t-butyl-8-ethyl-4-isopropyl-5,9-dimethylundecane) is the


correct IUPAC name.
(7.5) What is the structural formula of 2,6,10-trimethyldodecane which occurs in slate oil?
The skeletal formula of 2,6,10-trimethyldodecane (the sesquiterpene parent hydrocarbon, Chapter 76.1) provides
a more compact presentation.
CH 3 CH3 CH3
H 3C CH CH 2 CH2 CH2 CH CH2 CH2 CH 2 CH CH2 CH3 or

(8.1) Draw the skeletal formulas for cyclopropane, cyclopentane, cycloheptane, and cyclooctane.
Chapter 20.1, cycloalkane formulas
(8.2) Draw the condensed and skeletal structural formula of 2,6,10-trimethyldodecane.
answer (7.5)
(8.3) Draw the condensed and skeletal structural formula of 3-methylhexane and number the longest carbon chain.
answer (6.5a)
(8.4) Draw one of the tetrahedral projections of the CH carbon (C-3) in 3-methylhexane.
Chapter 8.4.1: The three-dimensional picture of tetrahedral bonds originating from a carbon facilitates the drawing
of FISCHER- and tetrahedral projections as in the case of 3-methylhexane.
FISCHER projection tetrahedral projections
C 3H 7 C3H 7 C3H7 C 3H7
H CH3 H H
H C CH 3 H C CH3 C C C C H
H C2H 5 H
2C
3
C 2H 5 C2H 5 H7C3 C2H 5 H3C H3C
1CH 3
3-methylhexane C-2C-3 bond

(8.5) Write the reaction equation for 1-bromobutane with sodium hydroxide by means of LEWIS formulas. What is the
origin of the CO bond in the product 1-butanol?
Chapter 8.3
(8.6) Draw NEWMAN projections of the central CC bond of hexane with staggered ethyl groups.
Chapter 8.4.2: C2H5 replaces CH3 in the NEWMAN projection of butane; the other two conformers with staggered
substituents arise from rotation of the tetrahedron behind by 120°.

(9.1) Why do alkanes form conformers? Explain the terms conformation and conformer.
Chapter 9.1
(9.2) How do conformers differ from structural isomers?
Chapter 9.2, last section
Working on Questions 253

(9.3) Draw NEWMAN projections of the C-2C-3 bond of butane for all conformers with staggered methyl groups.
Fig. 9.1: CH3 replaces staggered alkyl residues R (three conformers).
(9.4) Draw NEWMAN projections of the C-1C-2 bond of (a) 1-bromopropane and (b) 1,2-dibromoethane for all
conformers with staggered groups.
Fig. 9.1: (a) CH3 and Br replace staggered alkyl residues R (three conformers);
(b) Br replaces staggered alkyl residues R (three conformers).
(9.5) Draw the energy diagram of rotation about the central CC bond of butane and use NEWMAN projections to present
all conformers.
Chapter 9.2, Fig. 9.1: CH3 replaces alkyl residues R.
(9.6) Draw NEWMAN projections of the following conformers of 2-methylhexane and specify the spatial arrangements
of the alkyl groups.
C2H5 C2H 5
H H
C CH(CH 3) 2 C H
H C H C
H H
H CH(CH 3) 2

Chapter 9.2: NEWMAN projections clearly show when alkyl groups are syn(clinal, dihedral angle 60°) or
anti(periplanar, dihedral angle 180°).
C2H 5 C2H5
C 2H 5 direction of view C 2H 5
H CH(CH3) 2 H H
H H
C C H
H CH(CH 3)2 H C
C H
H H H H H
H H CH(CH 3)2 CH(CH3) 2
syn(clinal) anti(periplanar)

(9.7) What are steric interactions? Which conformers particularly suffer from these interactions?
Chapter 9.2: Eclipsed groups strongly repel each other and destabilize a conformer. This steric interaction is much
weaker for staggered groups.

(10.1) Give appropriate examples for a homolysis and a heterolysis of a  bond.


Chapter 10.1
(10.2) What are radicals, carbenium ions, carbanions, and carbenes? How are these species formed?
Chapters 10.1, 10.2, 10.3
(10.3) Draw the molecular orbital models of the methyl radical and the methyl cation. How do they differ?
Chapter 10.1, Fig. 10.1
(10.4) Define the terms electrophile and nucleophile and classify carbenium ions, carbanions, and carbenes as such.
Chapters 10.2, 10.3
(10.5) Suggest a molecular orbital model for the low-energy carbene with unpaired electrons.
The bond angle (136°) is not too far apart from the interorbital angle of sp2 hydrid orbitals (120°). Thus, it may be
assumed that the biradical carbon atom of hot carbene overlaps with two sp2 hybrid orbitals in order to form two
CH bonds. The remaining sp2 hybrid orbital and the unhybridized p orbital are singly occupied.

(11.1) Which kind of reactions are additions and eliminations? Formulate typical equations.
Chapters 11.1, 11.2
(11.2) Explain the term -elimination.
 Elimination involves separation of H and X from adjacent C atoms; in an  elimination H and X separate from
the same carbon atom (occuring infrequently, e.g. Chapter 42.6.1).
(11.3) Which kind of reactions are oxidations and reductions? Formulate typical equations.
Chapters 11.3, 11.4
(11.4) Which kind of substitution reactions exist? Formulate typical equations.
Chapter 11.5
254 Working on Questions

(11.5) What kind of rearrangements do you know? Formulate typical equations.


Chapter 11.6

(12.1) What is the transition state of a reaction?


Chapter 12.1, Fig. 12.1
(12.2) Explain the terms activation energy and heat of reaction.
Chapter 12.1, Fig. 12.1
(12.3) What are the effects of an exothermic and an endothermic reaction?
Chapter 12.1
(12.4) What are the characteristics of a kinetically and a thermodynamically controlled reaction? Draw an energy diagram
for explanation.
Chapter 12.3, Fig. 12.2
(12.5) Characterize the action of a catalyst.
Chapter 12.2

(13.1) Formulate all equations to describe the mechanism of the radical halogenation of alkanes.
Chapter 13.1
(13.2) Which kind of substituents can be introduced into alkanes by radical substitutions?
Chapters 13.1, 13.4: Radical halogenation, sulfochlorination and nitration permit substitution of hydrogen atoms
in alkyl groups (alkanes), introducing halogens, sulfonic acid and nitro groups.
(13.3) The 2-butyl radical is more stable than the 1-butyl radical. Why?
Chapter 13.2, Fig. 13.1
(13.4) Define the term regioselectivity. How are regioselectivity and reactivity related to each other in the photo-
halogenation of alkanes?
Chapter 13.3
(13.5) What product is expected from the bromination of butane? Explain, write equations and give the product’s name.
Chapters 13.2, 13.3: Replace propane by butane in Chapter 13.3; 2-bromobutane is the major product because a
2-butyl radical is more stable, longer living, thus having more time to react than the less stabilized 1-butyl radical.

(14.1) Which unbranched structural isomers exist for the molecular formula C8H16? Give their IUPAC names.
Chapter 14.1: 1-Octene, 2-octene (cis- and trans-), 3-octene (cis- and trans-) 4-octene, (cis- and trans) are
unbranched isomeric alkenes with the molecular formula C8H16. Answer (14.5) provides the skeletal formulas
of the cofigurational isomers.
(14.2) There is no free rotation about the  bond of alkenes. Look at Fig. 4.4 and provide an explanation.
Chapter 14.2
(14.3) What are configurational isomers? How do they differ? How are they specified? Why do they differ from
conformers?
Chapter 14.2
(14.4) Which of the alkenes (a) propene, (b) 1-bromopropene, (c) 1-butene, (d) 2-butene, and (e) 3-hexene exist as
configurational isomers? Draw condensed structural and skeletal formulas (Chapters 8.1, 8.2) of all compounds.
Only (b), (d) and (e) exist as cis- and trans- isomers (Z- and E- isomers).
(14.5) Draw condensed structural and skeletal formulas of all configurational isomers of the non-terminal octenes
C8H16 and name them.
The skeletal formulas of 2-octene (trans- and cis-), 3-octene (trans- and cis-), 4-octene (trans- and cis-) provide
a compact sketch of the molecular geometry.
Working on Questions 255

(15.1) What mechanisms exist for the dehydrohalogenation of alkyl halides? Formulate appropriate equations.
Chapter 15.1.1
(15.2) What is the mechanism for the acid-catalyzed dehydration of an alcohol? Write an appropriate equation.
Chapter 15.1.2
(15.3) Dehydrobromination of 2-bromobutane predominantly yields 2-butene. Why?
Chapter 15.1.1 in analogy to 2-bromopentane, set CH3 instead of C2H5
(15.4) 2,3-Dimethyl-2-butanol dehydrates to give 2,3-dimethyl-2-butene and not to 2,3-dimethyl-1-butene. Why?
Chapter 15.1.2
(15.5) Formulate the general conversion of a carbonyl compound into an alkene.
Chapter 15.2.4
(15.6) 1-Butene reacts with N-bromosuccinimide. What product do you expect? Formulate the reaction.
Chapter 15.3.1
(15.7) Methylpropene and ethene are heated under pressure. What product is likely to be formed?
Chapter 15.3.3: The ene reaction of methylpropene with ethene is expected to produce 2-methyl-1-pentene.
H H H
+

methylpropene ethene transition state 2-methyl-1-pentene

(15.8) Design a synthesis of (a) 3,4-diisopropyl-2,5-dimethyl-3-hexene and (b) cis- and trans-4,5-dimethyl-4-octene.
Chapter 15.2.3, answering (a): MCMURRY reaction of diisopropylketone produces 3,4-diisopropyl-2,5-dimethyl-
3-hexene (tetraisopropylethene).

TiCl4 (H3C)2HC CH(CH 3)2


Li Hg
C O + O C C C C C
(H 3C)2HC CH(CH3)2

diisopropylketone (2,4-dimethylpentan-3-one) tetraisopropylethene (3,4-diisopropyl-2,5-dimethyl-3-hexene)

Chapter 15.3.4, answering (b): Metathesis of 2-methyl-1-pentene produces 4,5-dimethyl-4-octene and ethene.

kat.
+ +
ethanol

2-methyl-1-pentene 4,5-dimethyl-4-octene, (E)- and (Z)- ethene

(16.1) Suggest the preparation of (a) 2,3-dimethylpentane and (b) 2,3-dibromohexane from appropriate alkenes.
Chapter 16.1, answering (a): 2,3-Dimethylpentane is obtained by catalytic hydrogenation of 2,3-dimethyl-
1-pentene (or regioisomers with the same carbon skeleton but CC double bonds in other positions).
CH 3
kat.
+ H2 H 3C CH CH CH 2 CH 3
CH 3
2,3-dimethyl-1-pentene 2,3-dimethylpentane

Chapter 16.2, answering (b): 2,3-Dibromohexane arises from the addition of bromine to 2-hexene (cis- or trans-).
H 3C H Br Br H 3C CH 2 CH2 CH3
+ Br2 + Br2
C C H 3C CH CH CH2 CH2 CH3 C C
H CH2 CH2 CH3 H H
trans-2-hexene 2,3-dibromohexane cis-2-hexene

(16.2) What is an electrophilic addition? Write mechanisms for the bromination and hydrobromination of alkenes.
Chapters 16.2, 16.3
256 Working on Questions

(16.3) What products are obtained by hydrobromination of (a) 2-methyl-2-butene and (b) 1-hexene?
Chapter 16.3: In accordance with MARKOVNIKOV´s rule, (a) 2-bromo-2-methylbutane
H 3C H CH3 CH3
C C + H Br H3C C CH 2 CH3 and H3C CH CH CH3
H 3C CH3 Br Br
2-methyl-2-butene 2-bromo-2-methylbutane (major) 2-bromo-3-methylbutane

and (b) 2-bromohexane are expected to arise as major products from hydrobromination.
H 2C CH CH 2 CH 2 CH 2 CH 3 + H Br H 3C CH CH 2 CH 2 CH 2 CH 3 and Br CH 2 CH 2 CH 2 CH 2 CH 2 CH 3
1-hexene Br 1-bromohexane
2-bromohexane (major)

(16.4) What products arise from (a) hydration and (b) hydroboration of 2-methyl-1-butene?
Chapter 16.4, answering (a): 2-Methyl-1-butene is expected to undergo acid catalyzed hydration to
2-methyl-2-butanol, involving the more stable tertiary carbenium ion according to MARKOVNIKOV´s rule.
H 3C H3C CH3
C CH2 + H3O C CH2H + H2O CH3 CH2 C CH2H
 [H ]
H3C H 2C H3C H2C OH
2-methyl-1-butene 2-methyl-2-butanol

Chapters 16.6, 33.3.4, answering (b): Hydroboration is expected to produce tri-(2-methylbutyl)borane which
undergoes oxidation to 2-methyl-1-butanol and boric acid with hydrogen peroxide, H2O2.
2-methyl-1-butene 2-methyl-1-butylborane tri-(2-methyl-1-butyl)borane 2-methyl-1-butanol
H 3C H 3C CH 3 CH 3
+ 3 H 2O2
3 C CH 2 via CH CH 2 (CH 3 CH 2 CH CH 2 )3 B 3 CH 3 CH 2 CH CH 2 OH
H 3C H 2C + H 3C H 2C
 B(OH) 3
BH 2
H BH 2

(16.5) Which methods can be used to prepare 1,2-diols from alkenes? Formulate general reaction equations.
Chapter 16.7
(16.6) Stereospecificity of hydrogenation, bromination and dihydroxylation for non-cyclic alkenes is not detectable. Why?
Chapters 16.1, 16.2, 16.7
(16.7) Which alkenes give only (a) propanone and (b) butanal upon ozonolysis followed by hydrolysis?
Chapter 16.8: Carbonyl compounds propanone (a) and butanal (b) permit reconstruction of the original
alkenes 2,3-dimethyl-2-butene and 4-octene (cis- or trans-), respectively.
H3C CH3 H3C CH 3
(a)
C O + O C C C
H3C CH3 H3C CH3
acetone (propanone) 2,3-dimethyl-2-butene
(b)
CH3 CH2 CH2 CH O + O CH CH2 CH2 CH3 CH 3 CH 2 CH2 CH CH CH2 CH2 CH3

butanal 4-octene (cis- or trans-)

(17.1) Which structural isomeric hexadienes exist and how are these classified? Write the formulas.
Chapter 17.1: 1,2- and 2,3-hexadiene have cumulated, 1,3- and 2,4-hexadiene have conjugated, 1,4- and
1,5-hexadiene have isolated double bonds. 1,3-, 2,4- and 1,4-hexadiene exist as E- and Z-isomers.
cumulated double bonds conjugated double bonds isolated double bonds
1 2 1 3 1 4
H2C C CH CH2 CH2 CH3 H2C CH CH CH CH2 CH3 H2C CH CH 2 CH CH CH 3
1,2-hexadiene 1,3-hexadiene 1,4-hexadiene
(E,Z-isomers) (E,Z-isomers)

2 3 2 4 1 5
H3C CH C CH CH 2 CH3 H3C CH CH CH CH CH3 H2C CH CH2 CH2 CH CH2
2,3-hexadiene 2,4-hexadiene 1,5-hexadiene
(E,Z-isomers)
Working on Questions 257

(17.2) What are the appropriate formulas of 1,3-butadiene? What is the meaning of the term resonance in this context?
Chapter 17.2.2
(17.3) There are several methods to prepare 1,3-butadiene. Formulate the equations for these reactions.
Chapters 17.3.1, 17.3.2, 17.3.3
(17.4) Suggest a resonable synthesis of 1,2-hexadiene.
Chapter 17.3.4: Dehydrobromination of 1-bromo-2-hexene is expected to produce 1,2-hexadiene.
Dehydrobromination of 3-bromo-1-hexene would yield the conjugated diene, 1,3-hexadiene.
base
CH 3 CH 2 CH 2 CH CH CH2 Br CH 3 CH 2 CH 2 CH C CH 2
 HBr
1-bromo-2-hexene (E- or Z-) 1,2-hexadiene

Memorizing Chapter 15.3.1, 1-bromo-2-hexene can be prepared by bromination of 2-hexene with


N-bromosuccinimide.

(18.1) What products are obtained by bromination and hydrobromination of 1,3-butadiene?


Chapter 18.1
(18.2) How can the selectivity of hydrobromination of 1,3-butadiene be controlled?
Chapter 18.1
(18.3) Write the equation for a [42]-cycloaddition. What is the preparative significance of this reaction?
Chapter 18.2.1
(18.4) What product is expected from the reaction of 1,3-butadiene and trans-1,2-dibromoethene? Write the equation.
Chapter 18.2.1: [4+2]-Cycloaddition (DIELS-ALDER reaction) of 1,3-butadiene and trans-1,2-dibromoethene is
expected to stereospecifically produce trans-1,2-dibromo-4-cyclohexene (1,2-dibromocyclohex-4-ene).
Br Br Br
+
Br Br Br
1,3-butadiene trans-1,2-dibromoethene transition state trans-1,2-dibromocyclohex-4-ene

(19.1) cis-trans-Isomers do not exist for alkynes. Why?


Chapter 19.1
(19.2) What structurally isomeric alkynes exist for the molecular formula C8H14? Name them.
Chapter 19.1: 1-, 2-, 3- and 4-octyne are skeletally isomeric alkynes with molecular formula C8H14.

H C C CH 2 CH 2 CH 2 CH 2 CH 2 CH 3 H 3C CH 2 C C CH 2 CH 2 CH 2 CH 3
1-octyne 3-octyne
H 3C C C CH 2 CH 2 CH 2 CH 2 CH 3 H 3C CH 2 CH 2 C C CH 2 CH 2 CH 3
2-octyne 4-octyne

(19.3) Which dihaloalkanes are starting materials for the preparation of 1-hexyne? Formulate the equations for this.
Chapter 19.2.3: 1,1- or 1,2-dibromohexane are expected to undergo dehydrobromination, yielding 1-hexyne.
NaNH2 NaNH 2
H2C CH R or Br CH CH 2 R HC CH CH 2 CH 2 CH2 CH3 H C C CH2 CH2 CH 2 CH 3
HBr HBr
Br Br R = n-C4H9 Br Br
1,2- 1,1-dibromohexane 1-bromo-1-hexene 1-hexyne

(19.4) How can the (E)- and (Z)-isomer of 2-hexene be prepared? Formulate the equations.
Chapter 19.3.1: Starting from 2-hexyne (R = CH3 and n-C3H7 in the reaction equations), catalytic hydrogenation
affords cis-2-hexene while reduction with sodium in liquid ammonia is expected to yield the trans-isomer.
(19.5) Terminal alkynes are CH acids. Why? Suggest a reasonable synthesis of 3,5-octadiyne from a terminal alkyne.
Chapter 19.3.6: 3,5-Octadiyne is feasible by oxidative (GLASER-) coupling of 1-butyne (R = C2H5).
2
2 R C C H + 2 OH + 2 Cu H 3C CH2 C C C C CH 2 CH3 + 2 H2O + 2 Cu
R = C2H5 : 1-butyne 3,5-octadiyne
258 Working on Questions

(20.1) Draw the skeletal formulas of (a) bicyclo[4.4.0]decane, (b) bicyclo[3.3.1]nonane, and (c) spiro[5.2]octane.
Chapter 20.1: Fused cycloalkanes are usually portrayed by means of skeletal formulas (Chapter 8.2).

(a) (b) (c)

(20.2) Name the bicycles (1-5). Which kind of isomers are these cyclic hydrocarbons?

1 2 3 4 5

The fused cycloalkanes are named as 1: bicyclo[5.1.0]octane; 2: bicyclo[4.2.0]octane; 3: bicyclo[2.2.2]octane;


4: bicyclo[3.3.0]octane; 5: spiro[4.3]octane. These are skeletal isomers with molecular formula C8H14.
(20.3) Draw the conformers of (a) cyclopentane, (b) cyclohexane, and (c) cyclohexene, and describe their shapes.
Chapter 20.2.3, concerning (a); Chapter 20.2.4, concerning (b); Chapter 20.3.3, concerning (c)
(20.4) What isomers exist for (a) dimethylcyclopropane, (b) 1,3-dimethylcyclobutane, (c) 1,2-, 1,3-, and 1,4-
dimethylcyclohexane, and (d) bicyclo[4.4.0]decane? Draw skeletal formulas of the isomers and specify them.
Chapter 20.3.1, answering (a); Chapter 20.3.1, Table 20.1, answering (c); Chapter 20.3.2, answering (d);
(b) cis- and trans-isomers exist in the case of 1,3-dimethylcyclobutane; those can be portrayed as conformers
or as projection formulas.
H 1,3-dimethylcyclobutane CH 3 CH3

fast CH 3 H fast CH3


H3C CH3 H H H3C
cis- H3C CH3 CH 3 CH3 trans- H 3C

(20.5) Cyclohexane undergoes ring inversion at room temperature. Which conformers are involved? What happens
to the axial and equatorial substituents?
Chapter 20.2.4
(20.6) Bromocyclohexane with equatorial Br and bromocyclohexane with axial Br cannot be separated by distillation.
Why?
Chapter 20.2.4: At room temperature, the kinetic energy of the molecules overcompensates the energy barrier of
ring inversion, meaning that separation (e.g. by distillation) is not possible.
Br
ring inversion
Br

(21.1) Formulate equations to suggest the preparations of (a) bicyclo[4.1.0]heptane, (b) cyclopentene, and (c) cyclononene.
Chapter 21.1, answering (a): Bicyclo[4.1.0]heptane arises from [2+1]-cycloaddition of diazomethane (carbene
precursor) to cyclohexene.

+ [ ICH 2 ]
carbene
cyclohexane biicyclo[4.1.0]heptane
 N2

diazomethane ICH 2 N NI

Chapter 21.3, answering (b)


Chapter 21.6, answering (c): Ring-closing metathesis of 1,10-undecadiene is expected to produce cyclononene.
kat.
+

1,10-undecadiene cyclononene
Working on Questions 259

(21.2) Trans-1,2-dibromoethene is irradiated with UV light. What product is expected?


Chapter 21.2: [2+2]-Photocycloaddition of trans-1,2-dibromoethene is expected to produce cis-trans-cis- and
trans-trans-trans-1,2,3,4-tetrabromocyclobutane according to the orientations of the reactimg alkenes.
Br Br Br Br
Br
Br h Br h Br
+ +
Br Br Br Br
Br Br Br Br
cis-trans-cis- trans-trans-trans-
1,2,3,4-tetrabromcyclobutane

(21.3) A structural isomer of limonene could be a minor byproduct of the dimerization of isoprene. Which one?
Chapter 21.4: [4+2]-Cycloaddition of isoprene permits two orientations of the starting dienes, producing the
natural product limonene or the regioisomeric 1-methyl-5-(2-propenyl)cyclohexene.

limonene = 1-methyl-4-(2-propenyl)cyclohexene (non-superimposable mirror images)


isoprene (2-methyl-1,3-butadiene)

1-methyl-5-(2-propenyl)cyclohexene (non-superimposable mirror images)

(21.4) cis-1,2-Dibromoethene reacts with 1,3-cyclohexadiene. What product is expected? Write the equation.
Chapter 21.4: [4+2]-Cycloaddition (DIELS-ALDER reaction, Chapter 18.2.1) of 1,3-cyclohexadiene and
cis-1,2-dibromoethene is expected to produce stereospecifically cis-1,2-dibromobicyclo[2.2.2]oct-4-ene.
The dienophile is able to approach with bromine atoms outside (exo) as shown or with bromine atoms inside
(endo), so that an exo- and/or an endo-stereoisomer may arise.
Br Br Br
+ exo- or endo-
Br Br Br
1,3-cyclohexadiene cis-1,2-dibromoethene cis-1,2-dibromobicyclo[2.2.2]oct-4-ene

(21.5) Write an equation to suggest a synthesis of 1,5-cyclooctadiene. What kind of reaction occurs?
Chapter 21.5, analogous reaction: A COPE sigmatropic rearrangement of 1,2-divinylcyclobutane (instead of
divinylcycopropane) is the key step to the target compound 1,5-cyclooctadiene.
COPE rearrangement

divinylcyclobutane 1,5-cyclooctadiene

(22.1) Hydrobromination of methylcyclopropane yields 2-bromobutane as the major product. Write the mechanism and
explain the regioselectivity. Look at Chapter 16.3.
Chapters 22.1, 16.3
(22.2) Bromine reacts with (a) cyclopropane, (b) cyclohexane, and (c) cyclohexene: What products are obtained?
Chapters 22.1, 22.2, 22.3.1
(22.3) Write the mechanism for the photochlorination of cyclohexane to give chlorocyclohexane. Look at Chapter 13.1.
Chapter 13.1: A cyclohexyl radical is the reactive intermediate in the chain reaction instead of the methyl radical.
(22.4) Suggest a one-step reaction to prepare pure cis-1,2-dimethylcyclopentane.
Chapter 22.3.2: Catalytic hydrogenation of 1,2-dimethylcyclopentene (pre-determining the cis-configuration)
produces cis-1,2-dimethylcyclopentane.
H
CH3
1,2-dimethylcyclopentene CH3
CH3 CH3
H
+ H H
cis-1,2-dimethylcyclopentane
Ni, Pd or Pt
260 Working on Questions

(22.5) Write equations to suggest the preparations of cis- and trans-1,2-cyclopentanediol.


Chapter 22.3.3, analogous reactions: Cyclopentene instead of cyclohexene reacts to cis- and trans-1,2-
cyclopentanediol, respectively.

(23.1) Which facts indicate that benzene is more than just a 1,3,5-cyclohexatriene?
Chapter 23.1.1, 23.1.2
(23.2) Explain the term resonance energy and its physical (thermodynamic) origin.
Chapters 23.1.2, 23.1.3
(23.3) Draw all formulas describing the benzene molecule and explain the term delocalized  bonding.
Chapter 23.1.3, Fig. 23.2
(23.4) Which properties characterize an aromatic compound?
Chapter 23.3
(23.5) Draw the skeletal formulas of (a) cyclobutadiene C4H4 and (b) cyclooctatetraene C8H8. Are these compounds
aromatic?
Chapter 23.3: The number of  electrons, 4 for cyclobutadiene (a) and 8 for cyclooctatetraene (b), does not follow
the HÜCKEL rule (4N + 2) meaning that both cycles are not aromatic. Moreover, bond distances in cyclobutadiene
are different, and cyclooctatetraene is not a planar molecule, existing as a crown and a boat conformer.

(a) cyclobutadiene (b) cyclooctatetraene: crown conformer boat conformer

(24.1) Draw the structural formulas of (a) 2,4,6-trinitrotoluene, (b) 4,4´-dihydroxydiphenylmethane, and (c) 4-nitro-o-cresol.
Chapters 24.1, 24.2: Formulas a-c are correct.
CH3 CH 3
O2N NO2 OH
4 1 1' 4'
HO CH2 OH
(c) 4-nitro-o-cresol
NO2 (a) 2,4,6-trinitrotoluene (b) 4,4´-dihydroxydiphenylmethane (p-nitro-o-cresol) NO2

(24.2) What are other names for the industrial chemical known as bisphenol A and the contact insecticide known as DDT?
CH 3 CCl3
4 1 1' 4' 4 1 1' 4'
bisphenol A HO C OH DDT Cl C Cl
CH 3 H

Chapters 24.1, 24.2: Bisphenol A is 2,2-bis(4-hydroxyphenyl)propane; DDT (from Dichlorodiphenyltrichlorethane)


is 1,1,1-trichloro-2,2-bis(4-chlorophenyl)ethane.
(24.3) Which structurally isomeric aromatic compounds with an OH group and the molecular formula C7H8O exist?
Chapters 24.1, 24.2: Benzylalcohol (-hydroxytoluene), o-, m- and p-cresol are skeletal isomers with molecular
formula C7H8O and hydroxy groups.
CH2OH OH OH OH
CH3

CH3
benzylalcohol CH3
(-hydroxytoluene) 1,2- or o- 1,3- or m- 1,4- or p-cresol

(24.4) What kind of chemicals are produced by the so-called platforming processes?
Chapter 24.3.1
(24.5) Suggest a synthesis of hexaethylbenzene from an alkyne. Formulate the equation.
Chapter 24.3.2, analogous reaction: Catalytic cyclotrimerization of 3-hexyne will produce hexaethylbenzene.
R
R Ni(CO) 4
R (pressure, heat)
R = CH 2CH 3 : 3-hexyne hexaethylbenzene
R
R
R
Working on Questions 261

(25.1) Formulate a detailed reaction mechanism for the electrophilic substitution of benzene by an electrophile E.
Chapter 25.1
(25.2) Write this mechanism for (a) a bromination to give bromobenzene and (b) an alkylation to give isopropylbenzene.
Chapter 25.2, answering (a): Electrophilic bromination of benzene with bromine (Br2 instead of Cl2) in the
presence of a LEWIS acid (FeBr3 instead of AlCl3) produces bromobenzene.
Chapter 25.3, answering (b)
(25.3) Starting with benzene, how would you prepare (a) t-butylbenzene, (b) nitrobenzene, and (c) acetophenone?
(a) Chapter 25.3: In analogy to the synthesis of isopropylbenzene, t-butylbenzene can be prepared by
electrophilic alkylation of benzene with methylpropene (isobutylene), the precursor of the intermediate
electrophilic t-butyl cation (t-butylium ion), and sulfuric acid as catalyst.
CH 3 CH 3
C + H 2SO4 C + HSO4
H3C CH2 H 3C CH 2 H
methylpropene t-butylium ion

CH 3
H + C C(CH3)3
H 3C CH 3  [H ]
t-butylbenzene

Chapter 25.5, answering (b)


Chapter 25.4, answering (c)
(25.4) Sulfur trioxide is electrophilic at the sulfur atom. Why? Write the mechanism of the electrophilic sulfonation of benzene.
Chapter 25.6

(26.1) Explain the ()-M and ()-M effect. In which manner do they influence the reactivity of substituted benzenes?
Chapters 26.1, 26.2
(26.2) Phenol reacts with (a) nitric acid and sulfuric acid and (b) fuming nitric acid. Which products are formed?
Chapter 26.2, answering (a)
(b): The (M) effect of the OH group activates the o-, o´- and p-position for electrophilic substitution.
Thus, fuming nitric acid nitrates phenol to yield 2,4,6-trinitrophenol (picric acid, Chapter 52.4.3).
(26.3) Salicyclic acid (o-hydroxybenzoic acid) is nitrated. Which products are expected?
Chapter 26.2: The OH group directs the electrophile to the o-, o´- und p- positions, the deactivating carboxy
group, however, permits the electrophile to enter m-positions. To conclude, electrophilic nitration of salicylic
acid is expected to produce 3-nitro-, 5-nitro- and 3,5-dinitrosalicylic acid.
CO2H CO2H CO2H CO2H
OH OH OH OH

NO2 O2N O2N NO2


salicylic acid 3-nitrosalicylic acid 5-nitrosalicylic acid 3,5-dinitrosalicylic acid

(26.4) Which alkene is an appropriate precursor of an intermediate electrophile to prepare 2,4,6-tri-t-butylphenol?


Chapter 25.3 and answer 25.3: Methylpropene (isobutylene) is protonated in situ to the t-butylium ion as
electrophile. Thus, 2,4,6-tri-t-butylphenol is produced by bubbling isobutylene into a solution of phenol.
(26.5) Which products arise from the nitration of (a) bromobenzene and (b) nitrobenzene?
Chapter 26.2: (a) Bromobenzene can be nitrated to 2-nitro-, 4-nitro-, 2,4-dinitro- and 2,4,6-trinitrobromobenzene
due to the (M) effect of Br; (b) nitration of nitrobenzene is expected to produce 1,3-dinitro- and 1,3,5-
trinitrobenzene due to the (M) effect of deactivating NO2.
(26.6) Which reactions are suitable for the preparation of 3-nitrobenzenesulfonic acid from benzene?
Chapter 26.2: 3-Nitrobenzenesulfonic acid arises either from nitration of benzene to nitrobenzene with nitric
acid/sulfuric acid and subsequent sulfonation with sulfur trioxide or from sulfonation of benzene and subsequent
262 Working on Questions

nitration. Both substituents deactivate due to their (M) effects, permitting second substitution in m-positions.
SO3H SO3H
[NO2 ] SO3 [NO2 ] SO3

NO2 NO2
3-nitrobenzenesulfonic acid

(26.7) Alkyl groups at benzene direct electrophiles to the o- and p-positions. How would you prepare 2,4,6-trinitrotoluene?
Chapter 26.2: Nitration of toluene with fuming nitric acid produces the explosive 2,4,6-trinitrotoluene (TNT).

(27.1) In aryl halides, carbonhalogen bonds are shorter than in alkyl halides. Look at Chapters 3.2, 26.1, and 27.1
and explain why.
It is assumed that the carbon atom attached to halogen in aryl halides provides sp2 hybrid orbitals for overlapping
and these are more compact than sp3 hybrid orbitals. In the same context, the carbonhalogen bond is a partial
double bond because of the (M) effect of the halogen atom as shown by the resonance formulas in Chapter 26.1,
replacing NH2 in aniline by halogen with three non-bonding electron pairs.
(27.2) 1-Fluoro-2,4-dinitrobenzene (2,4-dinitrofluorobenzene) reacts with (a) water and (b) ammonia. What products are
formed? Draw resonance formulas to explain the influence of the nitro groups on the reactive intermediate.
Chapter 27.1: In analogy to the nucleophilic substitution of the chloride anion in o-nitrochlorobenzene by
hydroxide, nucleophilic substitution of the fluoride anion in 2,4-dinitrofluorobenzene by water (a) as a
nucleophile produces 2,4-nitrophenol and by ammonia (b) as a nucleophile produces 2,4-dinitroaniline.
+ H2O + NH3
O2N OH O2N F O2N NH 2
 HF  HF
NO2 (a) NO2 (b) NO2
2,4-dinitrophenol 2,4-dinitrofluorobenzene 2,4-dinitroaniline

(27.3) Propylbenzene reacts with bromine in the presence of UV radiation (a) or ferric chloride in the cold (b).
Outline the mechanisms and explain the regioselectivity of both reactions.
Chapter 27.2: Propylbenzene reacts with bromine (a) under UV light to give -bromopropylbenzene (major
product, radical substitution) and (b) in the presence of ferric bromide to produce predominantly
p-bromopropylbenzene (electrophilic substitution).
Br
+ Br 2 , h + Br 2 , FeBr 3
Br
Br HBr HBr
-bromopropylbenzene (a) propylbenzene (b) p-bromopropylbenzene o-bromoproylbenzene
major minor

(27.4) Formulate equations to suggest the preparation of t-butylcyclohexane from benzene and methylpropene.
t-Butylbenzene is prepared from benzene and methylpropene (isobutylene, answer 25.3). Catalytic
hydrogenation of t-butylbenzene produces t-butylcyclohexane (Chapter 27.3).
CH 3 CH 3
C + H 2SO4 C + HSO4
H 3C CH 2 H 3C CH 2 H
methylpropene t-butylium ion

CH 3
+ 3 H2 (kat.)
H + C C(CH 3)3 C(CH 3)3
H 3C CH 3  [H ]
t-butylbenzene t-butylcyclohexane

(27.5) Write equations to describe the reactions of (a) benzene with sodium in liquid ammonia and a proton donor, and
(b) of toluene with chromium trioxide (CrO3).
Chapter 27.3
Working on Questions 263

(28.1) Draw the formulas of tetracene and its isomers containing benzenoid rings with autonomous electron sextets.
Two non-equivalent resonance formulas can be written for linearly fused tetracene, each having one autonomous
 electron sextet. Angularly fused chrysene and benzo[a]anthracene are hybrids of two resonance formulas,
each one containing two benzenoid rings with autonomous  electron sextets. One additional resonance formula
with one benzenoid autonomous  electron sextet can be written for benzo[a]anthracene.

benzo[a]anthracene
tetracene

chrysene

(28.2) What products are obtained when naphthalene is reacted with electrophiles under mild conditions? Give reasons
and examples.
Chapter 28.3
(28.3) What is the driving force behind the DIELS-ALDER reaction of anthracene with maleic anhydride?
Chapter 28.5: The generation of an additional benzenoid ring is the driving force for the DIELS-ALDER reaction of
anthracene and maleic acid anhydride.
(28.4) Anthracene and phenanthrene react with (a) bromine, (b) oxidizing reagents, and (c) reducing reagents. Write
equations for these reactions.
Chapter 28.5: 9,10-Positions are reactive centers so that the second benzenoid ring is generated or preserved.
(28.5) Polycyclic arenes are known to be carcinogenic. Which reactions produce the carcinogens? Formulate
these reactions.
Chapter 28.6

(29.1) Which types of non-benzenoid aromatic compounds do you know? Draw formulas.
Chapter 29.1, first section, Table 29.1
(29.2) Write equations describing the preparations of (a) potassium cyclopentadienide and (b) cycloheptatrienium bromide.
Chapter 29.1.2, answering (a); Chapter 29.1.3, answering (b)
(29.3) What are -electron-excessive and -electron-deficient aromatics? Give examples and draw formulas.
Chapter 29.1.3, last section
(29.4) (a) [10]Annulene does not exist. Why? (b) Write equations describing the preparations of [14]- and [18]annulene.
(a) [10]-Annulene could not be synthesized. Three configurational isomers can be formulated. The all-cis-
configuration would have an internal bond angle of 144° which is much larger than the interorbital angle of sp2
hybrid orbitals (120°). This large angle strain would destabilize the all-cis- as well as the mono-trans-
isomer, although the latter has partly relaxed. No angle strain would destabilize the bis-trans-isomer but strong
repulsion of the inner hydrogen atoms would prevent coplanarity of the ring as required for aromaticity.
Only bridged derivatives such as 1,6-methano[10]annulene have been prepared.

H
H H

all-cis- mono-trans- bis-trans10]-annulene 1,6-methano[10]-annulene

Chapter 29.2, answering (b)


264 Working on Questions

(30.1) Draw all structural isomers of (a) C3H7Br and (b) C4H9Cl, and classify them as primary, secondary, or tertiary.
Chapter 30.1, Tables 30.1 and 7.2 exemplify formulation, nomenclature and classification of alkyl halides (a)
and (b).
primary CH3 CH2 CH2 Br 1-bromopropane CH3 CH2 CH2 CH2 Cl 1-chlorobutane H3C CH CH2 Cl 1-chloro-2-methylpropane
(n-propyl bromide) (n-butyl chloride) (isobutyl chloride)
CH3
secondary H3C CH CH3 2-bromopropane CH3 CH2 CH CH3 2-chlorobutane
(isopropyl bromide) (sec-butyl chloride)
Br Cl
CH3
tertiary H3C C CH3 2-chloro-2-methylpropane
(a) (b) (t-butyl chloride)
Cl

(30.2) 2-Bromopentane can be prepared by substitution and addition. Write the equations for these reactions.
Chapters 30.2.1, 30.2.2: 2-Bromopentane can be prepared by bromination of pentane (radical substitution) or by
hydrobromination of 1-pentene (electrophilic addition). Radical substitution will also produce the regioisomers
3-bromopentane and 1-bromopentane (minor) while hydrobromination is expected to be more regioselective,
predominantly producing 2-bromopentane (MARKOVNIKOV´s rule, Chapters 16.3, 30.2.1).
 Br2 , h  HBr
CH3 CH2 CH2 CH2 CH3 CH3 CH 2 CH2 CH CH3 CH3 CH2 CH2 CH CH2
 HBr
pentane 2-bromopentane Br 1-pentene

(30.3) What kind of compounds are the so-called ozonekillers? Write equations for the ozone-killing reactions.
Chapter 30.2.4
(30.4) Explain the term inductive effect and describe its influence on the reactivity of alkyl halides.
Chapter 30.3
(30.5) 2-Bromopentane reacts with (a) potassium iodide, (b) potassium cyanide, and (c) ammonia. Write the equations.
Chapter 30.4.1: 2-Bromopentane undergoes nucleophilic substitution to (a) 2-iodopentane (FINKELSTEIN reaction),
(b) 2-cyanopentane (2-pentylcyanide, 2-pentylnitrile) and (c) 2-pentylammonium bromide.
I + NH 3
CH 3 CH 2 CH 2 CH CH 3 CH 3 CH 2 CH 2 CH CH 3 CH 3 CH 2 CH 2 CH CH 3
 Br
I Br NH 3 Br
(a) (c)
2-iodopentane (b) + CN  Br 2-pentylammonium bromide

CH 3 CH 2 CH 2 CH CH 3 2-cyanopentane
CN

(30.6) 2-Bromobutane reacts with potassium hydroxide. Which products can be formed?
Chapters 30.4.1 and 30.4.2: In keeping with the HSAB principle,  elimination (dehydrobromination) to 1-
and 2-butene (cis- and trans-) is expected to compete with nucleophilic substitution to 2-butanol.
 OH  OH
CH3 CH2 CH2 CH CH3 CH3 CH2 CH CH3 CH3 CH2 CH CH2
 Br  Br 1-butene
OH Br
2-butanol  H2O
H3C CH3 H3C H
nucleophilic substitution -elimination
C C C C
H H H CH3
(Z)- or cis- (E)- or trans-
2-butene

(30.7) Formulate the reaction of 1-bromobutane with magnesium in ether including the change of polarity of C-1.
Chapter 30.4.3: Magnesium metalates 1-bromobutane, producing 1-butylmagnesium bromide. Metalation
reverses the polarity of the electrophilic  carbon atom () of 1-brombutane to a nucleophilic center ().
electrophilic C atom Umpolung nucleophilic C atom

+   ++ 
ether
CH 3 CH 2 CH2 CH2 Br + Mg CH3 CH2 CH 2 CH 2 Mg Br
1-bromobutane 1-butylmagnesium bromide
Working on Questions 265

(30.8) Can the ()-I effect of a halogen influence the regioselectivity of the electrophilic substitution of halobenzenes?
The inductive effect, the ()-I effect, of halogens induces a partial positive charge at the carbon atom attached to
halogen in halobenzenes; this influence significantly decreases with distance. The longer-ranging ()-M effect of
halogen atoms is the predominating influence [()-M > ()-I ], enhancing electron density at o, o´ and p positions,
directing second substitution to these positions as outlined in Chapter 26.2.
_ _ _ _ _

X X = F, Cl, Br, I I XI I XI XI XI XI
+

()-I effect of halogen ()-M effect of halogen

(31.1) Draw equations to outline the mechanisms and rates of nucleophilic substitution.
Chapters 31.1 and 31.2
(31.2) Formulate the reactions of (a) t-butyl bromide with iodide (SN1), and (b) 1-bromobutane with hydroxide (SN2).
Chapter 31.2, answering (a); Chapter 31.1, answering (b): general mechanisms
(31.3) Which mechanism is probable for the reaction of -bromoethylbenzene with sodium hydroxide?
Chapter 31.1: SN1 is also reasonable. The intermediate benzylic carbenium ion is resonance stabilized as
formulated for the phenylethyl radical in Chapter 27.2; replace the unpaired electron by the positive charge,
a singly occupied p orbital by a vacant p orbital  to the phenyl ring.
(31.4) What happens in the so-called WALDEN inversion? Why does this not play a role in substitutions with an
SN1 mechanism?
Chapter 31.1, last section

(32.1) Which reactions open access to organometallic compounds? Formulate general equations.
Chapter 32.2
(32.2) Suggest reactions to prepare (a) butyllithium, (b) phenyllithium, and (c) butynylmagnesium bromide.
Chapter 32.2.1, answering (a) and (b); Chapter 32.2.4, answering (c): 1-Butynylmagnesium bromide is
prepared by hydrogen-metal exchange, reacting 1-butyne and ethylmagnesium bromide.

H 3C CH 2 C C H + H 3C CH 2 MgBr H 3C CH 2 C C MgBr + C 2H 6
1-butyne ethylmagnesium bromide 1-butynylmagnesium bromide

(32.3) Formulate all reactions necessary to prepare allylmagnesium bromide from propene. Look at Chapter 15.3.1.
Chapters 32.2.1, 15.3.1: Allyl bromide (3-bromo-1-propene), obtained by WOHL-ZIEGLER bromination of propene
with N-bromosuccinimide, is metalated with magnesium to produce allylmagnesium bromide.
O O
CCl4 ether
H 2C CH CH 3 + N Br H 2C CH CH 2 Br + N H H 2C CH CH 2 Br + Mg H 2C CH CH 2 MgBr
allylmagnesium bromide
O 3-bromo-1-propene O
propene N-bromosuccinimide (allyl bromide) succinimide

(32.4) Suggest syntheses of (a) allylcyclopropane and (b) phenylcyclopentadiene.


Chapter 32.3: Two pathways are possible for both target compounds. Allylcyclopropane (a) is expected to arise
from reacting allyl bromide with cyclopropylmagnesium bromide or bromocyclopropane with allylmagnesium
bromide. Phenylcyclopentadiene can be prepared from 1-bromocyclopentadiene and phenylmagnesium bromide
or from bromobenzene and cyclopentadienylmagnesium bromide (Chapter 32.2.4).
+   +
H2C CH CH2 Br + BrMg H2C CH CH2 H 2C CH CH 2 MgBr + Br
 MgBr2  MgBr2
allyl bromide cyclopropylmagnesium bromide allylcyclopropane allylmagnesium bromide bromocyclopropane
(a)

+   
Br + Br Mg Mg Br + Br +
 MgBr2  MgBr2
bromocyclopentadiene phenylmagnesium bromide phenylcyclopentadiene cyclopentadienylmagnesium bromide bromobenzene
(b)
266 Working on Questions

(33.1) 1-Butanol has a higher boiling point (74 g/mol, 118 °C) at normal pressure than pentane (72 g/mol, 36 °C). Why?
Chapter 33.2
(33.2) Draw the formulas of all structurally isomeric alcohols (7) with the molecular formula C5H12O and classify them
as primary, secondary, or tertiary.
Eight structurally isomeric alcohols (primary: 4, secondary: 3, tertiary:1) with the molecular formula C5H12O exist:
CH 3 CH 3 CH 3
H 3C CH 2 CH 2 CH 2 CH 2 OH H 3C CH CH 2 CH 2 OH HO H 2C CH CH 2 CH 3 H 3C C CH 2 OH
1-pentanol 3-methyl-1-butanol 2-methyl-1-butanol CH 3
2,2-dimethyl-2-propanol
H 3C CH 2 CH 2 CH CH 3 CH 3 CH 3 (neopentylalcohol)
2-pentanol OH H 3C CH CH CH 3 H 3C C CH 2 CH 3
OH OH
H 3C CH 2 CH CH 2 CH 3 3-methyl-2-butanol 2-methyl-2-butanol
3-pentanol OH

(33.3) Draw formulas of all structurally and configurationally isomeric methylcyclopentanols and name them.
Structurally isomeric 2- and 3-methylcyclopentanol exist as configurational isomers (cis- and trans-). One of the
envelope conformers (Chapter 20.2.3) is portrayed. Additionally, each cis- and trans-isomer exists as a pair of
non-superimposible mirror images (enantiomers: Chapter 44; cis-trans-pairs are diastereomers: Chapter 46).
OH OH OH OH
CH 3 H 3C

CH3 H 3C

OH HO OH HO OH HO OH HO

CH3 H3C CH3 H3C


H 3C CH3 H3C CH3
trans- cis- trans- cis-
2-methylcyclopentanol 3-methylcyclopentanol

(33.4) Write equations describing the preparations of (a) t-butyl alcohol, (b) 2-methyl-1-propanol, and (c) allyl alcohol.
(a) Chapter 33.3.3, answering (a); Chapter 33.3.4, answering (b); Chapter 33.3.6, answering (c)
(33.5) Suggest syntheses of (a) 1-pentanol (n-pentyl alcohol) and (b) 2-pentanol from 1-pentene.
Chapters 33.3.3, 33.3.4, analogous reactions
(a) Hydroboration of 1-pentene and subsequent oxidation is expected to produce 1-pentanol.
H3C CH2 CH2 CH CH 2 C 3H7 CHH CH2 + 2 C5H10 + 3 H2O2
+ (C 3H 7 CH 2 CH 2)3 B 3 H3C CH 2 CH2 CH2 CH2 OH
H BH 2 BH2 B(OH) 3
pentylborane tripentylborane 1-pentanol

(b) Acid-catalyzed hydration of 1-pentene regioselectively produces 2-pentanol (MARKOVNIKOV´s rule).


H 7C 3 C 3H 7
+ H2O
H 3C CH 2 CH 2 CH CH 2 + [H ] C CH 3 H C CH 3 H 3C CH 2 CH 2 CH CH 3
H  [H ]
OH 2 OH
1-pentene more stable carbenium ion 2-pentanol

(33.6) Write equations describing the preparations of (a) primary and (b) secondary alcohols by reduction.
Chapter 33.3.5: (a) Aldehydes are reduced to primary, (b) ketones are reduced to secondary alcohols.
(33.7) Which alcohols are obtained by reacting ethylmagnesium bromide with (a) benzaldehyde and (b) acetophenone?
Write equations.
Chapter 33.3.7: analogous reactions: Ethylmagnesium bromide reacts with (a) benzaldehyde to give
1-phenyl-1-propanol (secondary) and (b) with acetophenone to give 2-phenyl-2-butanol (tertiary alcohol).
O OMgBr OH
ether + H2O
(a) C + H 3C CH2 MgBr CH CH2 CH 3 CH CH 2 CH 3
 Mg(OH)Br
H
benzaldehyde 1-phenyl-1-propanol

acetophenone 2-phenyl-2-butanol
O OMgBr OH
ether + H2O
(b) C + H 3C CH 2 MgBr C CH 2 CH3 C CH2 CH 3
 Mg(OH)Br
CH 3 CH3 CH 3
Working on Questions 267

(33.8) Nucleophilic hydroxylation of allyl bromide follows a SN1 mechanism. Why? Look at Chapter 31.2.
A SN1 mechanism for the nucleophilic hydroxylation of allyl bromide to allyl alcohol is reasonable because the rate-
determining step involves dissociation of the bromide anion, leaving an intermediate resonance-stabilized allyl
cation.
slow + OH
H2C CH CH 2 Br H2C CH CH2 H 2C CH CH2 H 2C CH CH2 OH
 Br fast
allyl bromide resonance stabilized allyl cation allyl alcohol

(34.1) Six isomeric cyclohexanediols exist (structural and configurational). Draw all of them; look at Table 20.1 for
reference.
Chapter 20.3: Replace R by OH in Table 20.1
(34.2) Formulate a synthesis of (a) trans- and (b) cis-1,2-cyclopentanediol from cyclopentene.
Chapter 34.1.1, analogous reaction: CycIopentene reacts instead of cyclohexene with (a) peroxycarboxylic acid
via epoxide to produce trans-1,2-cyclopentanediol and with (b) osmiumtetroxide or permanganate to produce
cis-1,2-cyclopentanediol.
(34.3) Write equations to describe a synthesis of glycerol from propene.
Chapter 34.1.2
(34.4) What product is formed by reacting magnesium with acetone (propanone)? Write the equation.
Chapter 34.1.3
(34.5) Formulate the cleavage of an ,-diol with lead tetraacetate. Which products (formulas) do you expect from
the oxidative cleavage of (a) 1,2-cyclobutanediol, (b) 1,2-cyclopentanediol, and (c) 1,2-cyclohexanediol?
Chapter 34.2, analogous reactions: oxidative cleavage of 1,2-cycloalkanediols produces dialdehydes such as
butanedial (succindialdehyde, Table 47.1) from (a) 1,2-cyclobutanediol, pentanedial from (b) 1,2-cyclopentanediol
and hexanedial from (c) 1,2-cyclohexanediol.
O O O
OH (a) OH (b) OH (c)
CH CH CH

CH CH CH
OH O
OH O OH O
1,2-cyclobutanediol butanedial 1,2-cyclopentanediol pentanedial 1,2-cyclohexanediol hexanedial
cis- or trans- (succindialdehyde) cis- or trans- cis- or trans-

(35.1) Hydrogen chloride dissolves in cold ethanol. Which reaction takes place? What happens upon heating this
mixture? Look at Chapter 15.1.2.
Chapter 35.1: Ethyloxonium chloride is formed. It is stable in the absence of water and undergoes dehydration
upon heating to produce gaseous ethene which escapes (Chapter 15.1.2).
(35.2) Anhydrous ethanol reacts with sodium metal. Write the reaction equation. What happens upon addition of water?
Chapter 35.1
(35.3) Formulate the oxidation of (a) a primary alcohol (e.g. 1-pentanol) and (b) a secondary alcohol (e.g. 3-pentanol).
Chapter 35.2: 1-Pentanol (a) undergoes oxidation to pentanoic acid (valeric acid) via pentanal (set R = C4H9 in
the reaction equation); oxidation of 3-pentanol (b) yields 3-pentanone (set R = C2H5 in the reaction equation).
(35.4) Formulate the catalytic oxidation of a primary alcohol with copper metal as catalyst.
Chapter 35.2
(35.5) Pure methanol is reacted with (a) acetic acid and (b) fuming sulfuric acid. What products are obtained?
Chapter 35.4, analogous reactions: Methanol (R = CH3) reacts with (a) anhydrous glacial acetic acid (R' = CH3)
to produce acetic acid methyl ester (methyl acetate), with (b) fuming sulfuric acid (H2S2O7 = H2SO4 . SO3) to give
dimethyl sulfate (diester) via methyl hydrogensulfate (monoester).
 H2O O  H2O O
2 CH 3 OH + HO SO3H CH 3 O S OH + CH 3 OH CH3 O S O CH 3
O O
methyl hydrogensulfate dimethyl sulfate

(35.6) The name nitroglycerin (nitroglycerol) is misleading. Draw the formula in order to explain this.
Chapter 35.4, last section
268 Working on Questions

(36.1) Write the mechanism of alcohol dehydration.


Chapter 36.1
(36.2) Which one of the alcohols (a) t-butyl alcohol or (b) 1-butanol undergoes dehydration more readily? Why?
Chapter 36.1
(36.3) Three products are expected from the dehydration of 2-pentanol. Which one will be a minor product?
Chapter 36.1, analogous reaction: In keeping with SAYTZEFF´s rule, 2-pentanol is expected to undergo
dehydration to yield cis- and trans-2-pentene (major) and 1-pentene (minor).
H3C H2C H
trans- C C
+ [H ] H CH3
H3C CH2 CH2 CH CH3 H3C CH2 CH2 CH CH3 H3C CH2 CH2 CH CH2
 H2 O  [H ] H3C H2C CH3
OH 1-pentene
2-pentanol cis- C C
H H
2-pentene (major products)

(36.4) What product is obtained by the acid-catalyzed dehydration of 3,3-dimethyl-2-butanol? Formulate the mechanism.
Chapter 36.1, analogous reaction: WAGNER-MEERWEIN rearrangement, involving anionotropic 1,2-methyl shift of
the primarily formed carbenium ion to the higher alkylated and more stable carbenium ion, produces
2,3-dimethyl-1-butene and not 3,3-dimethyl-1-butene.
CH 3 CH 3 CH 3 H3C CH3 CH3
+ [H ]
H 3C C CH CH3 H 3C C CH CH 3 H 3C C CH CH 3 C C not H 3C C CH CH2
 H2O  [H ]
CH 3 OH CH 3 CH3 H3C CH3 CH3
3,3-dimethyl-2-butanol lower alkylated carbenium ion higher alkylated carbenium ion 2,3-dimethyl-2-butene
less stable more stable

(36.5) Suggest a method to prepare t-butyl methyl ketone from acetone? Which reactions are needed?
Chapters 36.2, 34.1.3, basic reaction: Bimolecular reduction of acetone (propanone) with magnesium yields
2,3-dimethyl-2,3-butandiol (pinacol), which undergoes pinacol rearrangement to the target t-butyl methyl ketone.
H3C CH3 H 3C CH3
H 3C CH 3 H 3C CH3 + 2 H2O [H ]
C C H3C C C CH3 C C CH3
+ Mg +  Mg(OH) 2  H2 O
O O HO OH O CH3
2,3-dimethyl-2,3-butanediol 3,3-dimethyl-2-butanone
(pinacol) (pinacolone, t-butyl methyl ketone)

(37.1) Formulate the synthesis of diethyl ether from ethanol. Which reaction, producing ethene, competes?
Chapter 37.3.1: Monomolecular dehydration of ethanol yielding gaseous ethene competes with the bimolecular
dehydration to diethyl ether.
(37.2) Write equations to suggest the preparation of (a) ethyl phenyl ether and (b) allyl propyl ether.
Chapter 37.3.2, analogous reactions: WILLIAMSON synthesis permits preparation of ethyl phenyl ether (a) from
sodium phenoxide (from phenol, Chapter 52.2) as the nucleophile and bromoethane (SN2); the bromide in
bromobenzene cannot be readily substituted by nucleophiles (Chapter 27.1). Allyl propyl ether (b) arises from
reaction of sodium popanolate (sodium propoxide) and allyl bromide (SN1), alternatively from sodium allyl
alcoholate (from allyl alcohol) and 1-brompropane.

_ (a) CH2 CH3 _ (b)


OI
_ Na + Br CH2 CH3 O _ Na + Br CH2 CH CH2
H3C CH2 CH2 OI O
 NaBr  NaBr
sodium phenoxide bromoethane ethyl phenyl ether sodium propoxide allyl bromide allyl propyl ether

(37.3) Ethers should be stored protected from light and air. If not, which reaction may take place?
Chapter 37.4.2
(37.4) What products are expected from the cleavage of (a) diisopropyl ether and (b) diallyl ether with hydrogen iodide?
Chapter 37.4.3 (mechanisms): Cleavage of diisopropyl ether (a), R = CH(CH3)2, with hydrogen iodide HI is
expected to produce 2-iodopropane and 2-propanol (SN2). Cleavage of diallyl ether (b), R = CH2CH=CH2,
with hydrogen iodide HI is expected to yield allyl iodide and allyl alcohol (probably SN1 involving resonance-
stabilized allyl cation).
Working on Questions 269

(38.1) Draw the structural formulas of all isomeric amines with the molecular formula C5H13N and classify them.
Chapter 38.1: Eight primary, six secondary and three tertiary amines with the molecular formula C5H13N exist.
primary amines CH 3
H3C CH2 CH 2 CH2 CH 2 NH2 H3C CH2 CH 2 CH CH 3 H 3C CH 2 CH CH 2 CH3 H 2N H 2C CH CH 2 CH 3
1-aminopentane NH2 NH 2 1-amino-2-methylbutane
(n-pentylamine) 2-aminopentane 3-aminopentane
CH 3 CH 3 CH 3 CH 3
H3C CH CH 2 CH2 NH 2 H3C C CH 2 CH 3 H 3C CH CH CH 3 H3C C CH 2 NH 2
1-amino-3-methylbutane NH2 NH 2 CH3
(isopentylamine) 2-amino-2-methylbutane 2-amino-3-methylbutane 1-amino-2,2-dimethylpropane
(t-pentylamine) (neopentylamine)

secondary amines CH 3 CH 3
H3C CH2 CH 2 CH2 NHCH 3 H3C CH 2 CH CH3 H 3C CH CH 2 NHCH 3 H3C C NHCH 3
1-(N-methylamino)butane NHCH 3 1-(N-methylamino)2-methylpropane CH3
(butylmethylamine) 2-(N-methylamino)butane (isobutylmethylamine) t-butylmethylamine
CH3
H3C CH2 NH CH 2 CH2 CH 3 H3C CH2 NH CH CH 3
ethylpropylamine ethylisopropylamine

tertiary amines CH 3 CH 3
H3C CH2 CH 2 N(CH 3)2 CH 3 CH N(CH3)2 H 3C CH 2 N CH2 CH3
1-(N,N-dimethylamino)propane 2-(N,N-dimethylamino)propane diethylmethylamine
(dimethylpropylamine) (isopropyldimethylamine)

(38.2) Explain the geometry of the trimethylamine molecule. Where is the non-bonding electron pair?
Chapter 38.2, Fig. 38.1
(38.3) Alkylation of ammonia is not a clean method to prepare amines. Why?
Chapter 38.3.1
(38.4) N-Methylpiperidine (Chapter 38.1) reacts with iodomethane. Formulate the reaction. What product is obtained?
Chapter 38.3.1, analogous reaction: N-Methylpiperidine as nucleophile substitutes iodide in methyl iodide (SN2).
A tetraalkylammonium salt, N,N-dimethylpiperidinium iodide crystallizes.
CH3
_ CH3
N + CH3 I N I N I
CH3 CH3
CH3
N-methylpiperidine N,N-dimethylpiperidinium iodide

(38.5) Suggest a method to prepare pure n-butylamine.


Chapter 38.3.2, analogous reaction:. GABRIEL synthesis permits the preparation of n-butyl amine by reacting
potassium phthalimide as the nucleophile with 1-bromobutane via 1-phthaloylaminobutane.
O O
+ H 2NNH 2
N K + Br CH 2 CH2 CH2 CH 3 N CH 2 CH 2 CH2 CH 3 H2N CH2 CH2 CH 2 CH 3
 KBr O

O 1-bromobutane O NH 1-aminobutane

potassium phthalimide (n-butyl bromide) 1-phthaloylaminobutane NH (n-butylamine)
O

(38.6) How would you prepare benzylamine (-aminotoluene) from (a) benzonitrile and (b) benzyl bromide?
Chapters 38.3.2, 38.3.3, analogous reactions: Benzylamine can be prepared from (a) benzonitrile by
catalytic hydrogenation (phenyl instead of o-toluyl) or by GABRIEL synthesis (b) from benzyl bromide and
potassium phthalimide (answer 38.5, benzyl bromide = -hydroxytoluene instead of n-butyl bromide).

(39.1) Aniline is a much weaker base than cyclohexylamine and other alkylamines. Why?
Chapter 39.1
(39.2) Formulate the mechanism of diazotization of primary amines. Explain the stability of arenediazonium ions.
Chapter 39.2
(39.3) Which simple test tube reaction can distinguish between (a) cyclohexylamine and (b) aniline?
Chapter 39.2: (a) Cyclohexylamine gives a positive VAN SLYKE reaction (evolution of gaseous N2). (b) Aniline
undergoes diazotization producing a stable diazonium salt (no evolution of nitrogen at room temperature).
270 Working on Questions

(39.4) What products are expected from nitrosation of (a) N-methylaniline and (b) N,N-dimethylaniline with nitrite and acid?
Chapter 39.3: (a) N-Methylaniline undergoes nitrosation to yield N-nitroso-N-methylaniline as formulated for
dimethylaniline; (b) N,N-dimethylaniline is electrophilically nitrosated producing p-nitroso-N,N-dimethylaniline.
(39.5) Starting from -aminoethylcyclopropane, formulate equations for all steps to synthesize vinylcyclopropane.
Chapter 39.4, 39.5: Exhaustive methylation of -aminoethylcyclopropane with iodomethane produces
(2-cyclopropylethyl)trimethylammonium iodide. Anion exchange converts iodide to hydroxide which undergoes
HOFMANN elimination to produce vinylcyclopropane.
CH3 CH3 anion exchanger CH3
  + OH  
CH2 CH2 NI + CH3 I CH2 CH2 N CH3 I CH CH2 N CH3
 I
CH3 CH3 H + OH CH3
(2-cyclopropylethyl) dimethylamine (2-cyclopropylethyl)- (2-cyclopropylethyl)-
trimethylammonium iodide trimethylammonium hydroxide

2 CH3 I heat
H
ethenylcyclopropane
CH2 CH2 NH2 -aminoethylcyclopropane (vinylcyclopropane) C CH2 + N(CH3)3 + H2O

(39.6) What products are expected from the reactions of cyclohexanone with (a) ethylamine and (b) diethylamine?
Chapters 39.6, 39.7, analogous reactions: Cyclohexanone reacts (a) with ethylamine, yielding the imine
N-ethyliminocyclohexane, (b) with diethylamine, affording the enamine 1-(N,N-diethylamino)cyclohexene.
(a) (b)
O + H2N N O + HN N
 H2O  H2O
cyclohexanone ethylamine N-ethyliminocyclohexane cyclohexanone diethylamine 1-(N,N-diethylamino)-
cyclohexene

(39.7) Draw resonance formulas of an enamine to explain the nucleophilicity of the carbon  to the nitrogen.
Chapter 39.7
(39.8) Using the concept of reductive amination of carbonyl compounds, draw equations to outline the syntheses of
(a) N-isopropylcyclohexylamine and (b) N,N-diethylcyclohexylamine.
Chapter 39.8, analogous reactions: Reductive amination of cyclohexanone with isopropylamine
(2-aminopropane) is expected to produce (a) N-isopropylcyclohexylamine via the imine. A synthesis of
N,N-diethylcyclohexylamine (b) is formulated in the last reaction equation of the chapter.
(a) + H2 , kat.
O + H2N N N H
 H2O
H
cyclohexanone isopropylamine N-isopropyliminocyclohexane cyclohexylisopropylamine

(40.1) Outline the mechanism of azo coupling of an arenediazonium chloride with a donor-substituted benzene.
Chapter 40.1
(40.2) Outline a synthesis of methyl orange and explain the function of this compound as a pH indicator.
Chapter 40.1
(40.3) Which steps are necessary to synthesize phenylhydrazine from benzene as starting material? Write equations.
Chapter 40.1: Electrophilic nitration of benzene produces nitrobenzene (1, Chapter 25.5) which is reduced to
aniline (2, Chapter 38.3.3). Diazotization affords phenyldiazonium chloride (3, Chapter 39.2) which is reduced
with sodium sulfite to produce phenylhydrazinium chloride (4, Chapter 40.1). Liquid phenylhydrazine is obtained
upon addition of sodium hydroxide and extraction of phenylhydrazine.
1 2 3 4
H NO2 NH2 N NI Cl NH NH3 Cl

nitrobenzene aniline phenyldiazonium chloride phenylhydrazinium chloride

(40.4) Formulate all equations to describe the preparation of azobisisobutyronitrile (AIBN) from acetone.
Chapter 40.2
(40.5) What happens upon heating AIBN?
Chapter 40.2
Working on Questions 271

(40.6) Write an equation to outline a preparation of diazomethane.


Chapter 40.3
(40.7) Which resonance formulas describe the bonding state of diazomethane and which one of these explains the
thermolysis to generate carbene?
Chapter 40.3
(40.8) Formulate the reaction of diazomethane with (a) phenol, (b) 1-butanol, and (c) benzene.
Chapter 40.3: (a) Diazomethane methylates phenol to methyl phenyl ether (anisol) as formulated.
(b) Alcohols such as 1-butanol are not acidic enough. Their methylation by diazomethane is achieved in the
presence of a LEWIS acid as catalyst. In the catalytic cycle (Chapter 12.2), dissociation of the proton from the
alcohol is facilitated (lower activation barrier, Figure 12.1) in the intermediate alkyloxonium borontrifluoride.
R R
O
_ H  CH2 N NI  BF3 O H + ICH2 N NI
F3B
(b)
e.g. R = 1-buty l R R R
H3C CH2 CH2 CH2 O CH3 O + CH3 O + CH3 N NI
 N2
F 3B F3B

(41.1) Draw the structural formula of the analgesic and antipyretic 2-(4-isobutylphenyl)propanoic acid (ibuprofen).
Chapters 80.3.3 and 81.5 deal with the synthesis and structural features of 2-(4-isobutylphenyl)propanoic acid:
CH3 CH3
2-(4-isobutylphenyl)propanoic acid H3C CH CH2 CH CO2H

(41.2) Explain why the CO single bond of a carboxylic acid is shorter than the same bond in an alcohol.
Chapter 41.2
(41.3) Formulate equations to describe three general methods to prepare carboxylic acids without starting from CO2.
Chapter 41.4.1, 41.4.3 and 41.4.4
(41.4) Write equations to suggest the preparation of carboxylic acids starting from CO2.
Chapter 41.4.2
(41.5) Calculate the pH value of a 1 N carboxylic acid with Ka  105 using the acidity constant formula.
Chapter 41.5: The logarithmic form of the equilibrium constant Ka correlates the pKa of a carboxylic acid with the
pH and the acid concentration c(RCO2H) in aqueous solution:
pKa = 2 pH  lg c(RCO2H).
The pH turns out to be
pH = 1/2 (pKa + lg c(RCO2H).
Given Ka = 105, meaning pKa = 5, and lg c(RCO2H) = 0 for a 1 N aqueous carboxylic acid solution, the pH is
pH = 1/2 (5 + 0) = 2.5.
(41.6) Arrange (a) acetic and chloroacetic acid, and (b) benzoic, p-hydroxybenzoic, and p-nitrobenzoic acid in order of
increasing acidity and explain your reasoning.
Chapter 41.5: (a) chloroacetic acid > acetic acid; (b) p-nitrobenzoic acid > benzoic acid > p-hydroxybenzoic acid

(42.1) (a) Write the mechanism of the acid-catalyzed esterification; (b) suggest three options to prepare n-butyl acetate.
Chapter 42.1, answering (a)
(b) n-Butyl acetate (acetic acid n-butyl ester) can be prepared from n-butyl alcohol (1-butanol) 1. by esterification
with acetic acid, 2. by acetylation with acetic anhydride (acetic acid anhydride), and 3. by acetylation with acetyl
chloride (acetic acid chloride).
CH 3 CH 3
C 1 C
H 3C CH 2 CH 2 CH 2 OH + HO O H 3C CH 2 CH 2 CH 2 O O + H 2O
CH 3 CH 3
2
+ C C + H3C CO2H
O O O
CH 3 3
C NaOH
+ Cl O + HCl
272 Working on Questions

(42.2) Outline the preparation of (a) acetyl chloride and (b) acryloyl chloride by using thionyl chloride as a reagent.
Chapter 42.2, analogous reactions: (a) acetyl chloride and (b) acryloyl chloride are prepared with thionyl chloride.
Gaseous byproducts (SO2 and HCl) leave the almost pure acid halides as residues.
O O O O
(a) (b)
CH3 C + SOCl2 CH3 C + SO2 + HCl H 2C CH C + SOCl2 H 2C CH C + SO2 + HCl
OH Cl OH Cl
acetic acid acetyl chloride acrylic acid acryloyl chloride
(acetic acid chloride) (propenoic acid) (acrylic acid chloride)

(42.3) Formulate the preparation of the mixed anhydride of formic and acetic acid (two options).
Chapter 42.3, analogous reactions: Mixed acid anhydride of formic and acetic acid (acetic formic acid
anhydride) can be prepared from acetyl chloride and sodium formate or from formyl chloride and sodium acetate.
O O O O O O
H 3C C + C H H 3C C O C H H 3C C + C H
 NaCl  NaCl
Cl NaO ONa Cl
acetyl chloride sodium formate acetic formic anhydride sodium acetate formyl chloride

(42.4) What products are formed by heating (a) maleic acid, (b) phthalic acid, and (c) diammonium succinate?
Chapters 42.3, 42.4
(42.5) Formulate the reaction of phthalic anhydride with one equivalent of ethanol. What product is obtained?
Phthalic anhydride reacts with one equivalent of ethanol to produce pure phthalic acid monoethyl ester.
O O
C
O CH 2 CH 3
O + HO CH 2 CH 3
OH
C
O O
phthalic anhydride phthalic acid monoethyl ester

(42.6) Outline the steps of a test tube color reaction to identify a carboxylic acid.
Chapter 42.5: Hydroxamic acid test tube reaction with ferric salt identifies carboxylic acids.
(42.7) What are the decarboxylation products of (a) trichloroacetic, (b) malonic, and (c) cyanoacetic acid? Formulate.
Chapter 42.6.3, analogous reactions: (a) trichloroacetic acid undergoes thermal decarboxylation to chloroform,
(b) malonic acid to acetic acid as formulated, and (c) cyanoacetic acid to acetonitrile (methylcyanide).
O O
heat heat
Cl3C C Cl3C H + CO2 N C CH2 C N C CH3 + CO2
O H
(a) O H
(c)
trichloroacetic acid chloroform cyanoacetic acid acetonitrile

(42.8) Formulate the preparation of (a) 2,4,6-trimethylphenyl cyanide and (b) n-butyl isocyanide with suitable reagents.
Chapter 42.6.4, analogous reactions: (a) 2,4,6-Trimethylphenyl cyanide (mesityl cyanide) is expected to arise
from the dehydration of 2,4,6-trimethylbenzoic acid amide ( eliminination of H2O).
CH3 CH3 CH3 CH3
O + 2 NH3 O tautomerism OH P4 O10
H3C C H3 C C H3 C C H3C C N
 NH4Cl  H2O
Cl NH 2 N
CH3 CH3 CH3 CH3
H
2,4,6-trimethylbenzoyl chloride 2,4,6-trimethylbenzamide 2,4,6-trimethylphenyl cyanide

(b) n-Butyl isocyanide is expected to arise from the dehydration of N-(1-butyl)formamide ( elimination of H2O).
O tautomerism 
OH POCl3 , KOR
N C N C H 3C CH 2 CH 2 CH 2 N C
  H2 O
H H H 1-butyl isocyanide
N-(1-butyl)formamide -hydroxyimine tautomer
Working on Questions 273

(43.1) Draw the structural formulas of (a) 2-oxopropanoic acid, (b) 3-oxobutanoic acid ethyl ester, (c) 2,2-dichloro-
pentanoic acid, (d) 4-hydroxy-5-phenylpentanoic acid methyl ester, and (e) trifluoroacetic acid N,N-diethylamide.
Chapter 43.1: Structural formulas of substituted carboxylic acids are (a-e).
O O Cl
O O O O O
CH3 C C CH3 C CH2 C CH3 CH2 CH 2 C C CH2 CH2 CH2 CH 2 C CF 3 C
OH OC 2H 5 Cl OH OH OCH3 N(C2H 5)2
(a) 2-oxopropanoic acid (b) 3-oxobutanoic acid (c) 2,2-dichloropentanoic acid (d) 4-hydroxy-5-phenylpentanoic (e) trifluoroacetic acid
(pyruvic acid) ethyl ester acid methyl ester N,N-diethylamide

(43.2) Formulate equations to suggest the preparations of pure (a) 2-bromobutanoic acid and (b) 3-bromobutanoic acid.
Chapter 43.2.1, analogous reactions: (a) 2-Bromobutanoic acid can be obtained by bromination of butanoic acid
(butyric acid) with bromine in presence of phosphorous tribromide (HELL-VOLHARD-ZELINSKY reaction).
Br
+ Br 2 , P
CH3 CH 2 CH2 CO2H H3C CH2 CH2 CH CO2H
 HBr
butanoic acid (butyric acid) 2-bromobutanoic acid (-bromobutyric acid)

(b) Electrophilic hydrobromination of E- or Z-2-butenoic acid is expected to produce 3-bromobutanoic acid.


Br
H3C CH CH CO2H + HBr H3C CH CH2 CO2H
(E- or Z-)-2-butenoic acid 3-bromobutanoic acid (-bromobutyric acid)

(43.3) What products are obtained by reacting chloroacetic acid with (a) potassium cyanide, (b) sodium hydroxide,
(c) ammonia, and (d) thionyl chloride. Write the equations.
Chapter 43.2.2, analogous reactions: Chloroacetic acid reacts with (a) potassium cyanide to yield cyanoacetic
acid, (b) sodium hydroxide to give hydroxyacetic acid (glycolic acid), (c) ammonia to afford aminoacetic acid
(glycine, Chapter 68.1) and (d) thionyl chloride to produce chloroacetic acid chloride (chloroacetyl chloride).
NC CH2 CO2H cyanoacetic acid

(a) + KCN ,  KCl


O
+ SOCl2 ,  SO2 ,  HCl + NaOH ,  NaCl
Cl CH 2 C Cl CH2 CO2H HO CH2 CO2H
Cl (d) (b) hydroxyacetic acid
chloroacetyl chloride (glycolic acid)
(c) + 2 NH3 ,  NH4Cl
(chloroacetic acid chloride)

H 3N CH2 CO2 aminoacetic acid (glycine)

(43.4) Ethyl bromoacetate reacts with zinc dust in toluene. What product is formed when benzaldehyde is added to the
mixture? Formulate the appropriate equations.
Chapter 43.3.1: last reaction equation, REFORMATSKY reaction of bromoacetic acid ethyl ester (ethyl bromoacetate)
with benzaldehyde
(43.5) What are (a) lactides, (b) lactones, and (c) lactams? Give examples.
Chapter 43.3.2: (a) Lactides are bislactones of -hydroxycarboxylic acids; (b) lactones are cyclic esters of
-, , and -hydroxycarboxylic acids; (c) lactams are cyclic amides of -, , -, and -aminocarboxylic acids.
(43.6) Which lactam gives -aminocaproic acid (6-aminohexanoic acid) upon acidic hydrolysis?
Chapter 43.3.2: Hydrolysis of -caprolactam produces -aminocaproic acid (6-aminohexanoic acid).
6 + H2O O    1
-caprolactam NH H2N H2N CH2 CH2 CH2 CH2 CH2 CO2H
1
6 4 2
1 OH
O 6-aminohexanoic acid (-aminocaproic acid)

(44.1) What is a stereogenic center? Define the term chirality. How do enantiomers of a specific compound differ?
Chapter 44.1
(44.2) Draw the enantiomers of 2-(4-isobutylphenyl)propanoic acid and assign the absolute configurations.
Chapter 81.5 answers this, including more information
274 Working on Questions

(44.3) Which structural isomers of heptane C7H16 exist as enantiomers? Draw, name, and specify them.
3-Methylhexane and 2,3-dimethylpentane (C7H16) are hydrocarbons with one stereogenic center. FISCHER and
tetrahedral projections formulas specify the absolute configurations following the FISCHER and CIP convention.
C 3H7 C 3H7 C3H 7 C 3H7
3-methylhexane H 3C CH 2 CH2 CH CH 2 CH3 H3C C H C C H C CH3
H CH 3 H3C H
CH3 C 2H5 H 5C2 C2H5 C 2H5
L- (S)- (R)- D-

CH(CH 3)2 CH(CH 3)2 CH(CH 3)2 CH(CH3)2


2,3-dimethylpentane H3C CH 2 CH CH CH 3 H3C C H C C H C CH3
H CH 3 H 3C H
CH3 CH3 C 2H5 H 5C2 C2H 5 C 2H5
L- (S)- (R)- D-

(44.4) Draw the appropriate projection formulas of (R)- and (S)-2,3-dibromopropanoic acid. Is L = (S) and D = (R) a rule?
FISCHER and tetrahedral projection formulas specify the absolute configurations following the FISCHER- and
the CIP convention, exemplifying that L = (S)- and D = (R)- is not a general rule.
Br CH2 CH CO2H CO2H CO2H CO2H CO2H
Br Br C H C C H C Br
BrH2C H H CH 2Br
(R)- and (S)-2,3-dibromopropanoic acid CH2Br Br Br CH2Br
L- (R)- (S)- D-

(44.5) What is a racemic mixture? Suggest a method to resolve racemic 2-butanol.


Chapter 44.3.4: Set R = 2-butyl in the scheme and read the introduction of Chapter 46.
(44.6) What product is expected from reacting (S)-2-bromooctane with sodium hydroxide? Characterize this reaction.
Chapter 44.4: (S)-2-Bromobutane is expected to undergo a stereospecific SN2 reaction, producing (R)-2-butanol
(WALDEN inversion of the absolute configuration; but dehydrobromination to E- and Z-2-butene may compete).
CH 3 CH 3 CH 3
 +   SN2
Br C + OH Br C OH C OH + Br
H H
H
C 2H 5 C 2H 5 C 2H5
(S)-2-bromobutane tranistion state (R)-2-butanol

(45.1) Which kinds of chiral compounds with heteroatoms as stereogenic centers do you know? Draw example formulas.
Chapter 45.1
(45.2) Which kinds of chiral compounds without stereogenic centers do you know? Draw example formulas.
Chapter 45.1
(45.3) Enantiomers of 1,3-diphenylallene exist. Why? Look at the molecular orbital model in Chapter 17.2.3.
Chapters 45.2, 17.2.3
(45.4) Which one of (a) hexa-2,3-diene and (b) hexa-2,3,4-triene exists as (E)- and (Z)-isomers, and which one
as enantiomers? Specify the enantiomers.
Chapter 45.2: (a) (aR)- or M- and (aS)- or P- enantiomers are expected for hexa-2,3-diene with an even number
of cumulated double bonds; (b) cis- and trans-isomers, however, are expected for hexa-2,3,4-triene with an odd
number of double bonds. The molecular orbital model provides an explanation (Chapter 17.2.3).

H H
 H3C CH3
cumulated diene C C C Chirality axis C C C C C C
H H
C2H5 H 5C2
(aR)- (aS)-
stretched tetraherdral shape (a) hexa-2,3-diene

H3C CH3 H3C H



cumulated triene C C  C C C C C C C C C C
H H H CH3
cis- = (Z)- trans- = (E)-
terminal bonds are coplanar (b) hexa-2,3,4-triene
Working on Questions 275

(45.5) What are atropisomers? Draw formulas of a pair of enantiomers and specify their absolute configuration.
Chapter 45.2, last section

(46.1) How do enantiomers and diastereomers differ?


Chapter 46.1
(46.2) Which stereoisomers exist for 3-bromo-2-butanol? Specify their configurations and classify them.
Chapter 46.1
(46.3) Which stereoisomers exist for 2,3,4-trihydroxybutanoic acid? Specify their configurations and classify them.
Chapter 46.1: Replace the aldehyde function by a carboxy group in threose and erythrose.
(46.4) Which stereoisomers exist for 2,3-dihydroxybutanedioic acid? Characterize a meso-isomer.
Chapter 46.2 (tartaric acids)
(46.5) Draw all stereoisomers of (a) 1,2-, (b) 1,3-, and (c) 1,4-diaminocyclohexane and fully name them.
Chapter 46.2, last section: (a) 1,2-, (b) 1,3- and (c) 1,4-diaminocyclohexane exist as cis- and trans-isomers.
Mirror images of cis-1,2- and 1,3-diaminocyclohexane are meso forms; mirror images of trans-1,2- and
1,3-diaminocyclohexane are enantiomers.
diaminocyclohexane cis- trans-
(S,R) (R,S) (R,R) (S,S)
NH 2 H 2N NH 2 NH 2
1,2- NH 2 H 2N NH 2 H 2N
NH 2 NH 2 NH 2 H 2N

NH 2 NH 2
1,3- H 2N NH 2 H 2N NH 2 NH 2 H 2N

meso forms H 2N enantiomers NH 2


NH 2 NH2
H 2N NH 2
NH 2 NH 2
1,4- NH 2 H 2N H2N NH2 H2N NH2
H 2N identical H 2N identical

(47.1) Draw the structural formulas of (a) cis- and trans-2-hexenal, (b) 5-hexenal, (c) 2-hexynal, and (d) hexanedial.
Skeletal formulas (Chapter 8.2) of unsaturated C6 aldehydes portray the molecular shape, e.g. for (a) and (c).
H
O
O O O O
O
H H H H H
cis- trans-
(a) 2-hexenal (b) 5-hexenal (c) 2-hexynal (d) hexanedial

(47.2) Draw the structural formulas of all aldehydes with the molecular formula C5H8O and name them.
There are nine aldehydes with the molecular formula C5H8O; cis- and trans-2-methylcyclopropanecarbaldehyde
(1-formyl-2-methylcyclopropane) are diastereomers; each one of those is a pair of enantiomers.
H H H
H
O O O O O O O O
O
H H H H H
cis- trans- cis- trans- cis- trans-
2-pentenal 2-methyl-2-butenal 3-methyl-2-butenal cyclobutane cyclopropyl- 2-methylcyclopropane
carbaldehyde acetaldehyde carbaldehyde

(47.3) Formulate equations to describe the preparation of benzaldehyde from toluene.


Chapter 47.2.2
276 Working on Questions

(47.4) Formulate equations to describe the preparation of hexanedial from adipic acid (hexanedioic acid).
Chapter 47.2.3: Hexanedial is expected to be produced by RoSENMUND reduction of adipic acid dichloride.
OH Cl + 2 H2 H
O + 2 SOCl2 O Pd (BaSO4 ) O
O O O
OH  2 HCl,  2 SO2 Cl  2 HCl H
adipic acid (hexanedioic acid) adipic acid dichloride hexanedial

Reduction of the bis-N-methylanilide of adipic acid with LiAlH4 (Chapter 47.2.3) would require an additional step.
(47.5) Which reactions permit preparation of p-methoxybenzaldehyde from methyl phenyl ether (methoxybenzene)?
Chapter 47.2.4: p-Methoxybenzaldehyde (anisaldehyde) is expected to arise from electrophilic formylation of
anisol (methyl phenyl ether, methoxybenzene) by following the GATTERMANN-KOCH or the VILSMEIER method.
(47.6) One test tube contains butanal, the other butanone (C4H8O, Table 48.1). Formulate three test tube reactions to
identify the aldehyde.
Chapter 47.4.1: Unlike butanone, butanal reduces: TOLLENS-, FEHLING- and NYLANDER reactions are positive.
(47.7) What happens when p-tolualdehyde (p-methylbenzaldehyde) is reacted with sodium hydroxide? Formulate
the mechanism.
Chapter 47.4.2: CANNIZZARO disproportionation of p-tolualdehyde is expected to produce p-methylbenzoic acid
(toluic acid, product of oxidation) and p-methylbenzylalcohol (-hydroxy-p-xylene, product of reduction). Set
Ar = p-methylphenyl in the reaction equations.

(48.1) Draw the structural formulas of all unbranched heptanones and name these compounds.
Chapter 48.1: Three unbranched heptanones exist.
2-heptanone 3-heptanone 4-heptanone
H 3C CH2 CH2 CH 2 CH 2 C CH 3 H3C CH 2 CH 2 CH 2 C CH 2 CH 3 H3C CH 2 CH 2 C CH 2 CH2 CH3
O O O

O O O

(48.2) Draw structural formulas of all isomeric ketones with the molecular formula C5H8O and name them.
There are 14 isomeric ketones with the molecular formula C5H8O; three of those are cis- and trans-isomers,
two are enantiomers (R)- and (S)- and (R,R)- and (S,S)-, respectively. Enol tautomers are not considered here.

O O
O
cis- cis- cis-
O O O
O
O O O O O O O
trans- trans- trans-
3-penten-2-one 4-penten-2-one 3-methyl- cyclopentanone (S)- and (R)-2- 3-methyl- acetylcyclo- 2,2-dimethyl- (S,S)- and (R,R)-trans-2,3-
3-buten-2-one methylcyclobutanone cyclobutanone propanone cyclopropanone dimethylcyclopropanone

(48.3) Which secondary alcohol can be oxidized to 3,3-dimethyl-2-pentanone? Formulate the reaction.
Chapter 48.2.1: 3,3-Dimethyl-2-pentanol undergoes oxidation to yield 3,3-dimethyl-2-pentanone.
CH 3 CH 3
CH 3 CH2 C CH CH3 CH 3 CH 2 C C CH 3
 2 [H ] ,  2 e0
H 3C OH H 3C O

(48.4) Write equations which suggest methods to prepare 1,2-cyclopentanedione from an appropriate (a) diol and
(b) ketone.
Chapters 48.2.1, 48.2.3: 1,2-Cyclopentanedione can be prepared by (a) oxidation of cyclopentanediol (cis-
and/or trans-) and (b) oxidation of cyclopentanone with selenium dioxide (RILEY oxidation).
(48.5) Acetylation of 1,3-dimethoxybenzene gives 2,4-dimethoxyacetophenone as the major product. Why?
Chapters 48.2.4, 26.2: Owing to their (M) effect, both methoxy groups direct electrophilic acetylation to
the o-, o´- and p-positions (positions 2-, 4- and 6-), but the bulky methoxy groups shield the 2-position.
5-Acetyl-2,4-dimethoxyacetophenone is expected to be a minor product.
Working on Questions 277

(48.6) Look again at Chapter 36.2 and suggest a method to prepare t-butyl methyl ketone (3,3-dimethyl-2-butanone).
Chapter 36.2: Pinacol rearrangement of 2,3-dimethyl-2,3-butandiol will produce 3,3-dimethyl-2-butanone.

(49.1) Formulate a general equation that accounts for the typical reactions of aldehydes and ketones.
Chapter 49, first section
(49.2) What products arise from the acid-catalyzed reaction of ethanol with (a) benzaldehyde and (b) propanone (acetone)?
Chapter 49.1.2, analogous reaction: Acid-catalyzed reaction of ethanol with (a) benzaldehyde produces
benzaldehyde diethyl acetal via the hemiacetal; with (b) acetone (propanone), acetone diethyl ketal
(2,2,-diethyoxpropane) is expected via the hemiketal (2-ethoxy-2-hydroxypropane).
H H H
H + C2H5OH
(a) C + O C OC2H5 C OC2H5
H2O
O C2H5 OH OC2H5
benzaldehyde diethylacetal

H3C H CH3 CH3


+ C2H5OH
(b) C O + O H3C C OC2H5 H3C C OC2H5
H2O
H3C C2H5 OH OC2H5
acetone diethylketal (2,2-diethoxypropane)

(49.3) Formulate the reactions of (a) benzaldehyde with n-butylamine and (b) cyclopentanone with diethylamine.
Chapter 49.2, analogous reactions: (a) Benzaldehyde and the primary amine n-butylamine react to produce
n-butylbenzaldimine (an azomethine, SCHIFF base). (b) Cyclopentanone and the secondary amine diethylamine
undergo dehydration to produce the enamine 1-(N,N-diethylamino)cyclopentene.
H H C2H5 C2H5
(a) C + C (b) O + HN N
H2 O H 2 O
O H 2N N C4H9 C2H5 C2H5
n-butylbenzaldimine 1-(N,N-diethylamino)-
cyclopentene

(49.4) Formulate the reactions of cyclohexanone with (a) 2,4-dinitrophenylhydrazine and (b) hydroxylamine.
Chapter 49.2, analogous reactions: Cyclohexanone reacts with (a) 2,4-dinitrophenylhydrazine to yield the
2,4-dinitrophenylhydrazone and with (b) hydroxylamine to produce cyclohexanone oxime.
O2N
(a) O2N (b)
O + H2N NH NO2 N O + H2N OH N
H2O H2O
NH NO2 OH

cyclohexanone-2,4-dinitrophenylhydrazone cyclohexanone oxime

(49.5) Formulate the cyanohydrin reaction of butanal. Which kind of isomers are formed?
Chapter 49.3.2, analogous reaction: Cyanohydrin reaction of butanal generates a stereogenic center (chirogenic
reaction), producing both enantiomers of the cyanohydrin (DL-1-cyano-1-hydroxybutane = -hydroxyvaleric acid
nitrile) as a racemic mixture (DL).
H H H O
KCN , CH3CO2H + 2 H2O
C + H C N C C N C C
 NH3
O OH OH OH
butanal (butyraldehyde) DL-butyraldehyde cyanohydrin DL--hydroxyvaleric acid
(DL--hydroxyvaleronitrile)

(49.6) Propyne reacts with cyclopentanone in liquid NH3 and NaNH2 as base. Write the equation and name the product.
Chapter 49.3.1, analogous reaction: Nucleophilic alkynylation of cyclopentanone with propyne in liquid ammonia
and with sodium amide as deprotonating reagent is expected to produce 1-propynylcyclopentanol.
NaNH 2 HO
liquid NH3
O 1-propynyl-
CH3 C C H + CH 3 C C
 78 °C cyclopentanol
278 Working on Questions

(49.7) Formulate all steps necessary to convert cyclobutanol into methylenecyclobutane, using the WITTIG alkenylation.
Chapter 49.3.4, analogous reaction: The secondary alcohol cyclobutanol is oxidized to the ketone cyclobutanone
which is reacted with methylenetriphenylphosphane to undergo WITTIG alkenylation to produce methylene-
cyclobutane. The "ylene" arises from deprotonation of methyltriphenylphosphonium iodide, obtained by
methylation of triphenylphosphane with methyl iodide.
OH O CH 2
 CH2 P(C6H5) 3

 2 [H ] ,  2 e0  O P(C6H5) 3
cyclobutanone methylenecyclobutane

+ Li C6H5 _
CH3 X + P(C6H5)3 CH 3 P(C6H5)3 X CH 2 P(C6H5)3 CH 2 P(C6H5)3
 Li X ,  C6H6
X = Cl , Br , I ylide ylene
methylltriphenylphosphonium halide methylenetriphenylphosphane

(49.8) Suggest reactions for the preparation of (a) 1-methylcyclopentanol and (b) racemic 2-pentanol from appropriate
carbonyl compounds.
Chapters 49.3.3, 49.4, analogous reactions: 1-Methylcyclopentanol (a) can be prepared by methylation of
cyclopentanone with methylmagnesium bromide.
CH 3 + H2O CH 3
+ H 3C MgBr
O  Mg(OH)Br
O MgBr OH
cyclopentanone 1-methylcyclopentanol

Racemic 2-pentanol (b) is accessible by reduction of 2-pentanone with complex metal hydride such as LiAlH4
or by methylation of butanal (butyraldehyde) with methylmagnesium bromide.
H
H H 3C + H2O S R LiAlH4
H 3C MgBr +
 Mg(OH)Br
O OMgBr OH OH O
butanal racemic 2-pentanol 2-pentanone

(49.9) Which methods are available to convert butyrophenone into n-butylbenzene?


Chapter 49.4: CLEMMENSEN- and WOLFF-KISHNER reductions of butyrophenone

(50.1) Explain why carboxylic acid derivatives, aldehydes, and ketones are CH acidic.
Chapters 50.1, 50.2
(50.2) Write equations for (a) a CLAISEN condensation and (b) an aldol reaction with subsequent dehydration.
Chapters 50.1.1, 50.2.1
(50.3) Formulate the reaction of ethyl formate and ethyl acetate in the presence of sodium ethoxide and name the product.
Chapter 50.1.1, analogous reaction: CLAISEN condensation of ethyl formate (formic acid ethyl ester) and ethyl
acetate (acetic acid ethyl ester) in the presence of a base is expected to produce formylacetic acid ethyl ester.
OC 2H 5 H O OC2H 5
NaOC 2H 5
H C + IC C H C CH 2 CO2C 2H 5 H C CH 2 CO2C2H5
O  C 2 H5 O
H OC 2H5 IOI
_ O
ethyl formate from ethyl acetate formylacetic acid ethyl ester

(50.4) Write equations to describe a synthesis of cyclopentanone from adipic acid.


Chapter 50.1.2, last section
(50.5) Suggest a simple synthesis of 2-methyl-2-pentenal (trans- and cis-isomers) by aldol reaction and dehydration.
Chapter 50.2.1, analogous reaction: Propanal (propionaldehyde) undergoes aldol reaction and subsequent
dehydration to produce trans- and cis- 2-methyl-2-pentenal.
O CH3 H OH CH 3 H H 3C H 2C CH 3 H CH 3
OH
H 3C H 2C C + H 2C C H3C H 2C C C C C C C C
 H2O
H O H H O H C O H 3C H2C C O
propanal H trans- and cis- H
(propionaldehyde) aldol (2-methyl-3-hydroxypentanal) 2-methyl-2-pentenal
Working on Questions 279

(50.6) Which reaction permits production of 2,4-pentanedione? Suggest an analogous preparation of 3,5-heptanedione.
Chapter 50.2.2, analogous reaction: 3,5-Heptanedione is accessible by CLAISEN condensation of ethyl
propanoate (propionic acid ethyl ester) and butanone. Two regioisomeric diketones may be produced.
3,5-Heptanedione as desired (major product) arises from deprotonation of the methyl group of butanone by the
base; 3-methyl-2,4-hexanedione is obtained when deprotonation of the methylene group of butanone takes
place (regioselectivity: Chapter 78.2).
H H H CH 3
OC2H 5 H O OC2H 5 H O
NaOC2H5 H 5C 2 C C2H 5 H 5C2 C CH 3 NaOC2H5
H 5C 2 C + IC C C C C C H 5C 2 C + IC C
O H C2H 5  C2H5O O O O O
 C2H5O O H3C CH 3
propionic acid from butanone 3,5-heptanedione 3-methyl-2,4-hexanedione propionic acid from butanone
ethyl ester ethyl ester

(51.1) Draw the structural formulas of all kinds of 1,3-dicarbonyl compounds you know.
Chapter 51, first section
(51.2) Formulate and explain the CH acidity of 1,3-dicarbonyl compounds.
Chapter 51.1
(51.3) Formulate a sequence of reactions that permits the synthesis of cyclopropanecarboxylic acid.
Chapter 51.2.1, analogous reaction: Cyclopropanation is achieved by cycloalkylation of diethyl malonate with
1,2-dibromoethane. Hydrolysis (saponification) of the diester and decarboxylation of the cyclopropane-1,1-
dicarboxylic acid produces the target cyclopropane carboxylic acid.
Br CO2C2H5
CH2 + NaOC2H5 CO2C2H5 + NaOC2H5 CO2C2H5 + 2 H2O CO2H heat
+ H2C CO2H
CH2  NaBr  NaBr  2 C2H5OH CO2
Br COC2H5 Br H CO2C2H 5 CO2C 2H5 CO2H
 C2H5OH  C2H5OH
1,2-dibromo- diethyl cyclopropane-1,1- cyclopropane-1,1- cyclopropane-
ethane malonate dicarboxylic acid diethylester dicarboxylic acid carboxylic acid

(51.4) Propose a synthesis of 3-methyl-2-pentenoic acid. Which isomers can be formed?


Chapter 51.2.2, analogous reaction: KNOEVENAGEL alkenylation of butanone with diethyl malonate is the key step
of the synthesis. Primarily formed 2-ethoxycarbonyl-3-methyl-2-pentenoic acid ethyl ester is hydrolyzed to the
dicarboxylic acid wich undergoes thermal decarboxylation to trans- and cis-3-methyl-2-pentenoic acid.
H3C CO2C2H5 H3C CO2C 2H 5 H3C CO2H H3C CO2H H 3C H
base + 2 H2O heat
C O + H2C C C C C C C C C
 H2O  2 C2H5OH CO2
H5C2 CO2C2H5 H 5C 2 CO2C 2H 5 H5C2 CO2H H5C2 H H5C2 CO2H
butanone diethyl 2-ethoxycarbonyl-3-methyl- 2-carboxy-3-methyl- trans- cis-
malonate 2-pentenoic acid ethyl ester 2-pentenoic acid 3-methyl-2-pentenoic acid

(51.5) What product is obtained by MICHAEL addition of acetylacetone to butenone?


Chapter 51.2.3, last section
(51.6) Which test tube reactions indicate the presence of enol tautomers in samples of 1,3-dicarbonyl compounds?
Chapter 51.2.4
(51.7) What products are formed by reaction of hydrazine with (a) ethyl acetoacetate and (b) acetylacetone?
Chapter 51.2.5, analogous reactions: Binucleophilic hydrazine undergoes cyclodehydration with (a) ethyl aceto-
acetate, yielding 3-methyl-5-oxo-4,5-dihydropyrazole, and with (b) acetylacetone (2,4-pentanedione), yielding
3,5-dimethylpyrazole.
CH 3 CH 3 CH 3 CH3
(a) (b)
O O + NH 2 N H 3C O N
H 2N O NH 2 H 3C
OC 2H 5  C 2H5OH ,  H2O N OH + H 2N  2 H2O N
H acetylacetone H
ethyl
acetoacetate hydrazine 3-methyl-5-oxo-4,5-dihydropyrazole enol tautomer 3,5-dimethylpyrazole
280 Working on Questions

(52.1) Why are phenols much stronger acids than alcohols? Explain your reasoning with resonance formulas.
Chapter 52.2
(52.2) Formulate all equations necessary to outline the production of phenol and acetone from cumene.
Chapter 52.3.1
(52.3) What products are formed by reaction of acetic anhydride with (a) phenol and AlCl3, and (b) salicylic acid?
Chapter 52.4.2, answering (a) and (b)
(52.4) What product is formed by reaction of phenol with 3-bromo-1-propene (allyl bromide) in sodium hydroxide?
Chapter 37.3.2, last reaction equation
(52.5) What products are likely to be formed by nitration of (a) -naphthol and (b) -naphthol? Look at Chapter 28.3.
Chapter 28.3, analogous reaction: (a) -Naphthol undergoes electrophilic nitration with concentrated nitric acid
and sulfuric acid at positions 2 and 4; (b) nitration of -naphthol is expected to occur at position 1.
OH OH OH OH
NO2 NO2
+ [ O=N=O ] + [ O=N=O ] + [ O=N=O ]

 [H ]  [H ]  [H ]
NO2 2-nitro-1-naphthol NO2
4-nitro-1-napthol (a) 2,4-dinitro-1-naphthol
NO2
OH + [ O=N=O ] OH

 [H ]
(b) 1-nitro-2-naphthol

(52.6) What product is expected from the oxidation of 2,4,6-tri-t-butylphenol? Account for the stability of the product.
Chapter 52.4.4

(53.1) Which isomeric benzo- and naphthoquinones exist? Draw the structural formulas.
Chapter 53.1, Table 53.1
(53.2) Formulate the tautomerism of resorcinol.
Chapter 53.1
(53.3) Formulate the preparation of (a) o-benzoquinone, (b) p-benzoquinone, and (c) 1,4-naphthoquinone, using
appropriate phenols as starting material.
Chapter 53.2.1: Oxidation of (a) catechol, (b) hydroquinone, and (c) 1,4-dihydroxynaphthalene
(53.4) What is an EDA or CT complex? Draw the formula of quinhydrone to explain.
Chapter 53.3.1
(53.5) Formulate (a) the redox equilibrium between hydroquinone and p-benzoquinone and (b) the mechanism of
the electron transfer in quinhydrone.
Chapter 53.3.1, answering (a) and (b)
(53.6) Which reactions can be used to prepare 9,10-anthraquinone?
Chapters 53.2.3, 53.3.3, last section
(53.7) Formulate an industrial method to produce hydrogen peroxide.
Chapter 53.3.2
(53.8) What products are expected from the electrophilic sulfonation of 9,10-anthraquinone? Outline a synthesis of
1,2-dihydroxy-9,10-anthraquinone (known as alizarin).
Chapter 53.3.4

(54.1) Which organosulfur compounds do you know? Which ones have oxygen analogues?
Chapter 54.1, Table 54.1
(54.2) Formulate equations to suggest preparations of (a) 1-butanethiol and (b) thiophenol.
Chapter 54.2.1
Working on Questions 281

(54.3) What happens when thiols are exposed to air? Why are thiols much stronger acids than alcohols?
Chapter 54.2.1, last sections
(54.4) Suggest a preparation of ethyl phenyl sulfide. Ethyl phenyl sulfide reacts with iodomethane. Specify the product.
Chapter 54.2.2: Ethyl phenyl sulfide is prepared by WILLIAMSON synthesis from thiophenolate as nucleophile and
bromoethane (SN2) but not from ethane thiolate and bromobenzene (Chapter 27.1). Methylation of ethyl phenyl
sulfide with iodomethane produces (R)- and (S)-enantiomers of ethyl-methyl-phenyl-sulfonium iodide.

S Na + Br CH2 CH3 S + CH3 I


CH 2 CH3 H 3C S S CH
3
 NaBr I H3C H2C CH2 CH3
thiophenolate bromoethane ethyl phenyl sulfide (S)- and (R)-ethyl-methyl-phenyl-sulfonium ion

(54.5) Which products are obtained by the oxidation of thioethers with hydrogen peroxide? Formulate the reactions.
Chapter 54.3.1
(54.6) Which reactions open access to (a) sulfinic acids and (b) sulfonic acids?
Chapter 54.3.2, answering (a) and (b)
(54.7) Formulate equations to outline preparations of (a) sulfochlorides, (b) alkyl sulfonates, and (c) sulfonamides.
Chapter 54.3.3
(54.8) Sulfonamides and N-alkylsulfonamides dissolve in aqueous sodium hydroxide. Explain with resonance formulas.
Chapter 54.3.3, last section: Dissociation of an NH-proton from a sulfonamide leaves a resonance-stabilized
sulfonamide anion; the negative charge is distributed (delocalized) among the N-atom and both O-atoms.
_
R O R O R O R O
_ + OH _
N S N
_ S N
_ S N
_ S
 H2O
H O O O_ O
N-alkylbenzenesulfonamide N-alkylbenzenesulfonamide anion, resonance formulas

(54.9) Which thiocarbonyl compounds exist? Suggest syntheses of (a) thiobenzophenone and (b) phenyldithioacetic acid.
Chapter 54.2.4, answering (a); Chapter 54.2.5, answering (b), last reaction equation, set R = C6H5CH2 (benzyl).

(55.1) Which stable derivatives of carbonic acid do you know?


Chapter 55.1
(55.2) What products are obtained from the reaction of phosgene and (a) ammonia, (b) ethanol with and without base,
and (c) potassium t-butylmonocarbonate?
(a) Chapter 55.4.1, product: urea; (b) Chapters 55.2.2, 55.3.1, set R = C2H5, products: ethyl chlorocarbonate and
diethyl carbonate; (c) Chapter 55.3.1, second reaction equation, set R = C(CH3)3, product: di-t-butyl tricarbonate
(55.3) Explain why all CN bonds of the guanidinium cation have equal length.
Chapter 55.4.2
(55.4) Formulate equations to outline the syntheses of (a) tetramethylurea, (b) guanidine, and (c) thiourea.
Chapter 55.4.1, last reaction equation, answering (a), set R2 = (CH3)2; Chapter 55.4.2, last scheme, answering (b);
Chapter 55.4.2, last scheme, answering (c)
(55.5) What are the parent compounds of (a) urethanes and (b) ureides?
(a) Chapter 55.3.2: Urethanes are esters of carbamic acid. (b) Chapter 55.4.1: Ureides are acylated ureas.

(56.1) What is a heterocumulene? Give examples. How are carbodiimide and cyanamide related to each other?
Chapter 56.1: heterocumulenes; Chapter 56.4: cyanamide-carbodiimide-tautomerism
(56.2) Formulate the preparation of (a) phenyl isocyanate, (b) phenyl isothiocyanate, and (c) DCC.
(a) Chapter 56.3, first reaction equation: set R = C6H5 (phenyl); (b) Chapter 56.3, second reaction equation:
set Ar = C6H5 (phenyl); (c) Chapter 56.4, second and third reaction equation: set R = cyclohexyl
282 Working on Questions

(56.3) Formulate the reactions of phenyl isocyanate with (a) 1-butanol, (b) ammonia, and (c) diethylamine.
Chapter 56.3, analogous reactions: phenylisocyanate adds (a) 1-butanol to produce 1-butyl-N-phenylurethane,
(b) ammonia to produce N-phenylurea and (c) diethylamine to produce N,N-diethyl-N´-phenylurea.
H H
_
N _ C4H 9
+ HO + HN(C2H5)2 N
O C O C N O C
O CH2CH 2CH2CH 3
(a) (c) N(C 2H 5)2
(b) + NH3
1-butyl-N-phenylurethane N,N-diethyl-N´-phenylurea
H
N N-phenylurea
O C
NH2

(56.4) React DCC, first with a carboxylic acid, then with (a) an alcohol and (b) a primary amine. Formulate all reactions.
(a) Chapter 56.4, last reaction equation: activated esterification of the carboxylic acid;
(b) same reaction equation: activated amidation of the carboxylic acid (Chapter 69.4, peptide synthesis)
O R O O NH R
+ H 2N R2
R1 C + N C N R1 C N R R1 C + O C
O H R O C N R2 NH R
NH R H
carbodiimide addukt (isourea active ester) N-alkylcarboxamide

(57.1) Formulate the general mechanisms of (a) anionotropic and (b) cationotropic 1,2-shifts.
Chapter 57.1.1, answering (a); Chapter 57.2, answering (b)
(57.2) Formulate a sequence of reactions which results in homologization of a carboxylic acid (RCO2H  RCH2CO2H).
Chapter 57.1.2 (WOLFF rearrangement, ARNDT-EISTERT homologization of carboxylic acids)
(57.3) Formulate all reactions which can be used to convert a carboxylic acid into a primary amine.
Chapter 57.1.4, Fig. 57.1
(57.4) Write equations for the reactions of (a) 2-bromocyclohexanone, (b) benzyltrimethylammonium salt, and
(c) benzyl methyl ether with a base.
(a) Chapter 57.2.1: FAVORSKII rearrangement to yield cyclopentanecarboxylic acid ester, replace Cl by Br;
(b) Chapter 57.3: SOMMELET-HAUSER rearrangement to give N,N-dimethyl-2-methylbenzylamine, set R = CH3;
(c) Chapter 57.2.3: WITTIG rearrangement to produce 1-phenylethanol, set R = phenyl and R´ = CH3
(57.5) Sodium phenoxide is reacted with 3-bromo-1-propene, and the product is heated (200 °C). What happens?
Chapters 57.4, 37.3.2: 3-Bromo-1-propene (allyl bromide) reacts with phenoxide to produce allyl phenyl ether
which undergoes CLAISEN rearrangement to give o-allylphenol upon heating.
_ CLAISEN
OI
_ Na Br O heat O OH
+
 NaBr

sodium phenoxide allyl bromide allyl phenyl ether H o-allylphenol


from phenol

(58.1) What is a polymer?


Chapter 58.1
(58.2) What are (a) vinyl polymers and (b) diene polymers? How do they differ? Formulate examples.
Chapter 58.2, answering (a) and (b)
(58.3) What kinds of reactions are (a) polymerizations, (b) polycondensations, and (c) polyadditions? Formulate examples.
Chapters 58.1, 58.2, answering (a); Chapters 58.4, 58.5, answering (b); Chapter 58.6, answering (c)
(58.4) Formulate equations to explain the mechanism of a radical polymerization.
Chapter 58.2, p. 165
(58.5) Formulate equations to outline the formation of polyethers from oxiranes.
Chapter 58.3
(58.6) Draw the structure of a polylactide. Which reactions permit the production of polyesters? Formulate examples.
Chapter 58.4
Working on Questions 283

(58.7) Which types of polymers are (a) polyamides and (b) polyurethanes? What kinds of reactions permit their production?
(a) Chapter 58.5: polyamides by polycondensation; (b) Chapter 58.6: polyurethanes by polyaddition
(58.8) Which types of polymers are formed from the reaction of 1,4-diisocyanatobenzene with (a) 1,3-propanediol
and (b) 1,4-diaminobenzene (p-phenylenediamine)?
Chapter 58.6: Polyaddition of 1,4-diisocyanatobenzene and 1,3-propanediol (a) produces a polyurethane;
the analogous polyaddition with p-phenylenediamine (b) produces a polyurea.
O
H O O C H
(a)
N C O + n HO OH N C N N
1,3-propanediol O H C O
O
n1
n O C N O C N polyurethane OH

1,4-diisocyanatobenzene
H O
N C O + n H 2N NH2 (b) N C O
NH NH C H
p-phenylenediamine
N N
H C NH NH2
n O C N O C N
polyurea O n1

(59.1) Which organosilicon compounds do you know? Why don’t silacarbonyl compounds exist? What are silicones?
Chapters 59.1, 59.4 (silicones)
(59.2) Formulate all reactions you know to describe the preparative significance of chlorotrimethylsilane.
Chapter 59.3 (trimethylsilylations, reactions of silylenolethers)
(59.3) Design a synthesis of 4-pentenoic acid from 2-propen-1-ol and acetic anhydride.
Chapter 59.3.2, typical reaction: The IRELAND-CLAISEN rearrangement of allyl acetate (acetic acid allyl ester),
accessible by acetylation of allyl alcohol with acetic anhydride, is the key step of a synthesis of 4-pentenoic acid.
O O OH OSi(CH3)3 IRELAND- OSi(CH 3)3 OH
78 °C, LDA CLAISEN
C CH3 + (H3C) 3Si Cl rearrangement + H2O
OH + O O O O O
O
CH3CO2H  HCl  (H3C) 3Si Cl
C CH3
allyl- O oxo tautomer enol tautomer ketene-O-trimethylsilylallyl- ,-unsaturated 4-pentenoic
alcohol allyl acetate ketal trimethylsilyl carboxylate acid

(60.1) Which heteroalicycles in Table 60.2 are (a) peroxides, (b) disulfides, (c) acetals, (d) thioacetals, (e) secondary
amines, (f) enol ethers, and (g) enamines?
(a) peroxide: 1,2-dioxolane; (b) disulfide: 1,2-dithiolane; (c) acetals: 1,3-dioxolane, 1,3-dioxane;
(d) thioacetals: 1,3-dithiolane, 1,3-dithiane; (e) secondary amines: azirane, azetane, pyrrolidine, imidazolidine,
piperidine, piperazine, azepane; (f) enolethers: oxirene, oxete, furan, 2H-pyran, 4H-pyran, 1,4-dioxine, oxepine;
(g) enamines: 1H-azirine, azete (an imine as well), pyrrole, imidazole, 1,4-dihydropyridine, 1H-azepine
(60.2) Write equations to prepare (a) tetrahydrofuran, (b) tetrahydrothiophene, and (c) 7-thiabicyclo[4.1.0]heptane.
Chapter 60.2.1: (a) Tetrahydrofuran and (b) tetrahydrothiophene are obtained by intramolecular SN reaction of
4-halobutanol or 4-halobutanethiol, respectively. Tetrahydrofuran is industrially produced by cyclodehydration of
butane-1,4-diol. (c) 7-Thiabicyclo[4.1.0]heptane is the product of an intramolecular SN reaction
(dehydrochlorination) of 2-chlorocyclohexanethiol.
NaOH Cl
(a) X = O : tetrahydrofuran NaHCO3
Br (CH 2)4 XH X
(b) X = S : tetrahydrothiophene (c) S
 HBr  HCl
SH
7-thiabicyclo[4.1.0]heptane
284 Working on Questions

(60.3) What product is expected from the reaction of cyclohexene and benzophenone under UV light?
Write the equation?
Chapter 60.2.2, analogous reaction: When irradiated by UV, cyclohexene and benzophenone undergo
[2+2]-cycloaddition to produce 8,8-diphenyl-7-oxabicyclo[4.2.0]octane (PATERNO-BÜCHI reaction).

H 5C6 C6H5 C 6H 5
C h C 6H5
+
O O
cyclohexene benzophenone 8,8-diphenyl-7-oxabicyclo[4.2.0]octane

(60.4) What products are obtained from the reaction of azirane with (a) iodomethane, (b) acrylonitrile, and (c) butenone?
Chapter 60.3.1, analogous reaction: Azirane as a nitrogen nucleophile is N-methylated by (a) methyl iodide and
adds to the electron-deficient CC double bonds of (b) acrylonitrile and (c) butenone (MICHAEL addditions, Chapter
51.2.3).
O (c) O
base (b)
NH + CH3 I N CH3 NH + H 2C CH C N N CH 2 CH2 C N NH + H 2C CH C N CH 2 CH 2 C
 HI
CH3 CH3
(a)
1-methylazirane 1-(2-cyanoethyl)azirane 1-(2-acetylethyl)azirane

(60.5) Ethylmagnesium bromide is reacted with (R)-phenyloxirane. Draw the equation and name the product.
Chapter 60.3.2, analogous reaction: Ethylmagnesium bromide reacts with (R)-phenyloxirane to produce
(R)-1-phenyl-1-butanol.
OH
O + H 2O
CH 3 CH2 MgBr + H CH 3 CH 2 CH 2 C
2 H
 Mg ,  OH ,  Br

(R)-phenyloxirane (R)-1-phenyl-1-butanol

(60.6) 5,6-Epoxy-1,3-cyclohexadiene and oxepine equilibrate at 25 °C. Formulate and provide an explanation.
Chapter 60.3.3, analogous reaction: Benzene oxide and oxepine arise from each other by COPE rearrangement.

benzene oxide
5,6-epoxy-1,3-cyclohexadiene O O oxepine
7-oxabicyclo[4.1.0]cyclohepta-1,3-diene

(60.7) What happens when 2-hydroxymethyl)tetrahydrofuran reacts with an acid? Write the mechanism.
Chapter 60.3.3, last section

(61.1) Explain the term -excessive heteroaromatic compounds. Which representatives do you know?
Chapter 61.2
(61.2) Account for the acidity of pyrrole and cyclopentadiene in terms of resonance stabilization of the anions.
Chapters 61.4.1, 29.1.2
(61.3) Which aromatic heterocycles can be prepared by the PAAL-KNORR synthesis? Write the equations.
Chapter 61.3.1
(61.4) Devise syntheses of (a) 2,4-dimethylpyrrole, (b) 5-acetylpyrazole, and (c) 1,5-diphenyltetrazole.
Chapters 61.3.1, 61.3.2, analogous reactions: 2,4-Dimethylpyrrole (a) is accessible by KNORR synthesis from
ethyl acetoacetate.
R = C2H5 : ethyl acetoacetate 2,4-dimethylpyrrole
H3C H 3C H3C H 3C
O + HNO2 O tautomerism O
CH 2  H 2O OH O CH 3
RO2C RO2C N RO2C N N
H H
Zn , CH 3CO2 H + 4 [H ] , + 4 e0 , H2O

 2 CO2

H 3C CO2R H 3C CO2R H 3C CO2R H 3C CO2H


O + H2C ester hydrolysis
-aminocarbonyl H H
intermediate CH3 N CH 3 RO2C N CH 3 HO2C N CH 3
NH 2 O  2 H2O RO2C
RO2C H H
-oxoester 3,5-diethoxycarbonyl-2,4-dialkylpyrrole
Working on Questions 285

1,3-Dipolar cycloaddition of diazomethane and butynone produces 5-acetyl-1H-pyrazole (b); analogous reaction
of phenyl azide and benzonitrile affords 1,5-diphenyltetrazole (c), Ph = C6H5.
butynone O benzonitrile
H C C COCH 3 COCH 3 C CH 3 Ph C N Ph
(b) Tautomerie (c) N
+ +
H 2C N H N NH N N N N
N H N N Ph N Ph N
diazomethane 5-acetyl-1H-pyrazole phenylazide 1,5-dphenylltetrazole

(61.5) What products are obtained from (a) acetylation and (b) nitration of N-methylpyrrole?
Chapter 61.4.2, analogous reactions: (a) 2-Acetyl-N-methylpyrrole and (b) 2-nitro-N-methylpyrrole are expected.
(61.6) Formulate two mechanisms for the nitration of furan in anhydrous acetic acid. Which one is more likely?
Chapters 61.4.2, 61.4.3
(61.7) What product arises from reaction of furan with (a) bromine in methanol and (b) butynedioic acid dimethyl ester?
Chapter 61.4.3
(61.8) Formulate the hydrolysis of furan in aqueous acid. What is the name of the reverse reaction?
Chapter 61.4.5

(62.1) Explain the term -deficient heteroaromatic compound, using pyridine and the pyrylium ion as examples.
Chapter 62.2
(62.2) Formulate equations for the preparations of (a) 2,2´-bipyridine and (b) 2,4,6-trimethylpyridine.
Chapter 62.3.1, answering (a); 2,4,6-trimethylpyridine (b) is obtained by HANTZSCH synthesis starting from ethyl
acetoacetate, acetaldehyde and ammonia; set R1 = R3 = CH3 and R2 = C2H5 in the scheme.
(62.3) Devise syntheses of (a) 2,4,6-trimethylpyrimidine, (b) unsubstituted pyrimidine, and
(c) 2-amino-4,6-dimethylpyrimidine.
Chapter 62.3.2, analogous reactions: Pyrimidines are prepared by cyclodehydration of 1,3-dicarbonyl compounds
with amidines, e.g. 2,4,6-trimethylpyrimidine (a) from 2,4-pentanedione and acetamidine, unsubstituted pyrimidine
(b) from malondialdehyde (or the tetraethyl acetal) and formamidine, 2-amino-4,6-dimethylpyrimidine (c) from
2,4-pentanedione and guanidine.
CH3 CH 3 CH 3
oxo-enol-
NH 2
O tautomerism O (a) N
+ C
H HN CH3  2 H 2O
H3C O H3C O H 3C N CH 3
2,4-pentanedione (acetylacetone) acetamidine 2,4,6-trimethylpyrimidine
H H CH 3 CH3
oxo-enol-
NH 2 NH2
O tautomerism O (b) N O (c) N
+ C + C
H HN H  2 H 2O HN NH 2  2 H2O
H O H O N H 3C O H3C N NH 2
propanedial (malondialdehyde) formamidine pyrimidine guanidine 2-amino-4,6-dimethylpyrimidine

(62.4) Outline a synthesis of 2-phenyl-4,6-dimethylpyrylium perchlorate.


Chapter 62.3.3, analogous reaction: Insert R = CH3 (for 2,4-pentanedione, enol tautomer) in the reaction
equation.
(62.5) 4-N,N-Dimethylaminopyridine (DMAP) is much more basic than pyridine. Give an explanation.
Chapter 62.4.1: Replace NH2 in 4-aminopyridine by N(CH3)2.
(62.6) Formulate the reaction of pyridine with (a) sodium amide in liquid NH3 and (b) butyllithium under nitrogen.
Chapter 62.4.2, (a): first reaction equation; (b): second reaction equation: Replace phenyllithium by butyllithium,
C6H5 by n-C4H9, and obtain 2-n-butylpyridine.
(62.7) Draw the resonance formulas of pyridine N-oxide. What can be concluded concerning reactivity?
Chapter 62.4.1
(62.8) Suggest reactions to prepare (a) 3-nitropyridine and (b) 4-nitropyridine.
Chapter 62.4.3, answering (a) and (b)
286 Working on Questions

(62.9) Draw resonance formulas of the intermediates resulting from deprotonation of (a) acetone and (b) 2-methylpyridine.
Chapter 50.2, concerning (a) and Chapter 62.4.4, concerning (b)
(62.10) 2-Methylpyrimidine is reacted with benzaldehyde in the presence of ZnCl2. Which (E)-isomer is the major product?
Chapter 62.4.4

(63.1) Why are benzo[c]furan and its analogues not aromatic? Write the equation for their reaction with maleic anhydride.
Chapters 63.3, 63.3.2
(63.2) Which reactants are required to synthesize 5-methoxyindole? Write the mechanism of the reaction.
Chapter 63.2.2: Following the mechanism of FISCHER´s indole synthesis, 5-methoxyindole can be synthesized
from p-methoxyphenylhydrazine and acetaldehyde (or acetaldehyde diethylacetal).
H3CO H 3CO
H 3C H
NH2 + C
N  H2O N
H O  NH3 H
p-methoxyphenylhydrazine acetaldehyde 5-methoxyindole

(63.3) What products are obtained by nitration of (a) benzo[b]furan and (b) benzo[b]thiophene? Explain your answer.
Chapter 63.3.1, answering (a) and (b)
(63.4) Suggest reactions to prepare (a) indole-3-carboxylic acid and (b) 3-(N,N-dimethylaminomethyl)indole from indole.
Chapter 63.3.1, answering (a); Chapter 50.2.3 and Fig. 63.1, answering (b)
(63.5) Propose a reaction to convert a substituted indole into a substituted quinoline.
Chapter 63.3.2
(63.6) Formulate (a) the tautomerism and oxidation of 3-hydroxyindole and (b) the dyeing of fabrics with indigo.
Chapter 63.3.3, answering (a) and (b)

(64.1) Draw the structural formulas of all benzodiazines with the molecular formula C8H6N2 and name them.
Chapter 64.1, Table 64.1; [C8H6N2 includes 1,5-, 1,6-, 1,7-, 1,8-, 2,6-, and 2,7-naphthyridine (diazanaphthalenes)
with nitrogen atoms in different rings, representing structural isomers containing differently fused pyridine rings.]
(64.2) What compound arises from reacting aniline and butenone in nitrobenzene with ZnCl2? Write the mechanism.
Chapter 64.2.1: SKRAUP´s quinoline synthesis with butenone (R1 = R2 = H, R3 = CH3) produces 4-methylquinoline.
(64.3) Write equations to suggest two syntheses of the alkaloid parent compound 1-benzylisoquinoline (Chapter 70.1).
Chapter 64.2.2: analogous reactions: 1-Benzylisoquinoline can be prepared from phenylethylamine and
phenylacetyl chloride (BISCHLER-NAPIERALSKY synthesis, R = CH2C6H5, benzyl) or from phenylethylamine and
phenylacetaldehyde (PICTET-SPENGLER synthesis, R = CH2C6H5).
(64.4) Recalling Chapter 44, two products are obtained by the catalytic hydrogenation of 1-benzylisoquinoline? Draw them.
Catalytic hydrogenation of 1-benzylisoquinoline produces the racemate (R,S)-1-benzyltetrahydroisoquinoline.

+ H2 (Pt) H H
N NH HN
S R

1-benzylisoquinoline (R,S)-1-benzyltetrahydroisoquinoline

(64.5) Isoquinoline reacts with (a) NaNH2 in liquid NH3 and (b) n-butyllithium under nitrogen. What are the products?
Chapter 64.3.4, answering (a); Chapter 64.3.3, answering (b): n-butyllithium is expected to undergo nucleophilic
addition to isoquinoline, producing (R,S)-1-n-butyl-1,2-dihydroisoquinoline upon addition of water.
(64.6) 2-Chloroquinoline reacts with (a) water and (b) sodium ethoxide. Formulate the reactions and name the products.
Chapter 64.3.4, answering (a) and (b)
(64.7) What products arise from the nitration of (a) quinoline and (b) isoquinoline?
Chapter 64.3.5, answering (a); (b) electrophilic nitration of isoquinoline is expected to produce 5-nitroisoquinoline.
(64.8) Write an equation to propose a synthesis of 2-quinolylacetone.
Chapter 64.3.6: 2-Quinolylacetone (quinolin-2-yl-propanone) is expected to arise from the ester condensation of
2-methylquinoline with ethyl acetate in the presence of sodium ethoxide as a base.
Working on Questions 287

(65.1) Draw the structural formulas of (a) 3H-pyrrolo[1,2-a]pyrrole, (b) pyrrolo[1,2-a]pyridine, and (c) pyrido[2,3-d]pyrimidine.
Structural formulas of the heterobicycles are:

N
N N N
N
(a) 3H-pyrrolo[1,2-a]pyrrole (b) pyrrolo[1,2-a]pyridine (c) pyrido[3,4-d]pyrimidine

(65.2) Which of the heterobicycles in Table 65.1 are aromatic?


Chapter 65.1, Table 65.1: 3H-Pyrrolizine (electron sextet in one pyrrole ring), indolizine (electron sextet in
one pyrrole ring) and dehydroquinolizinium ion (electron sextet in one pyridine ring) are aromatic.
(65.3) Design a synthesis of 2-methylindolizine.
Chapter 65.1, analogous reaction: 2-Methylindolizine can be synthesized from -picoline (2-methylpyridine) and
bromoacetone. Set R = CH3 in the reaction equation.
(65.4) Draw the structural formulas of two purine isomers that contain a fused pyrazole instead of an imidazole ring,
and name them.
Chapter 65.2.1: Pyrazol[4,3-d]pyrimidine and pyrazolo[3,4-d]pyrimidine are isomers of purine containing a fused
pyrazole instead of the imidazole ring. Provided ring numbering is the same as in purine, the bicycles are more
precisely denoted as 9H-pyrazolo[4,3-d]pyrimidine and 7H-pyrazolo[3,4-d]pyrimidine.
H
N N N
pyrazolo[4,3-d]pyrimidine N pyrazolo[3,4-d]pyrimidine N
N N N
H

(65.5) Which tautomers exist for (a) purine and (b) 2,4,5,6-tetraoxohexahydropyrimidine (alloxan)?
Chapter 65.2.1, concerning purine (a); Chapter 65.2.3, concerning alloxan (b)
(65.6) Which (a) purine nucleobases and (b) purine stimulants do you know?
Chapter 65.2.1, Table 65.2, answering (a) and (b)
(65.7) Formulate equations to describe TRAUBE’s guanine synthesis.
Chapter 65.2.2
(65.8) Formulate an equation to suggest the preparation of pterine from an intermediate in TRAUBE’s guanine synthesis.
Chapter 65.3.2, analogous reaction: Pterine is synthesized by cyclodehydration of 2,4,5-triamino-6-oxo-
1,6-dihydropyrimidine and glyoxal (ethanedial or the mono- or diacetal).
O O
NH 2 O H N
HN HN
+
2 H2 O
H 2N N NH 2 O H H2N N N
2,4,5-triamino-6-oxo-1,6-dihydropyrimidine glyoxal pterine

(65.9) Formulate equations to describe the preparations of (a) adenine and (b) 6,7-dimethyl-2-aminopteridine.
Chapter 65.2.2, concerning adenine (a); Chapter 65.3.2, concerning 6,7-dimethyl-2-aminopteridine (b), obtained
from 2,4,5-triaminopyrimidine and diacetyl (butanedione); set R1 = R2 = CH3 in the second reaction equation.
(65.10) Formulate an equation to synthesize the heterocycle of riboflavin (Table 65.3).
Chapter 65.3.2, last reaction equation

(66.1) Which structural features cause the color of a compound?


Chapter 66.1
(66.2) List the chromophores in organic compounds giving rise to UV and visible light absorption and classify them with
respect to the electronic transitions they may undergo upon excitation.
Chapter 66.1
(66.3) Explain why trans-stilbene (1,2-diphenylethene) has a longer wavelength UV absorption than the cis-isomer.
Chapter 66.1: As described for azobenzene, steric repulsion of the hydrogen atoms at the o-positions of the
288 Working on Questions

phenyl rings in cis-stilbene prevents coplanarity required for undisturbed conjugation of all double bonds and the
smaller max (278 nm instead of 298 nm for trans-stilbene) reflects this.

trans -stilbene cis -stilbene


coplanarity h H no coplanauty
conjugation H disturbed conjugation

(66.4) What are the structural elements of a dye? Draw the structural formulas of some azobenzene derivatives to explain.
Chapter 66.3.1
(66.5) Draw resonance formulas of (a) pinacyanol, (b) indigo, (c) crystal violet (= gentian violet), and (d) phenolphthalein
at pH > 8. Classify these dyes.
Chapter 66.3.2, answering (a); Chapter 66.3.4, answering (b) as follows; Chapter 66.3.3, answering (c) as follows
and (d)
O O O
D D D

D D D
O O O
(b) indigo, D = NH, resonance formulas

IN(CH3 ) 2 N(C H3 ) 2 IN(C H 3 ) 2

C C C Cl

(H3 C) 2N N(CH 3) 2 (H 3 C) 2 N N(C H 3 ) 2 (H3 C) 2N N(CH3 ) 2

(c) crystal (gentian) violet, resonance formulas

(66.6) How do dyes differ from pigments?


Chapter 66.2

(67.1) How do you account for the outstanding stability of porphyrins and phthalocyanines?
Chapter 67.1
(67.2) What is the difference between heme and hemin? How is hemin obtained?
Chapter 67.2.1
(67.3) What is the difference between porphine and chlorin? Draw formulas.
Chapters 67.2.1, 67.2.2
(67.4) Briefly outline the biological function of (a) heme and (b) chlorophyll.
Chapters 67.2.1, 67.2.2

(68.1) How do you account for the high melting points of amino acids?
Chapter 68.1
(68.2) What are essential amino acids and which ones do you know?
Chapter 68.1, Table 68.1
(68.3) Specify the absolute configuration of (a) L-serine and (b) L-cysteine following the CIP convention.
Chapter 68.1, Table 68.1: (a) L-serine = (S)-serine, but (b) L-cysteine = (R)-cysteine (S-atom > O-atoms)
(68.4) Write equations to suggest a synthesis of racemic phenylalanine from a derivative of diethyl malonate.
Chapter 68.2, first reaction equation: set R = C6H5CH2 and X = Br for benzyl bromide.
(68.5) Write equations to suggest a synthesis of (R)-phenylalanine from benzyl bromide (-bromotoluene).
Chapter 68.2, second reaction equation: set R = C6H5CH2 , X = Br and start with the dioxopiperazine
of L-valine.

(69.1) The CN bond in amides and peptides is shorter (132.5 pm) than in amines (148.7 pm). Why?
Chapter 69.1
Working on Questions 289

(69.2) What is an -helix? Explain the terms primary, secondary, tertiary, and quaternary structure of a protein.
Chapter 69.3, Fig. 69.2
(69.3) Look at Table 68.1 to draw the structural formula of the peptide with the sequence LATE.
The sequence LATE abbreviates the following (primary) structure:
CH(CH3)2 CH 3
CH2 H O CHOH H
LATE = Leu-Ala-Thr-Glu CH N C CH N CO2
H3N C CH N C CH
O CH 3 H O CH 2
CH 2 CO2H

(69.4) Which protective groups are used for (a) amino and (b) carboxy functions? Outline their introduction and removal.
Chapter 69.4.1, answering (a) and (b)
(69.5) Formulate the reaction of a Boc-amino acid with an amino acid ester. Explain the purpose of ester activation.
Chapter 69.4.2
(69.6) Write the equations for all steps necessary to synthesize the dipeptide Phe-Leu (FL).
Chapter 69.4.3, Fig. 69.3: Set phenylalanine for N-terminal, leucine for C-terminal amino acid.
Find formulas of the amino acids in Table 68.1.

(70.1) What are alkaloids and which biological functions do they perform?
Chapter 70.1
(70.2) Which amino acids are alkaloid precursors and which alkaloid classes are derived from them?
Chapter 70.1, Table 70.1
(70.3) Draw the structural formulas of (a) ()-cocaine, (b) ()-nicotine, and (c) ()-LSD. Classify these alkaloids.
Chapter 70.2, Table 70.2, answering (a), (b) and (c)
(70.4) Which alkaloid subunit do you recognize in the illegal drugs "speed" and "ecstasy"?
Chapter 70.2, last section, the phenylethylamine alkaloids and "drugs"
(70.5) Formulate the reaction of ()-morphine with acetic anhydride.
Acetylation of morphine with acetic anhydride produces diacetylmorphine, known as heroin.
OH O O CH 3
C
C CH 3 O
()-morphine O + 2 O O ()-diacetylmorphine
 2 CH3CO2H
H 3C C CH 3 H 3C O
N N
O C
OH O CH3

(70.6) Count the number of stereogenic centers in morphine. How many stereoisomers are possible? Look at Chapter 46.1.
Five carbon atoms and the nitrogen atom in morphine are stereogenic centers. Thus, 26 = 64 stereoisomers are
possible, comprising 32 diastereomers which are pairs of enantiomers.

(71.1) Which compound is named ()-(2R,3S,4R,5R)-2,3,4,5,6-pentahydroxyhexanal? How many stereoisomers exist?


Chapter 71.1
(71.2) Write FISCHER projections of all (a) aldohexoses, (b) aldopentoses, and (c) hexuloses, all with the D configuration.
Chapter 71.1, 71.2, answering (a) and (b) in Fig. 71.1, answering (c) in Fig. 71.2
(71.3) Draw the formulas of (a) - and -D-mannopyranose and (b) - and -D-ribofuranose. Define the term mutarotation.
Chapters 71.3, 71.4 (mutarotation)
(a) - and -D-mannopyranose and (b) - and -D-ribofuranose (Chapter 71.3) can be clearly depicted by means
of their skeletal formulas.
HO HO OH
HOH2C OH HOH2C OH
4 O 1 4 O 1
HO O HO O
HO HO OH OH
HO OH HO OH
a) OH- and -D-mannopyranose b) - and  -D-ribofuranose
290 Working on Questions

(71.4) What is a glucoside? Draw the structural formula of 2-nitrophenyl -D-glucopyranoside and design a synthesis.
Chapter 71.5.1, analogous reaction: Glycosidation of o-nitrophenol (2-nitrophenol) with acetobromo--D-glucose
produces 2-nitrophenyl -D-glucopyranoside (KÖNIGS-KNORR glycosidation). SN2: dictates inversion of
configuration at C-1 ( glycosidation because of stereospecificity). SN1: The O-acetyl group at C-2 shields the
 position of the intermediate carbenium ion from nucleophilic attack ( glyosidation because of stereoselectivity).
CH2OAc CH2OAc CH2OH
NO2 + Ag2CO3 NO2 + 4 NaOH NO2
O AcO O HO O
2 AcO + 2 HO 2 AcO O 2 HO O
AcO 1  2 AgBr  4 NaOAc
Ac = COCH3 O Br  CO2 OAc OH
H3C  H 2O 2,3,4,6-tetra-O-acetyl-
O o-nitrophenol 2-nitrophenyl -D-glucopyranoside 2-nitrophenyl -D-glucopyranoside

(71.5) D-Glucose is exposed to ethanol containing some anhydrous hydrogen chloride. What products are formed?
Chapter 71.5.1, analogous reaction: Glycosidation of D-glucose with ethanol and anhydrous hydrogen chloride
is expected to produce a mixture of ethyl - and ethyl -D-glucopyranoside (FISCHER glycosidation).
HOH2C HOH2C HOH2C
O [H ] , heat O O
HO HO + HO
2 HO OH + 2 C 2H5OH HO HO OC2H 5
OH OH OH
OC 2H5
- and -D-glucopyranose ethyl - and ethyl -D-glucopyranoside (anomeric acetals)

(71.6) What products are obtained from glucose by (a) reduction and (b) oxidation?
Chapter 71.5.3

(72.1) What compound is named -D-fructofuranosyl--D-glucopyranoside? Draw the structural formula.


Chapter 72.1, Fig. 72.1
(72.2) What is the difference between maltose and cellobiose? Draw the structural formulas to explain.
Chapter 72.1, Fig. 72.1
(72.3) What is the difference between maltose and trehalose? Suggest a test tube reaction to identify one of them.
Chapter 72.1, Fig. 72.1
(72.4) What products are obtained by enzymatic degradation (hydrolysis) of starch?
Chapter 72.1, last section
(72.5) Draw the structures of (a) amylose, (b) cellulose, and (c) chitin. What are the sources of these polysaccharides?
Chapter 72.2, answering (a), (b) and (c)

(73.1) Disconnect a nucleic acid stepwise via larger fragments to the monomeric components. Formulate and name them.
Chapter 73.1
(73.2) Adenosine 5´-triphosphate (ATP) is an energy source, phosphorylation reagent, and coenzyme in biosyntheses.
Draw the structural formula and classify this compound.
Chapter 73.1, Fig. 73.1: Adenosine-5´-triphosphate (ATP) is a mononucleotide.
NH 2

N
N
HO HO HO
HO O O O 5´ N N
adenosine-5´-triphosphate (ATP) P P P O
4´ 1´
O O O 3´ 2´

HO OH

(73.3) Draw the DNA sequence ACGT in detail and abbreviated.


Chapter 73.1, Fig. 73.1: Exchange thymine (T) and guanine (G).
(73.4) What are base pairs and how are they held together?
Chapter 73.2, Fig. 73.2
(73.5) An open-chain DNA strand would have a length of about 2 m. In fact, the length is in the nm range. Why?
Chapter 72.2, last section
Working on Questions 291

(74.1) (a) Explain the term lipid. (b) What are fats and which kinds of other lipids do you know? (c) What are RMEs?
Chapter 74.1, Table 74.1, answering (a) and (b); Chapter 74.2.2, answering (c)
(74.2) Which compound is described by the abbreviation C18:2(9c,12c)? Draw the formula. What is an 6 fatty acid?
Chapter 74.2.1, Table 74.2: linoleic acid, 18 C atoms, 2 double bonds, 9-cis-, 12-cis-, 6
(74.3) Suggest a test tube reaction to distinguish a saturated from an unsaturated fatty acid. Look at Chapter 16.2.
Unsaturated fatty acids decolorize a solution of bromine (addition of Br2 to CC double bonds, Chapter 16.2).
(74.4) What is the difference between a fat and a wax? How are soaps related to fats and how does a soap clean?
Chapters 74,2.1, 74.2.2

(75.1) Polyketide is a misleading term. Why? Explain your reasoning by briefly formulating the polyketide pathway.
Chapter 75, introducing section and Chapter 75.1
(75.2) Explain the biogenetic relationship of polyketides and fatty acids.
Chapter 75.1
(75.3) What is the smallest polyketide? Which biologically active tetraketides do you know?
Chapter 75.2, first section, Table 75.1
(75.4) How are the benzenoid rings of naturally occurring aromatic compounds and quinones formed?
Chapter 75.2, Table 75.1

(76.1) What is a terpenoid structure? Which classes of terpenes do you know?


Chapter 76.1, Table 76.1
(76.2) Suggest a synthesis of limonene (Chapter 21.4). Specify which stereoisomers are formed.
Chapters 21.4, 76.3.1: [4+2]-Cycloaddition (DIELS-ALDER reaction) of isoprene (2-methyl-1,3-butadiene) produces
the racemate (R,S)-limonene.

S R
+

1,3-diene dienophile
isoprene (2-methyl-1,3-butadiene) (R,S)-limonene (racemate)

(76.3) How many diastereomers and enantiomers of menthol exist? Draw the structures and specify configurations.
Chapter 76.3.1: Three C atoms of menthol are stereogenic centers. Thus, 23 = 8 stereoisomers exist, comprising
four diasterereomers which are pairs of enantiomers. Diastereomers are referred to as isomenthol, neomenthol
and neoisomenthol.

R S S R R S S R
R S R S S R S R
S R S R S R S R
OH HO OH HO OH HO OH HO

OH HO OH HO

OH HO OH HO
menthol isomenthol neomenthol neoisomenthol

(76.4) What is squalene? How is it related to tetracyclic triterpenes and to steroids? Try to formulate.
Chapter 76.3.4
(76.5) What is a carotenoid? What causes the color of carotenoids? Look again at Chapter 66.1.
Chapters 76.3.5, 66.1

(77.1) (Almost) all steroids possess a hydroxy or carbonyl function at position 3. Why?
Chapter 76.3.4: 2,3-Epoxysqualene is the biogenetic precursor of tetracyclic triterpenes and steroids with the
gonane skeleton; it introduces the hydroxy function at position 3 of the steroids.
(77.2) Briefly formulate the genesis of cholesterol in mammals.
Chapter 77.2
(77.3) Which steroid hormones do you know and what are their biological functions?
Chapter 77.4
292 Working on Questions

(77.4) Look at Chapter 49.3.1 to suggest a partial synthesis of 17-ethynylestradiol, a component of contraceptives.
Alkynylation of estrone with ethyne (acetylene) and sodium amide as a base in liquid ammonia (Chapter 49.3.1)
produces 17-ethynylestradiol. Spacefilling of the methyl group at C-18 shields the  position from nucleophilic
addition.
(+)-estrone O 17-ethynylestradiol OH C H
C

H NaNH 2 H
liquid NH3
+ H C C H
H H  78 °C H H
HO HO

(78.1) Explain (a) chemoselectivity, (b) regioselectivity, (c) stereoselectivity, and (d) stereospecificity. Provide examples.
Chapter 78.1, answering (a); Chapter 78.2, answering (b); Chapter 78.3, answering (c); Chapter 78.4,
answering (d)
(78.2) 2-Pentanone and allyl bromide yield 1-octen-5-one when reacted with LDA. Formulate and characterize this reaction.
Chapter 78.2
(78.3) Formulate examples for the (a) regioselectivity, (b) stereoselectivity, and (c) stereospecificity of [4+2]-cycloadditions.
Chapter 78.2, answering (a); Chapter 78.3, answering (b); Chapter 78.4, answering (c)
(78.4) (S)-2-Bromobutane yields (R)-2-butanol when reacted with hydroxide. Formulate and characterize this reaction.
Chapters 31.1, 44.4 and 78.4

(79.1) Explain the term prochirality at a tetrahedral and a trigonal carbon atom.
Chapters 79.1, 79.2
(79.2) Draw the tetrahedral projection formula of the methylene C of ethanol and assign the enantiotopic H atoms.
Chapter 79.1
(79.3) Formulate a reaction which converts enantiotopic groups into diastereotopic ones.
Chapter 79.1, last section
(79.4) Chirogenic reactions produce racemic mixtures under usual conditions. Why? Formulate examples.
Chapters 79.1, 79.2
(79.5) Formulate examples of enantioselective catalytic hydrogenations.
Chapter 79.3

Design syntheses of the following target compounds:


(80.1) (a) (E)-Butenal and (b) (E)-2-ethyl-2-hexenal;
Chapter 80.3.1, answering (b)
(a) In analogy to (b), retrosynthetic disconnection of (E)-butenal via racemic aldol 3-hydroxybutanal exposes
aetaldehyde as the starting compound, exemplified in Chapter 80.2.
H OH H OH H OH H
C + C C + C
O O H 3C H ICH 2 O H 3C H H 2C O
 H2O base
aldol reaction carbanion (nucleophile) enolate
(E)-butenal 3-hydroxybutanal acetaldehyde

(80.2) (a) 1-methylcyclohexene-4-carboxylic acid methyl ester and (b) bicyclo[2.2.1]heptane-2-carboxylic acid ethyl ester;
Chapter 80.2, Table 80.2: 1-Methylcyclohexene-4-carboxylic acid methyl ester (a) arises from a [4+2]-
cycloaddition of methyl acrylate to 2-methyl-1,3-butadiene (isoprene) as exemplified. (b) Bicyclo[2.2.2]heptane-
2-carboxylic acid ethyl ester (endo- or exo-) is produced by [4+2]-cycloaddition of ethyl acrylate (dienophile) to
cyclopentadiene (1,3-diene):

CO2C 2H 5 CO2C 2H 5
CO2C 2H5
+
synthesis
CO2C 2H 5
endo- exo-bicyclo[2.2.1]heptane-2-carboxylic acid cyclopentadiene acrylic acid
ethyl ester ethyl ester
Working on Questions 293

(80.3) (a) ethyl 2,4-dioxoheptanoate, (b) 2,4-pentanedione, and (c) 2-(4-isobutylphenyl)propanoic acid;
Chapter 80.3.2, answering (a); Chapter 80.3.3, answering (c)
Retrosynthetic disconnection of 2,4-pentandione (b) to the acetyl cation (from ethyl acetate) and the acetylmethyl
carbanion (from acetone) explains the synthesis of the target by CLAISEN condensation of ethyl acetate and
acetone as outlined in Chapter 50.2.2.
H H H H H H
NaOC 2 H 5
H 3C C CH 3 H3C + C CH3 H 3C OC 2H 5 + H 3C CH3 H3C C CH 3
C C retrosynthetic C C C C C 2 H 5 OH C C
O O disconnection O O synthesis
O O O O
2,4-pentanedione

(80.4) 9-tetrahydrocannabinol from (R)-p-mentha-2,8-dien-1-ol.


Chapter 80.3.4

(81.1) How many double bond equivalents hide behind the molecular formulas (a) C5H5N, (b) C6H12O6, and (c) C6H7NO2?
Chapter 81.1: The numbers of double bond equivalents are (a) 4, (b) 1 and (c) 4.
(81.2) Give examples of (a) all kinds of skeletal isomers and (b) all kinds of stereoisomers.
Chapter 81.2, answering (a); Chapters 81.4, 81.5, answering (b)
(81.3) Which (a) skeletal isomers and (b) stereoisomers exist for C4H8O? Draw and classify all of them.
(a) 22 skeletal and configurational isomers with the molecular formula C4H8O exist, also comprising three enol
tautomers of butanal (one, cis- and trans-) and butanone (two, one cis- and trans-); (b) cis- and trans-isomers
exist for 2-buten-1-ol, methyl propenyl ether and methylcyclopropanol (different relative configuration); cis- and
trans-methylcyclopropanol are diastereomers, and each one of those is a pair of enantiomers (with different
absolute configurations).
H OH OH OH
OH OCH3
O
butanal O butanone O trans- cis- 2-methyl- ethyl vinyl allyl methyl
3-buten-1-ol 2-buten-1-ol 2-propen-1-ol ether ether
H H

OH OH OH OH
cis-enol trans-enol cis-enol trans-enol

OH HO OH HO
OH OH
R S R S
OCH3 OCH3 CH2OH OCH 3 S R R S
CH3 H3C CH 3 H3C
cis- trans- cyclobutanol hydroxymethyl- cyclopropyl methyl cis- trans-
methyl propenyl ether cyclopropane ether methylcyclopropanol

(81.4) What significance does the absolute configuration have in biochemical processes? Give examples.
Chapter 81.5

(82.1) The molecular formula of a compound can be determined by mass spectrometry. Explain how.
Chapter 82.2, second and third section
(82.2) Explain how you recognize a nitrogen compound with an odd number of nitrogen atoms in a molecule.
Chapter 82.2, last section
(82.3) Formulate (a) the generation of the molecular ion, (b) an  fragmentation, and (c) a benzyl fragmentation.
Chapters 82.2, 82.3, answering (a); Chapter 82.3, answering (b) and (c)
(82.4) Comprehend the elucidation of the skeletal structure from Fig. 82.1. Which structural aspect remains unresolved?
Chapter 82.3: The absolute configuration of amphetamine (racemate, pure enantiomer, enantiomeric excess)
cannot be decoded by mass spectrometry and other methods of molecular spectroscopy (in non-chiral environ-
ment). Specific rotation and other chiroptic methods (Chapter 44.2) are appropriate methods to solve this problem.
294 Working on Questions

(83.1) Which molecular vibrations do you know and what property is required for them to be IR-active?
Chapter 83.2
(83.2) IR absorptions of alkynes (CC 2200 cm1) are very weak; those of nitriles (CN 2250 cm1) are very strong. Why?
Chapter 83.2: Nitrile groups display much more intense IR absorption bands because the polar CN triple bonds
are vibrating dipoles in contrast to the unpolar CC triple bonds of alkynes.
(83.3) Which IR absorption bands can be used to identify a primary amino group?
Chapter 83.3, first section
(83.4) Which IR absorption bands characterize a carboxylic acid ester?
Chapter 83.3, Fig. 83.4, last section

(84.1) Measure the integral steps (mm) in Figs. 84.2, 84.3, and 84.6 and evaluate the results.
Chapter 84.3, Fig. 84.2
(84.2) Try to predict the proton NMR spectra of 2- and 3-pentanone. All coupling constants are 7 Hz (why?).
Chapters 84.4.1, 84.4.2, Figs. 84.4, 84.5
2-Pentanone displays four signals: one intense singlet for the protons of the methyl group attached to the
carbonyl function; one triplet for the protons of CH2 attached to carbonyl (X2), one sextet (n = 5 coupling protons
so that n+1 = 6) for the protons of the central CH2 group (M2) and one triplet for the protons of the terminal CH3
group (A3) belong to the n-propyl group (CH3CH2CH2, an A3M2X2 spin system, ratio of integral steps: 3:2:2).
Because of molecular symmetry, 3-pentanone displays only two signals for two chemically equivalent ethyl
groups, comprising one triplet for the methyl protons and one quartet for the methylene groups of ethyl
(CH3CH2, A3X2 spin system, ratio of intergral step heights: 3:2). Look at Chapter 84.4.2 and Fig. 84.5 to
explain the averaged coupling constant (7 Hz) of chemically non-equivalent alkyl protons.
(84.3) Specify the spin systems in Figs. 84.3 and 84.6. Assign all coupling constants.
Chapters 84.4.1, 84.4.2: AMX spin systems characterize vinyl in acrylic acid ethyl ester (Fig. 84.3)
and ethynyl attached to ethenyl in 1-methoxy-1-buten-3-yne (Fig. 84.6)
(84.4) Which kind of NMR data reflects the relative configuration? Provide some examples.
Chapters 84.4.2, 85.3 including examples

(85.1) Homonuclear 13C13C couplings are usually not observed in 13C NMR spectra. Why?
Chapter 85.1
(85.2) How do aldehydes and ketones differ in carbon-13 NMR spectroscopy?
Chapters 85.2, 85.3: Unlike ketones (R2C=O), aldehydes display a doublet splitting of the 13C signal (RCH=O) in
the 13C NMR spectrum with coupling constant JCH  190 Hz due to their CH bonds.
(85.3) How do ethyl, ethenyl and ethynyl groups differ with respect to (a) 13C chemical shifts and (b) CH couplings?
Chapter 85.2, Fig. 85.1, answering (a): 13C chemical shifts increase in the order ethyl < ethynyl < ethenyl;
Chapter 85.3, answering (b): CH coupling constants of ethyl: JCH  125 Hz, ethenyl: JCH  165 Hz and
ethynyl: JCH  250 Hz follow the rule JCH = 500 s, where s = 0.25, 0.33 and 0.5 for sp3, sp2 and sp hybridized
C atoms, respectively.
(85.4) Summate all C, CH, CH2, and CH3 in Fig. 85.2. Why is the result different to the molecular formula?
Chapter 85.3, last section

(86.1) Briefly describe the concept of two-dimensional shift correlations and their use in molecular structure elucidation.
Chapter 86, introduction
(86.2) Evaluate the HH COSY plot in Fig. 86.1 including the integral steps (on top) to decode the skeletal structure.
Chapter 86.1.1, Fig. 86.1
(86.3) Evaluate the CC shift correlations in Fig. 86.2 in order to establish the carbocyclic molecular structure.
Chapter 86.1.2, Fig. 86.2
(86.4) Assign all CH bonds of the sample molecule in Fig. 86.3. One proton does not give a cross signal. Why?
Chapter 86.2, Fig. 86.3
Subject Index 295

Subject Index
Subject Index
acidity adenosine 214
A carboxylic acids 109 monophosphate, 5´- (AMP) 214
CH 136 triphosphate, 5´- (ATP) 290
absorption maximum cyclopentadiene 74 adipic (hexanedioic) acid
UV-vis spectra 194 dicarbonyl compounds diethyl ester
acetaldehyde (ethanal) 85, 126 1,3- 138 DIECKMANN
aldol reaction 137 methyl groups 189 cyclocondensation 136
molecular models 128 methylpyridine preparation 108
production 50 2- and 4- 181 aglycon 210
acetals 132 constant 109 AIBN (azobisisobutyronitrile) 103
acetic (ethanoic) acid 106 phenols 142 alanine 200
activated (acetyl-S-CoA) 152, 218 sulfonic acids 153 alcoholates (alkoxides) 90
alkylated 138 acridine 186 alcoholic
amide 111 acrolein (propenal) 126 beverages 85
anhydride 111 dimerization 172 fermentation 85
acetylation reagent 111, 144 acrylic (propenoic) acid 106 alcohols 84
butyl ester, t- chloride 111 acidity 90, 142
IR spectrum 241 ethyl ester basicity 90
chloro- 109, 115 proton NMR spectrum 244 dehydration 36, 92
esters 111 acrylonitrile (cyanoethene) 50 bimolecular 94
ethyl ester 91 poly (PAN) 164 relative reactivity 92
CLAISEN condensation 136 activation energy 30, 81 enantiomers 119
hydrazide 112 active esters 159, 204 nomenclature 84
acetoacetic acid acyl cations preparation 134
diketide 218 electrophiles 65 primary 84
ethyl ester 136, 138 acyl halides 111 oxidation 90, 108, 127
methyl ester hydrogenation 112 preparation 86
oxo-enol tautomerism 243 acylation secondary 84
proton NMR spectrum 243 electrophilic oxidation 90
acetobromoglucose 211 of benzene 65 preparation 86
acetogenins (polyketides) 218 of naphthalene 71 tertiary 84
acetone (propanone) 130 poly- 166 preparation 87
cyanohydrin 103 acylcarbenes 161 aldaric acids 211
production 131, 143 acylnitrenes 160, 161 aldehydes 126
acetonylacetone acyloin(s) 113, 134 acetals 132
hexanedione, 2,5- 130 reaction (condensation) 113 bisulfite adducts 129
heterocyclizations 175 addictive carbonyl umpolung 133
acetophenone 65, 130 compounds 85, 206, 207, 219 homologization 105
hydroxy-, p- 144 addition(s) 28, 40 hydrates 132
acetylacetone anti or trans 40, 58 oxidation 128
pentanedione, 2,4- 130, 138 electrophilic 41, 46 oxo-, 3- 138
heterocyclizations 179 ring opening 1,3- 58 preparation by
metal chelates 141 to alkynes 50 formylation 127
oxo-enol tautomerism 140 nucleophilic oxidation 90, 127
preparation 137 MICHAEL 139 reduction 112, 127
acetylene (ethyne) 15 to heteroaromatics 188 reactions 132
acetylsalicylic acid radical 47 alditols
ASS, aspirin 144 syn- or cis- 40, 59 polyols (sugar alcohols) 211
acetyl-S-coenzyme A 218 addition-elimination mechanism aldohexoses 208
of electrophilic substitution 177 aldol 137
adenine (A) 191, 214 condensation 137
TODD synthesis 192 reaction 137, 232
MUKAIYAMA 169
aldonic acids 211

Efficiently Studying Organic Chemistry: Exam training for chemists, biochemists, pharmacists, life and health scientists,
Third Edition. Eberhard Breitmaier. © 2022 WILEY-VCH GmbH. Published 2022 by WILEY-VCH GmbH.
296 Subject Index

aldopentoses 209 nucleophilic substitutions 79, 81 diazotization 98


aldoses polarity 78 from carbonyl compounds 101
diastereomers preparation 76 N-alkylation
and enantiomers 208 primary, secondary, tertiary 76 exhaustive 99
O-acylation, O-alkylation 211 reactions 78 nomenclature 96
reduction to polyols 211 alkyl shift, 1,2- 29, 93, 161 phthaloyl- 97
alizarin 149 alkylation preparation 97, 101
alkaloids 171, 206 of dicarbonyl compounds 139 primary 96
tetraketide 219 of malonic acid diesters 138 IR absorptions 241
alkanes 16 alkylbenzenes 62 preparation 97, 161
branched and unbranched 18 oxidation 69 secondary 96
separation 156 photohalogenation 76 cyclic 171
cracking 37 side chain substitution 68 preparation 101
energy sources 17 alkylmagnesium halides 79, 82 tertiary 96, 162
fluorination 77 addition to preparation 101
from petroleum 16 carbonyl compounds 134 amino acids 200, 237
nitration 33 carboxylation 108 Boc-protected 204
nomenclature 20 alkylzinc halides 116 carboxy-protected 204
oxidation (combustion) 17 alkynes 48 enantiomers, D- or L-
phenyl- 62 cyclooligomerization 51 preparation 201
preparation cyclotrimerization 63 essential
KOLBE electrolysis 17 hydration 50 and non-essential 200
WURTZ synthesis 17 hydroboration 49 esters 204
sulfochlorination 33 hydrohalogenation 50 ninhydrin reaction 201
alkenes 34 partial hydrogenation 37, 49 racemic
additions 40 phenyl- 62 preparation 115, 201
bromination 40 reduction 49 amino group(s) 96
catalytic hydrogenation 40 terminal 48 ()-M effect 66
hydration 41 CH acidity 51 aminolipids 216
hydrohalogenation 41 GLASER coupling 51 ammonia
cis- (Z)- and trans- (E)- 35 alkynoic acids 108 alkylation to amines 97
identification by NMR 245 alkynols 134 ammonium
selective preparation 49 alkynylides 51, 133 hydroxides
cycloadditions 42 carboxylation 108 tetraalkyl- 99
dialkylamino-, 1-N,N- allene(s) 44 ions and salts
enamines 100 disubstituted, 1,3- quaternary (tetraalkyl-) 96
dihydroxylation 42 axial chirality 122 rearrangements 162, 163
ene reaction 39 molecular shape 45 amphetamine 207
halogenation 40 allose 208 IR spectrum 241
metathesis 39 alloxan 192 mass spectrum 239
phenyl- 62 allyl N-methyl-
preparation 100, 135 alcohol (2-propen-1-ol) 86 speed, meth, crystal 207
structural isomers 34 bromide 38, 45 amylopectin 213
synthesis by cation 46 amylose 213
dehydrogenation 37 chloride (3-chloro-1-propene) 88 anabolic steroids 225
elimination 36 radicals 38, 163 analgesics 144, 207, 210, 226, 227,
partial hydrogenation 37 altrose 208 231, 235, 237
terminal and ambident reactivity androsterone 225
non-terminal 34 of hydroxy and amino acids 117 anesthetics 206, 222
alkoxides (alcoholates) 90 amides angelic acid 221
alkoxy groups carboxamides 111 angle strain 52
nomenclature 21 sulfonamides 153 angular fusion
alkyl groups 21 amidines 113 of benzene rings 70
branched and unbranched heterocyclizations 179 aniline(s) 62, 96
nomenclature 20 amination, reductive 101 basicity 98
IR absorptions 241 amine N-oxides dimethyl-, N,N-
alkyl halides 32, 76 enantiomers 122 azo coupling 102
alkylation reagents 64, 138 amines 96 nitroso-, p- 99
dipole moment 78 basicity 98 production 97
metalation 79, 82 cyclic 170 resonance formulas 98
Subject Index 297

anionotropic rearrangements 29 from coal and petroleum 70 azoles 174


annulenes 75, 263 preparation 71 preparation 175
[14]- 75 substitutions azomethines (imines) 100, 133
[18]- 75 electrophilic 71
di- and hexaaza- 198 porphyrins 198 B
anomers 209 production 63
anthocyanidins 186 aromaticity 61 BAEYER-VILLIGER oxidation 161
anthracene 70 benzenoid 61 BALZ-SCHIEMANN reaction 103
cycloadditions, [4+2]- 73 and proton NMR 243 barbiturates 141
reactions 72 heterocycles barbituric acid
anthrahydroquinone five-membered 174 preparation 141
endoperoxides 148 six-membered 178 base pairing 215
anthraquinone(s), 9,10- 72, 146 non-benzenoid 74 basicity
biosynthesis 218 porphyrins 198 amines 98
dyes 197 aroxyl radicals 145 constant 98
electrophilic substitution 149 aryl halides 64 bathochromic (red) shift 195
emodins 219 metalation 82 BECKMANN rearrangement 162, 167
preparation 147 nucleophilic substitution 68 beers 85
sulfonic acids 149 preparation 64 beeswax 217
tetrahydro- 148 arylhydrazines 133, 141 behenic acid 216
antiauxochrome 196 preparation 102 BEILSTEIN test 76
antibody proteins 202 asparagine 200 benzaldehyde 126
antifeedants 221 aspartame cyanohydrin 134
anti-periplanar (anti-) 25 enantiomers 237 preparation 127
arabinose 208 aspartic acid 200 reactions 134
arachidic acid 216 asphalt 16 benzene(s) 60
arachidonic acid 216 asymmetric carbon atom diazonium cation
arenediazonium cations see stereogenic centers 118 resonance stabilization 99
electrophiles 102 atomic orbitals 8 from ethyne 51
resonance stabilization 99 hybridization 12 KEKULÉ and DEWAR formulas 61
arenes 61 overlapping 10 molecular orbital model 61
polycyclic 70 p 8 molecular shape 60
arginine 200 symmetry 11 oxide 284
ARNDT-EISTERT synthesis 161 s 8 polyalkylated
aromatic compounds 61 atropine 206 preparation 63
benzenoid atropisomerism 123 production 63
from coal and petroleum 63 autoxidation reduction 69
nomenclature 62 of anthrahydroquinones 148 resonance stabilization 60
rearrangements 163 of cumene 143 substituted
substitutions of ethers 95 nomenclature 62
electrophilic 64, 66 auxochrome 196 preparation 64, 66
nucleophilic 68 axial and equatorial bonds 53 substitutions
heterocyclic azides electrophilic
five-membered 174 1,3-dipoles 175 mechanism 64
benzo-fused 182 aziranes (aziridines) 170 nucleophilic 68
substitutions preparation 172 sulfonic acid 65
electrophilic 176 reactions 173 benzenoid aromatic compounds
fused 190 azo biosynthesis 219
six-membered 178 coupling 102 nomenclature 62
benzo-fused 186 dyes 102, 196 substitution pattern
substitutions azoalkanes 103 from IR absorptions 241
electrophilic 181 radical precursors 103 benzimidazole(s) 183
nucleophilic 180 azobenzene(s) 102 benzo[a]anthracene 70
 electron deficient 178 cis- and trans- 102 benzo[a]pyrene 70
UV absorption 195 enzymatic epoxidation 73
 electron excessive 174
dimethylamino-, 4-N,N- 196 benzo[b]furan (coumarone) 182
non-benzenoid 74
nitro-, 4´- 196 nitration 184
 electron deficient 75
light absorption 196 benzo[b]thiophene (thionaphthene)
 electron excessive 75 nitration 184
polycyclic benzenoid 70 azobisisobutyronitrile (AIBN) 103
radical precursor 103 benzo[c]thiophene
carcinogenic 70 cycloaddition, [4+2]- 184
298 Subject Index

benzo-1,3-azoles bonds prochirality 230


preparation 183 covalent 10 structural isomers 18
benzoannelations 71 amide (peptide) 112 butanedial
benzoic acid(s) 62, 69 resonance formulas 202 succindialdehyde 126, 177
acidity 109 angles 14 butanediol, 2,3-
azide 112 carbon-carbon dimethyl-, 2,3- (pinacol) 89
hydroxy-, p- 109 determination 248 pinacol rearrangement 93
nitro-, p- 109 double 14 butanedione (diacetyl) 130
nomenclature 106 formation 83 butanol
preparation 108 single 14 1- 84
benzoin 134 triple 15 2- 84
reaction 134 carbon-hydrogen 13 3-bromo-
benzophenone 130 determination 249 stereoisomers 124
benzopyrylium ion 186 lengths 14, 15 3-methyl-2-phenyl- 87
benzoquinone  11, 15 enantiomers 118, 231
o- and p- 146 delocalized 44, 61 absolute
preparation 147  11, 14 configuration 119
benzothiazoles 183 ionic 10 butanone (ethyl methyl ketone) 130
benzoxazoles 183 borate, trialkyl- 86 butene
benzoyl chloride bornane (camphane) 222 1- 34
dinitro-, 3,5- 111 brandy 85 2- 34
benzyl bromination configurational isomers 35
alcohol 142 of alkenes 40 butenone (methyl vinyl ketone) 130
cation 81 radical (WOHL-ZIEGLER) 38 butyl alcohol
chlorocarbonate 154, 226 of benzene (aromatics) iso- 84, 86, 87
halides electrophilic 64 n-, sec- 84
-halotoluenes 76 of cycloalkenes 58 t- 41, 84
radical 68 radical (WOHL-ZIEGLER) 76 elimination
bicyclo[4.4.0]dec-1-en-3,7-dione of naphthalene and substitution 92
ketalization 226 electrophilic 71 preparation 85
bicycloalkanes 52 bromobutane butyl chloride, t- 92
bile acids 224 1- butyl halides
BINAP nucleophilic substitutions 78 n-, iso-, sec-, t- 76
enantiomers and chelates 231 1- and 2- 76 butyllithium 82
BINOL 231 bromoethanol, 2- butyloxycarbonyl (Boc)
binaphthyl, dihydroxy-, 2,2'- ethylene bromohydrin 42 protective group 204
enantiomers 123 bromonium ion, cyclic 40 butynedioic acid diester
biopolymers bromooctane, 2-, (R)-()- dienophile 177
nucleic acids 214 WALDEN inversion 121 butyraldehyde (butanal) 126
polysaccharides 212 bromopropane, 1- and 2- 41 butyric (butanoic) acid 106
polyterpenes 223 brucine 207 bromo-, - 115
proteins 202 but-1-en-3-yne ethyl ester 110
biphenyl(s) 62 methoxy-, 1-, cis- methyl-, - 108
substituted proton NMR spectrum 245 butyrolactam, - 117
enantiomers 123 butadiene, 1,3- butyrolactone, - 117
bipyridine, 2,2'- (,'-) addition, 1,2- and 1,4- 46
preparation 179 conformers 44
BIRCH reduction 69, 72 cycloadditions, [4+2]- 47, 229
C
BISCHLER-NAPIERALSKI synthesis hydrobromination 46 caffeine 191
of isoquinolines 187 resonance formulas 44 CAHN-INGOLD-PRELOG
bislactam 201 butanal (butyraldehyde) 126 convention (CIP) 119, 230
bislactim ether 201 hydroxy-, 3- (aldol) camphane (bornane) 220, 222
bisphenol A preparation 137 camphor
bis(4-hydroxyphenyl)- methyl-, (2S)-2- enantiomers 222
propane, 2,2- 63 cyanohydrin reaction 228 Cannabis drugs 219
boat conformers butane 16 CANNIZZARO
of cyclohexane 54 bromo-, 2- disproportionation 129
Boc (butyloxycarbonyl) enantiomers 230 canonical formulas
protective group 204 methyl 220 benzene 61
molecular models 18 butadiene, 1,3- 44
Subject Index 299

caoutchouc 45, 164, 223 carbonyl compounds 43, 126 methylation, O- 104
caproic (hexanoic) acid 106 alkenylation 38, 135, 139 nomenclature 106
caprolactam, - 167 carbon-13 shifts (NMR) 246 peroxy- 161
carbamate CH acidity 136, 138 preparation by
ammonium 155 reactions with nucleophiles 132 carbonylation 107, 235
carbamic acid 155 reductive amination 101 carboxylation 107
esters (urethans) 155 reductive coupling oxidation 90, 108
carbamoyl chlorides 158 MCMURRY reaction 37 reduction 112
carbanions 27, 133, 136, 138, 181, pinacol reaction 89 substituted 114
227 thio- 152 unsaturated
acyl 136 carbonyl dyes 197 ,- 139
carbazole (dibenzopyrrole) 182, 183 carbonyl group 130, 132 ,- 169
carbene(s) 27 polarization 128, 132 carcinogens 73
acyl- 29, 160, 161 reduction to methylene 135 carotene,- 223
dichloro- 128, 185 resonance formulas 86, 128, 132 UV-vis spectrum 194
for cyclopropanation 56 reversal of polarity carotenoids 223
insertion reaction 104 umpolung 133 catalysis
low- and high-energy 27 carbonyl reactions 128, 132 cycles 30, 51, 90
carbenium ions 36 carbonylation 107 catalysts 30, 40
carbon-13 shifts (NMR) 246 carboxamides 111 LINDLAR 37, 49
classical and non-classical 26 dehydration 113 NOYORI 231
electrophiles 64 peptides 202 catalytic hydrogenation 17, 40, 59
relative stability 41 preparation 111, 162 catechol 142, 146
resonance-stabilized 46, 100, 133 reduction 112 cathartic agents 219
carbide process 48 resonance formulas 202 cationotropic rearrangements 29
carbobenzoxy chloride carboxy group 106 cellobiose 212, 213
benzyloxycarbonyl chloride resonance formulas 106 celluloid 222
benzyl chlorocarbonate 154 carboxylates 109 cellulose 213
carbocations 26 KOLBE electrolysis 17 chain reactions
carbodiimide(s) 158 carboxylation photohalogenation
preparation 159 electrophilic 107, 184 of alkanes 32
carbohydrates 208 carboxylic acid(s) 106 polymerization 165
carbon acidity constant 109 chair conformers
atom, tetravalency 7 amides (carboxamides) 111 cyclohexane 53
hybridization 12 amino-, - 115, 200 glucopyranose 236
influences anhydrides charge-transfer (CT) complex 148
coupling constant (NMR) 247 acylation reagents 65 chelating ligands 96, 112, 141, 179,
isotopes, stable mixed 111 198
12C and 13C 246 azides 112 chemical shift (NMR)
modifications cyano-, - 115 carbon-13 (13C) 246
graphite and diamond 7 decarboxylation 113 correlation
NMR, 13C 246 derivatives 110 two-dimensional 248
nucleophiles 134, 136 hydrolysis 108 proton (1H) 242
carbon dioxide 199 rearrangements 161 chemoselectivity 226
electrophile 107 dimers 107 chenodeoxycholic acid 225
hothouse gas effect 17, 158 dithio- 152 CHICHIBABIN amination 180, 188
carbon disulfide 157 esters 110 chiral recognition 237
production 158 acyloin reaction 113 chirality 118, 122
carbon monoxide 48, 107 CLAISEN condensation 136 axial 122
toxicity 198 reduction 112 helical 123
carbon tetrachloride fatty 110 planar 123
tetrachloromethane 32 halides 111 stereo descriptors 119, 122
carbonic acid 154 acylation reagents 65, 111 chitin 213
derivatives 154 halo-, - and - 115 chlorin 199
dichloride (phosgene) 154 homologization 161 chlorination
diesters 155 hydrazides 112 of alkanes
ester amides 155 hydroxy- radical 32
ester chlorides 154, 226 -, -, and - 116 of benzene
carbonium ions 26 - and - electrophilic 64
lactone formation 117
300 Subject Index

chlorobenzene 64 relative crystal violet (gentian violet) 197


nucleophilic substitution 68 alkenes 35 cumene (isopropylbenzene) 65
chlorofluoroalkanes cyclohexanes, disubstituted 54 hydroperoxide 143
ozone killers 77 descriptors 237 cumulation
chloroform (trichloromethane) 32 cis- = (Z)- and trans- = (E) 35 of double bonds 44
chloroformates determination (NMR) 245, 247 cumulenes 44
carbonic ester chlorides 154 diastereomers 124 axial chirality 122
chloromethane (methyl chloride) 32 erythro- and threo- 125 hetero- 158
chlorophyll a and b 199 conformation 24, 236 molecular shape 45
chlorotrimethylsilane secondary structure cyanamide 157, 159
TMSCl 168 of proteins 202 cyanide anion
cholesterol 224 conformers 24, 236 nucleophile 134
cholic acid 225 alkanes 24 toxicity 198
chromophores 194 cycloalkanes 52 cyanohydrin reaction 103, 134
chrysene 70 dienes, 1,3- diastereoselectivity 228
cinnamic acid, trans- 139 s-cis and s-trans 44 cycloaddition(s)
ethyl ester 116 glucopyranose 236 [2+1]- 42, 56, 59, 171, 233
cinnoline 186 population 245 [2+2]- 56, 172, 233
CIP syn and anti 25 [3+2]- 42, 59, 172
CAHN-INGOLD-PRELOG coniine 219 [4+1]- 47
convention 119 conjugation [4+2]- 47, 57, 172, 177, 233
citral 221 of double bonds 44 regioselectivity 227
citronellol and light absorption 195 stereoselectivity 228
enantiomers 237 cyclic 60 stereospecificity 229
CLAISEN connectivities 236, 248 dipolar, 1,3- 43, 172, 175
condensation skeletal structure 18, 236 cycloalkanes 52
of esters 136, 137, 189, 218 determination preparation 56
rearrangement 163 NMR spectroscopy 248 reactions 58
CLAR’s rule 70 contraceptives 225 cycloalkanols 84
clathrates 156 COPE rearrangement 57, 104, 163 cycloalkanones 130
CLEMMENSEN reduction 135 diaza- 182 cycloalkenes 52
coal 16 oxa- 163 catalytic hydrogenation 59
tar 63, 70 corticosterone 225 dihydroxylation 59
cocaine 206 cortisone 225 larger rings 57
codeine 207 COSY (NMR) cycloalkylation 138
collagen 202 correlation spectroscopy 248 cycloalkynes 52
COLLINS oxidation reagent 90 coumarone (benzo[b]furan) 182 cyclobutane(s)
collision complex 30 coupling constants (NMR) carboxylic acid 138
color carbon-proton 247 folded conformers 52
and light absorption 194 proton-proton 243, 244 preparation 56
complementary 194 vicinal cyclodehydrogenation 63
combination and relative cyclodextrins 213
of radicals 32, 165 configuration 245, 247 cyclohemiacetals 209
complementary covalent bond cycloheptadiene, 1,4- 57
color 194 C2 hydrocarbons 14 cycloheptatriene, 1,3,5- 104
nucleobases 215 hydrogen molecule 10 cycloheptatrienium
complex methane 13 cation 75
 and  64 cracking cyclohexadiene, 1,4- 69
configuration of alkanes 37 cyclohexane(s)
absolute 118 CRAM’s rule 228 conformation 53
biochemical significance 237 cresol dibromo-, 1,2-, trans- 58
descriptors o-, m-, and p- 62, 142 dimethyl-, 1,4-, cis- and trans- 69
(aR) and (aS) 122 CRIEGEE cleavage halogenation 58
(M-)- and (P-)- 123 of 1,2-diols 89 molecular models 53
(R)- and (S)- 119 cross signals (NMR) 248 preparation 56
D- and L- 120 crosslinking 165, 166, 167 ring inversion 53
inversion 121 crotonaldehyde cyclohexanediol
of two stereogenic centers 124 butenal, 2-, (E)- 126, 137 cis- and trans-1,2- 59
specification 118, 122, 237 crotonic acid dimethyl, 1,2-
butenoic acid, 2-, (E)- 106 pinacol rearrangement 93
Subject Index 301

cyclohexanedione, 1,4- D diazotization 98


tetrabromo-, 2,3,5,6- 148 dibenzenechromium 82
cyclohexanol DABCO dibenzofuran 182
butyl-, 4-t-, cis- and trans- 227 diazabicyclo[2.2.2]octane dibenzothiophene 182
ethynyl-, 1- 134 1,4- 155 DIC
methyl Dacron 166 diisopropyl-
1- 84 DCC carbodiimide 159, 204
2-, 3-, and 4- dicyclohexylcarbodiimide 159 dicarbonate
cis- and trans- 84 DDT di-t-butyl 155, 204
cyclohexanone dichlorodiphenyl- dicarbonic acid
trichloroethane, p,p´- 63
alkylation, - 101, 169 diesters 155
DEAD dicarbonyl compounds
butyl-, 4-t-
diethyl azodicarboxylate 103 1,2- 131
reduction
decalin, cis- and trans- 55, 72 heterocyclizations 192
stereoselectivity 227
decane 16 1,3- 138
reductive amination 101
decarboxylation 113 acidity, CH 138
cyclohexene
dehalogenation alkenylation 139
bromination, radical 76
reductive 37, 56 alkylation 139
conformation 55
dehydration chelate ligands 141
dicarboxylic acid, 4,5-, cis-
of alcohols 36 heterocyclizations 141
anhydride 229
of diols 45, 93 MICHAEL addition 139
dimethyl-, 1,2- 93
dehydrohalogenation 36, 79 preparation 137
piperidino-, 1- 101
deoxycholic acid 225 1,4-
trimethylsilyloxy-, 1- 169
deoxyribofuranose, 2-, D- heterocyclizations 175
cyclohexenone
2- and 3- 130 -and- 210 dicarboxylic acids 106
cyclooctatetraene 260 deoxyribonucleic acid (DNA) 214 acidity 109
from ethyne 51 deoxyribose, 2-, D- 209, 214 cyclic anhydrides 111
cyclooctene, cis- and trans- 55 descriptors cyclic imides 112
cyclopentadiene (aR) and (aS) 122 monoesters 226
CH acidity 74 (M-)- and (P-)- 123 preparation 108
DIELS-ALDER reaction 57, 228 (R)- and (S)- (CIP) 119 dichlorodifluoromethane (Freon) 77
dimerization 57 (Z)- and (E)- 35 dichloromethane
cyclopentadienide cis- and trans- 35 methylene chloride 32
D and L (FISCHER) 120 dicyclopentadiene 57
anion 74, 174
cyclopentane designer drugs 207 DIECKMANN
carboxylic acid ester 162 DEWAR formulas cyclocondensation 136
conformation 53 of benzene 61 DIELS-ALDER reaction 47, 57, 172
phenyl- 83 dextrorotatory 118 of furan 177
cyclopentanol, 2-methyl- diagonal signals (NMR) 248 regioselectivity 227
shift correlations (NMR) 249 dialdehydes, 1,3- 138 retro- 57
cyclopentanone dialkyl carbonates 155 stereoselectivity 228
preparation 136 diamines 96 stereospecificity 229
cyclophanes 123 diastereomers 124, 228, 237 with quinones 148
cyclopropane(s) aldoses and ketoses 208 dienes
banana bond model 52 epimers 124, 208 1,2- (allenes)
diethenyl- (divinyl-) 57 esters 121, 124 molecular shape 45
ethenyl- (vinyl-) 56 salts 207 preparation 45
preparation 56 diastereoselectivity 201, 228 1,3-
ring opening 58 diastereotopic 230 additions, 1,2- and 1,4- 46
cyclopropenium diazoalkanes 103 cycloadditions, [4+2]- 47, 57
cation 74 carbene precursors 103 polymerization 164
cycloreversion 57 dipoles, 1,3- 175 preparation 45
cysteine 200 resonance formulas 103 1,5-
seleno- 200 diazomethane 27 COPE
cytidine methylation reagent 104 rearrangement 29, 163
deoxy-, 2-, tritylation 226 preparation 104 cumulated, isolated,
cytosine (C) 214 diazonium ions and salts 98 conjugated 44
arene- dienophiles 47, 57, 73, 228
azo coupling 102 diethyl ether 94
conversion into phenols 144 diethylenetriamine 96
resonance stabilization 99
302 Subject Index

diethylzinc 82 DNA (deoxyribonucleic acid) 214 absolute configuration 119


dihaloalkanes, ,- dodecane 16 aldoses and ketoses 208
reductive dehalogenation 37 double bond equivalents 236 amino acids 200
dihedral (torsional) angle 23, 24 double helix of DNA 215 physiological activity 237
and vicinal coupling (NMR) 245 dyeing process separation 121, 213
dihydroxylation vatting/reoxidation 185 enantioselectivity 230
of alkenes 43, 88 dyes 149, 195 enantiotopic 230
diketides 218 azo 102, 196 faces 230
diketones carbonyl 197 endo and exo 228
1,2- 131 phthaleins 197 endothermic 30
1,3- 138 polyaza[18]annulene 198 ene reaction 39
heterocyclizations 179 polymethine 196 energy
oxo-enol tautomerism 140 structural elements 196 diagram of a reaction 30
preparation 137 triarylmethine 197 fossil sources 16
reactions 139 dynamite 91 levels of the electron 8
1,4- production 17
heterocyclizations 175 E enol 50
1,5- 169 ethers 50
dimethyl ether 85, 94 E1 and E2 silyl 168
molecular models 94 elimination mechanisms 36 tautomers 50
dimethyl sulfate 91 eclipsed 24 identification 140
methylation reagent 151 ecstasy 207 proton NMR 243
dimethyl sulfoxide (DMSO) 152 EDA complex 148 enolate anions 136, 138, 227
dimethylamine 133 eicosane 16 enones 130
dinitrobenzene, m- 67 ELBS reaction 71 enthalpy of reaction 30
diols 88 electrofuge 29 envelope conformers
1,2- (glycols) 42 electron sextet,  of cyclopentane 53
cis- and trans- 59 autonomous 70 enzymes 202
oxidative cleavage 89 of benzene 61 epimers 124, 208, 209
preparation 89 electron(s) epoxidation, PRILEZHAEV 42, 59, 171
dehydration 45 energy levels 8 epoxides (oxiranes) 43, 59, 170
dioxane(s) excitation 194 preparation 171
1,2-, 1,3-, and 1,4- 171 solvated 69 reactions 173
dioxine(s) spin 8 polymerization 165
1,4- 171 electronegativity 78 equilibrium constant
dioxoheptanoate, 2,4-, ethyl influences acidity 109
design of synthesis 234 chemical shift (NMR) 242, 246 basicity 98
dioxopiperazine 201 coupling constant (NMR) 247 esterification 110
dipole moment 78 reactivity 78 redox 147
furan, pyrrole, and thiophene 174 electronic ERLENMEYER’s rule 132
pyridine 178 configuration of atoms 9 erucic acid 217
disaccharides 212 transitions of molecules 195 erythro- and threo- 125
reducing and non-reducing 212 electrophile(s) 26, 29, 41, 64, 78, erythrose 124, 208
disconnection, retrosynthetic 232 133 essential oils
examples 233, 234 electrophilic substitution see ethereal oils 221
disproportionation see substitution(s) 29 esterification 91, 110
self-oxidation-reduction 129 elimination(s) esters 91, 110, 144
disulfide(s) 151 HOFMANN 100 active
bridges in rubber 165 - 113, 128 hydroxysuccimidyl-, N- 204
diterpenes 222 - 28, 113 nitrophenyl-, o- 204
dithiane(s) mechanisms poly- 165
1,2-, 1,3-, and 1,4- 171 E1 and E2 36 protective groups 204
1,3- 132 regioselectivity 36, 92, 100 estradiol 225
dithiocarbonic acid 157 stereospecificity 229 estriol 225
dithiocarboxylic acids 152 emodins 219 estrone 225
diynes 51 enamines 100 ethanal (acetaldehyde) 50, 126, 137
DMAP carbon nucleophiles 100 ethane 16
dimethylaminopyridine enantiofacial 230 CC single bond 14
N,N-, 4- 180 enantiomeric excess (e.e.) 231 molecular models 14
DMSO (dimethyl sulfoxide) 152 enantiomers 118, 151
Subject Index 303

ethanol (ethyl alcohol) 85 farnesol 222 FRIES rearrangement 144, 163


molecular models 84 fats (triglycerides) 110, 216 fructofuranose
production 85 transesterification 217 - and -D- 210
ethene (ethylene) 34 fatty acids 110 fructopyranose
CC double bond 14 biosynthesis 218 - and -D- 210
dichloro-, 1,2- saturated 216 fructose 209
configurational isomers 35 unsaturated fruit sugar (fructose) 209
molecular models 14 essential 216 fuels 17
polymerization 164 FAVORSKII rearrangement 162 renewable 217
ethereal oils 221 FEHLING reaction 128 fumaric acid 106
bay 221 ferrocene 74, 82 diester
bergamot 222 aza- 176 dienophile 229
citrus 221 FINKELSTEIN reaction 77 functional groups 7, 28
fir cones 221 FISCHER convention 119 identification
lavender 221 amino acids 200 IR spectroscopy 241
mandarin peel 221 carbohydrates 124, 208 NMR spectroscopy
neroli 222 FISCHER projection 23, 119 carbon-13 (13C) 246
peppermint 221 FISCHER synthesis proton (1H) 242
rose 221, 222 of glycosides 211 interconversions (FGI) 232
ethers 94 of indole 182 ranking 114
allyl flavors, fragrances 221 functional isomers 94, 236
rearrangements 163 almond 126 aldoses and ketoses 208
aryl bay 221 furan(s) 174
preparation 144 blackcurrant 150, 221 nitration 176
autoxidation 95 citrus 221 preparation 175
cleavage 95 coffee 150 resonance formulas 174
cyclic 170, 171 grapefruit 150 tetrahydro- (THF) 170
hydroperoxides 95 lavender 221 furanoses 209
nomenclature 94 mandarines, oranges 221 furfural (furfurol) 174
preparation 94 peppermint 221
trimethylsilyl 168 pineapples 110
WILLIAMSON synthesis 95 rosemary 222
G
WITTIG rearrangement 162 roses 221 GABRIEL synthesis
ethyl acetate 91 sage 222 of primary amines 97
CLAISEN condensation 136 vanilla 126 galactose 208
ethyl acetoacetate flavylium salts 186 gas oil 16
synthesis 136 nucleophilic addition 188 gasoline 16
tautomerism 140 preparation 187 GATTERMANN-KOCH
ethylbenzene 64 fluorination formylation 127
bromination of alkanes 77 geminal 244
electrophilic 69 fluorobenzene genetic code 214
radical 68 dinitro-, 2,4- 63 gentian violet (crystal violet) 197
bromo- folic acid 193 geranial 221
p- 69 folinic acid 193 geranyl diphosphate 219
- 68 formaldehyde (methanal) 126, 133 GLASER coupling 51, 75
ethylenediamine 96 formamides glucaric acid 211
ethyloxonium chloride 90 alkyl-, N- glucitol (sorbitol) 211
ethyne (acetylene) 48 dehydration 113 gluconic acid 211
CC triple bond 15 formic (methanoic) acid 106 glucopyranose, - and -D- 209
dimerization 45 formyl (aldehyde) group 126 chair conformers 209, 236
hydration 50 formylacetone 138 mutarotation 210
molecular models 15 formylation 127 glucopyranoside(s) 210
production 48 four-center mechanisms 40, 42 methyl,- and -D-
exhaustive alkylation Freon identification by NMR 245
of amines 99 dichlorodifluoromethane 77 glucosamine
exothermic 30 FRIEDEL-CRAFTS acetyl-, N- 213
acylation 65, 71, 131, 147 glucose 208
alkylation 64 photosynthesis 199
F FRIEDLÄNDER synthesis glucosides, glycosides 210
farnesane 222 of quinolines 186 glucuronic acid 211
farnesene,- 222
304 Subject Index

glutamic acid 200 HARTWIG-SCHÖLLKOPF substitution


glutamine 200 diastereoselective synthesis electrophilic 181
glycans 212 of amino acids 201 nucleophilic 180
glyceraldehyde 119, 208 hashish 219  electron deficient 178
glycerol 88 heat heterolysis 26
esters (fats) 110, 216 of combustion 17 hexahelicene 123
production 88 of hydrogenation hexane 16
glyceryl trinitrate benzene 60 structural isomers 19
nitroglycerin 91 butadiene, 1,3- 44 hexenal, 2-, 2-ethyl-
glycine 200, 224 of reaction 30 design of synthesis 234
glycolipids 216 HECK reaction 38 hexene
glycols (1,2-diols) helix,- 202 1-, 2-, and 3- 34
oxidative cleavage 89 HELL-VOLHARD-ZELINSKY 2-
glycon 210 halogenation configurational isomers 35
glycosidation 211 of carboxylic acids 115 hexenol, 3-, cis-
glycosides, glucosides 210 heme 203 shift correlation
glycosyl halides 211 in hemoglobin 198 (NMR, HH COSY) 248
glyoxal (ethanedial) 126 hemiacetals 132 hexuloses (ketohexoses) 208
gonane 223, 224 cyclo- 209 HINSBERG separation
gramine 184 hemiaminal 100 of amines 153
grape sugar (glucose) 208 hemiketals 132, 143 histidine 200
GRIGNARD reaction 87, 134 cyclo- 210 in hemoglobin 198
GRIGNARD reagents 79, 82 hemin 199 HOCK process 143
carbon nucleophiles 79, 87 hemiterpenes 221 HOFMANN elimination 100
carboxylation 108 hemoglobin 198, 203 homochirality 201
guanidine 156 heptane 16 homologization
preparation 157 heroin with diazomethane 104, 161
guanidinium cation diacetylmorphine 207 homologous series
resonance formulas 156 heteroalicycles 170 alkanes 16
guanine (G) 191, 214 nomenclature alkanols 84
TRAUBE synthesis 191 HANTZSCH-WIDMAN 170 alkenes 34
gulose 208 preparation 171 alkynes 48
GUSTAVSON synthesis reactions 172 homolysis 26
of cyclopropane 56 ring hormones
gutta-percha 223 expansion and opening 173 peptides 202
heteroatoms 7 steroids 225
H stereogenic centers 122 HSAB principle 79
heterocumulenes 158 HÜCKEL rule 61, 74, 198
half-chair conformer heterocycles, aromatic HUND rule 9
cyclohexane 54 five-membered 174 hybrid orbitals
cyclohexene 55 benzo-fused sp, sp2, and sp3 12
hallucinogens nomenclature 182, 186 properties 13
heroin 207 preparation 182 hybridization
LSD 207 reactions 183 of atomic orbitals 12
phenylethylamines 207 nomenclature 174 influences
THCs 219 preparation 175 CH coupling constant 247
halogenation substitution chemical shift 242, 246
of alkanes electrophilic 176 hydroboration
radical 32 nucleophilic 177 of alkenes 42, 86
of benzene (aromatics)
 electron excessive 174 of alkynes 49
electrophilic 64 hydrocarbons
fused
halogen-metal exchange 83 saturated
nomenclature 190
halohydrins 42, 88 and unsaturated 34, 48
with bridgehead N 190
halomethylsilanes 168 hydrogen
six-membered 178
halonium ions molecule 10
benzo-fused
electrophiles 64 hydrogen bridges
nomenclature 186
HANTZSCH synthesis alcohols 85
preparation 186
of pyridines 179 carboxylic acids 107
reactions 187
HANTZSCH-WIDMAN nomenclature intramolecular 140
nomenclature 178
of heteroalicycles 170 nucleobases 215
preparation 178
Subject Index 305

hydrogen peroxide indolizine 190 isoprenoids (terpenes) 220


production 148 alkyl-, 2- isopropylbenzene (cumene) 65
hydrogenation synthesis 190 hydroperoxide 143
catalytic 17, 40 indoxyl 185 isopropylmagnesium bromide 79
enantioselective 231 inductive effect 78, 84, 265 isoquinoline(s) 186
of acyl halides 112, 127 infrared (IR) spectroscopy 240 amination 188
of alkenes 40 identification of basicity 187
of alkynes 49 functional groups 241 benzyl
stereospecificity 58 initiation alkaloids 206
hydrogen-metal exchange with of polymerization 165 hydrogenation 188
organometallic compounds 83 inositol, chiro- and scyllo- 122 methyl-, 1-
hydrohalogenation insulin 202 alkenylation 189
of alkenes 41 intermediates nucleophilic additions 188
of alkynes 50 reactive 26, 160 oxidation 188
hydroperoxides 95, 143 interorbital angle 12 syntheses 187
hydrophobism, hydrophobic 216 iodoalkanes isothiocyanates 158
hydroquinone 142, 146 preparation 77 production 158
hydroxamic acids 112 ion exchange(r) 99 isothiourea 156
hyperchromic effect 195 ionic bond 10 isothiuronium salts
hyperconjugation 32 IRELAND-CLAISEN alkyl-, S- 157
hypoxanthine 191 rearrangement 169 isovaleric acid 221
hypsochromic (blue) shift 195 isoalloxazine(s) 193 isoxazole(s) 174
isocyanates 158, 159 preparation 141
I by rearrangement 161 IUPAC rules
from acylnitrenes 160 of nomenclature 20
ibuprofen production 158 izines and izidines 190
design of synthesis 235 isoleucine 200
enantiomers 237 isomers
identification reactions J
atrop- 123
aldehydes 128 configurational 237 JONES oxidation reagent 90
aldehydes and ketones 133 alkenes 35 juglone 219
alkenes 40 anomers 209
alkyl halides 76, 157 azobenzenes 102 K
amino acids 201 cycloalkanes, disubstituted 54
amylose (starch) 213 KEKULÉ formulas
cyclohexanols, methyl- 84 benzene 60, 61
carboxylic acids 112 diastereomers 124, 208
enols 140 pyridine 178
epimers 124 keratin 202
primary amines 98 enantiomers 118
idose 208 kerosine 16
fatty acids, unsaturated 217 ketals 132
imidazole(s) 174 oximes, anti- and syn- 133
preparation 175 ketene(s) 29, 158, 160, 161, 218
functional 94, 236 keto esters, - 136, 138
imides, cyclic 112 skeletal
imidoesters 113 tautomerism 140
alkanes, branched ketohexoses (hexuloses) 208
imines 100, 133 and unbranched 18
iminium ions ketones 130
separation 156
resonance-stabilized 100, 133 alkylation, - 101, 169
alkynes and dienes 48
inclusion compounds 156 regioselectivity 227
heteroaromatics
indicators BAEYER-VILLIGER oxidation 161
azines 178
methyl orange 102 bimolecular reduction 89
azoles 174
phenolphthalein 197 halo-, -
benzo-fused 182, 186
indigo 185, 197 regio- 33, 41, 236 FAVORSKII
indole (benzo[b]pyrrole) 182 substituted benzenes 62 rearrangement 162
alkaloids 206, 207 isonitriles (isocyanides) hemiketals 143
carboxylation 184 preparation 113 homologization 105
FISCHER synthesis 182 isopinocampheol hydrates 132
hydroxy- carbon-13 NMR spectra 247 hydroxy-
tautomerism 185 isoprene 164 - (acyloins) 113, 134
substitutions rule 220 - 169
electrophilic (SE) 184 units 220 ketals 132
indolizidine 190
306 Subject Index

ketones LSD mescaline 207


preparation by lysergic acid meso-isomers 125
acylation of arenes 131 N,N-diethylamide 207 mesomeric (resonance) effects
alkylation of aldehydes 133 lubrication oils 16 ()-M and ()-M 66
oxidation 90, 131 lumiflavine 193 metabolites 190
reactions 132 lupinine 206 metal chelates 141
ketoses 209 lycopene 223 ligands
cyclohemiketals 210 lysergic acid 207 bipyridine, 2,2'- 179
diastereomers lysine 200, 206 di- and polyamines 96
and enantiomers 209 lyxose 208 dicarbonyl
reduction to polyols 211 compounds, 1,3- 141
kinetically controlled 31, 46 M hydroxamic acids 112
KNOEVENAGEL phthalocyanines 198
alkenylation 38, 139, 189, 218 macromolecules porphyrins 198
KNORR synthesis polymers 164 metalation
of pyrroles 175 macroradicals 165 of alkyl
KOENIGS-KNORR synthesis Makrolon 166 and aryl halides 82
of -glycosides 211 MALAPRADE cleavage metallocenes 74, 82
KOLBE synthesis of 1,2-diols 89 metathesis 39
of nitriles 109, 113 maleic acid 106 ring-closing 57
KOLBE-SCHMITT anhydride 111 meth (speed, crystal) 207
carboxylation of phenoxides 108 dienophile 73, 228, 229 methane 16
malondialdehyde bond data 12
propanedial 126, 138 molecular models 12
L malonic (propanedioic) acid 106 molecular orbital model 13
lactams 117, 192 activated (malonyl-S-CoA) 218 methanol (methyl alcohol) 84
lactic acid 117 decarboxylation 113 bond data 84
L-(+)- 120 diesters 138 production 85
polyesters 166 alkenylation 139 methionine 200
lactides 117 alkylation 138, 201 methyl
lactims 192 maltose (malt sugar) 212 anion 26, 27
lactones 117 mandelic acid 134 cation 26
lactose (milk sugar) 212 (S)-, diastereomeric esters 124 radical 26
lanosterol 223, 224 MANNICH methyl chloride (chloromethane) 32
lauric acid 216 electrophile 133 methyl ketones (2-alkanones) 130
LAWESSON’s reagent 152 reaction 137, 184 from terminal alkynes 50
LDA mannopyranoside methyl orange
lithium diisopropylamide 227 methyl - and -D- 211 indicator 102
leaving group 28 mannose 208 methylene chloride
lecithins 216 margaric acid 216 dichloromethane 32
leucine 200 marihuana 219 methylmagnesium iodide 79
leucopterin 193 MARKOVNIKOV rule 41, 85 methylpropane (isobutane)
levorotatory 118 mass spectrometry 238 molecular models 18
LEWIS acids 50 fragment ions 239 methylpropene (isobutene) 92
catalysts 50, 64 fragmentations 239 hydration 41
LEWIS formula 22 high-resolution 238 MICHAEL addition 139, 173
lignoceric acid 216 mass spectrum 238 MUKAIYAMA- 169
ligroin 16 molecular formula 238 milk sugar (lactose) 212
limonene 57, 259 molecular ion (peak) 238 molecular formula 16
enantiomers 221, 291 MCMURRY reaction 37 determination
LINDLAR catalyst 49, 127 mechanisms carbon-13 NMR 247
linear fusion E1 and E2 36 mass spectrometry 238
of benzene rings 70 mono- and bimolecular 36, 80 double bond equivalents 236
linoleic acid 216 SN1 and SN2 80 hydrogen deficit 236
linolenic acid, - 217 SN1 pull and SN2 push 173 molecular orbital models
lipids 216 menthane(s), p- 220 allene 45
lipophilism, lipophilic 216 menthol, enantiomers 221, 291 benzene 61
lithium aluminum hydride mercaptals multiple bonds 14
reducing reagent 86 thioacetals and thioketals 132 single bonds 13, 14
lithocholic acid 225 MERRIFIELD solid-phase
peptide synthesis 205
Subject Index 307

molecular orbitals nucleobases 214 nitrenium ions 160, 161


bonding and antibonding 10 nucleosides 214 nitriles (cyanides) 108
 nucleotides 214 hydrolysis 109
delocalized 44, 61 polyketides 218 preparation 113
 and  11 proteins 202 reduction 97
molecular spectra steroids 224 nitro group
electronic (UV-vis) 194 terpenes 220 ()-M effect 66
infrared (IR) 240 natural products from nitrogen rule 238
mass 238 black pepper 171 nitroglycerin
NMR 243, 246, 248 camphor tree 222 glyceryl trinitrate 91
molecular vibrations carrots 223 nitronium ion
bending (deformation) 240 cereals 213 electrophile 65
IR active 240 Cinchona bark 207 nitrophenol
stretching (valence) 240 coca bush 206 o- and p- 66, 142, 145
moniliformin 218 cocoa 191 nitrosation
monomers 164 cod-liver oil 223 electrophilic 99
monosaccharides 212 coffee 191 nitrosoamines, N- 99
monoterpenes cotton 213 NMR 242
acyclic 221 cottonseed 223 carbon-13 (13C) 246
mono- and bicyclic 221 fir cones 221 subspectra 247
morphine 207 fungi 207, 213 chemical shift
MUKAIYAMA Indian hemp 219 carbon-13 (13C) 246
aldol reaction 169 linseed 217 proton (1H) 242
MICHAEL addition 169 maté 191 standardized  242
MÜLLER-ROCHOW process 168 meteorites 222 coupling constants
multiplets (NMR) mold 218 carbon-proton
carbon-13 247 olive 216 one bond
proton 243 peppermint 221 and carbon
multiplicity rule (NMR) 244 peyote cactus 207 hybridization 247
mutarotation 210, 212 pines 221 proton-proton 243
myrcene poison hemlock 219 geminal 244
- and - 221 poppy (opium) 207 vicinal 244
myristic acid 216 potatoes 213 integral curves 243
rape 217, 223 multiplicity rule 244
rhubarb root 219 proton (1H) 242
N rosehips 223 quantitative analysis 243
naphthalene 70 senna leaves 219 signal 243
catalytic hydrogenation 72 slate oil 222 multiplicity 243
electrophilic substitution 71 sugar beet 213 spectrum 243
molecular orbital model 70 sugar cane 213 spin-spin coupling 243
oxidation and reduction 72 tea 191 two-dimensional 248
resonance formulas 70 tobacco 206 nomenclature
naphthol tomatoes 223 alcohols 84
- and - 142 turpentine oil 222 aldehydes 126
naphthonium ions vegetable oils 217, 223 alkanes 16, 20
resonance formulas 71 walnut shells 219 alkenes 34
naphthoquinone neral 221 alkynes 48
1,2- 146 nerolidol, (S)-(+)- 222 amines 96
1,4- 146 NEWMAN projection 23, 245 aromatic compounds
hydroxy-, 5- 219 nicotine 206 benzenoid 62
tetrahydro- 148 ninhydrin reaction aromatic heterocycles 174, 178
2,6- 146 of amino acids 201 benzo-fused 182, 186
naproxen, (S)-()- 231 nitration carboxylic acids 106
natural gas 16 electrophilic cycloalkanes 52
natural products benzene 65 ethers 94
alkaloids 206 heteroaromatics 176, 181, 184, ketones 130
amino acids 200 189 multifunctional compounds 114
carbohydrates 208, 212 naphthalene 71 phenols 142
fatty acids 216 nitrenes terpenes 220
nucleic acids 214 acyl- 160, 161 nonane 16
308 Subject Index

norcaradiene 104 oxetane(s) 170 phenazine 186


nuclear preparation 172 phenol(s)
magnetic resonance (NMR) 242 oxidation(s) 28 acidity 142
spin anodic 17 allyl-, o- 163
precession 242 oximes 133 dinitro-, 2,4- 68
quantum number 244 BECKMANN rearrangement 162 methylation, O- 104
nucleic acids oxiranes (epoxides) 43, 59, 170 nomenclature 142
DNA, RNA 214 preparation 42, 171 oxidation
structure from carbonyl compounds 105 to aroxyl radicals 145
DNA 215 reactions 173 to quinones 146
nucleobases 190, 214 polymerization 165 preparation 68, 143
complementary 215 oxonium ions and salts 36, 95 reactions 144
nucleofuge 28 oxygen phenolphthalein 197
nucleophile(s) 27, 28, 78 biradical 95, 143, 145, 148, 151 phenones 130
nucleophilic substitution by photosynthesis 199 preparation 131
see substitution(s) 28, 80 ozone phenonium ions
nucleosides 214 canonical formulas 43 resonance stabilization 64
nucleotides 214 killers 77 spirocyclic 143
NYLANDER reaction 128 ozonides, ozonolysis 43 phenoxide (phenolate)
Nylon anion
-6 (Perlon) 167 P resonance formulas 142
-6,6 166 carboxylation 108
PAAL-KNORR synthesis phenoxyl radical
of furan, pyrrole, tri-t-butyl-, 2,4,6- 145
O and thiophene 175 phenyl group 62
octane 16 reversal 177 phenylacetic acid
bromo-, 2-, (R)-()- 121 palmitic acid 216 chloro- 115
dimethyl-, 2,6- 220 PAN (polyacrylonitrile) 164 phenylalanine 200
octanol, 2-, (S)-()- 121 papaverine 206 phenylenediamine(s)
olefins paracetamol o-, m-, and p- 96
see alkenes 34 acetylaminophenol, p- 226 phenylethylamine
oleic acid 216 PATERNO-BÜCHI reaction 172 alkaloids 207
oligomers 164 PAULI principle 9 phenylhydrazones
oligopeptides 202 PAULING electronegativities 78 dinitro-, 2,4- 133
oligosaccharides 212 PE (polyethene) 164 phenyllithium 82
olivetolic acid 219 pelargonic acid 216 pheromones 221
opium 207 pentacene bark beetle 221
OPPENAUER oxidation 131 preparation 71 honeybee 221
optical activity 118 pentadecane 16 phloroglucinol 219
orbital symmetry 11 pentadiene phosgene 154
organometal compounds 82 1,2-, 1,3-, and 1,4- 44 phosphatides 216
aluminum 82 pentaketides 219 phosphonium salts
cadmium 82 pentane 16 alkyltriphenyl 134
carbon nucleophiles 83, 87 structural isomers 19 photocycloaddition(s)
lithium 82 pentene, 1- and 2- 34 [2+2]- 56
magnesium 79, 82 peptide(s) 202 photodissociation
metallocenes bond 202 of halogens 32
 complexes 74, 82 sequence 202 photohalogenation
preparation 82 synthesis (coupling) 204 of alkanes 32
sodium 17 perfumes 221, 222 of carboxylic acids 115
zinc 82, 116 peroxides 145 photosynthesis 199, 208
organosilicon compounds 168 endo- 148 phthalaldehydes
ornithine 206 hydro- 95 o-, m-, and p- 126
orthocarboxylic acids 110 petroleum phthalazine 186
oxalic (ethanedioic) acid 106, 109 coke 16 phthalic acid 106
oxaziranes 171 ether 16 anhydride 72, 111
oxazole(s) 174 fractions 16 acylation reagent 147
preparation 175 phenanthraquinone phthalimide 112
oxenium ions 143, 160 9,10- 72, 146 nucleophile 97
oxepine, dihydro-, 4,5- 173 phenanthrene 70
reactions 72
Subject Index 309

phthalocyanines 198 polyphenyls 62 purine(s) 190


metal chelates polypropene (polypropylene) 164 nucleobases 214
pigments 198 polysaccharides 212 preparation 191
phytane 222 polyterpenes 223 PVC (polyvinyl chloride) 164
phytol 222 polytransesterification 166 pyran
in chlorophyll 199 polyureas 167 dihydro-, 3,4-, 2H- 173
phytopharmaceuticals 221 polyurethanes 167 formyl-, 2- 172
picric acid polyvinyl chloride (PVC) 164 tetrahydro- 170
trinitrophenol, 2,4,6- 145 POMERANZ-FRITSCH synthesis pyranoses 209
PICTET-SPENGLER synthesis of isoquinolines 187 pyrazole(s) 174
of isoquinolines 187 porphine 198 preparation 141, 175
pigments 195 porphyrins 198 pyrazolines 172
pinacol 89 PP (polypropene) 164 pyrene 70
reaction 89 pregenolone 225 pyridine(s) 178
rearrangement 93, 161 pregnanes 225 alkylation, arylation 180
pinacyanol 196 PRILEZHAEV epoxidation 42, 59, 171 amino- 180
pinane 222 primary structure basicity 180
pinene, - and - 222 of proteins 202 carbaldehyde, 2- 127
piperidine(s) 96, 117, 170 prochirality 230 catalytic hydrogenation 172
from pyridines 172 progesterone 225 dicarboxylic acids 188
piperine 171 projection formulas dimethylamino-, N,N-, 4- 180
pivalic acid 108 FISCHER 23 HANTZSCH synthesis 179
platforming processes 63 NEWMAN 23, 245 N-oxides 180
polyacylation 166 tetrahedral 23 electrophilic
polyaddition 167 proline 200 substitutions 181
polyalkenes 164 propagation preparation 178
polyamides 166 of polymerization 165 resonance formulas 178
polyamines 96 of radical substitution 32 substitutions
polyaza[18]annulenes 198 propane 16 electrophilic 181
polycarbonates 155, 166 radical halogenation 33 nucleophilic 180
polycondensation 166 propanol pyridinium salts
polyenes 44 1- 87 alkyl-, N- 180
fatty acids 216 2-methyl-1-phenyl- 87 pyrimidine(s) 141, 178
light absorption 195 2- (isopropyl alcohol) 87 amino-, 2- 179
polyesters 165 propen-1-ol, 2- (allyl alcohol) 84 nucleobases 214
biologically degradable 166 propene 34 preparation 179
polyethene (polyethylene) 164 diphenyl-, 1,2- pyrimidones, 2- 179
polyisoprene cis- and trans- 229 pyrrole(s) 174, 175
cis- and trans- 223 propionaldehyde (propanal) 126 acetylation 176
polyketides (acetogenins) 218 propionic (propanoic) acid 106 acidity 176
biosynthesis 218 production 107 basicity 176
polylactides (polylactates) 166 pro-R and pro-S 230 KNORR synthesis 175
polymerization 164 prosthetic group 203 resonance formulas 174
radical protective groups 168, 204 ring opening 177
mechanism 165 proteins 202 pyrrolides 176
ring opening 166 amino acids 200 pyrrolidine(s) 117, 171
vinyl 164 biological functions 202 alkaloids 206
polymers 164 structure pyrrolines, 3- 173
bio- 202, 212, 214, 223 primary 202 pyrrolizidine 190
diene 164 quaternary 203 pyrrolizinium ion
polyamides 166 secondary 202 dehydro- 190
polyesters 165 tertiary 203 pyrylium ions, salts 178
silicones 169 proton (hydrogen 1H) preparation 179
unsaturated 164 NMR 242
vinyl 164 protoporphyrin 199 Q
polymethine dyes 196 psicose 209
polynucleotides 214 pteridine(s) 192 quantitative analysis
polyols 88 preparation 192 by proton NMR 243
alditols (sugar alcohols) 211 PTFE (polytetrafluoroethene) 164 quantum numbers 8
polypeptides 202 quaternary structure
of proteins 203
310 Subject Index

quaternization reactions phenol, phenoxide 142


of amines 99 basic types 28 stabilization
quinazoline 186 chirogenic 121, 230 benzene 61
basicity 187 control butadiene, 1,3- 44
quinhydrone 148 kinetic and thermodynamic 31 of dyes 196
quinine 207 reactive intermediates 26, 160 resonance formulas
quinoline(s) 186 rearomatization 64 benzene 61
alkaloids 207 rearrangements 160 butadiene, 1,3- 44
amination 188 anionotropic 29, 160 carbanion and
basicity 187 BAEYER-VILLIGER oxidation 161 enolate anion 136, 227
halo- BECKMANN 162, 167 carbenium ions
nucleophilic substitution 188 cationotropic 29, 162 allylic 46
hydrogenation 188 CLAISEN 163 benzylic 81
methyl-, 2- COPE 29, 104, 163 furan, pyrrole, thiophene 174
ester condensation 189 diaza- 182 naphthalene 70
nucleophilic addition 188 oxa- 163 phenonium ions 64
oxidation 188 CURTIUS 161 pyridine 178
preparation 186 FAVORSKII 162 radicals
substitutions FRIES 163 allylic 47
electrophilic 189 HOCK process 143 benzylic 68
nucleophilic 188 HOFMANN 161 substituted benzenes
quinolizidine 190 IRELAND-CLAISEN 169 dipolar (zwitterionic) 66
quinolizinium ion LOSSEN 161 sulfur trioxide 65
dehydro- 190 pinacol 161 ylides and ylenes 134
quinolone 188 sextet 160 resorcinol 142
quinones 146 sigmatropic 29, 163 tautomerism 146
additions 148 SOMMELET-HAUSER 163 retinal 222
cycloadditions, [4+2]- 148 STEVENS 162 retinol 222
nomenclature 146 WAGNER-MEERWEIN 93, 161 retron 232
o- and p- 146 WITTIG 29, 163 retrosynthesis 232
preparation 146 WOLFF 29, 161 reversal of polarity
oxidation of arenes 72 reduction(s) 28 alkyl halides 79, 83
reduction 147 reductive amination carbonyl group 133
quinoxaline(s) 186 of carbonyl compounds 101 pyridine 180
preparation 192 REFORMATSKY reaction 116 riboflavin 193
regioisomers 33, 41, 84, 236 ribofuranose, D- 209
R cyclohexenes 227 deoxy-, 2- 210
ketones 227 in DNA 214
racemates 121 substituted benzenes 62, 66 - and - 210
resolution 121, 213 regioselectivity 226 in RNA 214
radical substitution additions - and - 209
see substitution(s) 29 hydration 41, 85 ribonucleic acid (RNA) 214
radicals 32 hydrohalogenation 41, 115 ribose 208, 209
initiating 32, 165 eliminations ribulose 209
methyl 26 dehydration 92 RILEY oxidation 131
oligomeric and polymeric 165 dehydrohalogenation 36 ring
resonance-stabilized 47, 68, 145 HOFMANN 100 contraction 162
thiolyl 151 substitutions expansion 56, 57, 173, 185
raffinose 213 electrophilic 66, 69, 71, 145, inversion
rape oil methyl esters (RMEs) 217 149, 176, 181 of cyclohexane 53
RASCHIG process 68, 143 nucleophilic 180 opening 58, 108, 173, 177
rate radical 33, 68 ring size
constant 36, 80 REIMER-TIEMANN formylation 128 determination 248
-determining steps resonance RNA (ribonucleic acid) 214
of reactions 36, 81 effects (mesomeric effects) ROBINSON synthesis
law ()-M and ()-M 66 of benzopyrylium salts 187
first- and second- energy 61 ROSENMUND reduction 112, 127
order 36, 80 benzene 60 rotation, optical
Re and Si face 230 butadiene, 1,3- 44 specific 118
reaction enthalpy 17 fused aromatics 70 rubber 165
Subject Index 311

S SOMMELET-HAUSER stilbene (1,2-diphenylethene)


rearrangement 163 cis- and trans- 62, 287
saccharides sorbose 209 stimulants 206
mono-, oligo-, and poly- 212 specific rotation 118 purine 190
saccharose (sucrose) 213 speed (meth, crystal) 207 STORK enamine reaction 101
salicin 210 spin structural formulas
salicyl alcohol 210 electron condensed 22
salicylic acid 106 antiparallel and parallel 8 LEWIS 22
dinitro-, 3,5- 63 nuclear 242 projections 23
production 108 -spin coupling (NMR) skeletal 22
SANDMEYER reaction 103 carbon-carbon (13C13C) 246 structure elucidation 43, 237
sandwich complexes 75 carbon-proton (13C1H) 247 IR spectroscopy 241
saponification 91, 110, 217 proton-proton (1H1H) 243 mass spectrometry 239
SAYTZEFF rule 36, 92 systems (NMR) 244, 248 NMR spectroscopy 245, 248
SCHIFF bases (imines) 100, 133 spirocycles 52 strychnine 207
SCHLENK equilibrium 79 axial chirality 122 styrene 62
secondary structure squalene 223 substituent(s)
of proteins 202 epoxy-, 2,3- 223 alkyl groups
self-oxidation-reduction squaric nomenclature 20
of aromatic aldehydes 129 and semisquaric acid 218 at stereogenic centers
semiquinone staggered 24 ranking 119
radical anion 148 starch 213 donors and acceptors 66
sequence stearic acid 216 effects
nucleic acids 214 stereocontrol 42 ()-M and ()-M 66
peptides and proteins 202 stereogenic centers 118 ()-I 78
serine 200 heteroatoms 122, 151 electron-releasing
sesquiterpenes 222 stereoisomers 237 and -withdrawing 66
sheet structure anomers 209 increments
of proteins 202 configurational isomers 35, 54, chemical shifts (13C NMR) 246
shifts 55, 125 substitution(s) 28
anionotropic 1,2- 29, 160 descriptors 35, 119, 120, 122 electrophilic (SE) 29
cationotropic 1,2- 29, 162 diastereomers 124, 228 acylation 65
chemical enantiomers 118 alkylation 64
see chemical shift 242 endo-, exo- 228 aromatic compounds
sigmatropic 163 epimers 124 anthraquinone, 9,10- 149
signal multiplicity (NMR) 243 stereoselectivity 227 benzenoid 64
silanediols 168, 169 diastereoselectivity 201, 228 regioselectivity 66
silanes 168 enantioselectivity 230 naphthalene 71
enantiomers 122 stereospecificity 40, 228 aromatic heterocycles
silanols 168 additions five-membered 176, 177
silazanes 168 bromination 40, 58 benzo-fused 184
silicones 169 dihydroxylations 43, 59, 88 indole 184
siloxanes 168 hydrogenation 40, 49, 58 six-membered 181
poly- 169 cycloadditions 47, 56, 229 benzo-fused 189
silyl enol ethers 168 eliminations, E2 229 quinoline 189
skeletal formula 22 glycosidation 211 pyridine 181
skeletal isomers 18, 236 substitutions, SN2 121 azo coupling 102
skeletal structure 18
steric () effects carboxylation 108
determination
chemical shifts 247 formylation 127
IR spectroscopy 241
steric interaction 25 halogenation 64
mass spectrometry 239
steroids nitration 65
NMR spectroscopy 244, 248
androstanes 225 nitrosation 99
skew conformers 24
bile acids 224 regioselectivity 66, 67
SKRAUP synthesis
estranes 225 sulfonation 65
of quinolines 186
parent skeletons 224 nucleophilic (SN) 28, 95
SN1 and SN2
pregnanes 225 alkyl halides 79, 80
substitution mechanisms 80
ring fusion 224 chlorobenzenes 68
soaps
sterols 224 cleavage of ethers 95
hard and soft 110
STEVENS rearrangement 162 isoquinoline 188
anions 217
312 Subject Index

substitution(s) tartaric acid theobromine 191


nucleophilic (SN) enantiomers theophylline 191
mechanisms 80 and meso-isomer 125 thermodynamically controlled 31, 46
SN1 81 taurine 224 THF (tetrahydrofuran) 170
SN2 80 tautomerism, tautomers thiazole(s) 174
stereospecificity 121 carbodiimide-cyanamide 159 preparation 175
pyridine 180 hydroxamic acids 112 thietane 170
quinoline 188 hydroxyindole-oxindole 185 reactions 172
radical (SR) 29, 68 lactam-lactim 188, 192 thiirane 170
halogenation 32, 38 oxo-enol or keto-enol 50, 140 thioacetals 132
regioselectivity 33 analysis by NMR 243 thioaldehydes 152
nitration 33 porphyrins 198 thiobenzopyrylium ion 186
sulfochlorination 33 thiourea-isothiourea 156 thioethers 150
substrate 28 TCDD cyclic 170, 171
succindialdehyde tetrachlorodibenzo[b,e][1,4]di- preparation 151
butanedial 126, 177 oxine, 2,3,7,8- 171 thioketals 132
succinic (butanedioic) acid 106 Teflon (polytetrafluoroethene) thioketones 152
anhydride 111 PTFE 164 thiolic acids 152
succinimide 112 terephthalic acid thiols 150
bromo-, N- 38, 76 polyester 166 acidity 150
hydroxy-, N- 204 termination natural occurrence 150
sucrose (saccarose) 213 polymerization 165 oxidation 151
sulfanilic acid radical substitution 32 preparation 150
azo coupling 102 terpenes 220 thionaphthene
sulfenic acids di- 222 benzo[b]thiophene 182
derivatives 151 hemi- 221 thionic acids 152
sulfenyl chlorides 151 mono- 219, 221 thiophene(s) 174
sulfinic acids 152 parent hydrocarbons 220 desulfurization
sulfolane 47 poly- 223 by hydrogenation 177
sulfonamides 153 sesqui- 222 preparation 175
sulfonation tetra- 223 resonance formulas 174
electrophilic tri- 223 thiophenols 150
benzene 65 terphenyl 62 thiopyrylium ions, salts 178
naphthalene 71 tertiary structure thiourea(s) 159
sulfones 152 of proteins 203 nucleophile 157
sulfonic acids Terylene 166 preparation 157
amides 153 testosterone 225 tautomerism 156
aromatic 65 tetraalkylammonium salts thiourethane(s) 158
chlorides 153 enantiomers 122 threo- and erythro- 125
esters 153 tetraalkylstannanes 83 threonine 200
preparation 153 tetracene 70 threose 124, 208
sulfonium salts tetrachloromethane thymine (T) 214
enantiomers 122 carbon tetrachloride 32 tiglic acid 221
sulfonyl chlorides 153 tetradecane 16 tilidine 227
sulfoxides 152 tetrahedral bond angle 12 TISHCHENKO reaction 129
sulfur trioxide tetrahydrocannabinols TMSCl (trimethylsilylchloride)
electrophile 65 design of synthesis 235 chlorotrimethylsilane 168
sulfuric acid 8- and 9- 219 tobacco smoke 70, 73
mono- and diester 91 tetrahydrofuran (THF) 170 TODD synthesis
sweeteners 212, 213 tetraketides 219 of adenine 192
non-nutritional tetralin and isotetralin 72 tolane 62
aspartame 237 tetramethylsilane (TMS) 168 TOLLENS reaction 128
glucitol (sorbitol) 211 NMR standard 242 toluene 62
syn-clinal (gauche) 25 preparation 83 bromination, radical 76
synthetic equivalent 232 tetraterpenes 223 production 63
synthon 232 tetrazole(s) trinitro-, 2,4,6- 63
preparation 175 toluidines
T tetrulose 209 o-, m-, and p- 96
THCs
tagatose 209 tetrahydrocannabinols 219
talose 208
Subject Index 313

tosylates U W
toluenesulfonic acid, p-
esters 153 umpolung WACKER process 131
toxins alkyl halides 79 WAGNER-MEERWEIN
bacterial 191 carbonyl group 133 rearrangement 93, 161
myco- 218 pyridine 180 WALDEN inversion 80, 121
transesterification 217 undecane 16 water
poly- 166 uracil (U) 214 photolysis 199
transition states 30 urea(s) 154 wave functions
cyclic 40 acyl- (ureides) 156 of electrons 8
dehydrohalogenation 36 alkylated 156 linear combination 10
SN2 reactions 80 basicity 155 wavenumber 240
transmetalation dialkyl-, N,N´- 159 waxes 217
of organometallic compounds 82 from isocyanates 159 whisky (whiskey) 85
TRAUBE synthesis poly- 167 WILLIAMSON synthesis
of guanine 191 syntheses 156 of ethers 95
trehalose 212, 213 WÖHLER 7 aryl 144
triacontane 16 ureides 156 cyclic 171
trialkylboranes 42 cyclic 141 of thioethers 151
trialkylsulfonium salts 151 urethane(s) 154 wines 85
triarylmethine dyes 197 ethyl 155 WITTIG alkenylation
triazolines 172 poly- 167 of carbonyl compounds 38, 135
trichloromethane (chloroform) 32 preparation 158 WITTIG rearrangement 163
tridecane 16 protective group 204 WÖHLER synthesis
trifluoroacetic acid uric acid 191 of urea 7
anhydride 111 oxidative cleavage 192 WOHL-ZIEGLER
triglycerides (fats) 110, 216 uronic acids 211 bromination 38, 76
transesterification 217 uronium cation WOLFF rearrangement 161
triketides 219 resonance formulas 155 WOLFF-KISHNER reduction 135
trimethylamine WURTZ synthesis
shape and MO model 96 V of alkanes 17
trimethylsilyl ethers 168 valeraldehyde (pentanal) 126
trimethylsilylation 168 valeric (pentanoic) acid 106 X
triols 88 bromo-, - 115 xanthine 191
triphenylphosphane 134 valerolactam, - 117 xanthogenates 157
trisaccharides 213 xanthogenic acids 157
valerolactone, - 117
triterpenes 223 xanthopterin 193
valine 200
trithiocarbonic acid xylene
VAN SLYKE reaction 98
salts and esters 157 o-, m-, and p- 62, 69
of urea 156
trityl (triphenylmethyl) xylose 208
vanillin 126
chloride 226 xylulose 209
vatting 185
ether 226
vibrational spectroscopy 240
tropane
alkaloids 206
VILSMEIER formylation 127 Y
vincamine 206
tryptamine 206 ylenes 134
vinyl
tryptophane 200, 206 ylides 134
group and compounds 34, 244
turpentine 220 nitrogen 162
polymers 164
twist-boat conformers
visible light absorption
cyclohexane 54 Z
and color 194
tyrosine 200, 206
vitamins ZEREWITTENOFF reaction 79
A 222 zingiberene, ()- 222
B 193 zwitterions 200
vulcanization 164
314 Periodic Table

Periodic Table
Periodic Table
of the Elements 315

of the Elements
of the Elements
316 Selected Reference Sources

Selected Reference Sources


Selected Reference Sources

M.B. SMITH
MARCH´s Advanced Organic Chemistry
7th ed., Wiley & Sons, Hoboken (N.J., USA), 2013
comprehensive textbook with references to the primary and review literature

The MERCK Index


An Encyclopedia of Chemicals, Drugs, and Biologicals
16th ed., MERCK Research Laboratories, MERCK & Co., Inc., Whitehouse Station (N.J., USA), 2020
more than 10 000 monographs, organic name reactions,
with references to the primary literature

RÖMPP Online Encyclopedia


almost all topics of chemistry including natural products
in German language, English nomenclature of compounds,
with references to the primary and review literature
access via www.thieme-chemistry.com

THE AMERICAN CHEMICAL SOCIETY


Chemical Abstracts Service (CAS) data base
abstracts of primary chemical literature, permanently updated
access via "SciFinder Scholar" and www.cas.org

D. RIDLEY
Information Retrieval: SciFinder
2nd ed., Wiley, 2009

source of Figs. 2.1, 3.2, 3.3, 3.4, 6.1, 14.1, 67.1, 69.2, 84.1, 86.1, 86.2:
E. BREITMAIER, G. JUNG, Organische Chemie, 8th ed., Wiley VCH, Weinheim (Germany), 2021
access via www.wiley-vch.de
source of the Periodic Table of the Elements
C.E. MORTIMER, U. MÜLLER, Chemie, 13th ed., Thieme, Stuttgart (Germany), 2019
translated and modified
WILEY END USER LICENSE AGREEMENT
Go to www.wiley.com/go/eula to access Wiley’s ebook EULA.

You might also like